Download as pdf or txt
Download as pdf or txt
You are on page 1of 497

MIR

PUBLJ:SHF.I(S
r. c. 5apaHeHK06. 5. n DeMUiJOf1U't, B. A Ef/JuMeHfCO,
C. M. Kocan. r Jl Jlynu,, E lP flopumeea, E fl. Cwteea,
C. B. l/>po/108, P. fl. lllocmaK, A. P. fl nnollbCKUiJ.

3AJIA4M 11 YnPA)I(HEH11S1
no
MATEMATM4ECKOMY

AHAJ1113Y

lloiJ penaKl{Ut>iJ.
B. n. A EM 11 A 0 B H q A

TocgiJapcmBeHHO!! U3iJameAbCmi>O
t/Ju3uKO-MameMamu 'leeK oil. 11u mepamypw
MocKtHI
G. Baranenkov, B. Drmidovich V. Efimenko, S. Kogan,
G. Lunt~. E. Por!:.hmva, E. ~!;t:hrta, S. frolw, R. ~ho!>tak,
A. Yanpolsky

PROBLEMS
IN
MATHEMATICAL
ANALYSIS

Under the editor.,hip


of
B. DEMI DOVICH

Translated from the Russian.


by
G. YAN KOVSKY

Mil{ PUBLISHEI{S
Moscow
TO THE READER
Ml R Publishers would be glad to have your
opinion of the translatwn and the design of thls
book.
Please send uour suggest tons to 2, Pervy Rtzh:.ky
Pereulok, Mo:.ww, U. S. S. R.

Second PrintiR/l

Prlntecl ln the Unlon of Soviet Socialist Republics


CONTENTS

Preface • • . • , ••••• 9

Chapter I. INTRODUCTION TO ANALYSIS


Sec. l. Functions . . . . . . 11
Sec. 2 Graphs of Elementary Functions 16
Sec. 3 Limits . . . . . . . . . . . . 22
Sec. 4 Infinitely Small and Large Quanltties 33
Sec. 5. Continuity of Functions . . . . . . 36

Chapter 11 DIFFERENTIATION Of .fUNCTIONS


Sec 1. Calculating Derivativl's Directly . . . • • . . . . . • • 42
Sec 2 Tabular Differentiation . . . . . . . . . . . . . . 46
Sec. 3 ThE.' Derivat•ves of Functions Not RE.'presented Explicitly 56
Sec. 4. GE.'oml'trical and Mechanical Applications of the Derrvative 60
Sec 5 Denvallveo; of Hig1er Orders ..... 66
Sec 6 Differentials of First and Higher Orders 71
Sec 7 Mean VahlE.' Theorems . 75
Sec. 8 Taylor'5 Formula 77
Sec 9 The L'Hospital-Bernoulli Rule for Evaluating Indeterminate
Forms .......•. . . . . • . . 78

Chapter Ill THE EXTREMA Of A FUNCTION AND THE GEOMETRIC


APPLICATIONS Of A DERIVATIVE
Sec. l. The Extrema of a Function of One Argument 83
Sec. 2 The Direction of Conravity Points of I nllection 91
Sec 3 Asymptote<; 93
Sec 4. Graphmg Functions by Characteristic Points 96
Sec. 5. DiiTerential of an Arc Curvature 101

Chapter IV IIIIDEFINITE INTEGRALS


Sec. l Direct Integration . . 107
Sec 2 Integration by Substitution • 113
Sec 3 1ntegral•on by Parh 116
Sec. 4 Standard Integral~ Containing a Quadratic Trinom1al 118
Sec. 5. Integration of Rational Functions • , , • • • , • • • • 121
Contents

~ec. 6. I ntq1ratin11 Certain lrraflonal Functions •• 125


~Ec 7. lr.tq:rating lrironc-rretric Functions . . 128
~ec. 8 ln1e!lration of Hyrerbolic Functions . . 133
Sec 9. Usmg lr1gonometric and Hyperbolic Substitutions for Finding
Integrals of the Form ~ R (x. Vax 2
+bx+c) dx. Where R IS a Ra·
tional Functwn . . . . . . . . . . . . . . . . . . . 133
~ec 10 I ntq:rat ion of Vanou• Transcendental Functions 135
Sec II Usin!l Reduction Formulas . . . . . . 135
Sec. 12. Miscellaneous Examples on Integration 136

Chapter V DEFINITE INTEGRALS


Sec. 1. The Definite Integral as the Limit of a Sum • , , • • • 138
~Ec 2 fvaluata;p [, firite lntrgrals by N.eans of Indefinite Integrals 140
Sec. 3 Improper Integrals . . . • 143
Sec 4 Charge ~f Vari2ble in a Ddinite Integral • 146
~ec. 5. lntrrration by Parts ••• , • , . • \49
Sec 6 Mean-Value Theorem . , • • • • 150
Sec. 7. The Areas of Plane Fi~tures • • 153
Sec 8. The Arc Length of a Curve , 158
Sec 9 Volumes of Sol ids . . • 161
~ec 10 The Area of a ~urface of Rt>volution , 166
Sec 11 JVorrents Centres of Cravity CJuldm's Throrrms • 168
~ec 12. Applyir.g I:ef.r.ite IntEgrals to the Soluhon of Physical Prcb·
Iems 173

Chapter VI. FUNCTIONS OF SEVERAL VARIABLES


Sec. 1. Basic Notions • , , 180
Sec. 2. Contmu1ty 184
Sec 3 P.art1al Derivatives 185
Sec 4 Total Dlf'lerential of a Function 187
Sec 5 Differentiation of Comros1te f-unctions 190
Sec. 6, Dt-nvat1ve m a G1ven Direct•on and the Gradient of a Function 193
Stc. 7 H 1gH'e, -Crder Cenvat1ves and Difterentials 197
Sec 8 Integration of lotal Dif:erentials 202
Sec 9 Dif.erentiation of lmpllc1t Functions . • . 205
~ec 10 Change of Variables • 211
Sec. 11. The Tangt'nt Plane and the Normal to a Surface 217
Sec 12 1 aylor'~ Formula lor a f-unction of ~everal Variables 220
Sec. 13 The Extremum of a Functwn of Several Variables • 222
5t>c 14 Firdlr.!l tht Creates! and ~n·allest Values of Functions 227
Sec 15 Sml'ular Pomts ot Plane Curves . 230
Sec 16 Envelope 2.32
Sec. 17, Arc Length of a Space Curve • , . 234
Contents 7

&c. 18. The Vector Function of a Scalar Argument 2J5


Sec. 19 The Natural Trihedrnn of a Space Curve 2J8
Sec. 20. Curvature and Torsion of a Space Curve 2-12

Chapter VII. MULTIPLE. AND LINE INTEGRALS


Sec. 1 The Double Integral in Rectangular Coordinates 246
Sec. 2 Change of Variables in a Double Integral 252
Sec. 3. Computing Areas • . • • • • • • 256
Sec. 4. Computing Volumes . • • • . • • • • • • 258
Sec. 5. Computing the Areas of SurfacE's 259
Sec. 6 Applications of the Double Integral in Mechanics 230
Sec. 7. Triple Integrals . . . . . . . • . . . . . . . . . 262
Sec. 8. Improper Integrals Dependent on a Parameter. Improper
Multi tie Integrals. . •• • 269
Sec. 9 Line Integrals . • • • • . . . . . 273
Sec. 10. Surface Integrals • . . . . . . 284
Sec. 11. The Ostrogradsky-Gauss Formula 286
Sec. 12. Fundamentals of Field Theory 288

Chapter VIII. SERIES


Src. I. Number Series 293
Sec. 2. Functional Series 304
Sec. 3. Taylor's Series • 311
Sec. 4. Fourier's Series . 318

Chapter IX DIFFERENTIAL EQUATIONS


Sec. 1. Verifying Solutions. Forming Differential Equations of Fami·
lies of Curves. Initial Conclitions . . . • • • • • . . . . . . . . 322
Sec. 2 First-Order Differential Equations • • • . • . • • . • . . . 324
Sec. 3. First-Order Diflerential Equations with Variables Separable.
Orthogonal Trajer.tories . • . . . . . . . . . . . . . . . . . . . 327
Sec. 4 First-Order Homogeneous Differential Equ:ttlons • • • . . . 330
Sec. 5. First-Order Linear Differential Equations. Bernoulli's
Equation . . . . . . . • • . • • • . • . . . . . • . . • • • . • 3~2
Sec. 6 Exact Differential Equations. Integrating Factor • . • • . • 335
Sec 7 First-Ordrr Differential Equations not Solved for the Derivative 337
Sec. 8. The Lagrange and Clairaut Equations . . • . . . . . . . • 339
Sec. 9. Miscellaneous Exrrcises on First-Order Differential Equations 340
Sec. 10. Higher-Order DiiTerential Equations • • • • . • • • • • • . 345
Sec. 11. Linear Differential Equations • . • • • • • . . . . • • . . 349
Sec. 12. L1near Differential Equations of Second Order with Constant
Coefficients • • • • • • • • • • • • • , • • • . • • . . . • • • • 351
8 Contents
Sec. 13. LinE>ar Differt'ntial Equations of Order Higher than Two
with Constant Coefficients . . . . • • 356
Sec 14. Euler's Equations . . . . . . . • • • , , ••• , ••••• 357
Sec 15. Systems of Differential Equations • . • . . . . • . . . . . 359
Sec. 16. Integration of Diffen•ntial Equations by Means of Power Se·
ries . . . . . . . . . . . . . . 361
Sec 17. Problems on Fourier's Method • • . • • • . • , , , •• 363

Chapter X. APPROXIMATE. CALCULATIONS


Sec. I Operations on Approx.rnatc Numbers • 367
Sec. 2. InterpolatiOn of FunctiOns . . . . . 372
Sec. 3. Computtng thc,Rcal Roots of Equations .•• , • , • • 376
Sec. 4 NunwriLal, lntel!ration of Functions . . . . . • . . . . 382
Sec. 5. 1'\un er:ral lnteFlllion of Crd1rary Dd'ltrtntial Equations 384
Sec. 6. Appro,.tmating h urtu's Cceflictents 3Q3
ANSWERS • 396
APPENDIX 475
I. Greek Alphabrt 475
II. Some Con~tants 475
III. Inverse Quantities, Powers, Roots, Logarithms 476
IV Trigonometric Funct 1011~ • • • • • • • • • • • • 478
V. Exroret~tJal, Ilnerbolic and Trigono111etnc Functions 479
VI. Some Curves . • . • , , . • . . • . . . . . • , , , 480
PREFACE

This collection of problems and exercises in mathematical anal-


ysis covers the maximum requirements of general courses in
higher mathematics for higher technical schools. It contains over
3,000 problems sequentially arranged in Chapters I to X covering
all branches of higher mathematics (with the exception of ana-
lytical geometry) given in college courses. Particular attention is
given to the most important sections of the course that require
established skills (the finding of limits, differentiation techniques,
the graphing of functions, integration techniques, the applications
of definite integrals, series, the solution of differential equations).
Since some institutes have extended courses of mathematics,
the authors have included problems on field theory, the Fourier
method, and approximate calculalions. Experience shows that
the number of problems given in this book not only fully satisfies
the requirelr:£11 s of the student, as far as practical mas~ering of
the various sections of the course goes, but also enables the in-
structor to supply a varied ch01ce of problems in each section
and to select problems for tests and examinations.
Each chap.er begins with a brief theoretical introduction that
covers the basic definitions and formulas of that section of the
course. Here the most important typical problems are worked out
in full. We believe that th1s will greatly simplify the work of
the student. Answers are given to all computational problems;
one asterisk indicate'> that hints to the solution are given in
the answers, two astensks, that the solution is given. The
problems are frequently illustrated by drawings.
This collection of problems is the result of many years of
teaching higher mathematics in the technical schools of the Soviet
Union. It includes, in addition to original problems and exam-
ples, a large number of commonly used problems.
Chapter I

INTRODUCTION TO ANALYSIS

Sec. 1. Functlcns
1°. Rnl nurrl:ers. Rational and irrational numbers are collectively known
as real numbers 1 he ab,otutf value of a real number a IS undt-rstood to be
the nonnegative numb~r I a 1 d~fined by the conditions· 1 a 1 =a if a :;;.o, and
laJ =-a if a< 0. lhe following in(quallty holds for all real numbers a
an b:

2°. DeHnition of a function. If to every value*) of a variable x, which


belongs to son.e collection (set) E. there corresponds one and on I y one linite
value of the quantity y, then y is sa1d to be a functwn (~1ngle-valuedl of x
or a dependent 1 artable defined on the ~et E. x is the a•gument or tndepen-
dent vanable The fact that 11 1s a lunct10n of x IS expressed 10 bnef form
by the notation y =I (x) or y = F (~ ), and the like
If to every value of x belonging to some set E there corresponds one or
several values of the vanablt> y, then y is called a multtple-valued functiOn
of x defined on E. From now on we shall u~e the word "funrt10n" only in
the meanl!lg of a ~tngle-valurd functwn, 1f not otherw1~e stated
3° The domain of dellnition of a function. The collectiOn of values of x lor
which the g1ven funct1on IS defined IS called the domatn of defirutwn (or the
domain) of th1s functiOn. In the sm1pl~st cas~s. the doma1n of a function IS
either a clo~ed tnterval [a. bl. which is the set of n•al numbPrs x that sat1sfy
the inequalities ae;;;;;~~b. or anopen tntenal (a.b). wh1ch :s the set of real
numhers that satisfy the 1nl'qual1t.es a< x <b. Al~o poss1ble IS a more com-
plex structure of the domam ol dehmt10n of a function (see, for mstance, Prob·
lem 21)
Example 1. Determine the domam of delimtion of the function
I
y= Vx 2 -l ·

Solution. The function is defined if


x2 - l > 0,
that is, if I xI> l. Thus, the domam of the function Is a set of two inter-
vals:- oo < x <-I lind I < x < + oo
4°. Inverse functions. If theequationy=f(x) may be solved un1quely for
the variable x, that is, if there is a function x =g (y) such that y ==
f lg (y)J,

*) Hencetorth all values will be cons1dered as real, if not otherwise


stated,
12 Introduction to Analy.~is (Ch. I

!hen the function x = g (y), or, in standard notation, y =g (x), is the mverse
of y = f (x). Obviously, g If (x)) == x, that is, the function f (x) is the tnverse
of g (x) (and vice versa).
In 11 t> 1eret a! ca~e. th rquation y = f (x) refines a multiple-valued m-
verse furct1on x=f-•(y) wch that y==flf-'(y)j for ally that are values of
ihe functiOn f (x)
l:.nn pIe 2. Ceterrnine the inverse of the lunchon
y= 1-2-x. (1)
Solution. Solving equation (I) for x, we have
2-X=J-y
and
X=- log(l-y) "')
(2)
log 2 ·
Obviou~ly, tredorrain of c'efinition of the function (2) is- oo < y < 1.
5°. Corrroslte and irr.plicit functicns. A function y of x defined by a se-
ries of equalitu~sy=f(u), whereu=q>(x), etc., is called a comoosite function,
or a functwn of a fun:-tio't.
A function defined by an rqu11tion not solved for the derencent variable
+
is call~cl an tmpllCll function. For example, the equation x1 y 1 = 1 defines
y as an imp1ic1t function of x.
6°. The graph of a function. A set of points (x, y) in an xy-plane, whose
coordinates are connecttd by the equation y = f (x}, is called the graph of
the given funct:on.

1**. Prove that if a and b are real numbers then


!!al-l b II~ la-b I~ ial+lb 1.
2. Prove the following equalities:

a) Iab I= I a 1·1 b I; c) I: I= (:) (b + 0);


b) lal =a
2 2
; d) V£i2=~\aj.
3. Solve the inequalities:
a) lx-11<3; c) l2x+ 11< 1;
b) lx--1-1/>2;
d) jx-1/<lx+1l.
4. Find f(-1), /(0), {(1), {(2), {(3), {(4), if /(x)=x'-6x•+
+ llx-6.
5. Find /(0), t(-!). f(-x),t(+). f~x)'iff(x)=Vl+x•.
6. f (x) =arc cos (log x). Find f ( kl , f (1), f (10).
7. The function {(x) is linear. Find this function, if f(-1)=2
and f (2) = - 3.

*) Log x is the logarithm of the number x to the base 10.


Sec /] Functions 13

8. Find the rational integral function f (x) of degree two, if


f (0) = 1' f (1) = 0 and r(3) = 5.
9. Given that f (4) = - 2, f (5) = 6. Approximate the value of
f (4,3) if we consider the function I (x) on the interval 4 ~ x ~ 5
linear (linear interpnlation of a function).
tO. Write the function
f (x) = { 0, ~f x ~ 0,
X, If x>O
as a single formula using the absolute-value sign.
Determine the domains of definition of the following functions:
t t. a) y = Vx + 1; 16. y= V x-x•.
b)y=Vx+l. 2+x
I
17. Y= log- -.
2 -x
12. y = 4----z.
-X x2 -3x+2
18. y=log x+l .
13. a) y= Vx 2 -2;
2x
b) y=xVxz-=2. 19. y = arc cos 1 + x .
14**. y_.:V2+x-x•.
-.1- I 20. y =arc sin (log ~) .
15. y= r -x+ Y 2 +x'
21. Determine the domain of definition of the function
y= Vsin2x.
4 1
22. f(x)=2x -3x'-5x +6x-10. Find
1 1
q> (x) = 2 If (x) + f (-x)J and [f (x) -f (-x)].
'i'( x) = 2

23. A function f (x) defined in a symmetric region - l < x < l


is called even if f ( - x) = f (x) and odd if f ( -x) = - f (x).
Determine which of the following functions are even and which
are odd:
a) f(x)=; (a>:+a-x);

b) f(x)= Vt + x-t-x•-Vt-x -t-x•;


c) f(x)= V<x+ 1)'+ V(x-1) 1 ;
d) f (x) =log -l+x- ;
1 -x
e) f (x) =log (x+ Vt -t-x•).
24. Prove that any function f (x) defined in the interval
- I < x < l may be represented in the form of the sum of an
even function and an odd function.
14 Introduction to Analysis [Ch. I

25. Prove that the product of two even functions or of two odd
functions is an even fvnctton, and that the product of an even
function by an odd function is an odd function.
26. A function f (x) ts called periodic if there exists a positive
numter T (the period of the Junction) such that f (x + T) =f (x)
for all val res of x within the de main of definition of f (x).
Ce1ermine \\hirh of the follcwing functions are penodic, and
for U.e r.;encdic ft:ncticns f.nd U.eir least period T:
a) f (x) = 10 sin 3 x, d) f (x) = sin 1 x;
b) f (x) =a sin 'Ax+ b cos 'Ax; e) f (x) = sin <VX).
c) f (x) = Vtan x;

27. Express the length of the segment y = M N and the area S


of the figure AMN as a fundton of X= AM (Fig 1). C:onstruct
the graphs of these functions.
-;,_ _ _ _:;..c___,. 28. The linear density (that is,
mass per unit length) of a rod AB = l
b (Fig. 2) on the segments AC = l,.
CD=l 2 and DB=l,(l,+l.+l,=-1)
is equal to qP q2 and q1, respec-
B
l, l2 lJ
I(.--- ' j < - - - - ·t---
IJ
-'I
A ..... __ _ C_ .,. ;,o B
1---- a _ ____.
X
Fig. 2

tively. Express the mass m of a variable segrr.ent AM =x of this


rod as a functiOn of x. Construct the graph of this function.
29. Find q;l'ljJ(x)l and "IJJjq;(x)l, if q;(x)=K and "¢(x)=2x.
30. Find f tf If (xlH. if f (x) = 1 ~x •
31. Find f(x+ 1), if f(x-l)=x 1 •
32. Let f (n) be the sum of n terms of an arithmetic progression.
Show that
f (n+ 3)-3f (n + 2) + 3f (n+ 1)-f (n) = 0.
33. Show that if
f (x)=kx+ b
and the numbers x,, x., x, form an arithmetic progression, then
the numters j (x,), f (x1 ) and f (x,) likewise form such a pro-
gression.
Sec. I] Functions 15

34. Prove that if f (x) is an exponential function, that is,


f(x) =ax (a> 0), and the numters x,, x 2 , x. form an arithmetic
progression, then the numbers f (x,), f (x 2 ) and f (x,) form a geo-
metric progression.
35. Let
f (x) =log 1l-x
+x .
Show that
f (x) + f (y) = f (;:%Y) .
1 l
36. Let «p(x)= 2 (a"'+a-"') and 'lj)(x)= (a"'-a-"').
2
Show that
rp (x + y) = «p (x) <p (y) + 'lj) (x) ljJ (y)
and
'lj) (x + y) = «p (x) 'lj) (y) + «p (y) ljJ (x).
37. Find f(-1), /(0), f(l) if
f (x) = { arc sin x for-I~ r :e;;;; 0,
arc tan x for 0 < x .: + oo,
38. Determine the roots (zeros) of the rrgion of positivity and
of the region of negativity of the function y if:
a) y = I + x; d) y = x•- 3x;
b) y=2+x-x1 ;
e) y- 1o g2x --
c) y=l-x+x 2 ; - l+x ·
39. F;nd the inverse of the function y if:
a) y = 2x + 3·, d) y=log 2x ;
b) y=x 2 -l;
c)y=t/l-xJ; c)y=arctan3x.

In what regions will these inverse functions be defined?


40. Find the inverse of the function
I X, if x~o.
Y= I x\ if x > 0.
41. Write the given functions as a series of equalities each
member of which contains a simple elementary function (power;
exponential, tngonometric, and the like):
X
a) y = (2x-5) 10 ; c) y = log tan ;
2
b) y= 2cosx; d) y =arc sin (3-x'),
16 lnt roductton to Analysts [Ch I
--------------~----------~------------~~·--
42. Write as a single equation the composite functions repre-
sented as a series of equalities:
a) y=u•, u= sinx;
b) y= arc tan u, u= v = logx;Vv,
c) = { 2u, if 11 ~ 0,
Y 0, if u>O;
1
tt=X -l.

43. Write, explicitly, functions of y defined by the equations:


a) x'-arccos y=n;
b) lOX+ l()Y = 10;
c) x+IYI=2y.
Find the domains of definition of the given implicit functions.

Sec. 2. Graphs of Elementary Functions


Graphs of functions Y=f {x) are mainly constructed by marking a suffi-
ciently dt>me net of points M;(x;. 1/;). where y 1 =/(x;) (i=O, I, 2, ... ) and
by conuecti ng the pomts with a Iine that takes account of intermediate pomts.
Calculations are best done by a ~hde rule.
y
......................
---.}It
·.
\

Fig. 3

Graphs of the basic elementary functions (see Appendix VI) are readily
learned through their construction. Proceeding from the graph of
Y=f (x), (f)
we get the graphs of the following functions by means of simple geometric
construct ions:
1) y, = - f (x) is the mirror image of the graph r about the x-axis;
2) fla=/(-;c) is the mirror image of the graph r about they-axis;
Sec. 2) Graphs of Elementary l'•mctwni 17

3) y1 = f (x-a) Is the r graph displaced along th~ x-axls by an amount a;


4) y4 = b +I (x) Is the r graph displaced along the y·axls by an amount b
(Fig. 3).
Example. Construct the graph of the function

y =sin ( x- : ) .

Solution. The desired line Is a sine curve y =sin x displaced along the x·axis
to the :right by an amount 1- (Fig. 4)
y
!f• sin (:r- f)
X

Fig. 4

Construct the graphs of the following linear functions


(straight linPs ):
44. y=kx, if k=O, 1, 2, 1/2,-1, -2.
45. y=x+ b, if b=O, 1, 2, -1, -2.
46. y = 1.5x + 2.
Construct the graphs of rational integral funetions of degree
two (parabolas).
47. y=ax•, if a=1, 2, 1/2,-1, -2, 0.
48. y = x• + c, if c = 0, 1, 2, - 1.
49. y=(x-x.) 2 , ii x.=O, 1, 2, -1.
50. Y =Yo + (x- 1)2 , if Yo= 0, 1, 2, - 1.
51*. y=ax•+bx+c, if: 1) a=1, b=-2, c=3; 2) a=-2,
b=6, C=O.
52. y=2 t·x-x•. Find the points ot intersection of this pa-
rabola with the x-axis.
Construct the graphs of the following rational integral lunc-
tiom of degree above two:
53*. y=X 3 (cubic parabola).
54. y = 2 + (X- 1)3 •
55. !J=X3 -3x-t-2.
56. y= x•.
57. y = 2x1 - x•.
Construct the graphs of the following linear fractional func·
tl ons (hyper bolas):
68*. Y=+.
18 Jntr~duct10n to Analysis (Ch. 1

1
59. y= 1-x·
x-2
60. Y= x+2.
m
61*. y=y o +--,if
x - Xo
x.=1, y.=-1,

m=6.
62 *· y=3x+2'
2x-3

Construct the graphs o[ the [ractional rational functions:


1
63. y=x+x-·
xz
64. Y= X+ 1'
65*. y=:..
1
66. Y=-..
X

67*. y = ~}~ 1 (Witch of Agnes£).


68. y = x 2 ~ 1 (Newton's serpentine).
2

1
69. Y= x+xz·
70. y = x• + _!_
X
(trident of Newton).
Construct the graphs of the irrational functions:
7t*. y =Vi
J/-
72. IJ = V X.
73*. y= V? (Niele's parabola).
74. y = ± x Vx ('>emicubical parabola).
f
75*. y= ± V25-x (elltpse).
2

76. y~ ± V X -1 2
(hyperbola).
I
77. Y= V 1- x 2 .

78*. y= ~X v~ (cissoid of Diocles).


79. y= ± xV'25-x•.
Conslruct the graphs of the trigonometric functions:
80*. y = sin x. 83*. y =cot x.
81*. y=cosx. 84*. y=sec x.
82*. y=tanx. 85*. y=cosec x.
86. Y= A sin x, if A= 1, 10, 1/2, -2.
87*. y= sin nx, if n= 1, 2, 3, 1/2.
88. y=sin(x-q>), if q>=O, ~. ; . n, -~.
3

89*. y=5sin{2x-3).
Sec. 2] Graphs of Elementary f•mclions 19

90*, y=a sin x+ b cosx, if a=6, b=-8.


91. y = Sin X -j- COS X. 96. y= l-2cosx.
92*. y=co~•x. . x- I sm
97 . y= sm . 3x.
3
I
93*. y= x+ sin x. 98. y= cosx+ 2 cos2x.
1t
94*. y = x sin x. 99*. y = cos-.
X
95. y=tan 1 x. tOO. y=± Vsinx.
Construct the graphs of the exponential and logarithmic fune-
tions:
101. y=a", if a=2, ; , e(e=2, 718 ... )*).
102*. y= toga x, if a= 10, 2, ; , e.
103*. y= sinh x, where sinh X= lj2 (e"-e-~.
104*. y=co~hx, where co'>hx=l/2(e"+e-").
sinh x
105*. y= tanh x, where t an h X=-h-.
COS X
I

106. y= w-x.
2
107*. y = e-" (probability curve).
I 1
108. y=2-Xi. 113. y=log-.
X
109. y=logx 2 • lt4. y=log(-x).
110. y= log' x. 115. y= log 2 (l + x).
ttl. y =log (log x). 116. y =log (cos x).
I
112. ·l j = -og
,- .
x
Construct the graphs of the inverse tngonometric functions~
118*. y =arc sin x. 122. y =arc sin f.
I
119*. y=arc cosx. 123. y=arccos .
7
120*. y= arc tan x. 124. y=x+arccotx.
121*. y=arccotx.
Construct the graphs of the functions:
125. Y=lxl.
I
126. y= 2 (x+lxj).
127. a) y=xlxl; b) y=logv!lxl.
128. a) y=sinx+lsinxl; b) y=smx-lsinxj.
3-X2 when 1x1~1.
129. y= { 1~1 when lxl> l.

*) About the number e see p. 22 for more details.


20 Introduction to Analusis (Ch. 1

130. a) y=[x], b) y=x-[x], where [x] is the m~egral part


of the numLer x, that is, the grea~est in:eger less than or equal
to x.
Constn:ct the graphs of the following functions in the polar
coordina.e system (r, <p) (r,;;. 0):
131. r=l.
132*. r = t (spiral of Archimedes).
133*. r-=e'P (log·zrithmic spiral).
134*. r=!!:.. (hyperbolic sptral).
q>
135. r=2cOSijl (circle).
1
136. r = -SIO- q> (straight line).
137. r=sec ~ (parabola).
1

138*. r = 10 sin 3<p (three-leafed rose)


139*. r=a(l +-cos<p) (a>O) (cardioid).
143*. r• = a 2 cos 2<p (a> 0) (lemniscate).
CJns~ruct the graphs of the functions represented parametri-
cally:
141*. X=/ 3 , y=t" (semicubical parabola).
142*. X= 10 cost, y= sin t (ellipse).
143*. X= lOcos• t, y= 10 sin• t (astroid).
144*. X=a(cost+ t smt), y=a(smt-tcost) (involute of a
circle).
145*. x = 1 ~ 130 y = 1~:a (folium of Descartes).
146. X= Va , y= 1
at (semicircle).
I+ 12 ~ I+ 12
147. x= ~~ + 2- , y= 2 -2- (branch of a hyperbola).
1 1 1

149. x=2co!-> 2 t, y=2sm 2 t (segment of a straight line).


149. X=t-t 2 , y=/ 2 - - / 3 •
150. x=aJ2cost-co-;2t), y=a(2sint-sin2t} (cardwid).
Cmstruct the graphs of the following functions defined implic-
itly:
151*.x'+y2 =25 (circle).
152. xy = 12 ( hyperbGla ).
153*. l=2x (parabola).
y• .
154. 100 + = 1 (elltpse).
1('

64
155. y' = x• (100- x").
156*. x•
I 2 2

+ yl =a> (astroid).
15/*. x+y= \O!ogy.
158. X2 =CO!->!J.
Sec. 2) Graphs of Elemtntary Functions 2t

arc tan..!!.. •
159*. Vx 2 + y 1 =e x (logarithmtc spiral).
160*. x 8 +y8 -3X!J=0 (folium of Descartes).
161. Derive the conversion formula lrom the Celsius scale (C)
to the Fahrenheit scale (F) if it is known that ooc corresponds
to 32°F and 100°C corresponds to 2l2°F.
Construct the graph of the function obtained.
162. Inscribed in a triangle (base b = 10, altitude h = 6) is a
rectangle (Fig. 5). Express the area of the rectangle y as a func-
tion of the base x.

Fig. 5 F11~ 6

Construct the graph of this function and find its greatest


value.
163. Given a triangle ACB with BC =a, AC =band a variable
angle <;: ACB = x (Fig. 6).
Express y =area D. ABC as a function of x. Plot the grapn
of this function and find its greatest value.
164. Give a graphic solution of the equations:
a) 2x 1 -5x+2=0; d) w-x=x;
b) X 8 +X-1=0; e) X=1-+ 05sinx;
c) log x = O.lx; f) cot x = x (0 < x < n).

165. Solve t)le systems of equations graphically:

a) xy = 10, x-+ y = 7;
b) xy=6, X 2 +Y 2 = 13;
2
c) X -x+y=4, y 1 -2x=0;
+
d) x 2 y = 10, x + y• = 6;
e) Y= sinx, y=cosx (0<x<2n).
22 Introd•lction to Analysis (Ch. 1

Sec. 3. Limits
1°. The limit of a sequence. The number a is the limit of a sequence
x1, x1, • • • , x,., . . . , or
lim x,.=a,
n .. <~>
If for any e > 0 there is a number N = N (e) such that
lx,.-al<e when n>N.
Example 1. Show that
Jim 2n + '=2. (I)
n-+oon+l
Solution. Form the difference
21'1 +1_2 _ _ _1_
n+l - n+l'
Evaluating the absolute value of this difference, we have:
21'1+1 I I
1 n+l - 2 =n+l <e, (2)

If
I
n > --1 = N (e).
e
Thu5, for every positive number e there will be a number N=_!_-1 such
e
that lor n > N we will have irequality (2) Consequently, the number 2 is
+
the I imit of the ~equence xn = (2n + I )/(n I). henc~. formula (I) is true.
2". The limit of a function. We say that a function f (x) ..... A as x--. a
(A and a are numbers), or
lim f (x) =A,
x -..a
if lor every e > 0 we have 6 = 6 (e) > 0 such that
If (x)- A I< e lor 0 <I x-a I< 6.
Simtlarty;
lim f (x)=A,

if If (x)- A I < e for I x I > N (e).


The following conventional notation ts also used:
lim f(x)=co,
x-..a
which means that If (x) I > E for 0 < I x-a I < 6 (E), where E is an arbitrary
posttive number
3°. One-sided limits. If x <a and x ...... a, then we write conventionally
x ...... a-0; s!mtlarly, il x >a and x ..... a, then we wnte x ..... a +0. The numbers
f (a-0) = lim f (X) and J (a+ 0) = lim f (x)
x~a~-o x-+a+o
.are called, respectively, the ltmtl Ol'l the left ol the function f (x) at the point a
and the limtt on the rtght of the function f (x) at the point a (if these
numbers ex1st).
Sec. 3] Limits 23

For the existence of the limit of a function f (x) as x ..... a, it Is necessary


and sufficient to have the follcwmg equality:
f(a-O)=f(a+O).
If the limits lim f 1 (x) and lim { 2 (x) exist, then the following theorems.
x-..a x-.a
I old:
1) lim [f 1 (x)
x~a
+ fz (x)] = lim {1 (x)
x~a
+ lim {1 (x);
x~a

2) lim [{ 1 (x) f 2 (x)) = lim {1 (x) • lim f a(x);


x~a x~a x~a

3) lim [f.(x)/f 1 (x)J = lim {.(x)/lim {2 (x) (lim f 1 (x) :F 0).


JC-+a JC-+a x -+a

The following two limits are frequently used:


lim sin x = 1
JC-+ o X
and
I

lim
X--+ (I)
(1 +..!_)x = Jim
X a ...... o
(1 + a)a =e=2 71828 .. ,

Example 2. Find the limits on the right and left of the functioii
I
f (x) =arc tan-
X
as x .... o.
Solution. We have
f(+O)= lim (arctan ..!_)=2:
X-++0 X 2
and
f(-0)= lim (arctan..!..)=-~.
X--+- 0 X 2
Obviously, the function f (x) in this case has no limit as x -..0.

166. Prove that as n-- oo the limit of the sequence


I I
1
• 4 • · · · • nz • · ..
is equal to zero. For which values of n will we have the inequal-
ity
I
nz< 8

(e is an arbitrary positive number)?


Calcula e numerically for a) 8=0.1; b) 8=0.01; c) 8=0.001~
167. Prove that the limit of the sequence
n
X"=n+l (n=1, 2, ... )
!4 Introduction to Analysis [Cia, I

as n- oo is unity. For which values of n > N will we have


the inequality

(e is an arbitrary positive number)?


Find N for a) e = 0.1; b) e = 0.01; c) e = 0.001.
168. Prove that
lim x1 =4.
"-+ 2

How should one choose, for a given positive number e, some


positive number {) so that the inequality
lxz-41< e
should follow from
IX-21<il?
Compute {) for a) e = 0.1; b) e = 0.01; c) e = 0.001.
t 69. Give the exact meaning of the following notations:
a) lim logx=-oo; b) lim 2"= +oo; c) limf(x)=oo.
"-• +o x + +oo
170. Find the limits of the sequences:
I 1 I ( - l)n-t
a) 1• -2' 3' -T' "· ' n ' "· '
b) !I ' .!3' 6
5• ... •2i!=1• ... ;
2n

c) V2, V2 V2. 2-.( 2 V2, ... ,


d) 0.2, 0.23, 0.233, 0.2333, ...
V
Find the limits:
171 . nIllll 2 n-1) .
3 ... +-~-
. (I2+2+---z+
"'"' _n n n 11

172 . lim (n+l)(n~2)(n+3).


n ..... oo n
173 . lim ll+3+5+7+ ... +(2n-1)_2n+IJ
n-+oo n+l 2 '
. n+(-l)n
174. I tm - ( - I ) " '
n. -+ t:t'J n
2n+l + 3"+1
175. lim 2n+·n
n-+<X> J

176. }i~ ( ~ +! +-} + ... + ~).


. [
177. ~1_!1!, l-3-J- g--27+ "' +
I 1 1 ( -l)n-t]
3n-J •

178*. lim ••+2z+az; .. +n•.


n..... n
Sec. 3) _____________________L_,_·m_i_t_s___________________________

179. lim
n-oo<»
<V n+ 1- Vn).
• n ~in nl
180. hm --r--+
n_,.r.t'J n 1

When seekiPg the limit of a ratio of two lntegr& I polynomials In x as


x .... oo, it is useful first to divide both terms of the ratio by xn, where n is
the highest drr:ree of these rolyr.omia!s.
A similar procedure is also possible in many cases for fracti008 eontaiu-
lng irrational terms.
Example t.
(2x-3) (3rc+ 5) (4x-6)
lim
x_,.ao 3x 1 +x-l

( 2 - 3- ) ( 3+- 6)
5 ) ( 4--
• X X X 2·3·4
~c hm =--=B.
~--"' 3-f-..!.._..!.. a
x2 x3
Example 2.
I 1111
x ->CJO
. X
3/Aa+JO
v
=
x-.ao
I'nn
3v l
1+-
.\J
)Q
=l

. 2x"-3x-4
181. }~~~ <.:.:rlt I 86. I tm --;-;--~
<->co rx 4 +1
.
182. I't:n -:21.
IOOOx 2x+3
X-i-OOX-

. x"-5rc+ I
187. I IIIl
x-> oo X-1-
J

x2
X v
183. 1llll 3 + 7 188. lim
X ~en X x -• oc J0 +X YX
184. .
189. Iltll V x- + 1
---
2

"--->"' x+l
185.
190. lilll .
v-x
Htcoy x+Vx-j tl•
If P (x) and Q (x) arP integral polynomials and P (a) =1= 0 or <.;_(a) =1= 0,
then the limit of the rational fraction
lim p (x)
a Q (x)
t .....

is obtained directly.
But if P (a)= Q (a) =0, then it is advisable to LanLel the binon!lal x--a
out of the fraction ~ ~;~ once or several times.

Example 3.
lllll X
1
-4 = lim (x-2) (x t- 2)
"-+ •x•-Jx+2 JH· a (x-2) (x-1)
Introduction to Analysis (Ch. I

. x8 -3x+ 2
195. hm 4 _ 4 + 3 •
~-I A X
.x 2 -(a+ I) x+a
196 • I IITI •
X-+ a x3 -a1
197. lim (x+h~'-x'
h-+0
1 3
198. lim ( 1 -
1 1) •
X-+1 X X

The exrressions containing irrational terms are tn many cases rational-


bed by introducing a new variable.

Example 4. Find
lim Y'f"+X-1 •
x-+o VI +x-1
Solution. Putting
I +x=y',
'We have
Jim Yi+X-1
X-+t tll+x-1

.
.
199. I IITI
~-+I
Yx-1
x-
I

vx--s .
• 201. lim
~-~
r.-l
V-
x-1
200. 1tm
~+14
V x-4
202. lim -2 v-; +I
V7 (x-1) 2
~-+ I

Another way of finding the limit of an irrational expresston is to trans-


fer the trralional term from the numerator to the denominator, or vice versa,
from the denominator to the numerator.

Example 5.

lim
"-+a
v-x- Va- lim x-a
x-a .-z-+a(x-a)(Yx+Ya)
=
lim 1 - 1 (a > 0).
X-+ a y + v a - 2 Va
X

203. rtm 2- Yx-3


206. r 3-Vs+x
¥.....,., X
~-49 . tm
~-·1-
V5-x-·
r
204. I.TI
X-+8
vx-ll
X -2
• 207. lim VI +x- Vt-x
Jt-+0 X

.205. r Vx-t
1m -;-:-t-- • 208 . rtm Yx+h-
h.
VX •
x-+li/x-1 11-+0
Sec. ~~l Limits

.
209. 11m V -x+h- v-x 212. li:n (Vx(x+a)-x].
x ...... +ct~
h ~0
11
213. lLn <V x --5x-+ 6-x).
2

+ + 2x-6
.
210. 11m
Jfx -2t+6- Vx
2 2
X.....,.+ ao
2_ 4X 3
X-+3 X
214. li.n x <V x + 1-x). 2

211. iim cV X ~ a- Vx). , ..... +oo


x~+oo
215. li.n (x -t V 1 - x').
x~oo

The formula
lim sh1 x= 1
X-+ 0 X'

io frequently usl'd when solving the following examples. U Is taken for


granted that lim sm x = sm a and lim cos x = cus a.

Example 6.
lim sin5x=lim (~in5x·5)=1·5=5.
x-+o x x-o\ 5x

216. a) tm-;
)' SillX
221. a) r1111 X Sin-;
. t
.¥ ....,.2 X X--+ a X

b) rLll -
sinx, b) rI'll X Sin-.
. t
"-+ <:IJ t .t-+<D X

217. r1:11--.
sin 3x
228. r1m ( 1--x) tan-.
nx
X-+ 0 X .t-+1 2

218. 1'1111--
sill 5x
..

~ 0 Sili2X
229. li;n cot
.t-+0
2x cot ( i- x).
)' Sill
219. t : n -:JlX -. 1-sin~
....... Sln3nx 2
230. lim
.t-+ n n-x
220. lim ( n sin _::) •
1-2 crsx
n-.oo n 231. li:n
1-Cf'SX :rt n-3x
221. lim x2
x~-

.t .... 0 ' cos mx-c0s nx


222.
rlnl sin x-sin a • 232. lim xz
..: ......
.C-+ 0
x-a
tan x-~tn t
223. I't.n crs x- CPs a 233. lim --A.--
x-a
. .C-+0
X-+"

r1111 tan n.t 234. r1:11---.


arr SitJ X
224. -- .C-+0 X
.¥-+-z x+2
1' sin(x+h)-sinx 235.
rtm arc
----
tan 2t
225. tm h • t-+o '>Ill 3x
I) -+0
rtrn l-x2
226. lim
.11-+-
Jl
Sill X-Cf1S X
l-lanx . 236.
.c --+ 1
--
su1 nx ·

'
Introduction to Analysis (Ch. I

. x-~in 2x 1- VC<iSX
237 I tm + Sill 3X • 239. lim
~- 0 X
.t-+ 0
xz
cos-
l"!X
240. lim
Yl +sinx- Y l-sinx
238. lim / . .t-+0 X
• +I 1- X

When laking limits of the form


lim («p (x)J'~' <"'=C (3)
x -+a

<me should bear in mind that:


I) 1! there are linal lim1ts

lim «p (x) =A and lim ¢ (x) = 8,


Jt-+a
then C= A11;
2) if lim <p (X)= A =1: I and lim lJl (x) = ± co, then the problem of finding
f~a X-+n
the lim1t of (3) is ~olved in straightforward iash10n;
3) il lim«ptx)=l and hm¢(x)=co, then we put «p(x)=l+a(x),
x~a x -+a
where a tx) .... 0 as x _,.a and, hen ... e,
I lim a (X) 111 (X) lim [!j) (X) - l j 111 (X)
C= lim {[l+a(x)]a<xl~z(.~l'!l<xl=ex-+a =r-+a
x +a
where e=2.718 ... is Napier's number.
I

Eumple 7. Find
sin2x' 1 +-"
lim ( - ) •
X-+ 0 X

Solution. Here,
lim
.t-+0
(sin 2x) =2 and
X
lim (I +x) =I;
.t-+0
hence.
lim (sin2x)l +" =2' =2.
X-+0 X
Example 8. Find
lim ( x+ I )xa
x-+oo 2x+l •
Solution. We have

and
Sec 3] Limits 29
There lore,

Example 9. Flncl
lim (x-1}\".
x-ooo x+l
Solution. We have
1- ..!._
Jim x-1_ Jim _ _ x -1
X-+oox+l t-+oo I+_!_-'
X
Transforming, as indicated above. we have

lim
X-+- <»
-=-I)" = l + (X + I )J" =
( XX+ I lim
JC .... "'
1 -=--1
X 1

In this case it is ea>ier to find the lim1t without re~orting to the general
procedure:
1 (x-1)" . ( 1
.. ~m"' x+ I = xll:!, F~
-+rr= x ~~l(~-+r"l-l
1
x~'~ (I++ r
e-•
=-e=e .
...

Generally, it is useful to remember that

lim (
X-+ ao
1+~)"
X
=e~

241. li:n (32+x)·" 248. lim ( --;- )!r .


c-.o \ -X
t-."' x+
·. (~)X+I,
242. 11.n 2
¥-toJ X- 1

J)
243. lim (
%-+:»
1
X
X+l
250. lim ( 1 + !..
n-+CD n
)n
slnx 1
.(x•-3
244 . Iun -2.t+3) "
+2 .
1
251. lim (l +sin x) ".
X-+0 X X

. (x +2 )""•
245. }~~ 2 + 1 x1
1

252**. a) lim (cos x) x;


1

246. lim ( 1 - _!_


n~e~t n
)n I

b) lim(cosx)ii'.
247 lim
¥-+..
(t t !)x. "
X-+0
30 Introduction to Analysis (Ch. I

When solving the problems that follow, it is useful to know that if the.
limit lim f (x) exists and is positive, then
~-a

lim [In f (x)] =In (Jim f (x)] .


.¥-+a %-+A

Example 10. Prove that


Jim In (1 +x) I. (*)
~-+o X
Solution. We have

Jim
In (I + x) =Jim ..!._ ..!...
(ln(l+x)~J=ln[lim(l+x)xl=lne=l.
%-..0 X X-+0 ¥"-+0

Formula (*) is frequently used in the solution ol problems.

253. lim [In (2x + 1) -In (x + 2)].

254 . lim log (I+ lOx) •


x-+e x

255. ~~ (+In Y:~;). 260*. limn (


n-oo
Va -1) (a>O).
. eax_e'lx
256. lim x (In (x+ 1) -In x]. 261. lun •
.1:-++GD x-o X
. In (C~"S x) I -x
257 . I1m 1 • 262. lim __:::;.!.- •
x-+0 X X-+O Slll X

258*. lim ,:x - 1 • 263. a) lim c;ir h x ;


.Y-+G X X-+o X
a:x 1 b) limcosh~-1
259*. lim-=- (a >0).
.¥-+G X ¥'-+0 X

(see Problems 103 and 104).


Find the' following limits that occur on one side:
264. a) lim x 1
...... -~"' y.\2+ 1 b) lim
x-++o
- -
~

b) lim x • 1 +ex
x -+ + oo Y x• + I
265. a) li.n tanh x; 267. a) lim In (I + e") ;
"_,. -QO
X

b) lim tanh x, b) lim In (I+ e") •


.t-++ QO X
e"' -e-"
where tanh x = e~
-~--­
+e-" •
268. a) lim I <~in xI ;
X-+ - 0 X

.... -.
. 1
266. a) I1 m - - · b) lim lsinxl.
I '
¥-.+G X
Limits 31

. x-1
270. a) lim _!__ ;
269. a) 1t m _-;
X-+1-0 1X 11 x-+z-o x- 2
. x-1
b) lim~ •
b)X-+11tm l-=-1I .
+O X x-+z+o x- 2
Construct the graphs of the following functions:
271 **. y =lim (cos•" x).
n-+oo

272*. y=lim !+x n (x~O).


n-J>oo X

273. y =lim Vx-..,':-+-a-=- 2•

274. y = li:n (arc tan nx).


n-+oo
275. y =lim
n-+oo
v+ 1 xn (x ~ 0).
276. Transform the following mixed periodic fraction into
a common fraction:
a= 0.13555 ...
Regard it as the limit of the corresponding finite fraction.
277. What will happen to the roots of the quadratic equation
ax 1 +bx+c=0,
if the coefficient a approaches zero while the coefficients b and c
are constant, and b=I=O?
278. Find the ltmit of the interior angle of a regular n-gon
as n- oo.
279. Find the limit of the perimeters of regular n-gons inscribed
in a circle of radius R and circumscribed about it as n- oo.
2W. Find the hmit of the sum of the lengths of the ordinates
of the curve
y = e-"' cos :tx,

drawn at the points x = 0, 1, 2. . .. , n, as n- oo.


~ft. Find the limit of the mm of the areas of the squares
constructed on the ordinates of the curve
Y=2'-"'
as on bases, where x = 1, 2, 3, ... , n, provided that n- oo.
282. hnd the limit of the perimeter of a broken line M,M, .. . M,.
inscnbed in a logarithmic sptral
r -=- e-'P
lntroductron to Analysis (Ch. 1

(as n -• oo), if the vertices of this broken line have, respectively,


the polar angles
n nn
cp, = O, cp, = 2 , • . . • cp,. = 2 .

283. A segment AB =a (Fig. 7) is divided into n equal parts,


each part serving as the base of an isoscelf's triangle wtth base
angks u ~o- 45°. Show th:1l the limit of the peruneter of the bro-
ken line thus formed rliiTers from the
length of AB despite the fact that in
the litmt the broken line "geornetrtcally
nwr;~es With the segment AB".

A l3
t-~r---- a ~
_J
Fig. 7 Fig 8

284. The point C, divides a segment AB ---l in half; the


point C 2 dtvides a segment AC, in half; the point C, divides a
segment c.c, in half; the point C, divides C,C, in half, and so
on. Det<'rmine the limiting position of the pomt C,. when 11- • ou.
285. The side a of a right triangle is divided into n equal
parts, on each of which is constructed an inscribed rectangle
(Fig. 8). Determine the limit of the area of the step-like figure
thus formed if n-+ ou.
286. Find the constants k and b from the equation
(1)

What is the geometric meaning of (I)?


287*. A certain chemical process proceeds in such fashion
that the increase in quantity of a substance during each interval
of time 't out of the infinite seq11ence of intervals (t't, (i-t I) 't)
(i =- 0, 1, 2, ... ) is proportional to the quantity of the substance
available at the commencement of each interval and to the length
of the interval. Assuming that the quanttty of substance at the
initial time is Q0 , determine the quantity of substance Qln 1 after
the elapse of time t if the increase takes place each nth part of
the time interval 't = ~,
Find Q1 = li·n Q~11 l,
n-+•
Sec. 4] Infinitely Small and Large Quantities 33

Sec. 4. Infinitely Small and Large Quantities


t•. Infinitely small quantities (lnfinitesimals). If
lim a (x)=O,
X-+a
i.e., if 1a (x) I< e when 0 < 1>:-a 1< fJ (e), then the function a (x) is an
rnfinitesimal as x - a. In mnilar fashion we define the infinitessmal a (x)
as x - co.
The sum and product of a limited number of infinitesimals as x-a are
also infinitesimals as x _,.a.
If a (x) and ~ (x) are infinstessmals as x-+ a and
lim a (x)=C
X-+a ~(X) '

where C is some number diffE'rent from zero, then the functions a (x) and ~(x)
are called infinitesimals of the same order; but if C=O, then we say that the
function a (x) is an infinitesimal of l11glzer order than ~ (x). The function
u (x) is called an infinitesimal of order 11 compared with the function ~ (x) if

lim a (x) -C
Ha (~ (x)]n- '
where 0 <I C I < +co.
If
lim a (x) - I
Ha~(x)-'

then the function~ a (x) and ~ (x) are called equit•alent functions as x-+ a:
a (x)-~ (x).
For example, lor x--+ 0 we have
sin x-x; tan x-x; In (I +x)-x
and so forth.
The sum of two infinitesimals of different orders is equivalent to the
term whose order is lower.
The limit of a ratio of two infinitesimals remains unchanged if the terms
ol the ratio are replaced by equivalent quantities. By virtue of this theorem,
when taking the limit of a fraction
lim a(x)
"-+" ~ (x) '
where a (x)-+ 0 and p (x) _,. 0 as x - a, we can subtract from (or add to)
the numerator or denominator infinitesimals of higher orders chosen so that
the resultant quantities should be equivalent to the original quantities.
Example I.
. J/,? !-2~~ . vx-a ,
llm ;-V....,..,-..,....,- hm --=-.
x-+o In (l + 2x) X-+0 2x 2
2°, lnftnltely large quantities (inOnites). If for an arbitrarily large num-
ber N there exists a l'l (N) such that when 0 < I x-a ( < l'J (N) we have the
i nequallty
lf(x) I> N,
then the function f (x) is called an infinite as x-+ a.

2-1900
34 Introduction to Analysis [Ch. 1

The definition of an infinite f (x) as x-->- QO is analogous. As in the case


of infinitesimals, we Introduce the concept of infinites of different orders.

288. Prove that the function


f (x) = s!nx
X

is an infinitesimal as x-- oo. For what values of x is the ine-


quality
If (x) I< e
fulfilled if e is an arbitrary number?
Calculate for: a) e = 0.1; b) e=0.01; c) e=-0.001.
289. Prove that the function
f (x) = 1-X2
is an infinitesimal for x-- 1. For what values of x is the ine-
quality
If (x) I< e
fuliilled if e is an arbitrary positive number? Calculate numeri-
cally for: a) e = 0.1; b) e = 0.01; c) e = 0.001.
290. Prove that the function
1
f (x) = x-2

is an infinite for x _.. 2. In what neighbourhoods of Ix- 2/ < 6 is


the inequality
I f(x) I >N
fulfilled if N is an arbitrary positive number?
Find 6 if a) N = 10; b) N = 100; .
D c) N = 1000.
A ~::;;...-o~--.....;;::::t~~o... 8 291. Determine the order of smallness
of: a) the surface of a sphere, b) the volume
of a sphere if the radius of the sphere r
is an infinitesimal of order one. What
will the orders be of the radius of the
sphere and the volume of the sphere with
respect to its surface?
292. Let the central angle a of a cir-
Fig. 9
cular sector ABO (Fig. 9) with radius R
tend to zero. Determine the orders of
the infinitesimals relative to the infinitesimal a: a) of the
chord AB; b) of the line CD; c) of the area of L1 ABD.
Sec. 4) Infinitely Small and Large Quantities 35

293. For x ___.. 0 determine the orders of smallness relative to


x of the functions:
2x d) 1- cos x;
a) 1 +x ; e) tan x- sin x.
b) Yx-t-Vx;
c) Vx~- Vx';
294. Prove that the length of an infinitesimal arc of a circle
of constant radius is equivalent to the length of its chord.
295. Can we say that an infinitt!simally small segment and
an infinitesimally small semicircle constructed on this segment
as a diameter are equivalent?
Using the theorem of the ratio of two infinitesimals, find
. sin 3x·sin 5x 298 • l'tm -1In- x •
29 6 . Itm ( •)z •
x~o x-x x-+~ -x
arc sin Y x 299 • lim co~ x-cos2x •
297. lim 1 -x• x~o -cosx
x~o In (1 --x)

300. Prove that when x-----0 the quantities and VI +x~i -i


are equivalent. Using this result, demonstrate that when I xI is
small we have the approximate equality
Vl+x:::::;l+~. (1)

Applying formula (1), approximate the following:


a) VL06; b) Vo.97; c) Vw; d) Vl20
and compare the values obtained with tabular data.
301. Prove that when x-> 0 we have the following approxi·
mate equalities accurate to terms of order x•:
l
a) 1 +x :::::; 1- x;
b) Va -j-x;:::;;a+ta (a>O);
1

c) (1-t-x)n:::::;1-j-nx (n is a positive integer);


d) log(l-t-x)=Mx,
where M=loge=0.43429 ...
Using these formulas, approximate:
1
1) l.0 2
; 2) _197 ; 3) 1 ; 4)
0 105
v-15; 5) 1.04'; 6) 0.93'; 7) log 1.1·
Compare the values obtained with tabular data.
2*
36 Introduction to Analysis (Ch. 1

302. Show that for x--.. oo the rational integral function


P (x) = a0X 11 + a,x"- 1 + ... +a 11 (a 0 =F0)
is an infinitely large quantity equivalent to the term of highest
degree aox".
303. Let x--. oo. Taking x to be an infinite of the first order,
determine the order of growth of the functions:
2
a) X -l00x- 1,000;
xs
b) x-j-2

Sec. 5. Continuity of Functions


1°. Definition of continuity. A function f (x) is continuous when x = s
(or "at the point s"), if: l) this function is defined at the point 6. that is,
there exists a number f (6); 2) there exists a finite limit lim f (x); 3) this lim-
x .... ;
it is equal to the value of the function at the point 6. i.e.,
lim f (x) = f (s). (1)
x-+1;
Putting
x=s+M.
where As- 0, condition (1) may be rewritten as
lim Af<s>= lim lf<s+As)-f(s))=o. (2)
Lll;->0 Lll;-+0
or the function f (x) is continuous at the point 6 if (and only if) at this point
to an infinitesimal increment in the argument there corresponds an infinitesi-
mal increment in the function.
If a function is continuous at every point of some region (interval, etc.),
then it is said to be continuous in this region.
Example t. Prove that the function
y=sinx
ts continuous for every value of the argument x.
Solution. We have
. Ax
Ay=sin(x+Ax)-sinx=2sin Ax cos ( x+y
Ax) = - 2 .cos ( x+ Ax ) ·Ax.
sinAx
2 2
2
Since
Ax
~~ slny
0
I (
~=1 and cos x+ A2x ) o;;;;;;l, I
2
it follows that for any x we have
lim f).y=O.
dX-+0
Hence, the function sin x is contin1.1ous when - oo<x< + oo,
Sec. 5] Continuity of Functions 37
2°. Points of discontinuity of a function. We say that a function f (x) has
a dtscontinuity 'at x=x0 (or at the point x 0) within the domain of definition
of the function or on the boundary of this domain if th~re is a break in the
continuity of the function at this point.
1
Example 2. The funct.ion f (x) = (l x)z (Fig. 10 a) is discontinuous
when x=l. This function is not defined at the point x=l, and no matter

y
y=E(x)
---~
I I
--~ I
I I _I

X
X

(a) (b)

y
I

-I

(C)
F1g. 10

how we choose the number f (I), the redefined function f (x) will not be con·
tinuous for x= 1.
If the function f (x) has finite limits:
lim f(x)=f(x 0 -0) and lim f(x)=f(x 0 +0),
X-+X 0 -0 X-+X 0 +0

and not all three numbers f (x 0 ), f (x0 -0), f (x0 +0) are equal, then x 0 is called
a discontinuity of the first kind. In particular, if
f (x0 -0) =f (x0 +O),
then x0 is called a removable discontinuity.
For continuity of a function f (x) at a point x0 , it is necessary and suf·
ficient that
38 Introduction to Analysis [Ch. I

Example 3. The function 1(x) = st:; has a discontinuity of the first kind
at x = 0. Indeed, here,
f (+ 0) = lim sin x = + I
X-++0 X
and
1(-0)= lim sinx=-1.
X-+-0 - X

Example 4. The tunction y = E (x), where E (x) denotes the integral part
of the number x [i.e., E (x) is an integer that satisfies the equality x= E(x) + q,
where O<;q < 1], is discontinuous (Fig. lOb) at every integral point: x=O,
± 1, ±2, ... , and all the discontinuities are of the first kind.
Indeed, if n is an integer, then E (n-O)=n-1 and E (n+O)=n. At all
other points this function is, obviously, continuous.
Discontinuities of a function that are not of the first kind are called
discontinuities of the second kind.
Infinite discontinuities also belong to discontinuities of the second kind.
These ar.e points x 0 such that at least one of the one-sided limits, f (x0 -0) or
f (x0 + 0), is equal to co (see Example 2).
Example 5. The function y=cos.!!: (Fig. lOc) at the point x=O has a
X
discontinuity of the second kind, since both one-sided limits are nonexistent
here:
lim cos~ and lim cos~ ,
X-+-0 X X-++0 X

3°. Properties of continuous functions. When testing functions for conti-


nuity, bear in mind the following theorems:
l) the sum and product of a limited number of functions continuous in
some region is a function that is continuous in this region;
2) the quotient of two functions continuous in some region is a continuous
function for all values of the argument of this region that do not make the
divisor zero;
3) if a function f (x) is continuous in an interval (a, b), and a set of its
values is contliined in the interval (A, B), and a function cp (x) is continuous
in (A, B), then the composite function cp [f (x)) is continuous in (a, b),
A function f (x) continuous in an interval [a, b] has the following proper-
ties:
1) I (x) is bol!lnded on [a, b[, i.e., there is some number M such that
ll(x) I<M when a<x<;b;
2) f (x) has a minimum anrl a maximum value on [a, b);
3) l (x) takes on all intermediate values between the two given values;
that is, if f (a)= A and f (~) = B (a.;;;;: a<~,.;;; b), then no matter what the
number C between A and B. there will be at least one value x=y (a<y<~)
6UCh that f (y)=C.
In particular, if l(a) f (~)<0, then the equation
[(x) =0

has at least one real root in the interval (a, ~).

304. Show that the function y=X 2 is continuous for any value
of the argument x.
Sec. 5) Continuity of Functions 39

305. Prove that the rational integral tunction


p (x) = aoxn + a,xn-J + ... +an
is continuous for any value of x.
306. Prove that the rational fractional function
R ( ) _ a0 x n+ a,x n-t+ . . . + an
X -b 0 xm+b,xm-•+ ... +bm
is continuous for all values of x except those that make the de-
nominator zero.
307*. Prove that the function y = Vx is continuous for x ;;;;:oO.
308. Prove that if the function f (x) is continuous and non-
negative in the interval (a, b), then the function
F(x) = Vt(x)
is likewise continuous in this interval.
309*. Prove that the function y =cos x is continuous for any x.
310. For what values of x are the functions a) tan x and
b) cot x continuous?
311 *. Show that the function y =I xI is continuous. Plot the
graph of this function.
312. Prove that the absolute value of a continuous function
is a continuous function.

f (x) =
,
313. A function is defined by the formulas
x 2 -4
--
x- 2
for x=r62,

t A for x=2.
How should one choose the value of the function A= f (2) so
that the thus redefined fundi on f (x) is continuous for x = 2?
Plot the graph of the function y = f (x).
314. The right side of the equation
f (x) = l-x siri _!_X
is meaningless for x = 0. How should one choose the value f (0)
so that f (x) is continuous for x = 0?
315. The function
f (x) =arc tan x-1 2
is meaningless for x = 2. Is it possible to define the value of f(2)
in such a way that the redefined function should be continuous
for x = 2?
40 Introduction to Analysis [Ch. 1

316. The function f(x) is not defined for x=O. Define f(O)
so that f(x) is continuous for x=O, if:

a) f(x) =(I +x)n- 1 (n is a positive integer);


X

b) f(x)= 1-xczosx;

c) f(x)=ln(I+x)-:ln(l-x)_;
ex-e-x
d) f (x) = x ;

e) f (x) = X
2
sin _!_
X
;
f) f(x)=xcotx.

Investigate the following functions for continuity:


317. u=/:._2. 324. y=lnJtanil·
I+~ 1
318. y=l+x. 325. y=arctanx.

Y~_;-- 3
1
319. y= 326. y= (I +x) arc tan 1 x•.
X
320. Y= iXl . 327. y= eX+ 1 •
I
321. a) y= sin~; -Xi"
X 328. y= e
b) y=x sin~.
X 329. y = ---=-,-
x- . I +e'-x
322. y=..:-.
Stn X
323. y=ln(cosx).
X
1
• for x ~ 3,
330. y= { Plot the graph of this function.
2x+l for x>3.
331. Prove that the Dirichlet function 'X. (x), which is zero for
irrational x and unity for rational x, is discontinuous for every
value of x.
Investigate the following functions for continuity and construct
their graphs:
332. y=lim 1+
1
n (x~O).
n-+ao X

333. y =lim (x arc tan nx).


n-+ao
Sec. 5] Continuity of Functions 41

334. a) y = sgn x, b) y = x sgn x, c) y = sgn (sin x), where the


function sgn x is defined by the formulas:
+ 1, if x>O.
sgn x= 0, ~f X= 0,
{ -1, 1f x<O.

335. a) y=x-E(x), b) y=xE(x), where E (x) is the integral


part of the number x.
336. Give an example to show that the sum of two discontin-
uous functions may be a continuous function.
337*. Let a be a regular positive fraction tending to zero
(O<a< 1). Can we put the limit of a into the equality
E (1 +a)= E (1-a) +I,
which is true for all values of a?
338. Show that the equation
x'-3x.+ 1 =0
has a real root in the interval (1 ,2). Approximate this root.
339. Prove that any polynomial P (x) of odd power has at
least one real root.
340. Prove that the equation
tan x=x
has an infinite number of real roots.
Chapter II
DIFFERENTIATION OF FUNCTIONS

Sec. 1. Calculating Derivatives Directly


1°. Increment of the argument and increment of the function. If x and X 1
are values of the argument x, and y=f(x) and y1 =f(x1) are corresponding
values of the function y = f (x), then
ll.x=x 1 -x
is called the increment of the argument x in the interval (x, x1), and
ll.Y=Yt-Y
or
ll.y=f (x~)-f (x) =f (x +Ax) - f (x) (l)
y

Fig. 11

is called the increment of the function y in the same interval (x, x1) (Fig. 11,
where Ax=MA and lly=AN). The ratio

ll.y =tan a
ll.x
is the slope of the secant MN of the grafh of the function Y=f (x) (Fig. 11)
and is called the mean rate of change {) the function y over the interval
(x, x+ ll.x).
Example 1. For the function
y=x 2 -5x+6
Sec. 1] Calculating Derivatives Directly 43

calculate tu and fly, corresponding to a change in the argument:


a) from x= 1 to x= l.l;
b) from x=3 to x=2.
Solution. We have
a) flx= l.l-1=0.1,
fly=(l.1 2 -5·1.l +6)-(P-5·1 +6) = - 0.29;
b) flx=2-3=-l,
t:..y = (21 -5· 2 + 6)- (31 -5 ·3 + 6) =0.
I
Example 2. In the case of the hyperbola y=-, find the slope of the
. I\
X
I t)
secant passing through the points M ( 3,
3) and N ( 10,
10
.
) I 7
Solution. Here, t:..x=I0-3=7 and fly= -
10 3
=- . Hence,
00
fly I
k= t:..x=-ao·
2°. The derivative. The derivative y' = ~ of a function y = f (x) with re-

spect to the argument x is the limit of the ratio !~ when t:..x approaches zero;
that is,
y' = lim fly.
Ax-+ o t:..x

The magnitude of the derivative yields the slope of the tangent MT to the
graph of the function y = f (x) at the point x (Fig. II):
y' =tan <p.
Finding the derivative y' is usually called diOerentiatwn of the functton. The
derivative y' =f' (x) is the rate of chanj!e of the functiOn at the point x.
Example 3. Find the derivative of the function
y=x•.
Solution. From formula (I) we have
fly= (x+ flx) 2 -x 1 =-2xAx+ (flx)•
and

Hence,
y' = lim fly= lim (2x + flx) = 2.c.
Ax -+ o llx Ax-+ o

a•. One-sided derivatives. The expressions


('_ (x) = lim f (x + llx)- f (x)
Ax-+-o IJ.x
and
~~ (x) = lim f (x+ t:..x)-f (x)
Ax-++o llx
44 · Differentiation of Functions [Ch. 2

are called, respectively, the left-hand or rtght-hand dertvative of the function


f (x)at the point x. For f' (x) to exist, it is necessary and sufficient that
t'_ (x) = f~ (x).
Example 4 Find f~ (0) and f~ (0) of the function
f (x) =I x l·
Solution. By the definition we have

/~(0)= lim IMI=-1,


AX->-0 AX
f~ (0) = lim I Ax I= 1.
t!.x ->+o Ax
4°. lnftnite derivative. If at some point we have
lim f (x + Ax)- f (x) _
t!.x-> o Ax -co,
then we say that the continuous function f (x) has an infinite derivative at x.
In this case, the tangent to the graph of the function y = f (x) is perpendicu-
lar to the x-axis.
Example 5. Find f' (0) of the function

Solution. We have
y= vx:
Jim v-=co.
VAx 1
f' (0)= lim
t!.x->o
-.-=
ux 6.x->o Ax 2

341. Find the increment of the function y= x• that corresponds


to a change in argument:
a) fromx=1 to X 1 =2;
b) fromx= 1 to X1 =I. I;
c) fromx= 1 to X1 = 1 +h.
342. Find t!y of the function y = V x- if:
a) x=O, t!x=O.OOI;
b) X=8, L\x=-9;
c) x=a, t!x=h.
343. Why can we, for the function y = 2x 3, determine the +
increment t!y if all we know is the corresponding increment
t!x = 5, while for the function y = x• this cannot be done?
344. Find the increment t!y and the ratio ~Y for the func-
tions: x
1
a) y=(x•- 2). forx=l and t!x= 0.4;
b) y= Vx for x=O and t!x = 0.0001;
c) y =log x forx= 100,000 and t!.x = - 90,000.
Sec. 1] Calculating Derivatives Directly 45

345. Find 6.y and!~ which correspond to a change in argu-


ment from x to x+ 6.x for the functions:
a) y=ax + b; d) y= Vx;
b) y=x'; e) y=2x;
c) y=~;
.X
f) y=lnx .

346. Find the slope of the secant to the parabola


y =2x-x 2 ,
if the abscissas of the points of intersection are equal:
a) X 1 = 1, X2 =2;
b) X 1 = 1, X 2 =0.9;
C) X 1 = 1, X 2 = 1 +h.
To what limit does the slope of the secant tend in the latter case
if h -+0?
347. What is the mean rate of change of the function y = x•
in the interval 1 ~ x ~ 4?
348. The law of motion of a point is s = + 3t + 5, where
2r
the distances is given in centimetres and the time t is in seconds.
What is the average velocity of the point over the interval of
time from t= 1 to t=S?
349. Find the mean rise of the curve y = 2x in the interval
I~x~S.
350. Find the mean rise of the curve y = f (x) in the interval
(x, x+ 6.x}.
351. What is to be understood by the rise of the curve y = f (x)
at a given point x?
352. Define: a) the mean rate of rotation; b) the instantaneous
rate of rot at ion.
353. A hot body placed in a medium of lower temperature
cools off. What is to be understood by: a) the mean rate of
cooling; b) the rate of cooling at a given instant?
354. What is to be understood by the rate of reaction of a sub-
stance in a chemical reaction?
355. Let m = f (x) be the mass of a non-homogeneous rod over
the interval (0, x). What is to be understood by: a) the mean
+
linear density of the rod on the interval {x, x 6.x]; b) the Iinear
density of the rod at a point x?
356. Find the ratio ~~ of the function y =.!. X
at the point
x=2, if: a) 6.x= 1; b) 6.x=0.1; c) 6.x = 0.0 1. What is the deri v-
ative y' when x -'" 2?
46 Dif!erentiation of Functions [Ch. 2

357**. Find the derivative of the function y =tan x.


358. Find y' = lim ~ of the functions:
11x-+ o

a) y = x';
1
c) y = vx;
b) y = Xi; d) y =cot x.

359. Calculate f' (8), if f (x) = Vx.


360. Find f' (0), f' (1), f' (2), if f(x)= x(x-1)1 (x-2)'.
361. At what points does the derivative of the function
f (x) = x' coincide numerically with the value of the function itself,
that is, f (x) = f' (x)?
362. The law of motion of a point is s = 5tZ, where the dis-
tance s is in metres and the time t is in seconds. Find the speed
at t =3.
363. Find the slope of the tangent to the curve y = 0.1x1
drawn at a joint with abscissa X= 2.
364. Fin the slope of the tangent to the curve y = sin x at
the point (tt, 0).
365. Find the value of the derivative of the function f (x) = _!_
X
at the point x = X0 (X 0 =I= 0).
366*. What are the slopes of the tangents to the curves y = l.
X
and y = x• at the point of their intersection? Find the angle be-
tween these tangents.
367**. Show that the following functions do not have finite
derivatives at the indicated points:

a) Y= V-
x• at X=O;
b)y= x-1 V at X= 1;
2k+l
c) y = l cos xl at X=-rtt, k=O, ± 1, ±2, ...

Sec. 2. Tabular Differentiation


0
I Basic rules for finding a derivative. If c is a constant and u = q> (x),

v = 'ljl (x) are functions that have derivatives, then


1) (c)'= 0; 5) (uv)'=u'v+v'u;
2) (x)' = 1; 6) (.!:.)'
v,
=vu'-v'u
-~~-....- (u -1: 0);
3) (u ± v)'=u' ± v'; 7) ( ~ )' = --~~-· (u =F 0).
4) (cu)' =cu';
Sec. 2] Tabular Dlf!erentiation 47

2°. Table of derivatives of basic functions


I. (xn)' =nxn-1.
1
II. ( Vx)' = ,r- (x > 0).
2 I' X
III. (sin x)' =cos x.
IV. (cos x)' =-sin x.
V. (tan x)' = COS
-\-.X
-1
VI. (cot x)' = Stn
-.- 1- •
X
1
VII. (arc sin x)' = .r (I xI < 1).
r 1-x•
-1
VIII. (arc cosx)' = .r (I xI < 1).
r 1-x•
, I
IX. (.trCiar x) = 1 +x•·
-1
X. (arc cot x)' = x•+ 1 •
XI. (aX)'=axlna.
XII. (ex)' =ex.
XIII. (In x)' =~ (x > 0).
X

X IV. (log 4 x)' =


x 1na
- -1- = loga
x
e (x > 0, a> 0).
XV. (sinh x)' =cosh x.
XVI. (cosh x)' =sinh x.
1
XVII. (tanh x)' = - h
2 •
COS X
-1
XVIII. (cothx)'=~h• .
sm x
XIX. (.ncsinhx)'= ~·1
I +x
1
XX. (arc cosh x)' ,r (I xI> 1).
r x1 -1
XXI. (arc tanh x)' =
1~
-x
(I xI< 1).
-I
XXII. (arccothx)'=-1- (lxl>1).
x- 1
30. Rule for differentiating a composite function. H y=f(u) and u=q>(x),
that is, y = f [q> (x)], where the functions y and u have derivatives, then
,
Yx=YuUx (1)
or in other notations
dy dy du
dx= du dx'
This rule extends to a series of any finite number of differentiable functions.
48 DtUerentiation of Functions [Ch. 2

Example I. Find the derivative of the function


y= (x1 - 2x+ 3)5•

Solution. Putting y=u', where u=(x1 -2x+3), by formula (1) we will


have
y' = (u 5 )~ (x1 -2x + 3)~ = 5u 4 (2x-2) = lO (x-1) (x 2 -2x 3)'. +
Example 2. Find the derivative of the function
y=sin1 4x.
Solution. Putting
y=u'; u=sinv; v=4x,
we find
y' =3u 2 ·cos v·4 = 12 sin 2 4x cos 4x.

Find the derivatives of the following functions (the rule for


differentiating a composite function is not used in problems
368-408).

A. Algebraic Functions
I ~

368. y=x'-4x'+ 2x-3. 375. y=3£i -2xl -t-x-•.


369. y=!- ~ x+x'-0.5x'. 376*. y=x· Vx•.
370. y= ax 2 + bx +c. 377. Y= v-- v-·
a
X2 X
b
X

-5x' a+bx
371. y=--.
a 378. Y = c+dx.
372 y = atm + btm+n. 2x+3
379 · Y = x1 -5x +5 ·
ax'+b .11 2 I
373. Y= Ya•+b•' 380. y= -
2X-
-1
- -X .

374. y=..::..+1n2.
X
381 . Y-
_t+
.r
vz-
1- t' z

B. Inverse Circular and Trigonometric Functions


382. y=5 sinx+3cosx. 386. y =arc tan x + arc cot x.
383. y =tan x- cot x. 387. y=xcotx.
384 . =slnx+cosx
y sin X--COS X' 388. y = x arc sin x.

385. y=-2t sin t-{t'-2) cost. 389. y=


(1 +x 2
) arc tan x -x
2
Sec. 2) Tabular DiOerenttation 49

C. Exponential and Logarithmic Functions


7
390. f/ = X ·ex. 396. y =ex arc sin x.
x•
391. y=(x-l)ex. 397. y=-1
nx .
ex
392. Y=xz. 398. y=x • lnx- 3
x• .
xs I lnx
393. Y= ex. 399. y=-+21nx--.
X X
394. f (X) =ex COS X. 400. y=inxiogx-ina logax.
391i. y=(X 2 --2x+2)ex.

D. Hyperbolic and Inverse Hyperbolic Functions


401. y= X Stnh X. 405. y =arc tan x- arc tanh x.
x•
402 · Y= cosh x · 406. y =arc sin x arc st nh x.
_arc cosh x
403. y=tanhx-x. 407 . Y- X •

404 Y = 3coth x _arc coth x


\nx 408 ' Y- 1-x•

E. Composite Functions
In problems 409 to 466, use the rule for differentiating a composite func-
tion with one intermediate argument.
Find the derivatives of the following funchons:

409**. y= (1 + 3x-- sx•r.


Solution. Denote I+ 3x-5x2 = u; then y = u••. We have:
y~ = 30u 11 ; u~ = 3 -lOx;
~~~ c= 30u 21 ·(3-10x) =30 (I+ 3x-5x 2) 21 ·(3-10x).

410. u= (axe+ b)'.


411. f(y)=(2a+3by)'.
412. y = (3 2x1 )&. +
413 · y =56 (2:-1) 7 24 (2~-1)1 40 (2x-1)1 '
414. Y= vr::xr.
415. y= Va+bx'.
416. Y= (a''•-x''•)'/•.
50 DifJerenttation of Functions [Ch. 2

417. y=(3-2 sin x)'.


Solution. y' = 5 (3-2 sin x)'·(3-2 sin x)' =5 (3-2 sin x)' (- 2 cos x) =
- 10 cos x (3-2 sin x)'.

418. y=tanx- ~ tan'x+ ~ tan'x.


419. y= -- v--
Vcotx- cot a. 1 1
423. y= ---~.
3 cos' x cos x
420. y= 2x + 5 cos• x. _ ,/3sinx-2c osx
424 · Y- V 5 '

421*. x=cosee t +sec• t. 425. y= V sin' x+ co!•x.


I
422 · /(X)=-6(1 -3cosx) 1 '

426. y= Vt
+arc sinx.
427. y = V arc tan x- (arc sin x)'.
I
428 · Y =arc tan x ·
429. y=Vxe"'+ x.
430. y= t/2ex-2x 1 +In' x. +
431. y=sin3x+ cos ~ +tanVx.

Solution. y' =Cos 3x·(3x)' -sin ~ ( i )' +cos• 1


}" x (y.X)' =3 cos 3x-
I . x
- - sm -
+ --:=---=
I
5 5 2 V x cos• Vx ·

432. y= sin (x'-5x+ l) +tan~. X


433. f (x) =cos (ax+~).
434. f (t) = sin t sin (t + <p).
2x
435 . y =II +cos
-"-COS 2x.

436. f (x)= a cot~.


a
1
437. y=- 20I cos (5x1) - 4 cos x•.
438. y =arc sin 2x.
1 2 .
Solution. y' • (2x)' =
YI-(2x) 8 Yl-4x 8
439. y =arc sin~
l
. 441. y= arc tan-.
X X
l+x
440. f (x) =arc cos Vx. 442. y=arcco t-.
1-x
Sec. 2) Tabular Differentiation 51

443. y=Se-"•. 447. y=arccose".


1 448. y=ln(2x+7).
444. y= • · 449. y = log sin x.
5"
445. y=x•tou. 450. y=ln(l-x1 ).
446. f (t) = t sin 2t. 451. y = tn• x-ln (ln x).
452. y = In (e" + 5 sin x- 4 arc sin x).
453. y =arc tan (ln x) + ln (arc tan x).
454. u=Vlnx+l+ln(Vx+l) .

F. Miscellaneous Functions

455**. y= sin'Sxcos• ~.
11 4
456. Y = - 2 (x-2) 1 - x-2 ·
15 10
457 · Y = - 4 (x-3) 4 3 (x-3) 1 2 (x-3) 1 '

x•
458. y = 8 (I -x•)• .
_ V2x'-2x+ I
459 . Y- X •

X
460. y= .r .
a• y a 2 -t-x•
x•
461. y= .r -.
3 y (1-!-x 2 ) 1

462. Y='f3 v· + 1 x
X
18 y6r;_ 9 X
x+; v· +
X
6 ...~ 6f;;
13 ..{. V X.

463. Y= {· V (1 + x')'- ~ V (1 +x')'.


464. Y=a 4 vx-1
x+2.
465. y = x• (a- 2x 1 ) 1 •
a+bxn)m
466. y= ( a-bxn •
9 3 2 I
+
467. Y = 5 (x+2)s- (x +2) 4 (x+ 2) 1 -2 (x+2) 1 •
468. y=(a+x)Vii=X.
469. Y= Y(x+a)(x-j-b) (x+c).
470. z=Vu+Vy.
47t. t <t> = (2t + t> (3t + 2> V3t + 2.
62 Dtfjerentiation of Functions [Ch. 2

1
472. x=,r~·
r 2ay-y•
473. y=ln(Yl+e"-1)-ln(Vl +e"+l).
1
474. y = ~ cos' x (3 cos• x- 5).
_ (tan 2 x-l)(tan 4 x+IOtan 2 x+l)
475 · Y- 3 tan'x
. x2-l
476. y =tan' 5x. 485. y =arc sm ---,x2.

477. y = ; sin (x 2 ). 486. y= arc sin y x •


I +x 1

arc cos x
478. y = sin• W>· 487.
Y=Vl-xz'

479. y= 3 sinx cos• x+ sin' x. 488. Y= ;b_arc sin (x f !) .


1 1/'--
480. y= 3 tan' x-tanx+x. 489. y = r a•- x• +a arc sin -Xa .
481. y =3-.- a
Slll X
COS X
+-34 cotx. 490. y=xVa"-x"+a•arcsin..:..
a
V
482. y = a sin 2 x + ~ cos• x. 491. y=arcsin(l-x)+Y2x-r.
483. y =arc sinx• + arccosx•.
484. y=~(arcsinx)•arccosx.

492. v=(x- ;)arcsinVx+ ~ Vx-r.


493. y =In (arc sin 5x).
494. y =arc sin (In x).
x sin a
495. y = arc tan 1 -xcosa •

X
5tan
2 2 +4
496. y = 3 arc tan 3

497. v=3b•arctan yb~x-<3b +2x) Vbx-x•.


498. y = - V2- arc cot tan x
Y2' -x.
499. y= Yea".
500. y = e•ln•x.
501. F(x)=(2mam"+b)P.
502. F (t) =e"t cos ~t.
= (a sin ~x- ~cos ~x) e~"
503
•y a• + ~· •
Sec. 2) Tabular Differentiation 53
--~~------------------~----------------------

504. y = -1 e-x (3 sin 3x-cos 3x). 507. Y= 3 cot ~x.


10
1
505. y= xna-" • 508. y =In (ax 2 + bx +c).
506. y=VcosxaVC'O'SX. 509. y=ln(x+Va•+x•).
510. y=x-2Vx+2ln(l+Vx}
511. y=ln(a+x+V2ax+x•). * (x-2) 5
1
514. y=ln(x-t-l)''
512. Y=tn•x· 515 = 1 (x-1)' (x-2)
x-1 • Y n x-3
513. y=lncos-. l
X 516. y=-- - 2 X +lntanx.
2 SHI
517. Y= ~ Vx•-a•-a; ln(x+ Vx•-a•).
518. y= In In (3-2x').
519. y=5ln'(ax+b).
-I V~+x
520 · Y- n .r
r x2 -t-a 2 -x
·
m I n(x a-a z) +-
521. y=- n Inx-a
-.
2 ~ x+a
522. y=x·sin (lnx- ~).
I X I cos X
523. y= -2 In tan -2 - - -.-~-.
2 SHI X

524. f (x) =---= VX2~0 -In 1 + VX"+l . X

1 x2 -2x-H
525. Y=;rln .\•-t-x+l .
526. Y = 2arc sln IX -1- (1- arC COS 3X) 1 ,
sln ux . 1
527. lj = 3cos bt + ; ~~~~ ~: •
x ,r-
2 -t- :2- r 3
1 tan
528. Y=--=ln---------
V3 tan i+ 2-t- V3
529. y =arc tan In x.
530. y=lnarcsinx+~ln 2 x-1 arcsinlnx.
I
531. y =arc tan In-.
X

V2
532. y = - 3- arc tan V.x2. +1fI In x-1
x+i .
54 DiOerentiation of Functions [Ch. 2

+ ~+ 2 arc tan V sin x.


1
533. y= In
1- sin x
x1 t1 x-1 I
534. y= 4 tn
3
x•-I + 41 tn xt 2 arc tan x.
1+
1
535. f (x) = 2 tn (1 + x)- 61 In (x 1 -x+ 1) + Va I t 2x-l
arc an y";r .
536. f (x) = ;;rc sin x + In V 1- x•.
1-x•
537. y = sinh• 2x. 542. y =arc cosh In x.
538. y = e"" cosh ~x. 543. y = arc tanh (tan x).
539. y = tanh• 2x. 544. y =arc coth (sec x).
2x
540. y = In sinh 2x. 545. y = arc tanh 1
+ x• .

541. y=arcsinh ::. 546. y= ~ 2


(X -1)arctanhx+; x.
547. y= (; x"+!) arcsinhx--{ x V1 + x•.
548. Find y', if:
a) y=lxl;
b) y=xlxl.
Construct the graphs of the functions y and y' •
.549. Find y' if
y=ln/xl (x=foO).
550. Find f' (x) if
1- x for x ~ 0,
f (x)= { e-~ for x>O.
551. Calculate f' (0) if
f (x) = e-~ cos 3x.
Solution. f' (x) =e-~ (-3 sin 3x)-e-~ cos 3x;
f' (0)=e0 (-3sin O)-e0 cos 0 =-1.
*
552. f (x) =In (1 +x) +arc sin ~ . Find f' (1).
653. y= tan• n;. Find (:~)x=•'
554. Find {+(0) and f'_(O) of the functions:
a) f (x) = V sin (x1 ) : d) f (x) = x• sin..!..,
X
X :;b 0; f (0) = 0;
a1 x•
b) f(x)=arcsln a•t x 1 ; e) f (x) = x sin ..!..
X
X :f= 0; f (0) = 0
c) f(x) = - x -
1
, x=foO;
\-
{(0) = 0;
1 +ex
Sec. 2] Tabular Differentiation 55

+
555. Find f (0) xf' (0) of the function f (x) = e-".
+
556. Find f (3) (x-3) f' (3) of the function f (x) = VT!=X.
557. Given the functions f (x) = tanx and <p (x) =In (1-x).
. f' (0)
fmd <p' (O).
558. Given the functions f (x) = 1-x and <p (x) = 1- sin ~x,
• <p' (I)
fmd f' (I) .
559. Prove that the derivative of an even function is an odd
function, and the derivative of an odd function is an even func-
tion.
560. Prove that the derivative of a periodic function is also
a periodic function.
561. Show that the function y=xe-" satisfies the equation
xy' = (1-x) y.
x•
562. Show that the function y = xe -~ satisfies the equation
xy' = (l-x 1 ) y.
I
563. Show that the function y = 1 +x+In x satisfies the equa-
tion xy' = y (yIn x-1).

G. Logarithmic Derivative
A logarithmic derioative of a function y=f (x) is the derivative of the
logarithm of this function; that is,
(In y)' =IL.=f' (x)
y f (x)
Finding the derivative Is sometimes simplified by first taking logs of the func-
tion.
Example. Find the derivative of the exponential function
y=uv,
where u = <p (x) and v = 'lj1 (x).
Solution. Taking logarithms we get
lny=v In u.
Differentiate both sides of this equation with respect to x:
(lny)'=v' In u+ v (In u)',
or
I I
-y'=v' lnu+v- u',
y u
whence
y' = y ( v' In u+ ~ u') ,
56 DiOerenttation of Functions [Ch. 2

or
y'=uv(v'lnu+~ u')
564. Find y', if
- 1-x
y= Vx 1 1
1 + x• sin x cos
1
x.

2 1
Solution. In y=
3 1n x +In (1-x)-ln (1 +x )+31n sin x +2ln cosx:
(-1) _ -~ 3 _1_ cosx- 2smx .
_!_ , =~ ..!._
y y3 x + 1-x l+xz+ sinx cosx

1 ~x 1 +3cotx-2
2 1
whence y' =Y ( 3 1
x- x- tan x).
565. Find y', if y=(sinx)x.
Solution. In y =x In sin x; ..!._ y' =In sin x +x cot x;
y
y' =(sin x)x (In sin x + x cot x).

J.n the following problems find y' after first taking logs of the
function y = f (x):
566. y=(x+l)(2x+ 1)(3x+l).
(x +2)1
574. !J= v-x.
567· Y = (x+ 1)' (x+3)' · 575. !J= xV%.

568. !J= yx~--=-21). 576. lj=XxJt..

569. y= X V-.
xl-:-l. 577. Y=Xslnx.

570. = . (x-2)9 578. y =(cos x)sin x.


y V(x-1) 1 (x-3)1' ·

571 - Jfx=I
' Y- Vtx+2)1 Y(x+3) 1 579. y= ( 1 ++ r.
572. !J=Xx. 580. Y= (arc tan x)~.
573. y=xx•.

Sec. 3. The Derivatives of Functions Not Represented Explicitly


1°. The derivative of an Inverse function. II a function y =f (x) has a
derivative y~ ¢0, then the derivative of the inverse function x=f- 1 (y) is
I
XII=-;
Yx
Sec. 8] The Derivatives of Functions Not Represented Expltcitly 57

or
dx l
dy= dy'
dx

Example 1. Find the derivative x~. if


y=x+lnx.
· l x+l • x
Solution. We have Yx = 1 + - = - - ; hence, xu=-+ •
X X X J
2°. The derivatives of functions represented parametrically. If a function !!
Is related to an argument x by means of a parameter t,
X=cp (t),
{
y=¢ (t),
then
, Yt
.
Yx=--; •
xt
or, in other notation,
dy
dJJ di
ifx= dx ·
dt
Example 2. Find :~, if
x=a cost, }
y=a sin t

Solution. We find :~=-asint and ~~=acost. Whence

dy a cost
dx = - a sin t =-cot I.

3°. The derivative of an implicit function. If the relationship between X


and y is g1ven in implicit form,
F(x, y)=O, (\)
then to find the derivative y~ = y' in the simplest cases It is sufficient: l) to-
calculate the derivative, with respect to x, of the left side of equation (I),
taking y as a function of x; 2) to equate this derivative to zero, that is, to put
d
dx F (x, y) =0, (2)

and 3) to solve the resulting equation for y'.


Example 3. Find the derivative y~ if
x• + y1 -3axy = 0. (3)
Solution. Forming the derivative of the left side of (3) and equating it.
to zero, we get
3x1 +3yly' -3a (y +xy') = 0,
58 DiOerentiation of Functions [Ch. 2

whence
, r-ay
Y = ax-y1 '

681. Find the derivative x~ if


a) y=3x+x1 ;
b) y=x- ~ sinx;
X

c) y=O.lx+e•.
In the following problems, find the derivative y' = :~ of the
functions y represented parametrically:
x=2t-1, f x=a cos• t,
582. { 589. I
y=t'. tY=bsin•t.

583,
X = t ! I , { X =a COS f,
1

t )z
590, b . I(
y= sm .
{
Y= ( m.
cos' t
2at
X= I +I"' r X= Jfcos 2t
,
584.
a (1-1 2) 591. ~ sin 1 t
{
y = 1 + t• • y-
\ - Ycos 2t ·
3at 1
( x =arc cos y
585.
{
X=

y = l
1 +Ia'
3at 2
+t'.
592. t y =arc sm
. y I~~·+ ,

tz •
1

586.
f ~=Vt, x=e- 1
,
593. { zt
\ u=Vt. y=e.

587.
x -. V t• +I, { x =a ( In tan {+cost- sin t),
{ y =Vt•+l.
t-1 594.
y=a(sint+cost).
x=a(cost+tsint),
588. {
Y= a (sin t-t cost).
595. Calculate :~ when t = ~ if
X= a (t- sin t),
{ y=a(l-cost).
. dy a sin t sin t
I t 10n. - =
S OU =--
dx a (I-eos t) 1-cos t
Sec. 3] The Derivatives of Functions Not Represented Explicitly 59

and
0 11
dy) Stn 2
( dx n: = 11 = 1.
t=-;: 1-cos-
2
x= tInt,
596. Find ~~ when t = 1 if { y =I~ t .
597. Find dd..J!.x when t=~ if { x=e: c?st,
4 y=e sm t.
598. Prove that a function y represented parametrically by the
equations

satisfies the equation


y= (~r +2(:~r.
599. When x = 2 the following equation is true:
x' =2x.
Does it follow from this that
(x')' = (2x)'
when x=2?
600. Let y=Va•-x•. Is it possible to perform term-by-term
differentiation of
x' + Y2 = a•?
In the examples that follow it is required to find the deriva-
tive y' = :; of the imp! icit functions y.
601. 2x-5y+l0=0. 609. acos•(x+y)=b.
x' yz 610. tany=xy.
602. a1 + b = l.
1

603. x• +y• =a•. 611. xy =arc tan -.y


X

604. x• + x' y + y' = 0. 612. arc tan (x + y) = x.


605. Vx+V"Y=Va. 613. e>'=x+y.
_.J!_
606. Vii+Vu'=Vaa. 614. lnx+e X =C.

607. y• x-y
= - 615. !ny+..:..=c.
y
x+y'
608. y-0.3 siny=x. 616. arc tan f =~In (x + y•).
1
60 Differentiation of Functions tC/1. 2

617. Vx•+y 2 =carctanl!...


X
618. xY=y".
619. Find y' at the point M (1,1), if
2y= 1 + xy•.
Solution. Differentiating, we get 2y'=y 1 +3xyly'. Putting x=l and
y=l, we obtain 2y'=l+3y', whence y'=-1.

620. Find the derivatives y' of specified functions y at the


indicated points:
a) (x + y) 1 = 27 (x-y) for X=2 and Y= I;
b) yeY =e"+' for X=O and y= I;
c) y 1 = x +In lL
X
for X= I and Y= I.

Sec. 4. Geometrical and Mechanical Applications of the Derivative


1°. Equations of the tangent and the normal. From the geometric signifi-
cance of a denvative it follows that the equation of the tangent to a c~rve
y = f (x) or F (x, y) = 0 at a point M (x 0 , y0) will be

where y~ is the value of the derivative y' at the point M (x0 , y0). The straight
line passing through the point of tangency perpendicularly to the tang~nt is
called the normal to the curve. For the
8 normal we have the equation
y
x-xo+Y~(Y-Yo)=O.

2°. The angle between curves. The


angle between the curves
y=f, (x)
and
y=fz (x)
0 Zo X
at their common point M 0 (x 0 , y 0) (Fig. 12)
is the angle w between the tan~ents
Fig. 12 M 0 A and M 0 8 to these curves at the
point M 0 •
Using a familiar formula of analytic geometry, we get

tan w = f; (~o)- t:, (xo) .


1 + f, (xoH 2 (xo)
3°. Segments associated with the tangent and the normal In a rectangular
coordinate system. The tangent and the normal determine the following four
Sec 4) Geometrical and Mechanical Applications of the Deritalit·e 61

segments (Fig. 13):


t=TM is the so-called segment of the tangent,
S 1=TK is the subtangent,
n=NM is the segment of the normal,
Sn = KN is the subnormal.

X
Fig. 13

Since K M =I Yo I and tan q> = y0 , it follows that

t = TM I
= ~0~ VI + (y~) 1 I; n= NM = j Yo VI + (y~) 1 I;

St=TK=I ~J Sn=IYoY~j.
4°, Segments associated with the tangent and the normal In a polar sys-
tem of coordinates. If a curve is giv-
en tn polar coordinates by the equa-
tion r = f (q> ), then the angle 1.1.
formed by the tangent MT and the
radius vector r =OM (Fig. 14), is
defined by the following formula:
tan" =r dq> =.!_.
r dr r'
The tangent MT and the norma\ MN X
at the point M together with the radi-
us vector of the point of tangency
and with the perpendicular to the T
radius vector drawn through the pole Fig. 14
0 determine the following four seg-
ments (see Fig. 14):
t = MT is the segment of the polar tangent,
n = MN is the segment of the polar normal,
S 1=0T is the polar subtangent,
Sn =ON is the polar subnormal.
62 Differentiation of Functwns [Ch. 2

These segments are expressed by the following formulas:


,z
t= MT =-!,
Ir I
Yr + (r')
1 1
; St =OT = j?"j;
n = MN = y-=,z-+-:(-:,,:-:-::)1; sn =ON= I r' I·
621. What angles cp are formed with the x-axis by the tangents
to the curve y = x-x• at points with abscissas:
a) X=O; b) X= lj2; c) X= 1?
r Solution. We have y' = 1-2x. Whence
a) tan <p = 1, <p =45°; b) tan <p =0, c:p =0°;
c) tan <p=-1, <p=l35° (Fig. 15).

622. At what angles do the sine


curves y =sin x and y =sin 2x inter-
sect the axis of abscissas at the
origin?
Fig. 15 623. At what angle does the tan-
gent curve y =tan x intersect the
axis of abscissas at the origin?
624. At what angle does the curve y =eo. ax intersect the
straight line x = 2?
625. Find the points at which the tangents to the curve
y = 3x' + 4x'- 12x1 + 20 are parallel to the x-axis.
626. At what point is the tangent to the parabola
y=x1 -7x+3
parallel to the straight line 5x + y- 3 = 0?
627. Find the equation of the parabola y=x1 +bx+c that is
tangent to the straight line X=Y at the point (1,1).
628. Determine the slope of the tangent to the curve x 3 + y 3 -
- xy-7=0 at the point (1,2).
629. At what point of the curve Y2 = 2x 3 is the tangent per-
pendicular to the straight line 4x-3y+2=0?
630. Write the equation of the tangent and the normal to the
parabola
Y= X
v-
at the point with abscissa x = 4.
Solution. We have y' = ~-; whence the slope of the tangent is
k=(y'J.~= 4 =!. Since the ;oin/of tangency has coordinates x=4, y=2, It
follows that the equation of the tangent is y-2=l/4(x-4) or x-4y+4=0.
Since the slope of the normal must be perpendicular,
k,=-4;
whence the equation of the normal: y-2=-4 (x-4) or 4x+y-18=0.
Sec. 4J Geometrical and Mechanical Applications of the Derivative 63

631. Write the equations of the tangent and the normal to the
curve y=x'+2x 2 -4X-3 at the point (-2,5).
632. Find the equations of the tangent and the normal to the
curve
y= V-x-1
at the point (1,0).
633. Form the equations of the tangent and the normal to the
curves at the indicated points:
a) y =tan 2x at the origin;
1
b) y =arc sin x at the point of intersection with the
2
x-axis;
c) y =arc cos 3x at the point of intersection with the y-axis;
d) y=lnx at the point of intersection with the x-axis;
e) y=e1 -x• at the points of intersection with the straight
line y= 1.
634. Write the equations of the tangent and the normal at the
point (2,2) to the curve
1 +t
X=-~,-,

3 1
y=21 2 + 21.
635. Write the equations of the tangent to the curve
x = t cost, y = t sin t

at the origin and at the point t = ; .


636. Write the equations of the tangent and the normal to the
+
curve x' y 1 +2x-6=0 at the point with ordinate y= 3.
637. Write the equation of the tangent to the curve x' + y'-
--2xy=0 at the point (1,1).
638. Write the equations of the tangents and the normals to
the curve y = (x-1) (x-2) (x- 3) at the points of its intersection
with the x-axis.
639. Write the equations of the tangent and the normal to the
curve y4 =4x4 +6xy at the point (1,2).
640*. Show that the segment of the tangent to the hyperbola
xy = a 2 (the segment lies between the coordinate axes) is divided
in two at the point of tangency.
641. Show that in the case of the astroid x•t• + y•l• = a•t• the
segment of the tangent between the coordinate axes has a con-
stant value equal to a.
64 DtOerentialton of Functions [Ch. ~
---·-------- -----~------~------------------~~

642. Show that the normals to the involute of the circle


x=a(cost+t sint), y=a(sint-tcost)
+
are tangents to the circle x• y• = a•.
643. Find the angle at which the parabolas y = (x-2) 1 and
51= --4 + 6x- x• intersect.
644. At what angle do the parabolas y=x" and y=x• inter-
sect? ·
645. Show that the curves y=4x"+2x-8 and y=x'-x+lO
are tangent to each other at the point (3,34). Will we have the
same thing at (-2,4)?
646. Show that the hyperbolas
xy = a•; x•-y = b1
intersect at a right angle.
647. Given a parabola y•=4x. At the point (1,2) evaluate the
lengths of the segments of the subtangent, subnormal, tangent,
and normal.
648. Find the length of the segment of the subtangent of the
curve y = 2x at any point of it.
649. Show that in the equilateral hyperbola x•-y•=a• the
length of the normal at any point is equal to the radius vector
of this point.
650. Show that the length of the segment of the subnormal
in the hyperbola x•-y• = a• at any point is equal to the abscissa
of this point.
651. Show that the segments of the subtangents of the ellipse
x• y•
(j!+ b2 = 1 and the circle x•+ y• = a• at points with the same
abscissas are equal. What procedure of construction of the tan-
gent to the ellipse follows from this?
652. Find the length of the segment ol the tangent, the nor-
mal, the subta.ngent, and the subnormal of the cycloid
x=a (t- sin t),
{ y=a (1-cos t)
at an arbitrary point t = 10 •
653. Find the angle between the tangent and the radius vector
oi the point of tangency in the case of the logarithmic spiral
r=aek~.

654. Find the angle between the tangent and the radius vec-
tor of the point of tangency in the case of the lemniscate
r• = a• cos 2cp.
Sec. 4) Geometrical and Mechanical Applications of the Dertvative 65

655. Find the lengths of the segments of the polar subtangent,


subnormal, tangent and normal, and also the angle between the
tangent and the radius vector of the point of tangency in the
case of the spiral of Archimedes
r=acp
at a point with polar angle cp = 2n.
656. Find the lengths of the segments of the polar subtangent,
subnormal, tangent, and normal, and also the angle between the tan·
gent and the radius vector in the hyperbolic spiral r = ~at an
cp
arbitrary point cp=cp 0 ; r=r 0 •
657. The law of motion of a point on the x-axis is
X=3f-f 1 •

Find the velocity of the point at ! 0 =0, f 1 = l, and ! 2 =2 (x ts


in centimetres and t is in seconds).
658. Moving along the x-axis are two points that have the
+
following laws of motion: x = 100 5t and x = 1(2!\ where t ;:;;=: 0.
With what speed are these points receding from each other at
the time of encounter (x is in centimetres and t is in seconds)?
659. The end-points of a segment AB ~, 5 m are sliding along
the coordinate axes OX and OY (Fig. 16). A is moving at 2 m(sec.

r
y
8

X A X
Fig. 16 Fig. 17

What is the rate of motion of B when A is at a distance OA =3m


from the origin?
660*. The law of motion of a material point thrown up at an
angle a to the horizon with initial velocity v 0 (in the vertical
plane OXY in Fig. 17) is given by the formulas (air resistance is
3-1900
66 Differentiation of Functions [Ch. 2

disregarded):
x= V 0 t cos a,
where t is the time and g is the acceleration of gravity. Find the
trajectory of motion and the distance covered. Also determine the
speed of motion and its direction.
661. A point is in motion along a hyperbola y = 10 so that its
X
abscissa x increases uniformly at a rate of I unit per second.
What is the rate of change of its ordinate when the point passes
through (5,2)?
662. At what point of the parabola y 2 = lBx does the ordinate
increase at twice the rate of the abscissa?
663. One side of a rectangle, a= 10 em, is of constant length,
while the other side, b, increases at a constant rate of 4 cm.'sec.
At what rate are the diagonal of the rectangle and its area increas-
ing when b = 30 em?
664. The radius of a sphere is increasing at a uniform rate
of 5 cmfsec. At what rate are the area of the surface of the
sphere and the volume of the sphere increasing when the radius
becomes 50 em?
665. A point is in motion along the spiral of Archimedes
r= aqJ
(a= 10 em) so that the angular velocity of rotation of its radius
vector is constant and equal to 6° per second. Determine the rate
of elongation of the radius vector r when r = 25 em.
666. A nonhomogeneous rod AB is 12 em long. The mass of a
part of it, AM, increases with the square of the distance of the
moving point. M from the end A and is 10 gm when AM= 2 em.
Find the mass of the entire rod AB and the linear density at
any point M. What is the linear density of the rod at A and B?

Sec. 5. Derivatives of Higher Orders


1". Definition of higher derivatives. A derivattve of the second order, or
lhe second derivative, of the function Y= f (x) is the derivative of its deriva·
live; that is,
y" =(y')'.
The second derivative may be denoted as
' d2y
y' ' or dx2 ' or r (x).
If x=f (t) is the law of rectilinear motion of a point, then :;~ is the accel-
eration of th1s motion.
Sec. 5] Derivatives of Higher Orders 67

Generally, the nth derivative of a function y= f (x) is the derivative of


a derivative of order (n-1). For the nth derivative we use the notation
dny
Y <n> ' or -
dxn'
or f <n> (x)
·

Example 1. Find the second derivative of the function


y= In (1-x).
-I ( -1 )' I
Solution. y' = l-x ; y" = l-x = (1-x)•.

2°. Lelbniz rule. If the functions u = q> (x) and v = 'ljJ (x) have derivatives
up to the nth order inclusive, then to evaluate the nth derivative of a prod-
uct of these functions we can use the Leibniz rule (or formula):

(uv) <n> = u<n>v + n·u <n- 11 v' + n (~~I) u<-•> v• + ... + uv<n>.
3°. Higher-order derivatives of functions represented parametrical:y. If

X= q> (t),
{
y='\j) (t),

. t' • dy " d•y


t hen th e denva 1ves Yx= dx, y xx= dx• ' can successively be calculated
by the formulas

"' { Y:x)~
y xxx = - - ,- and so forth.
xt

For a second derivative we have the formula

" XI
..
Ytt - XttYt
.
Yxx= { x~)'
Example 2. Find y•, if
x=a cost,
{ y =b sin t.
Solution. We have
(b sin 1)~ b·COS t b
Y= 1
, --cot t.
(a cost)t -a sm t a
and

-~coft )~ --a. Sfrii7


b -1
( b
(a cost); -a sin t == - ai'Siiii7 •
68 Differentiation of Functions [Ch.

A. Higher-Order Derivatives of Explicit Functions


In the examples that follow, find the second derivative of th'
given function.
667. y=x'+7x 8 -5x+4. 671. y = ln (x + Va
2
+ x2 ).
2
668. y= e" • +
672. f (x) = (1 X )· arc tan x.
2

669. y = sin 2 x. 673. y =(arc sin x) 1 •


670. y=lnV1+x•. 674. y =a cosh..:. .
a
675. Show that the function y = xz + ~x + satisfies the differ
2
2
entia! equation 1 +y' =2yy".
676. Show that the function y =-} X 2 e" satisfies the differen
tial equation y"-2y' +y=e".
677. Show that the function y = C 1e-x + C 2 e-u satisfies th
equation y'' +3y' -l-2y=0 for all constants C 1 and C 2 •
678. Show that the function y = e2 " sin 5x satisfies the equa
tion y"-4y' +29y=0.
679. Find y"', if y=x"-5x 2 +7x-2.
680. Find f"'(3), if f(x)=(2x-3) 5 •
681. Find yv of the function y= In (1 +x).
682. Find yv' of the function y = sin 2x.
68:3. Show that the function y = e-" cos x satisfies the differ
entiat equation y'v + 4y = 0.
y 684. Find f (0), f' (0), f" (0) and f'" (O:
if f (x) = e" sin x.
685. The equation of motion of a poitl
along the x-axis is
X= 100+5t-0.00lt 3 •
M0 X
Find the velocity and the acceleration c
the point for times t 0 = 0, i 1 = l, an
t2 = 10.
686. A point M is in motion around
+
circle X2 y 2 = a2 with constant angula
Fig. 18 velocity ffi. Find the law of motion of i1
projection M 1 on the x-axis if at time t =
the point is at M 0 (a, 0) (Fig. 18). Find the velocity and the a<
celeration of motion of M 1 •
What is the velocity and the acceleration of M, at the in
tial time and when it passes through the origin?
What are the maximum values of the absolute velocity and H
absolute acceleration of M 1 ?
Sec. 6) Derivatives of Higher Orders 69

687. Find the nth derivative of the function u= (ax+ b) 11


,
where n js a q,atural number.
688. Find the nth derivatives of the functions:
a) u =
I
-;
1 -x and b) u= ~/-
v x.
689. Find the nth derivative of the functions:
I
a) u= sinx; e) Y= I +x ;
b) y=cos2x; f) =I +x .
Y 1-x '
c) y = e-•~; g) y = sin• x;
d) y=ln(l+x); h) y=ln(ax+b).
690. Using the Leibniz rule, find yen>, if:
l+x
a) y=x·e"; d) Y= Yx;
b) y=x•-e-zx; c) y=x'Inx.
c) y = (l-x 2 ) cos x;
1
691. Find [<n> (0), if f (x) =In 1
x

B. 1/tgher-Order Derivatives of Functions Represented


Parametrically and of Implicit Functions
In the following problems find :~ .
692. a) { x=ln t,. b) { x=arctant, c) { x=arcsint
u=t'; u=ln(t +t•>; u=Vt-t•.
693 . a) { x=acos t, c { x=a(t-sint),
y =a sin t; ) y =a ( 1-cost);
x=acos't, { x=a(sint-tcost),
b) { y=asin't; d) y=a(cost+tsint).
x=cos2t, { x=arctant,
694. a) { y __ si·n• t· 695. a) 1 t•
- ' y='i ;
x=e-at, { x=lnt,
b) { at b) I
y=e • Y=1-t'
. d'x . { x = e' cos t,
696. Fmd d-.,
Y
If y=et sm · t

70 DtUerentiation of Functions [Ch. 2

. day . { X = In ( 1 ), +/ 1

697. Fmd dx for 1=0, 1f


2 y=tz.
698. Show that y (as a function of x) defined by the equa-
tions x =sin t, y = aet Y2 +be-t Y!- for any constants a and b
satisfies the differential equation
2 d 2y dy
(1-x )dxz -xdx=2y.

In the following examples find y"' = :a;a .


x=sect, { x=e-t,
699. { t 701. 3
Y= ant. y=f.
1
x=e- cost, . dny . { X=lnmt,
700. { _ .• t 702. Fmd d-n, 1f
y = e 1 sm . x y=t .
703. Knowing the function y = f (x), find the derivatives x",
x'" of the inverse function x = f- 1 (y).
704. Find y", if x2 + y 2 = 1.
Solution. By the rule for differentiating a composite function we have
2x+ 2yy ' = Q; whence y ' =--anX
y
d !/' = - -X )' =-~
y X y2
l'
y -xy'-.

Substituting the value of y', we finally get:


, y2+x2 1
!I =---qs-=- LJ'.
In the following examples it IS required to determine the
derivative y" of the function y=f(x) represented implicitly.
705. Y2 -; 2px.
xz yz
706. (i2 + fj2 = 1.
707. y=x-tarctan y.
. d2 11 d2 x
708. Having the equation y = x +In y, fmd dx 2 and dya.
709. Find y" at the point (1,1) if
x' +5xy+y 2 -2x+y-6=0.
710. Find y" at (0, 1) if
x'-xy+y'= 1.
711. a) The function y is defined implicitly by the equation
x• + 2xy + y'- 4x + 2y- 2 = 0.
Find :~ at the point (1, 1).
b) • d d'y
Fm dx•,
'f
1
I
x t y =a. I I
Sec. 6] DiUerentials of First and Higher Orders 71

Sec. 6. Differentials of First and Higher Orders


1°. First-order differential. The dif!erential (first-order) of a function
y = f (x) is the principal part of its increment, which part is linear relative
to the increment t..x=dx of the independent variable x. The ditJerentlal of a

0
Fig. 19
function is equal to the product of its derivative by the differential of the
independent variable
dy=y'dx,
whence
t/=dy.
· dx
If M N is an arc of the graph of the function y = f (x) (Fig. 19), MT is the
tangent at M (x, y) and
PQ=t..x=dx,
then the increment in the ordinate of the tangent
AT=dy
and the segment AN= fly.
Example 1. Find the Increment and the differential of the function
y=3x 2 -x.
Solution. First method:

t..y = 3 (x+ flx) 1 -(x+ flx) -3x2 +x


or
fly=(6x-1) t..x+3(flx) 2 •
Hence,
dy = (6x-1) t..x = (6x -1) dx.
Second method:
y' =6x-1; dy=y' dx= (6x-1) dx.
Example 2. Calculate fly and dy of the function y=3x 2 -x for x= 1
and flx=O.Ol.
Solution. t..y= (6x-1)· t..x+ 3 (6.x) 2 =5·0.01 +3·(0.01) 1 =0.0503
and
dy=(6x-J) 6.x=5·0.01 =0.0500.
72 Differentiation of Functions [Ch. 2

2°. Principal properties of ditferentials.


1) dc=O, where c=const.
2) dx= !J.x, where x is an independent variable.
3) d (cu)=cdu.
4) d(u ± v)=du ± dv.
5) d (uv)=u dv + v du.
6) d (.!!...)=
v
v du-u dv
2
v
(v ¥= 0).
f' (u) du.
7) df (u) =
3°, Applying the dilferential to approximate calculations. If the increment
Ax of the argument x is small in absolute value, then the differential dy of the
function y = f (x) and the increment !J.y of the function are approximately
equal:
!J.y::::::dy,
that is,
+
f (x !J.x)- f (x) ::::::: f' (x) !J.x,
whence
+ +
f (x !J.x) ::::::: f (x) f' (x) dx.
Example 3. By how much (approximately) does the side of a square change
If its area increases from 9 m 2 to 9.1 m2 ?
Solution. If x is the area of the square and y is its side, then

Y=Vx.
It is given that x=9 and !J.x=O.l.
The increment !J.y in the side of the square may be calculated approxi-
matrly as follows:
!J.y::::::dy=y' !J.x= ~r- ·0.1=0.016m.
2 y 9
4°, Higher-order dilferentials. A second-order differential is the differential
of a first-order differential:

We similarly define the differentials of the third and higher orders.


If y = f (x) and x is an independent variable, then
d 2 y = y" (dx) 2 ,
d'y=y'" (dx) 1 ,

dny = y<n) (dx)n.

But if Y=f (u), where u=q> (x), then


d 2y=y" (du) 1 -t-y' d1u,
+
d'y= y'" (du) 1 +3y" du·d 2 u y' d 1 u
and so forth. (Here the primes denote derivatives with respect to u).

712. Find the increment ~y and the differential dy of the func-


tion u=5x-t x• for x=2 and L\x=O.OOl.
Sec. 6] DiOerenttals of First and Higher Orders 73

713. Without calculating the derivative, find


3
d (l-X )
I
for x = 1 and ~x = --;;-... .
714. The area of a square S with side x is given by S=x•.
Find the increment and the differential of this function and ex-
plain the geometric significance of the latter.
715. Give a geometric interpretation of the increment and
differential of the following functions:
a) the area of a circle, S = nx 2 ;
b) the volume of a cube, V=X 3 •
716. Show that when ~x-+0, the increment in the function
u= 2x, corresponding to an increment ~x in x, is, for any x,
equivalent to the expression 2"'"ln 2 t1x.
717. For what value of x is the differential of the function
y = X 2 not equivalent to the increment in this function as ~x-----.. 0?
718. Has the function u=\X\ a differential for X=O?
719. Using the derivative, find the differential of the function
Jl .1t
y =cos x for x = 6 and ~x --= 36
720. Find the differential of the function
2
Y=yx
for x=9 and ~x=-0.01.
721. Calculate the diiTerential of the function
y ==tan x
Jl .1t
for x= 3 and t1x= 180 •
In the following problems find the differentials of the given
functions for arbitrary values of the argument and its increment.
722.y,.-=}m· 727.y=xlnx-x.
1-x
723. lj = I X X • 728. y=ln 1 +x'
.
724. !J= arC Sltl a.
X
729. r =cot <p +cosec <p.
725. y=arc t an-.
x 730. s =arc tan e1•
a
726. y=e -xz .
731 Find dy if x 2 +2xy-y•=a1 •
Solution. Taking advantage of the invariancy of the form of a differential,
we obtain 2xdx-!-2(ydx+xdy)-2ydy=0
Whence
dy=--dx.x+u
x-y
74 Dif]erentiation of Functions [Ch .. ~

In the following examples find the differentials of the functions


defined implicitly.
732. (x-t-y) 1 • (2x+y)' = 1.
X
733. y=e-'Y.
734. In V x• + y* =arc tan.!!_. X
735. Find dy at the point (1,2), if y'-y=6x1 •
736. Find the approximate value of sin 31°.
Solution. Putting x=arc30°=T and 1\x=arc 1°= ~ • from formula (1)
10
1 ~0 cos 30°=0.500+0.017. ~
(see 3°) we have sin 31°:::::: sin 30° + 3
=0.515.

737. Replacing the increment of the function by the differen-


tial, calculate approximately:
a) cos 61 °; d) ln 0.9;
b) tan 44°; e) arc tan 1.05.
c) e0 •1 ;
738. What will be the approximate increase in the volume of
a sphere if its radius R = 15 em increases by 2 mm?
739. Derive the approximate formula (for 1!lx I that are small
compared to x)
Vx+flx ~ VX +
2
1\Vx.
Using it, approximate Y-5, VT7, V10, V 640.
740. Derive the approximate formula

V- -
x-t-L\x~
v- X
IU
-t- 3 v · =
x•
and find approximate values for Vto, V70, V200.
741. Approximate the functions:
a) y=x'-4x1 -t-5x+3 for x= 1.03;
b) f(x)=Vl +x for x=0.2;
C) f (X) = V 1
for x= 0 . 1 ;
l+xX
d) y=e -x• 1
f or x= 1.05.
742. Approximate tan 45°3'20".
743. Find the approximate value of arc sin 0.54.
744. Approximate Vf7.
Sec. 7) Mean-Value Theorems 75

E
745. Using Ohm's law, I= R , show that a small change in
the current, due to a small change in the resistance, may be
found approximately by the formula
I
1:!.1=- Rl:!.R.

746. Show that, in determining the length of the radius, a


relative error of P/. results in a relative error of approximately
2°/ 0 in calculating the area of a circle and the surface of a sphere.
747. Compute d 1 y, if y =cos 5x.
Solution. d 2y = y" (dx 2 ) = - 25 cos 5x (dx) 2•

748. u= V1-X 1 , find d2 u.


749. y=arccosx, find d2y.
750. y=sinxlnx, find d2 y.
In x
751. Z =X- , find d 2 z.
752. z = X 2 e-x, find d 8 z.
X~
753. Z = 2-x' find d 4 z.
+
754. u = 3 sin (2x 5), find d11 U.
755. y =ex cos sin (x sin u), find d"y.
(l

Sec. 7. Mean-Value Theorems


1°. ~olle's theorem. If a function f (x) is continuous on the interval
f' (x) at every interior point of thts interval, and
a,;;;;;. x ~ b, has a derivative
f (a)= f (b),
then the argument x has at least one value s. where a<~< b, such that
f'(~)=O.
2°. Lagrange's theorem. If a function f (x) is continuous on the interval
a,;;;;;. x,.;;; b and has a derivative at every interior point of this interval, then
f(b)-f(a)=(b-a)f' (s),
where a < 6 < b.
3°, Cauchy's theorem. If the functions f (x) and F (x) are continuous on the
interval a~ x,;;;;;. b and for a< x < b have derivatives that do not vanish
simultaneously, and F (b) =/= F (a), then
f (b) -f (a) f' <6>
F(b)-F(a)= F'(s)' where a<s<b.

756. Show that the function f (x) = x-x• on the intervals


-1 ~ x ~ 0 and 0 ~ x ~ 1 satisfies the Rolle theorem. Find the
appropriate values of 6.
76 Differentiation of Functions [Ch. 2

Solution. The function f (x) is continuous and differentiable for all values
of x, and f (-l)=f (O)=f (I) =0. Hence, the Rolle theorem is applicable on
the intervals -lo;;;;;xos;;;;O and Oo;;;;;x<l. To find 6we formtheequation
f' (x) = l-3x2 =0. Whence 61 = - Y ~ ;6 Y}, 2 = where -I<~~< 0
and 0 < 61 < I.

757. The function f(x)=V<x-2)' takes on equal values


f (0) = f (4) = V4
at the end-points of the interval [0.4]. Does
the Rolle theorem hold for this function on [0.4]?
758. Does the Rolle theorem hold for the function
{(x) =tan x
on the interval [0, n]?
759. Let
f (x) = x (x + 1) (x +- 2) (x + 3).
Show that the equation
f' (X) =0
has three rea i roots.
760. The equation
e"= 1 t-x
obviously has a root x = 0. Show that this equation cannot have
any other real root.
761. Test whether the Lagrange theorem holds for the function
f(x)=X-X 3
on the interval 1-2,1) and find the appropriate intermediate
value of s. ·
Solution. The function f (x) =x-x• is continuous and di!Yerentiable for
nil values of x, and f' (x)= l-3x 2 Whence, by the Lagrange formula, we
hJve f(l)-f(-2j=0-6=[l-(-2))f '(6), that is, f'(S)=~2 Hence,
1-362 =-2 and 6=±1; the only suitable value is 6=-1, for which the
inequality -2 < 6 < I holds

762. Test the validity of the Lagrange theorem and find the
<:ppropriate intermediate point s
for the function f (x) = x• 1• on
the interval [-1,1].
763. Given a segment of the parabola y = x 2 lying between
two points A (1,1) and B (3,9), find a point the tangent to which
is parallel to the chord AB.
764. Using the Lagrange theorem, prove the formula

sin (x +h)- sin X= h coss,


where x<6<x+h.
Sec. 8] Taylor's Formula 77

765. a) For the functions f (x) = r 2 and F (x) = x• -1 test +


whether the Cauchy theorem holds on the interval [ 1,2] and
find £;
b) do the same with respect to f (x) =sin x and F (x) =cos x
on the interval [ 0, ~ J.

Sec. 8. Taylor's Formula


If a function f (x) is continuous and has continuous derivatives up to the
(n-l)th order inclusive on the interval a~xo;;;;b (or b,..;;;x,..;;;a), and there
ts a finite derivative t<n> (x) at each interior point of the interval, then Tay·
lor's formula
f (x) = f (a)+ (x-a) f' (a)+ (x a)• f" (a)+ (x-;t)a f"' (a)+ ...
21
.. . + <~~G~-~ rn-l'(a)+ <x-:~>~ rn> (s).
where £=a+8(x-a) and 0<8<1, holds true on the interval.
In particular, when a=O we h:we (Maclaurin's formula)

f (x) =f (0) +xf' (0) +~ f" (0) + 0. 0 + (~n-:)1 rn-l) (0) +~ t<n) m.
where ~=Ox, 0<8< I.

766. Expand the polynomial f(x)=X 1 -2x1 +3x+5 in posi-


tive integral powers of the binomial x-2.
Solution. f' (x) = 3x 2 - 4x + 3; f" (x) = 6x- 4; f'" (x) = 6; t<nt (x) = 0
for n :;-, 4. Whence
f (2) =II; f' (2)=7; [" (2)=8; f"'(2) = 6.
[herefor~.

3X + 5 = II
1
1
X- 2X+
2
+(X - 2) • 7 +---· (x- 2) 6
(x- 2)" 8 +--•
21 31
or

767. Expand the function f(x) =ex in powers of x+ 1 to the


term containing (x+ 1) 1 •
Solution. t<nt (x) =eX for all n, r<n> (-I)=...!_. Hence,
e
2
ex=..!._+<x+ l)..!_+(x+ 1) _!_+(x+ I)__!_ +(x+ 1) e~
4

e e 21 e 31 e 41 '
where s= - I +8 (x+ I); 0<8<1.
768. Expand the function f (x) =In x in powers of x-1 up to
the term with (x-1) 1 •
78 DiUerentiation of Functions [Ch. 2

769. Expand f (x) = sin x in powers of x up to the term con-


taining x' and to the term containing x•.
770. Expand f (x) =ex in powers of x up to the term contain-
ing xn-l.
771. Show that sin (a+h) differs from
sin a+h cos a
by not more than I {2 h2 •
772. Determine the origin of the approximate formulas:
a) V-l+x~I+ 2I x- 8I x , )x)<l, 2

-- I I
b) V I+x~ I+ 3 x-- 9 x Jxi<I 2
,

and evaluate their errors.


773. Evaluate the error in the formula
I I , I
e~ 2+ 2! + 31 T 4i .
774. Due to its own weight, a heavy suspended thread lies
in a catenary line y =a cosh~ a
. Show that for small 1xI the
shape of the thread is approximately expressed by the parabola
xz
y=a+2a'
775*. Show that for
approximate eq·uality
IX I~a, to within ( ~ r, we have the

ea ~
X y--
-. a+x
a-x

Sec. 9. The L'Hospitai-Bernoulli Rule for Evaluating Indeterminate Forms


1°. Evaluating the indeterminate forms~ and ~. Let the single-valued
functions f(x) and q>(x) be differentiable for O<jx-aj <h; the derivative
of one of them does not vanish.
If f (x) and q> (x) are both infinitesimals or both inflnites as x---. a; that
is, if the quotient ~ (~), at x =a, is one of the indeterminate forms ~ or

~, then
co lim f (x) _lim f' (x)
x-+a q> (x) - x-.a q>' (x)

provided that the limit of the ratio of derivatives exists.


Sec. 9] L' Hospital-Bernoulli Rule for Indeterminate Forms 79

The rule is also applicable when a= oo.


~.<;;) again yields an indeterminate form, at the point
If the quotient
x=a, of one of the two above-mentioned types and f' (x) and q>' (x) satisfy
all the requirements that have been stated for I (x) and q> (x), we can then
pass to the ratio of second derivatives, etc.
However, It should be borne in mind that the limit of the ratio I ~x)
may exist, whereas the ratios of the derivatives do not tend to any 1Tm~~
(see Example 809).
2°. Other indeterminate forms. To evaluate an indeterminate form like
O·oo, transform the appropriate product f 1 (x)·l1 (x), where limf1 (x)=0 and
¥-+0
fa (x)
limf
-a 2 (x) = oo, into the quettent fdlx) (the form - °
0 I
1
(or I ( -) (the form
X
~ ).
00
fa (x)
In the case of the indeterminate form oo- oo, one should transform the
appropriate difference / 1 (x)- / 2 (x) into the product fdx) [ ~;n and 1-j:
12 1
first evaluate the indeterminate form ((x)); if lim • ((x)) =I, then we re·
11x 1
¥-+atX
duce the expression to the form
1-'• (x)
It (x) (the form 0 ).
I 0
f1 (x)

The indeterminate forms I"', 0°, oo 0 are evaluated by first laking loga•
rithms and then finding the limit of the logarithm of the power ({1 (x)) '• r~r'
(which requires evaluating a form like 0· oo).
In certain cases it is useful to combine the L'Hospital rule with the
finding of limits by elementary techniques.
Example I. Compute
inx oo
lim - - (form -).
¥-+0cot x oo

Solution. Applying the L'Hospital rule we have


lim In x =lim (In x)' == -lim sin ;c •
1

¥-+O cot x ¥-+o (cot x)' ¥-+O x


0
We get the indeterminate form
0 ; however, we do not need to use the
L'Hospital rule, since
sin1 x sinx
lim --=llm--sinx=l·O=O.
r-+o X ¥-+O x

We thus finally get


80 DiUerentiation of Functions [Ch. 2

Example 2. Compute
1
Jim ( ---_!._) (form oo-oo).
x .. o sin 2 x
2
x
Reducing to a common denominator, we get
2 2
Jim (-1- _ _!._)=Jim x -sin x (f 0)
x-..o sin 2 x x2 x-..o x 2 sin 2 x orm 0 ·
Before applying the L'Hospital rule, we replace the denominator of the lat-
ter fract10n by an equivalent infinitesimal (Ch. I, Sec. 4) x2 sin 2 x-x • We
4

obtain
. ( 1 1) .
hm - .2 - - -2 = hm
x 2 -sin

2
x (form - ). 0
X--+0 5111 X X X--+0 X 0

The L'Hospital rule gives


. ( -I- - -I )
11m . 2x-
= x--.o
11m sin 2x
-~,.--
Jim 2-2cos 2x.
x-..osin 2 x X
2 4x 8 x--.o 12x2

Then. in elementary fashion, we find


2
Jim ( - l___ .!.)= 1im l-cos2x=Jim 2sin x=_!_.
x--.o sin 2 x x2 x-+o 6x 2 x--.o 6x 2 3

Example 3. Compute
a
Jim (cos 2x) X> (form 1"')
X-->0

Taking logarithms and applying the L'Hospital rule, we get


a
lim In (cos 2x) x; =lim 31 n cos 2x -Glim tan 2x = _6
X-+0 X-+0 X2 x-.o 2x ·
a
Hence, Jim (cos 2x)Xi ,.e- 8 •
X-+0

Find the indicated limits of functions in the following exam-


ples.
. xa-2x1 -x+2
776 . IllTI '-7X +6
X->1 X

777. J'liTI X COS X-Sill


a
X
• 779. r1m cosh x-l
I •
x~a .t x-+o -cos x
1-x
778. lim
x-+J 1-sin~
2
tan x--sin x
780. lim x-:.inx
X-+0
.
Sec. 9] L' Hospital-Bernoulli Rule for Indeterminate Forms 81

. sec 2 x-2tanx 1t
781. hmn 1 -j-cos 4x

x .. - 785. lim-x-.
4 X-+-Ocojl'lX
tan x . 2
782 . I t m -5.
nan tx . In (stn mx)
X-+-- 786. I1m -,..'----'
2 x-.o In stn x
. ex
783. ltm 5 . 787. lim ( 1- cos x) cot x.
X \:'--1>00
X ->0
. In x
784 . IIll . ! / - .
X--~>':$.1 V X

. (I-eos x) cos x . (I-eos x)·l


Solution. lim (1-cosx)colx=hm =lun =
X-->0 x-.o SUI X X->0 Sill X

=lim sin '=0


x -.0 CtJS \'

788. lim ( 1 - x) tan ~x • 792. lim x" sin !:. , n>U.


x--.-,.1 x-r. x
789. lim arc sin x cot x. 793. hmlnxln(x-1).
X-->0

790. lim (x"e- x), n>O. 794. I .IITI ( -\- - -


I )
r--1-0 x +l x-I In x •
791. lim x sin !:..
X
.
. ( -x- - -I- ) =hm . x In x-x +I =
Solution. hm
x-.1 .\-I lnx x-..1 (x-l)ln\
I I
x--+lnx-1
x lnx .\
=Jim I =lim I = hm-~--~-=2.
x-.'ln.\ +-<x-1) X->lJnx---t I ~--'-+-2
x x x x

. ( ·-3-
795. I llTI I 5
. 2 _ -6
)
.
X ->3 X X .\

. [
796. I ltll
x-+-1
I .
2 ( I - }I x) 3(1-t/ x)
].
79~ hm
• ( -X - -l1- )
n cotx 2cosx
t->-
2

798. lim xx.


x->0

Solution. We iny=xlnx: lim lny=llmxlnx=-


x-..o x-..o

lnx x
== .¥-+-0
lim-
_1
= Jim--1 = 0,
X-+-0 _ _
whence limy= 1, that
.¥-+-0
ts, Inn.\"'= l •
X-+0
X X2
82 Differentiation of Functions [Ch. 2

1 1

799. limxi". 804. limx1-x.


f-+1
I tan~
800. limxHin
X-+0
x. 805. lim( tan
X-+1
7) 6

806. lim (cot x)iii%.


X-+0 X-+0
nx
802. lim (1-x)cos-z. 807. lim( .!_)tan x •
f-+1 X-+0 X
1

803. lim (1 + x2)"i. 808. lim (cot x)~ 10 %,


X-+0 X-+0
809. Prove that the limits of
l
x• sln-
1' X
a} 1~Si"ii"X=0;
b) lim x-s~n x
x-+ao x+smx

cannot be found by the L'Hospital-Bernoulli rule. Find these


limits directly.
/)

Fig. 20

810*. Show that the area of a circular segment with minor


central angle a, which has a chord AB=b and CD=h (Fig. 20), is
approximately

with an arbitrarily small relative error when a--0.


Chapter Ill

THE EXTREMA OF A FUNCTION AND THE GEOMETRIC


APPLICATIONS OF A DERIVATIVE

Sec. 1. The Extrema of a Function of One Argument


1°. lncrea~e <~nd decrease of !unctions. Till 1uuct10n y=f(x) is called
increuswg (decreusmg) on some mterval if, fo. any points x 1 and x 2 whtch
belong to this tnterval, from the inequaltty x 1 <x2 we get the tnequality 1 (x 1 )<
<f(x 2 ) (Fig 2la) [f(x 1 )>/(~ 2 ) (Ftg. 2\b)]. if f(x) is conttnuous on the
interval [a, b] and f' (x)>O (f' (x)<O] for a< .<b. then f (.\) increases (de-
creases) on the tnterval [a. bl.

y y

0 x, Iz X
(b) 0
(a)
Fig. 21 Fig. 22

In the simplest cases, the domam of detinillllll of f(x) may be subdtvid-


ed mto ;_, finite number of intervals of increase and decrease of the func-
tion (intervals of monotontcity). These mtervals are bounded by ct itic" ·
points x (where f' (x) =' 0 or f' (x) does not exist].
Example t. Test ,the following function for increase and decre:Jse:
y=x 2 -2x+5.
Solution. We find the derivative
y' = 2x-2=2 (x-1).
Whence y'=O for x= 1. On a number scale we get two intervals of monot·
onicity: (-co, I) and (1, +co). From (I) we have: I) if -co<x<l, then
y'<O, and, hence, the function t (x) decreases in the interval (- txl, I); 2)
if I <x< + txl, then y'>O, and, hence, the function f (x) increases in the in·
terval (I, +co) (Fig. 22).
84 Extrema and the Oeometrtc Applications of a Derivative [Ch. 3

Example 2. Determine the intervals of increase and decrease of the func·


tion
I
y=x+2 ·

Solution. Here, x=- 2 is a discontinuity of the function and y' =


= - _!_
)2 <0 for x=F -2. Hence, the function y decreases in the intervals
(x+ 2
-oo<x<-2 and -2<x<+ oo.
Example 3. Test the following function for increase or decrease:
I 1
Y=sxs-3x'.
Solution Here,
(2)
4
Solving the equation x -x =0, we find the points x 1 = - l , x2 =0, x1 =1
2

at which the derivative y' vanishes. Since y' can change sign only when
passing through points at which it vanishes or becomes discontinuous (in the
given case, y' has no discontinuities), the derivative in each of the intervals
(- oo, -I), (-I, 0), (0, I) and (I, + oo) retains its s1gn; for this reason, the
function under investigation is monotonic in each of these intervals. To
determine in which of the indicated intervals the function increases and m
which it decreases, one has to determine the sign of the derivative m each
of the intervals. To determine what the sign of y' is in the interval (-oo,
-1), it is sufficient to determine the sign of y' at some point of the inter·
val; !or example, taking x = -2, we get from (2) y' = 12>0, hence, y'>O 111
the interval (-oo, -1) and the function in this interval increases Similar·
ly, we find that y' <0 in the interval ( -1, 0) (as a check, we can take
x=-f). y'<O in the interval (0,1)
y (here, we can use x= lj2) and y'>O in the
interval (I, + ao ).
Thus, the function being tested in-
creases in the interval (-co, -I), decreases
in the tnterval (-I, I) and again increases
in the interval (1, + oo).
2°. Extremum of a function. If there
f(:ro} exists a two-stded neighbourhood of a point
x0 such that for every p0int x=Fx0 of this
0 neighbourhood we have the inequality
x, f (x)>f (x 0 ), then the point x0 is called the
Fig 23 minimum potnt of the fu net ion y = f (x),
whtle the number f (x 0) is called the mtni·
mum of the function y=f(x). Similarly, if
for any point x=Fx1 of some neighbourhood of the point x1 , the inequality
f (x)<f (x 1) Is fulfllled, then x1 is called the maximum point of the function
f (x), and f (x 1) is the maximum of the function (Fig. 23). The tTIInimum
point or maximum point of a function is its extremal point (bending point),
and the minimum or maximum of a function is called the extremum of thl'
function. If x0 is an extremal point of the function f (x), then f' (x 0 ) = 0, or
t' (x0 ) does not exist (necessary condition for the existence of an extremum).
The converse is not true: points at which f' (x)=O, or f' (x), does not exist
(critical potnts) are not necessarily extremal points of the function f (x).
Sec. I] The Extrema of a Function of One Argument 85

The sufficient conditions for the existence and absence of an extremum of a


continuous function f (x) are given by the following rules:
+
l. If there exists a neighbourhood (x 0 -6, x0 6) of a critical point x0
such that f' (x)>O for x 0 -6<x<x0 and f' (x)<O for x 0 <x<x0 +6. then x0 is
the maximum point of the function f (x); and if f' (x)<O for x0 -6<x<x0
and f' (x)>O for x 0 <x<x~+6. then x0 is thr minimum point of the function
f (x).
Finally, if there is some positive number 6 such that f' (x) retains its
sign unchanged for 0<1 x-x 0 1<6, then x0 is not an extremal point of the
functiOn f (x).
2. If f' (x0 ) =0 and f" (x0 )<0, then x 0 is the maximum point;
if f'(x 0 )=0 and f"(x 0 )>0, then x0 is the minimum point; but if f'(x 0 )=0,
f' (x0 ) = 0, and f"' (x0)#=0, then the point x0 is not an extremal point.
More generally: lrt the first of the derivatives (not equal to zero at the
point x0 ) of the functiOn f (x) be of the order k. Then, if k is even, the
point x0 1s an extremal point, namely, the maximum point, if [<k 1 (x0 )<0;
and it is the minimum point, if (<kl (x 0)>0 But if k IS odd, then x0 is not
, n extremal point.
Example 4. Find the extrema of the function
yo=2x+3 v;;.
Solution. Find the derivative
2
y'=2+ J/- = 2
J/- (
v-x+ 1). (3)
V X V t

Equating the derivativ~ y' to zero, we get:

V -x-+1=0.
\VhencL', we find the critical point x 1 =c -1. From formula (3) we have: if
x=- : - h, where h IS a sufficiently small positive number, then y'>O; but
if X c~ -1 + h, then y' <0*). lienee, X 1 = -1 IS the maxlml1111 point of the
funct1on 11· and !/max co_).
Equatmg thr denominator of the expression of y' in (3) to zero, we get

V- x~
o·'
whence \\e find the second critical point of the funclton x2 =0, where there
IS no der1 v at1 ve y' For x o=- h, we obvious! y have y' <0; for x = h we have
!(>0. Con,l·quently, x2 =0 is the mimmum point of the functiOn y, and
Yrnln=O (F1g. 24). It is also possible to test the behaviour of the function
at the point x= -1 by means of the second dertvative
!1'=- 2
3x Vx
Here, y" <0 for x1 = -1 and, hence, x1 = -1 is the max1mum point of the
function.
3°. Greatest and least values. The least (greatest) value of a continuous
function f (x) on a given interval [a, b] is attained either at the critical
points of the function or at the end-points of the interval [a, b].

*) If 1t is difficult to determine the sign of the derivative y', one can


calculate arithmetically by taking for It a sufl1cirntly small positive number.
86 Extrema and the Geometric Applications of a Derivative {Ch. 3

Example 5. Find the greatest and least values of the function


y=x 1 -3x+3
1
on the interval -l'j 2 ,.;;;;x,;;;;;;2 / 2•
Solution. Since
y'=3x1 ·-3,
it follows that the critical points of the function y are x, = - I and x2 = 1.

X - ,!_ -1 0
2
Fig. 24 Fig.~

Comparing the values of the function at these points and the values of the
function at the end-points of the given interval

y(-1)=5;f!(l)=l;y( -I ~)=4 ~: u(2 ~)=11 ~.


we conclude (Fig. 25) that the function attains its least value, m = l, at
I
the point x= I (at the minimum point), and the greatest value M = 11 B
at the point x=21 / 1 (at the right-hand end-point of the interval).

Determine the intervals of decrease and increase of the func-


iions:
8ll. y= 1-4x-x•. I
812. y= (x-2) 1 • 816. Y= (x-t)•.
813. y=(x+ 4)'.
814. y = x• (x- 3). 817. y= x• - : x- 16.
1

815. y=__!_2.
x- 818. y=(x-3)Vx.
__o~f_O_n_e_A_r~gu_m_e_n_t_________
~Se=c~.~~~~------~T~h~e~E~x~tr~e~m~a_o~t_a~F~un~c~ti_on 87

819. u=, 3X - v-x.


I
820. y=x t sinx. 824. y = 2x:=-a.
821. y=xlnx. e"
822. y =arc sin (I +x). 825. y=x-.

Test the following functions for extrema:


826. y=x•-t-4x-t-6.
Solution. We find the derivative of the given function, y' = 2x + 4.
Equating y' to zero, we get the critical value of the argument x= -2.
Since y'<O when x<-2, and y'>O when x>-2. it follows that x= -2 is
the minimum point of the function, and Ymln=2. We get the same result
by utilizing the sign of the second derivative at the critical point y"=2>0.
827. y=2+x-x•.
828. y=x'-3x'+ 3x +2.
829. y=2x'+3x1 -12x+5.
Solution, We find the derivative
y'=6x1 +6x-12=6 (x'+x-2).
Equating the derivative y' to zero, we get the critical points x 1 = -2
and x1 = I. To determine the nature of the extremum, we calculate the
second derivative u"=6(2x+l). Sinee y"(-2)<0, it follows that x1 =-2
is the maximum point of the function y, and Ymax=25. Similarly, we have-
if (1)>0; therefore, x1 = I is the minimum point of the function y and
Ymln= -2.
830. y=x 1 (x--12) 2 •
831. y=x (x-1) 1 (x-2)'.
840. y = 2 cos ~ + 3 cos i.
x'
832. y= x• + 3 . 841. y=x-ln(l+x).
1
x -2x+2
833. y= x-l 842. y=xlnx.
834. y = (x-2)x~8-x) • 843. y=x tn• x.
16
835. y -= x (4-x•). 844. y =cosh x.
4
836. y = vX~+B.
x• + 845. y=xe".
837. y= vx -4 . X
1

838. u= V (r -1)'.
839. y = 2 sin 2x +sin 4x. 848. y = x- arc tan x.
Determine the least and greatest values of the functions on the
indicated intervals (if the interval is not given, determine the
88 Extrema and the Geometric Applications of a Derivative [Ch. 8

greatest and least values of the function throughout the domain


.of definition).
X
849. y= 1+x*. 853. y = x• on the interval [ -1 ,3].
850. y = v·-x-;-:-(
1-:;:-0---x~). 854. y=2x1 -j-3x1 -12x+1
851. y = sin x + cos 4 x.
4
a) on the interval r-1 ,5];
b) on the interval [ -10, 12].
852. y =arc cos x.
855. Show that for positive values of x we have the inequality
I
x+-;;;z,2.
X

856. Determine the coefficients p and q of the quadratic tri-


·nomial y=x• +px +q so that this trinomial should have a min-
Imum y = 3 when x = 1. Explain the result in ~eometrlcal terms.
857. Prove the inequality
1
110
> 1+ x when x + 0.
Solution. Consider the function
f (x) =1110 -(1 +x).
Jn lhe usual wa"j we find that this function has a single minimum f (0) =0·
Hence,
f (x) > f (0) when x :1: 0,
and so e110 > l +x when x ::p 0,
as we set out to prove.

Prove the inequalities:


x•
858. x- 6 < sinx<x when x>O.
x•
859. cos x > 1- 2 when x+O.
x2
860. x- 2 <ln(l +x)<x when x>O.
861. Separate a given positive number a into two summands
·such that their product is the greatest possible.
862. Bend a piece of wire of length l into a rectangle so that
the area of the latter is greatest.
863. What right triangle of given perimeter 2p has the great-
·est area?
864. It is required to build a rectan~ular playground so that
it should have a wire net on three sides and a long stone wall
on the fourth. What is the optimum (in the sense of area) shape
.of the playground if l metres of wire netting are available?
Sec. 1) The Extrema of a Function of One Argument 89

865. It is required to make an open rectangular box of greatest


capacity out of a square sheet of cardboard with side a by cutting
squares at each of the angles and bending up the ends of the
resulting cross-like figure.
866. An open tank with a square base must have a capacity
of u litres. What size will it be if the least amount of tin is used?
867. Which cylinder of a given volume has the least overall
surface?
868. In a given sphere inscribe a cylinder with the greatest volume.
869. In a given sphere inscribe a cylinder having the greatest
lateral surface.
870. In a given sphere inscribe a cone with the greatest volume.
871. Inscribe in a given sphere a right circular cone with the
greatest lateral surface.
872. About a given cylinder circumscribe a right cone of least
volume (the planes and centres of their circular bases coincide).
873. Which of the cones circumscribed about a given sphere
has the least volume?
874. A sheet of tin of width a has to be bent into an open
cylindrical channel (Fig. 26). What should the central angle qJ be
so that the channel will have maximum capacity?

IJ c
0 N
A

/
/'-----'
'f '
'
A~B
a
Fig. 26 Ftg. 27

875. Out of a circular sheet cut a sector such that when made·
into a funnel it will have the greatest possible capacity.
876. An open vessel consists of a cylinder with a hemisphere
at the bottom; the walls are of constant thickness. What will the
dimensions of the vessel be if a minimum of material is used for
a given capacity?
877. Determine the least height h = OB of the door of a ver··
tical tower ABCD so that this door can pass a rigid rod MN of
length l, the end of which, M, slides along a horizontal straight
iine AB. The width of the tower is d < l (Fig. 27).
'90 Extrema and the Geometric Applications of a Derivative [Ch. 3

878. A point M0 (x 0 , Yo) lies in the first quadrant of a coordi-


nate plane. Draw a straight line through this point so that the
triangle which it forms with the positive semi-axes is of least area.
879. Inscribe in a given ellipse a rectangle of largest area with
sides parallel to the axes of the ellipse.
880. Inscribe a rectangle of maximum area in a segment of
the parabola y• = 2px cut off by the straight line x = 2a.
881. On the curve y = l·~x• find a point at which the tangent
forms with the x-axis the greatest (in absolute value) angle.
882. A messenger leaving A on one side of a river has to get
to B on the other side. Knowing that the velocity along the bank
is k times that on the water, determine the angle at which the
messenger has to cross the river so as to reach B in the shortest
possible time. The width of the river is h and the distance be-
tween A and B along the bank is d.
883. On a straight line AB=a connecting two sources of light A
(of intensity p) and B (of intensity q), find the point M that
receives least light (the intensity of illumination is inversely pro-
portional to the square of the distance from the light source).
884. A lamp is suspended above the centre of a round table
of radius r. At what distance should the lamp be above the table
so that an object on the edge of the table will get the greatest
illumination? (The intensity of illumination is directly proportion-
al to the cosine of the angle of incidence of the light rays and
is inversely proportional to the square of the distance from the
I ig ht source.)
885. It is required to cut a beam of rectangular cross-section
o11t of a round log of diameter d. What should the width x and
the height y be of this cross-section
so that the beam will offer maximum
resistance a) to compression and b) to
p bending?
Note. The resistance of a beam to compres-
~~::----r----_./3 sian is proportional to the area of its cross-
section, to bending-to the product of the
width of the cross-section by the square of
Its height.

F1g. 2 886. A homogeneous rod AB, which


can rotate about a point A (Fig. 28),
is carrying a load Q kilograms at a distance of a em from A
and is held in equilibrium by a vertical force P applied to the
free end B of the rod. A linear centimetre of the rod weighs
q kilograms. Determine the length of the rod x so that the force P
should be least, and find P min·
Sec. 2] The Direction of Concavity. Points of Inflection 91

887*. The centres of three elastic spheres A, B·, Care situated


on a single straight line. Sphere A of mass M moving with ve-
locity v strikes B, which, having acquired a certain velocity,
strikes C of mass m. What mass should B have so that C will
have the greatest possible velocity?
888. N identical electric cells can be formed into a battery
in different ways by combining n cells in series and then combin-
ing the resulting groups (the number of groups is ~ ) in par-
allel. The current supplied by this battery is given by the formula
I= NnC
NR+n 2 r'
where If is the electromotive force of one cell, r is its internal
resistance, and R is its external resistance.
For what value of n will the battery produce the greatest
current?
889. Determine the diameter y of a circular opening in the
body of a dam for which the discharge of water per second Q
will be greatest, if Q = cy V h- y, where h is the depth of the
lowest point of the opening (h and the empirical coefficient c are
constant).
890. If x 11 x 2 , ••• , xn are the results of measurements of equal
precision of a quantity x, then its most probable va! ue will be
that for which the sum of the squares of the errors
n
O=~(x-xd
i=t

is of least value (the principle of least squares).


Prove that the most probable value of x is the arithmetic mean
of the measurements.

Sec. 2. The Direction of Concavity. Points of Inflection


1°. The- concavity of the graph of a function. We say that the graph of a
differentiable function y = f (x) is concave down in the interval (a,b) (concave
up in the interval (a 1 ,b 1)] if for a< x < b the arc of the curve is below (or
for a < x < b1, above) the tangent drawn at any point of the interval (a, b)
or ol the interval (a 1 ,b/)] (Fig. 29). A sufficient condition for the concavity
downwards (upwards) o a graph y=f (x) is that the following inequality be·
fulfilled in the appropriate interval:
f" (x) < 0 [f" (x) > 0].
2°. Points of Inflection. A point (x , f (x0)] at which the direction of con-
cavity of the graph of some function ctanges is called a point of inflectzon
(Fig. 29).
'92 Extrema and the Geometnc Appltcattons of a Derivative [Ch. 3

For the abscissa of the point of inflection x0 of the graph of a function


y=f(x) there is no second derivative f"(x 0)=0 or f"(x 0). Points at which
r (x) =0 or f" (x) does not exist are called critical points of the second kind.
The critical point of the second kind x0 is the abscissa of the point of inflec-
tion if f" (x) retains constant signs in the intervals x0 -l\ < x < x 0 and
x 0 < x < x0 +11, where II is some posi-
y tive number; provided these signs are
opposite. And it is not a point of
r
inflection if the signs of (x) are the
same in the above-indicated intervals.
Example t. Determine the inter-
vals of concavity and convexity and
also the points of inflection of the
Gaussian curve
y=e-x2.
Solution. We have
0 a X y' = -2xe-x
and
!/=(4x2 -2) e-x•.

Equating the second derivative y• to zero, we find the critical points of tHe
second kind
1 1
X1 = - Y2 and X2= V2'

These points divide the number scale - oo < x < +oo into three intervals:
+
I (- oo, x1 ), II (x1 , x 2), and II I (x 2 , oo). The signs of !I' will be, re~pec-
y
y
1

Fig. 30 Fig. 31

1ively, +, -, + (this is obvious if, for example, we take one ,r,oint in each
of the intervals and substitute the corresponding values of x into y) Therefore:
I) the curve is concave up when -oo< x < - ;
2
and ;
2
< x< + oo; 2) the

curve ts concave down when- 1 < x < Y2".


Y2" I The points ( Yf,
±1 1 )
Ye are
points of inflection (Fig. 30).
It will be noted that due to the symmetry of the Gaussian curve about
the y-axis, it would be sufficient to investigate the sign of the concavity of
this curve on the semiaxis 0 < x < +
oo alone.
Sec. 3] Asymptotes 93

Example 2. Find the points of inflection of the graph of the function


Y= V -x+2.
Solution. We have:
I
2
y" = - - (x +2) - , = -2
. (1)
9 9 V<x+2) 5
It is obvious that y" does not vanish anywhere.
Equating to zero the denominator of the fraction on the right of (1), we
find that y" does not exist for x = -2. Since y" > 0 for x <- 2 and y" <0 for
x>-2, it follows that (-2,0) is the point of inflection (Ftg. 31). The tan-
gent at this point is parallel to the axis of ordinates, since the first derivative y'
is infimte at x= -2.

Find the intervals of concavity and the points of mflection


of the graphs of the following functions:
891. y=X 1 -6X2 +12x+4. 896. lj=COSX.
892. y=(x+l) 4 • 897. y=x-sinx.
1
893. y=x+ 3 . 898. y=x 2 lnx.
899. y =arc tan x-x.
900. y= (1 +X ex.2
)

Sec. 3. Asymptotes
)
0
• Definition. If a point (x,y) is in continuous motion along a curve
!I=-= f (x) in such a way that at least one of its coordinates approaches infinity
(aud at the same time the dtstance of the point from some stratght line tends
to zero), then this straight line is called an asymptote of the curve.
2°. Vertical asymptotes. If there is a number a such that
lim f ( •) = ± oo,
X-?a

then the straight line x=a is an asymptote (vertical asymptote).


3o Inclined asymptotes. If there are limits
lim f (x) =k
X-->+<» X I

and
lim [/ (x)-k 1x] = b1 ,
X-++ oo

then the straight line y = k 1x+ b1 will be an asymptote (a right inclined


asymptote or, when k 1 =0, a right horizontal asymptote).
If there are limits
94 Extrema and the Geometric Applications of a Denvatwe [CII . .1

and
Iirn [f(x)-kzX]=b 2 ,
X4-00

then the straight line y = k 2x+ b2 is an asymptote (a left inclined asymptote


or, when k2 =0, a left horizontal asymptote). The graph of the function y=f (x)
(we assume the function is single-valued) cannot have more than one right
(inclined or horizontal) and more than one left (inclined or horizontal) asymptote.
Example t. Find the asymptotes of the curve
x•
y= Vx 2 -1 ·
Solution. Equating the denominator to zero, we get two vertical asymp-
tote~·
x=-1 and x=l.
We seek the melt ned asymptotes. For x ~ + oo we obtain
2
k1 = lim }!_= lim x - I,
.:-->+<» X x_,.+cn r V X2 - l
.
.: ... +"'
. x•-x
b, ~~ ltm (y-x) = ltm
.:-->+oo
v· xvxz=t
2 -l
=0,

Fig. 32

h~nce,the straight line y=x is the right asymptote. Similarly, whenx--oo,


we have
k1 = lim .1!....=-1;
X-+-<» X

b1 = lim (y+x) =0.


X4-00

Thus, the left asymptote Is y= -x (Fig. 32). Testing a curve for asymp-
totes is simplified If we take into consideration the symmetry of the curve.
Example 2. Find the asymptotes of the curve
y=x+lnx.
Sec. 3] Asymptote:. 95
Solution. Since
lim y= -co,
¥·++0

the straight line x=O is a vertical asymptote (lower). Let us now test the
curve only for the inclined right asymptote (since x > 0).
We have:
k= lim .!= 1,
x-.+ao X

b= lim (y-x)= lim lnx=co.


X-++ ao X-.+ co

Hence, there is no inclined asymptote.


If a curve is represented by the parametric equations x=qJ (t), y='ljJ (t),
then we first test to find out whether there are any values of the parameter t
for which one of the functions qJ (t) or 'ljJ (I) becomes infinite, while the other
remains finite. When qJ (t 0 ) = oo and 'ljJ (1 0) = c, the curve has a horizontal
asymptote y=c. When 'ljJ{t 0 )=co and qJ(t 0 )=c, the curve has a vertical
asymptote x =c.
If <p(i 0 )='1JJ(to)=co and

lim 'ljJ ((~)) = k; lim ['ljJ (I)- k!jl (I)]=--' b,


t-..1 0 (jl I -+In

then the curve has an incl111ed asymptote y=kx+b.


If the curve is represented by a polar equation r = f (!jl), then we can
find its asymptotes by the preceding rule after transforming the equation of
the curve to the parametric form by the formulas x=rcosqJ=f(qJ)cosqJ;
y=r sin cp=f (qJ) sin cp.

Find the asymptotes of the following curves:


I xl
901. Y = (x-2)• . 908. y=x-2+ y~·
AI+
X
902. y= xz-4x+3' 909. y=e-x'+2.
x• I
903. y = x•- 4 • 910. y = -x•
1-e
x•
904. Y=Xi+g. 911. y=ex.
905, lj = v X
2
-- 1. 912 . Y-
_sin x
X •

X
906. y= .r . 913. y=ln(l-t-x).
r xz-+ 3
yxx+
2
I
907. y= 2-l
. 914. x=t; y=t+2arctant.
a
915. Find the asymptote of the hyperbolic spiral r=-.
(jl
96 Extrema and the Geometnc Applications of a Derivative [Ch. 3)

Sec. 4. Graphing Functions by Characteristic Points


In constructing the graph of a function, first find its domain of definition
and then determine the behaviour of the function on the boundary of this
domain. It is also useful to note any peculiarities of the function (if there
are any), such as symmetry, periodicity, constancy of sign, monotonicity, etc.
Then find any points of discontinuity, bending points, points of inflection,
asymptotes, etc. These elements help to determine the general nature of the
graph of the function and to obtain a mathematically correct outline of it.
Example 1. Construct the graph of the function
X
y=vx2-l .

Solution. a) The function exists everywhere except at the points x= ± 1.


The function is odd, and therefore the graph is symmetric about the point
0 (0, 0). This simplifies construction of the graph
b) The discontinuities are x=-l and x=l; and limY=± oo and
~~1±0
lim y= ±oo; hence, the straight lines x= ±I are vertical asymptotes of the
x~-1±0
graph.
c) We seck inclined asymptotes, and find
.
k 1 = Ilnt !I =0,
-
X-Jo+ oo X

b1 = lim y = oo,
X--++oo

thus, there is no right asymptote. From the symmetry of the curve it follows
that there is no left-hand asymptote either.
d) We find the critical pomts of the first and second kinds, that is,
r Olnts at whtch the first (or, respecttvely, the second) derivative of the giveu
unction vantshes or does not exist.
We have: .

y' = 3 V (x2- I )4 ' (I)

y" 2x (9-x 2)
(2)
9t/(x2-1)7
The derivatives y' andy" are nonexistent only at x= ±I, that is, only at
potnts where the function y itself does not exist; and so the critical points
are only those at which y' and y" vanish.
From (I) and (2) it follows that
y'=O when x= ± )1'3;
y"=O when x=O and x= ±3.
Thus, y' retains a constant sign in each of the intervals (- oo, - )1'3),
(-Y:f. -1), (-1, I), (1, )1'3) and (V3, +oo), and y"-in each of the
Intervals (-oo, -3), (-3, -1), (-1, 0), (0, 1), (I, 3) and (3, +oo).
To determine the signs of y' (or, respectively, y") m each of the indicated
intervals, it is sufficient to determine the sign of y' (or y") at some one point
of each of these intervals.
Sec 4) Graphina Functions by Characteristic Points 97

It is convenient to tabulate the results of such an investigation (Table I),


calculating also the ordinates of the characteristic points of the graph of the
function. It will be noted that due to the oddness of the function y, it is
enough to calculate only for x:;;:;o, 0; the left-hand half of the graph is con-
structed by the principle of odd symmetry.
Table I

X
I
0
I
(0, I)
I
I I(I. I
Jl3) V3::::::1. 731 (V3, 3) I 3 I(3, +co)
!J 0 - ±co + V3
v-~.
0 2
1 37
+ 1.5 +

y - -- I non- - 0 + + +
I I CXIS!
I I I I I
1/"
- - -I-
()
I
- 111011·
t'XI,t
I +
I + I +
I 0
I -

Point Function Funrt ton Functton Point Funclton


Con- of detrt::a'lc'1, Dhcon- dec• ed~C'S, Min. IOCrCa'-le'i; of incrcn"e";
clu- tntlcc- grdph h
llnui t, graph ts puinl graph intlec- gr•ph
ston.., !ton lOilt .1ve l'Onrave 1s LOill ave lion Is conoave
dU\\11 up up down
I

e) Using the results of the investigation, we construct the graph of the


function (Ftg 33).

Fig. 33
4-1900
9_B_ _ _ _E_x_t_re_m_a_an_d_t_he_G_eo_m_e_t_rt_·c_A_p:....:pc....l_ic_a_ti_o_ns_o'-f_a_D_er_i_va_t_iv_e_--!c(_Ch. 8]

Example 2. Graph the function


In x
y=-.
X

Solution. a) The domain of definition of the function is 0 < x < oo. +


b) There are no discontinuities in the domain of definition, but a~ we
approach the boundary point (x = 0) of the domain of definition we have
.
I1my= . In x
11m - = - oo.
X->-0 X->-0 X

Hence, the straight line x=O (ordinate axis) is a vertical asymptote.


c) We seek the right asymptote (there is no left asymptote, since x can-
not tend to - oo ):
k= lim 1!...=0;
X-++ <X> X

b= lim y=O.
X-++<X>

The right a:;ymptote is the axis of abscissas: y=O.


d) We find the critical points; and have
, 1-lnx
y =-X-.-,
!/'-2\nx-3 .
. - xs t

y' and if exist at all points of the domain of definition of the function and
y' = 0 when In x =I, that is, when x =e;

11'=0 when lnx=;, that is, when x=e 312 •


We form a tablt;, including the characteristic points (Table II). In addition
to the charactcrist ic points it is useful to find the points of intersection of

y
lnx
!J=-x

Fig. 34

the curve with the coordinate axes. Putting y=O, we find X= I (the point
of intersection of the curve with the axis of abscissas); the curve does not
;ntersect the axis of ordinates
e) Utilizing the results of investigation, we construct the graph of the
function (Fig. 34).
•* Table II

I I I II
(e, e-i-) I e- ::::::4.49 (e+.+co)
I '' 3
I '
X 0 ! (0. I) I I I I, e) e:::::: 2.72 2

I I
I
I
I
I
I I
: I

I I I I I
' I 3
y I
-cc I
I
- I 0 I
I
-::::::0.37
e + 2
y_::::::O 33
e' +
i I
I I
I I
I I
I I I
y' nonex1st. I -;- -i- T II 0 - - -
II i
:
I
I I I I
I I i
I I I
I I
y"
I nonexist
I - I - ! - - - I 0 +
I I I
I
iI I I i
!

I
I I
Boundary
I
I
Funct
I I
I I Funct Functton
I Conclu·
SlOOS
po1nt of
doma1n o:
dt.f. o' tun-
elton
'
i
1t1crea ... c~
gTdph 1'-
concave
I
F .•n·
,p, 1
g •. trll
t
I
:
Funct
Jr\- r .
JZf.!ph
I
i
Max
pOII1t
ot fund.
deer.,
graph
b (Oncave
Potnt or
in fleet ton
decreases;
graph
1:, concave
~ conra\c 1.5- lOncave
Ver!tcal do\\n do" n do\\n do-..n up
asymptote
100 Extrema and the Geometric Applications of a Derivative [Ch. 3

Graph the following functions and determine for each function


its domain of definition, discontinuities, extremal points, inter-
vals of increase and decrease, points of inflection of its graph,
the direction of concavity, and also the asymptotes.
916. y=x'-3x•.
-
939. y= V x+1-v x-1.
J;-

6x 2 -x 4 940. Y= V (x+4)•-V(X-=4fi·
94t. u=V<x--2>z+V<x-4>~
917
· u=-g-·
4
918. y = (x-1) 1 (x+ 2). 942. y= y .
2
_ (x-2) (x+4)
2 4-x
919 . y -- 4 • 8
2 1
943. Y =X Jf~•- 4 .
(x - 5)
920. Y= 125
2
92 1. Y= x -2x+2
944. y=
X

x2 - l
v .
922. y = x'- .
x-1
3 945. y=
X

(x-2) 2
v .
X
4 946. y = xe-x.
923. Y=x X+ 3 •

924. y=x
2
+-.
2 94 7. y = + (a x; ) e; .
X 948. y=eax-x•-u,
925. Y= x•-t- 3.
I
949. y = (2 + x 2 ) e-x•.
8
926. y = -y----4 • 950. y = 21 x 1- x•.
X - In x
4~
927. y = 4 + x• .
951. u= v--x·
4x-12 x2 x
928. y= (x • 2)•.
952. u=21na.
X
X
929. y = -.---4
x- .
953. u=rnx·
16 954. Y= (x-+ 1) Jn• (x+ 1).
930. y == x• (x-4). 955. Y= ln(x 2 -1) +:;-.~ •
4 1
931 • Y =3x x•+1 . 956. y= In V7+J-I
x .
932. u= Vx+ V4-x. 957. y= In (1 +e-x).
933. y= Vs +x-V-8-x.
958. y =In ( e + ~).
934. y=xVx+ 3. 959. y =sin x +cos x.
. sin2x
935. u=Vx·-~. 960. y= sm x + - -.
2
-- 961. y=cosx-cos• x.
936. y= V I-x 2

962. y = sin' x +cos' x.
937. y= V1-x'. I
938. y=2x+2-3 V<x t 1) 2 • 963. y = --.,.--
5111 x+cosx
S_e_c~·-fi~]___________
D~iff~e_re_n_tr_·a_l~o~f~a~n~A~r~c~·~C~u~rv~a~tu~r~e_____________l01

964. y = --,----,-
sin x
976. y = arc cosh ( x + ! ).
sin ( x+-f )'
965. y =sin X· sin 2x.
966. y = cos x ·co:; 2x. 978. y = earc sin v-x.
967. y=x+ sinx. 979. y = earc tanx.
968. y =arc sin (1- V x~). 98:1. y =In sin x.
969. y = .r
JJ('

r I -x
~ttl X
2
. 981. y =In tan ( T-i).
970. y = 2x-tan x. 982. y =In x-arc tan x.
971. y = x arctan x.
983. y =cos x-ln cos x.
I
972. y = x arc tan-
X
when x =FO 981. y =arc tan (In x).
and y=O when x=O. 985. y =arc sin In (x 1 + 1).
973. y = x-2 arc cot x.
X
986. y=x".
974. Y=:r+arc tanx.
987. y=xx.
975. y =In sin x.
A good exercise is to graph the functions indicated in Exam-
ples ~~6-84H.
Construct the graphs of the following functions represented
paramelrica Ily.
988. X=t•-2t, y~d -t-2f. 1

989. x=aco·,,/t, y=asint (a>O).


990. x=te 1, y=te- 1•
1
991. X=l +e- , y=2tt-e- 21 •
992. x=a (sinht-t), y=a(cosht-1) (a>O).

Sec. 5. Differential of an Arc. Curvc.ture


1°. Differential of an arc. The dr!Terential of an arc s of a plane curve
repre~ented by an equation in Cartesian coordrnates x and y is expressed by
the formula
ds= J((dx) 2 + (dy) 2 ;

here, if the equation of the curve rs of the form

a) y=f (x), then ds = VI+ (t)' dx;


b) x=f.(y), then ds= Vl+(~)' tly;
c) x = q> (I), y = 'i' (t), then ds = V-,-(~-,--7-;)-;;-+--,-(~-,-t--,)-: dt;
2 1

d) F ( 0 th d
V F ·• + r'' d
2
11 F"' + r''
IF~/ IF~/
II I II
x, y)= , en s= x= dy.
102 Extrema and the Geometric Applications of a Derwattve [Ch. 3

Denoting by u the angle formed by the tangent (1n the direction of


increasing arc of the curve s) with the positive x-direction, we get
dx
cos ct= ds,
. dy
Sill CL= ds.
In polar coordinates,

ds = V (dr) + (r drp)
2 2
= V r2 -1- ( ~,)" drp
Denotm~ by ~ the angle between the radius vector of the point of the
curve and the tangent to the curve at this point, we have
dr
cos~=-
ds'
sin B=-= r~!p
tiS
.

2°, Curvature of a curve. Thr curvature K of a curve at one of 11~


points M is the l!m1t of the ral!o of the angle between the posltiVl' din~c­
._.,. M and N of the curve (angle of conltn-
tions of the tangents at the points
gence) to thl· length of the arc i\1 N -~ 1\s when .V - · M (F1t;. 35), tlwt 1~,
K= lim .'\~- du.
~' ~n i\~- ds'

11h<'l-e u. 1s the angle bet\we!l the posJtl\'e d1rectwns of the tangent Jt the
po111t M and the x-ax1s.

The radius of curvature R is the reciprocal of the absolute value of the


curvature, i. e.,
I
R=jl(j·
The circle ( K =f, where a is the radius of the circle) and the straight
line (K = 0, are lines of constant curvature.
Sec 5J Differential of an Arc. Curvature 103

We have the following formulas for computing the curvature in rectan·


gular coordinates (accurate to within the sign):
I) if the curve is given by an equation explicitly, y=f (x), then
K- y" •
-(I+ yl2>"1.'

2) 1f the curve IS g1ven by an equation implicitly, f(x, y)o.=.O, then


F;x f:ll F~
F:x F~y F~
F~ F,, 0
K= {F'z + f'2)'/•
X • y

3) if the curve is represented by equation> in parametric form, x = !p (I),


y = ¢ (t), then
x' yl \
K= I
x" y" '
+
(x1z y'2)'1•
where
1 dx 1 dy d2 x
X = rft' IJ =dt' x" =(ill·
In polar coordinates, when the curve is given by the equation r=f (:p),
W(' have

where
d"r
r I
=-
dr an d ,. = d!p2.
dcp
3°. Circle of curvature. The ctrcle of curvature (or osculatitt~,? ctrcle) of a
curve at the po111t M IS the limittn~ positton of a circle drawn throul(h M
01nd two other pomts of the rurve, P and Q, as P--+ M and Q - • M.
The r;1dius of the ctrcle of curvature is equal to the radius of curvature,
and the centre of the circle of curvature (the centre of curvature) lies on the
nortnal to the curve drawn at the point M in the dtrectton of roncavity of
the curve.
The coordinates X and Y of the centre of curvature of lh<.! curve are
computed from the formula~
y' (I+ y'Z) l-j-y'2
)(=x- , , Y=y-J---,-.y
If
The et•olute of a curve is the locus of the centres of curvature of the
curve.
If in the formulas for determining the coordinates of the centre of curva·
ture we regard X and Y as the current coordinates of a point of the eva-
lute, then these formulas yield pnrametric equations of the evolute \\ ith
parameter x or y (or I, if the curvP itself is represented by equations in
parametric form)
Example 1. Fmd the equation of th<! evolute of the parabola y=x".
104 Extrema and the Geometnc Applications of a Derivative (eh. a·
2
Solution. X=-4x 3 , Y=
1
+26x • Eliminating the parameter x, we find

the equation of the evolute in explicit form, Y=


1+3 (X4 )2(a .
2
The tnvolute of a curve is a curve for which the given curve i~ an
evolute.

-
The normal Me of the involute f 2 is a tangent to the evolute r,; the
length of the arc eel of the evolute is equal to the corresponding increment
in the radius of curvature Ce 1 '-'M 1C1 -MC;
that is why the involute r2 is also called the
c, evolvent of the curve r 1 obtained by unwinding
a taut thread wound onto r 1 (Fig. 36). To each
evolute there corresponds an infinitude of invo-
lutes. which are related to d1fl'erent initial
lengths of thread.
4°. Vertices of a curve. The vertex of a curve
is a point of the curve at which the curvature
has a maximum or a minimum. To determine
the vertices of a curve, we form the expression
of the curvature K and find its extremal points.
In place of the curvature K we can take the
radius of curvature R=l~l andseekitsextremal
points if the computations are simpler in this case.
Fig. 36 Example 2. Find the vertex of the catenary
X
y=a cosh -(a> 0).
a
Solution. Since y' =sinh!__ and y•=..!..cosh_:_, it follows that K=
u a a
and, hence, R =a cosh 2 !__. We have dR X
-d =sinh 2- Equating
a cosh 2 _:_
a x a
a
the derivative ddR to zero, we get sinh 2 _::. = 0,
a
whence we find the ~ole
x
critical point x=Q Computing the second derivative ~x~ and putting mto

wegetddx~~ X= o=~cosh2!_1 o=~>O.


2

it the value x=O, Therefore,


a a X= a
x=O is the minimum point of the radius of curvature (or of thP maximum
of curvature) of the catenary. The vertex of the catenary y=acosh_::. is,
a
thus, the point A (0, a).

Find the differential of the arc, and also the cosine and sine
of the angle formed, with the positive x-direction, by the tangent
to each of the following curves:
993. X2 + Y2 = a 2 (circle).
xz yz
+
994. {i:i jj2 =I (ellipse).
995 y 2 = 2px (parabola),
Sec. 5] Di(Jerenttal of an Arc. Curvature 105

+
996. xzfa yzfa = a 2i 1 (astroid).
997. y =a cosh:__ (catenary).
a
998. x=a(t-sint); y=a(l-cost) (cycloid).
999. x=acos't, y=asin't (astroid).
rind the diiTerential of the arc, and also the cosine or sine
of the angle formed by the radius vector and the tangent to each
of the following curves:
1000. r = aqJ (spiral of Archimedes).
tOOt. r=!!..
cp
(hyperbolic spiral).
1002. r=a sec'; (parabola).

1003. r =a cos' ; (cardioid).


1004. r = aq> (logarithmic spiral).
1005. r 2 =a 2 cos2qJ (lemniscate).
Compute the curvature of the given curves at the indicated
points:
I 006. y = X 4 -4x'-l8x 2 at the coordinate origin.
1007. x 1 +xy+y"=3 at the point (1, 1).
xz 11 z
a
1008. 2 +ijz =I at the vertices A (a, 0) and 8(0, b).
1009. x=l\ y=t' at the point (I, 1).
1010. r 2 =2a 2 cos2qJ at the vertices <p=O and qJ=n.
tOt 1. At what point of the parabola 1/ = Bx is the curvature
equal to 0.12R?
1012. Find the vertex of the curve y=e"'.
Find the radii of curvature (at any point) of the g1ven hnes:
t013. y=x' (cubic parabola).
x2 r/ .
t014. a~+·12 = 1 (elltpse).
I 0 15 =~ ~~ - 1n !I
' X 4 2 '
t016. x=acos•t; y=asin't (astroid).
1017. x=a(cost-+ tsint); y=a(sint-tc;.ost) involute of a
circle).
tOtS. r = aekq> (logarithmic spiral).
10t9. r=a(I+coscp) (cardioid).
1020. Find the least value of the radius of curvature of the
para bola !J 2 = 2px.
1021. Prove that the radius of curvature of the catenary
y=acosh:_ is equal to a segment of the normal.
a
Compute the coordinates of the centre of curvature of the
given curves at the indicated points:
106 Extrema and the Geometric Applications of a Dertvalive (Ch. 3

1022. xy= 1 at the point (1, 1).


1023. ay 1 =X1 at the point (a, a).
Write the equations of the circles of curvature of the given
curves at the indicated points:
1024. y=x'-6x+ 10 at the point (3, 1).
1025. y =ex at the point (0, 1).
Find the evolutes of the curves:
1026. y2 = 2px (parabola).
x• y2
1027. a2 + b2 = 1 (ellipse).
1028. Prove that the evolute of the cycloid
x=a(t-sint), y=a(l-cost)
is a displaced cycloid.
1029. Prove that the evolute of the logarithmic spiral
r = aekcp

is also a logarithmic spiral with the same pole.


1030. Show that the curve (the involute of a circle)
x=a (~os t + t sin t), y=a (sin t-t cost)
is the Involute of the circle x=aco~t; y=asmt.
Chapter IV

INDEFINITE INTEGRALS

Sec. 1. Direct Integration


1°. Basic rules of integration.
I) If F' (x) c~ f (.\ ), tlwn
~ f(x)dx~F ~x) f-C,
where C is an arlJIIrary constant.
2) ~ Af(x)dxo-A ~ f(x)dx. where A is a constant quantity.

3) ~[f. (x):l: f, (\')) d\ .. ~ r. (x)dx±~ f.(x) dx.


4) If St(x)dx.··F(<)-!-C and u-~£f'(\'). then

Sf (11) du -~ F (u) +C.


In part tcular,
Sf
2°. Table of standard integrals.
(a.\ : b) d-..: · *
F (a.\. j b) j- C (a ,~0).

~
\ .\ll-t-1
!. '"d,·--~- '-C n#-1.
( ll ~: 1 I '

II.
J(' dX~~-=In I x I -: C.

5 dx I ' I -.:

_
Ill. ---;;-;::-2-=- arc tan- j- C = - -arc cot-+ C (a# 0).
x- · 1- a a a a a
lV s~o _l_ ln~x-ai+C (a-# 0).
2
x• -a :la x--j-a

S ~=_!__lnja+"]+c
x• 2a a-x
a~--
(a-:/::0).

V. CYdx =lnlx+Yx 2 -l-ai+C (a#O).


J x• +a
VI.
~ f .r y
dx
a•-x•
. X +C =-arc cos-+
=arc ~Ill-
a
X
a
C (a> 0).

VII. Saxdx~' \~xa +C (a> 0); S exdx=ex +C.


108 Indefinite Integrals [Ch. 4

VIII. ~ sin x dx=- cos x +C.

IX. ~ cosxdx=sinx+C.

X S d~COS X
=tan x-t-C.

XI. S _d: X =-cotx-t-C.


Sill

XII S5~:x = In I tan ; I + C = In 1 11osec x- cot x 1 + C.

XIII. Sc~:x=ln !tan ( i+ ~) I+C=Inltan~ -t-secxl-f-C.


XIV. ~ sinhxdx=coshx+C.

XV. ) coshxdx=sinhx-f-C.

XVI. r
.) COS
dxh2 =tanh X+ c.
X

XVII. S .dhx
Sin 2
X
= - cothx-t-C.
Example 1.
~ (ax 2 -t-bx-t-c} dx=) ax 2dx+ ~ b..tdx+) cdx=
=a
5x dx + b sx dx + c sdx
2
3
x~ +ex+ C.
=·a x• -1- b -i

Applying the basic rules 1, 2, 3 and the formulas of integra-


tion, find the following integrals:
2
1031. ~ 5d'x 6 dx.
2
1040 C (x -I.Jl (x 2 L dx
.J v~
-
.
1032. ~ (6x 2 +Rx+3)dx. (xrn- 1")2

1033. ~ x(x-t a)(x + b)dx.


1011.
S-v--:~ - dx.
.(v-Ii- v-xt
- va~-- dx.
1034. )(a+nx') 2 dx. 1042. J
1035. ~ V2pxdx. 1043. Sx 2d~ 7 .
1036.
dx
Sy;-;. 1044. x•:
S 10 .
1037. 5(nx) ~n 1

dx. 1045. sv:: x2.

1038. s(a+ -x+ rdx.


1046. r v8-x
J
dx .
2

t039. ~(Vx-1-l)(x-Vx+l)dx. 1047. J' Y2+~~-=- Y2 x2 dx.


V4-x 4
Sec 1] Direct Integration 109

1048*. a) ) tan• x dx; 1049. a) ) cot·· xdx;


b) ~tanh" x dx. b) ) coth" x dx.
1050. ) 3'~exdx.
3°. Integration under the sign of the differential. Rule 4 considl'rably
expands the table of standard integrals: by virtue of this rule the table of
integrals holds true irrrspecllve of whether the variable of mtegration is an
independent variable or a differentiable function.
Example 2.

_!}x = _!_S (5x-2)-+ d (5x-2> =


S Y5x-2 5

I
=-
5
s --
u
I
1
du=-
fi
I u2
· -+C=-
I 5
-
I

2 (5x-2) 2
I
-
I

2
+C=-
5
--
Y5x-2 t-C '
2 2
where we put u = 5x-2. We took advantage of Rule 4 and tabular integral I.
x dx
V I + x' =
I j'' V d (x = -I 2
In (x + VI + x•) +C.
)
1
__
Example 3.
S -2
. I + (x 2 2 2
)

We implied u =x•, and use was made of Rule 4 and tabular llllef.(ral V.
Example 4. Sx•ex" dxo= ~ S ex' d (x ~ ex +C by virtue of Rule 4 and
3
) =
1

tabular integral VI I.
In ('Xamples 2, 3, and 4 we reduced the given integral to the following
form before malong u~e of a tabular integral:

~f(<p(x))cr'(x)dx=~f(u)du, where u=<p(x).

This type of transformation is called cntegratwrt under the differential sign.


Some common transformat ton~ of d lffc·rcntials, wh1ch were used in Exam·
pies 2 and 3, are:
I I
a) dx=ad(ax-j b) (a#O); b) xdx= d(x 2 ) and so on.
2
ll~ing the basic rules and formulas of integration, find the following in·
tegrals:
1051*"'. s adx.
u-x 1055. Sax+
ax+b
[ dx. 1
1052t* ' S 2
x-l- ~ d 1056.
x"+ 1
dx.
2.1-1- I X. Sx-!
1053.
t-:\1
~~E dx. 1oQr:: 7• s 2
x -j-Gx+7d
-j-:~
S X JC,

1054. sa~-d:x' 1058. Sx ~.:t 1 dx.


4
110 Indefinite Integrals [Ch. 4

1059, s(a -1 X b a rdX. 1078. 52;2~3.


1060*. S(x~ 1)2dx. 1079. 5 ax-l-b
a 2x 2 + b• dx.
1080. r xdx .
bdy J fa4-x4
1061.
5.r-· y 1-y
1081. 51:•xs dx.
1062. ~Va-bxdx. x2 dx

5.r x
1082.
I

.r-=·
y x• - I
1063*.
y x2 -f- I
dx.
1083. JY aric si~2x dx.
1064. sYx; In x dx.
r arc tan!__
2
1065. s3x~~-5. 1084. J 4 -t-x 2 dx.

1085 ' Jlx-Varltan2xd


1066. 57x~~-8. I -t-4x 2 X.

1067. 5 (a-t b)-~:-b) x2 1086. f dx


. V (I + x") In (x + Jl-1 + x2)
,

(0< b <a). 1087. ~ ae-mx dx.

1068. 5x2 ~: 2 dx. 1088. ~ 42 -axdx.

1069. ~,dx.
Sa-x 1089. ~ (e 1-e- 1)dt.

x2-f-4 d X.
1070 • 5~x-t- 6
1090. 5(e-~ + e-~ Ydx.
(ax __ bx)'
1071. r dx . 1091. S a"bx dx.
J ¥1 -t-8~' r Ya""
a•x - I
(' dx 1092. J dx.
1072. 1
J
y 7 -5x . 2

1093. ~ e-<xz+l) xdx.


1073. S~;2 _ 2 dx. 5

1094. ~X· 7x dx.


2
3-2x
1074. 5 5x 2 -t- 7 dx.
1075. 5
3 1
Y5x -1-1
dx. x-t-
2 1095. s~ dx.
1076. 5Yx x-t-
2
3
-4 X.
d 1o96. Jr5v-x -v-x.
d~

xdx
1077.
5 -~-r..
x -o 1097. Se/' 1 dx.
Sec. 1) Direct Integration lll

1098. ~ex Va- bex dx. 1119. ~tan xdx.

1099. S(e-~ + i)~ e~dx. ll20. ~ cotxdx.


1121. scot a~b ax.
llOO*. s2_/~3.
~
tlOl. s ax dx
I +u•x ·
1122. dx
tan ~ .

e-bx 1123. 5tan v- Y-x. X dx


1102. Sl--e-'b·"dx.
1103.
e1 dt
.r -==:.t
. 1124. ~ x cot (x 2 + l) dx.
S r l-e·
1104. ~sin (a+ bx) dx.
1125. s dx
sin x cos x ·

1105. Jl' cos VX 2 dx.


1126. scos- sin - X
a
X
u
dx.

1127. ~ sin• 6x cos 6x dx.


1106. ~(cos ax+ sin ax) 2 dx.

ll 07. ' v- dx
,r--.
1128. s cos ax
sin 5 ax
d
X.

tl08.
•\
l'OS

Ssin(lgx)di·
X
t' X
1129. s sin3x d
3+cos 3x X.

tt 09*. ~ sin x dx. 2


1130. j' y sin.\
cos•
CflS X

x-s111 2 x
-dx.

1131. ~ Vt -i-3cos• x sin 2xdx.


1110*. ~ cos• xdx.
Ill 1. ~ sec• (ax f b) dx.
t 132. St an 3 seczXd
sX
3 x.
1112. ~cot•axdx. 1133. J Ytanx l
cos 2 x l X.

ttl3.)'~.
. X ~ -cot- lxX
23I
Slll-
tl
1134. sm 2 x ·
1114. (' dx
J 3 cos ( 5~:- ~ )
• 1135. s I +sin 3x d
cos 2 3x X.

1115 S (ux+b)
dx -
1136 . s (cos ax 1- sin ax)
smax
2
d
X.
• s111
xdx
·
1137 • s cosec• 3x d
1116. Scos• x• • b-acot3x X.

1ll7. ~xsin(l-x')dx. 1138. ~ (2 sinh 5x-3 cosh 5x) dx.


1118. 5(sin x 1y'2 l )•dx. 1139. ~ sinh• x dx.
112 Indefinite Integrals [Ch. 4

dx
1143. ~ tanh x dx.
1140
· 5 sinh x •
1141.5 COS
dxh
X
• tl44. ~ coth x dx.
dx ·
5. x cush x .
1142. smh

Find the indefinite integrals:

1145. ~ x V5-x 2 dx. 1163. ~ dxx •


cos-
1146. 5x4 ~~~~ dx. SV \+In x dx.
a

1147. 5xs~ 5 dx. 1164.


- dx
1148. ~ xe-" dx. Stan V x-l .r-.
2
ll65.
r x-1

1149. 5 3- V2+3x 2
2 +3x• dx. l166. s-xdx 2 .
Sill x
1150. 5;"-/ dx. 1167. S
earc tan x+x ln(l+x•)-t-1
1 +-\• dx.
~ dx
1151. j Jlex' 1168 , SSillX-CPSXdX.
su1 x +cos x
1152. 5l-sin x dx.
J2Y
1
x+cos x (1-sin

1153 . 5 tan 3~- cot 3x dx.


sm x 3
1169.
f.
.
Sill
X
J.f'j
dx.

dx
1 154. S-~-.- . x
X n X 1170. 5 x• x• 2 dx.
~ec 2
1155.
I
v
y'
tan 2 x-2
dx. 1171.
5 (l+o~)~
x (I +.t•) dx.
dx
1172. ~ e' 111'
1156 \ ( 2 X )
· J + 2x 2
+I 2x 2 +I • x sin 2x dx.
1157. ~ a•lnxcosxdx. 1173 . \ 5-3x dx.
x• ., Jf4-3x 2
1158. J/ dx. dx
SV xdx
.t"-1- I 1174.
Se"+ I·
1159. .r--=. 1175. r dx
v r l-x4 j (a+b)+(a-b)x1
1160. ~ tan• ax dx. (0< b <a).

5
11 61. ' sin
2
~ dx. 1176. 5Ve::_2dx.
1162. s sec2 .t d_x •
Y4-tan 1 x
1177 ·
5
dx
sin ax cos ax •
Sec. 2] Integration by Substitution 113

1178. s c;t + q>o)


sin dt. 1185. 5 sec x tan x dx.

1179. s dx
x (4- ln 2 x) · tt86. s
Y sec 2 x+ I
cos 2x d
4+cos 2 2x X.

tt80.
) arc cos
y
i dx. 1187. s dx
4-x• I +cos 2 x ·

1181. ~ e-tan x sec" X dx. ) ( I n (x+ Yx"+ I)


1188. 1 +~· dx.
1182. \ sin x ens x dx.
,. Jf2-sm 4 x tt89. ~ x• cos (x 3 ~ 3) dx.
1183. s dx
S!l1 2 X CCJS 2 X • ~ 3 tanhA

1184.
,
farc s111 x + x dx.
y' 1-x•
1190.
<
- - J x·
wsh 2 x

Sec. 2. Integration by Substitution


1°. Change of variable in an indefinite integral. Putting
X=<jJ(t),

wlll're t is a new variable and (jl is a continuously differentiable function. we


will have:
~ f (x) dx= ~ f [cp (t))IJl' (t) dt. d)

The attempt is made to choose the function cp in such a way that the right
side of (I) becoml'» more convenient for inlcgrahon.
Example 1. F1nd
~ x v~- 1 d.t.

Solution. It is natural to put t = Y x-1, whence x= t• +I ,md dx= 2t dt.


Hence,

Sometimes substitutions of the form

U=ljl (x)
are used.
Suppose we succeeded in transforming the integrand f (x) dx to the form

f(x)dx=g(u)du, where U=IJl(X).


114 Indefinite Integrals [Ch. 4

If ~ g (u) du is known, that is,

~ g (u) du = F (u) +a,


then
~ f (x) dx=F [cp (x)] +C.
Actually, we have already made use of this method in Sec. 1,3°.
Examples 2, 3, 4 (Sec. 1) may be solved as follows:
1
Example 2. u=5x-2; du=5dx; dx= du.
5

du
Example 3. u=x 2 ; du=2xdx; xdx=2.

S Yx dt
1 +x'
1
=-
2
sy·
du
I+ u2
I
2
( .r- 1 ( 2 .r-)
= - In u+ r l+u 2 )+C=- In x + r I+x4 +C.
2
du
Example 4. u=x'; du = 3x2 dx; x 2 dx = - ,
3

5 e-"
x2 3 l
dx = -
3
5e'' I e"
du = 3- +C = 3I e"'' + C.

2°. Trigonometric substitutions.


l) If an integral contains the radical Ya 2 -x 2 , the usual thin~ ts to put
x=a sin t; whence
Y a2 - x2 =a cos t.
2) lf an integral contains the radical Vx 2 -a 1 , we put X=tl 'cc t,
whence

3) If an integral contains the radical Y.t 2


+a 2 , we put x=a tan/; whence

Y x +a2 2
= a sec t_
It should be noted that trigonometric substitutions do not always turn
out to be advantageous.
It is sometimes more convenient to make use of hyperbolic substituttolls,
which are ~imilar to trigonometric substitutions (see Example 1209).
For more details about trigonometric and hyperbolic substitutions, see
Sec. 9.
Example 5. Find
Sec. 2) Integration by Substitution 115
--~~--------------~----~-------------------------

dt
Solution. Put x= tan t. Then• fore, dx= cos
- 2- .
1

dx= f Vtan• t -t- 1 ~=s sect cos•


Svxz::tT
t __!!:!__=
x• , tan t ens• t
2 sm 2 t cos 2 t
= s dt
sm 2 t cost
2
s 2
= sin 2t + cos t dt =
sin t -cost
s~ scost+
cos t dt =
sin 2 t

=~In I tan t +sect 1--.1


- + C =In 1tan t -1- Yl -!- tan 2 t I-
sm t

- Yf+t3rlit + C =In I x-t- ~I- Yx•-t- 1 +C.


tan t x

1191. Applying the indicated substitutions, find the followmg


integrals:
I
X=-·
t.

b) Sex~ 1 , X=-lnt,
c) ~ x (5x" -- 3) 7 dx, 5x 2 ---:3 = t;
d)
J
c.:drx+ -·
t I
f=l/XTJ;
r· cos x dx
l') \ -cc::..---=-=-= ' fo_ sinx.
v J-1 I +~Ill· X

Applying suitable substitutions, find the following integrals:

1192. ~x(2x--t-5) 1 "dx. 1197.


. I -1-\
1 W3. .r- dx. 1198 .
.\ H- t X
n dt
ll94. l
J x 2r+ I
y -. ' sin' x
1199. .r-dx.
1195. CY~x . ~\ " cos t
.I ex-1
1196. sln2xdx_ 1200*.5 dt .
In 4x x xYl+.,•
Applying trigonometric substitutions, find the following in-
tegrals:
. .r-.
x dx •2
1203. j' ~r_;_:g_az dx.
1201.
J x dx
r 1-x•
1
X

1204*. \' dx .
1202.
S.rr 2-x . 2 , .\ Vx•-1
116 lndefintte lntegral.s [Ciz. 4

dx
1205. 1206*.
Sx• Y4-x•'
1207. ~ Vl--K dx.
1208. Evaluate the integral
~ dx
j Yx(l-x)
by means of the substitution X= sin• t.
1209. Find
~ Va2 +x" dx,
by applying the hyperbolic substitution x=asinht.
Solution. We have: Jf a• + x• = Ya• +a• smh t=a cosh t and dx=a cosh tdt.
2

Whence

~ V a• + x 2 dx = ~ a cosh t ·a cosh t dt =
= a• Scosh t dt = a• 5cosh ;t + 1 dt =a; ({ sinh 2t + t) + C=
2

a•
=
2 (sinh t cosh t + t) +C.
Since

sinh I=; , "V"a•+x"


cosh t = -'-----'--
a
and

et = cosh t + sinh t = x+ Jfa• + x 2


__:.__;.__:__
a
we fin a 11 y get
,r=;;-;-::;2 x . r - a• . r - -2
5r C?'+x dx=2 r
2
u"+~·+
2 In (x+ r a'+x )+C1 ,
wherP. C, = C- a In a is a new arbitrary constant.
2
1210. Find

putting x=acosht.

Sec. 3. lntegrdion by Parts


A formula fer integration by parts. If u = <p (x) and v = 1Jl (x) are differen·
tiable functions, then

~ u dv = uv- ~ v du.
Sec. 31 Integratron by Parts 117

Example 1. Find
~ xlnxdx.
dx x2
Putting u=lnx, dv=xdx, we have du=x· v= Whence
2

S xlnxdx=
X
2
r dx x• s 2
x"
2 lnx- "2x= 2 Inr- 4 +C.
Sometimes, to reduce a given integral to tabular form, one has to apply the
furmula of mtegration by parts several times. In certain ca~es, integration
by parts yields an equation from which the des1red integral is determined.
Example 2. Find
~ex cos xdx.
We have

~eX cosxdx= ~ exd (sin x)=ex sin x- ~ e·-.: sin xdx=ex sitlX+

+~ex d (cosx)=-=ex s111 x+e·-.: cos x- ~eX cos xdx.


lienee,
~ eX COS X dx =eX Sill X+ eX COS X- ~ eX COS X dX,
whence

Applying the formula of integration by parts, find the following


integrals:
1211. ~In xdx. 1221. ~ x sin x co~xd.\
1212. ~arc tan x dx. 1222* ~ (x 2 +5x+6) cos 2x clx.
1213. ~arc sin xdx. 1223. ~ x"ln x dx.
1214. ~ rfx. 1224. ~ln"xdx.
rn.l
X Sill X

1215. ~ x cos 3x dx. 1225. l


-:i>CX.

1216. s exdx.
X 1226. 1lnx
v.-dx.
.\

1217. ~ X·2-XdX. 1227. ~ xarctanxdx.


1218**. ~ x•e•·-.:dx. 1228. ~ x arc sm x dx.
1219*. ~ (x"-2x+5)e-xdx. 1229. ~ In (x + V 1 +x") dx.
1220*. Sx•e -~ dx. 1230. .,;\ Slll(t'
xdx
118 Indefinite Integrals [Ch. 4

X COS X d 1234. ~ eax sin bx dx.


123 1 .
S sin x 2 X.

1232. ~ e" sin x dx. 1235. ~ sin (In x) dx.

t 233. ~ 3x cos x dx.


Applying various methods, find the following integrals:
Yx d
1236. ~ X
I <Jrc sin
2
3
e-x dx. 1246. ,F X.
• r l-x
1237. ~ eV"Kdx. 1247. ~ x tan 2 2x dx.
~ (x2 - 2x + 3) ln xdx.
~
sin 2 x
1238. 1248. -x-dx.
• e
1239. 5xln :+;dx. 1249. ~ cos (ln x) dx.
2

1240.
1n2 x
57 dx. 1250**. 5(x 2 ~ l) dX.
2

S'n(~nx)dx. dx
1241. 1251*.
5 (x2 -t a•)2 .

1242. ~ x2 arc tan 3x dx. 1252*. ~ Va 2 - X 2 dx.


1243. ~ x (arc tan x)" dx. 1253*. ~ VA -1- x• dx.

1244. ~ (arc sin x)' dx. 1254*.

1245 . 5-x-2-cx.l
arc sin x

Sec. 4. Standard Integrals Containing a Quadratic Trinomial


1°. Integrals of the form
d
5axmx+n
-f-bx+c x.
2

The principal calculation procedure is to reduce the quadratic trinomial to


the form
(I)

where k and l are constant~. To perform the transformations in (1), it is


best to take the perfect square out of the quarlraltc trinomial. The folio·
wing substitution may also be used:
2ax+b=t.

If m=O, then, reducing the quadratic trinomial to the form (1), we get
the tabular integrals Ill or IV (see Table).
Sec 4] Standard Integrals Containing a QuadratiC Trtnomwl 119·

Example I.

5
I I x-4
-----=arc tan --+C=
2 Jf31 Jf31
-4- --4-
2 4x-5
=---=arc tan~-+C.
Va1 Y31
If m ¥: 0, then from the numerator we can take the derivative 2ax+~
out of the quadratic tnnomial

m(2ax-f- b) -1- ( n-nzb)


i mx+ n dx=
ax• + bx + c i 2a
ax• + bx + c
_ m
2a dx=

mb ) 5ax•+bc+c
dx
- 2a In I ax + bx +c I+ n-2a
2 • (

and thus we arrive at the mtegral discussed above.


Example 2.

I
~, x--1
- - - - dx-=
\2-x-1
i~(2x--J)-;
12 -x-l
- d~ =-In I x•- x-11-
2
d(x-_!_) _
-2
1
l
2 I _ I 12x- 1- Y 51
2
(x-~)~-~ =21nl., -x-1/-2 V51n 2x-I+Y5 +C.
t.;\ 2 4

• mx+ n
2°. Integrals of the form
j V a.\ +bx+c dx.2
The methods of calculation
are similar to those analyzed above. The integral is finally reduced to tabu-
lar integral V, if a> 0, and VI, if t! <. 0.
Example 3.

Example 4.

' x+3 dx=I. 5 2x+2 dx+2 r dx


J Vx + 2x + 2 2 V x + 2x + 2 J V
2 2
(x + 1) 1 +1
= v X2 + 2x + 2 + 2\n (x + 1 + vx-=·,. . +-=2x=--+"""'2) +a.
120 Indefinite Integrals (Ch. 4

3°. Integrals of the form s dx


(mx+n) Vax 1 +bx+c
. By means of the in-
verse substitution
_l_=t
mx+n
these integrals are reduced to integrals of the form 2°,
Example 5. Find

Solution. We put
s+ (x I) ~ x +I .
2

1
x+l=T,
whence

We have:

4°. Integral~ of the form~ Yax 2 +bx+cdr. By taldngthe perfect square


out of the quadratic trinomial, the given integral is reduced to one of the
following two basic integrals (sec examples 1252 and 1253):
(' . r - - x -- a2
x
I) j r a 2 -x~dx= 2 Va 2 -x 2 +
2- arc sm-Zt-+C;
(a > 0);

2) 5Y x +A dx=; V.!c -j-A + ~ In/ x+ Yx +A /+C.


2 2 2

Example 6.
SVl-2x-x 2
dx= ~ V2-(l+x) 2 d(l-j-x)=
1--J-x .r . I +x
=- - r l-2x-x2 +arcsm V2'+C.
2
Find the following inte.;!rals:
dx
1255.
Sxz+2x+5.
1256. S xz ~:lx.
Sec. 5) Integration of Rational Functions 121

3x-2 dx
1269.
1259.
SX2--4x+5 dx. SxVx "+x-l" 1

I
~ d"'
1260. x (x-.!_L dx 1270.
S +3x ·1-4 ·
2
• {x-1) V.\ 2 -2.
x'dx (' d-<
126 1. x -6x + 10 1271.
S 2
j (x -1- 1) V x• 1- 2' ·
1262. •\
·• dx
v·2 t-3x-2x . 1272. ~ Vx"-J-2x+5dx.
2

1273. ~ yx-=-?dx
1263.
Jr vx-x•
dx -
1274. ~ V2-x-x 2 dY.
1264. SV dx
x"+ px J- q
.
1275.
xdx

1265. \ 3
x-
6
d
S° x -4x -J-3 ·
4 2

J V x -4x+5
2 r. 1276. J COS X
sm 2 x-6 su1 x+ 12 dx.
1266. ~S_-B - dx. exdx
SV I-x-x 2 1277.
) Vi -f-ex 1-c•x
1267. c X d.\
J Vsx·-~i=ld~.
Stll \
1278.
"I v- cn; +4 2
X liJS X ]- I
1268. C dx
1279.
' ln x d\
JrVl-x•" Jx VI- 11 n x -ln 2

Sec. 5. Integration of Rational Functions


1°. The method of undeiermined coefficients. Integration of a rational
function, after taktng out tlw whole part, reduces to mtehratwn of the proper
ra/tonal fructton
p (x)
(I)
Q (x)'
where P (x) and Q (x) are mtegral polynomtals, and the dt>gree of the nume·
rator P (x) is lower than that of the denominator Q {x).
If
Q {x)={x-u)~ .. .(x-l)',

where a, ... , l are real dtslinct roots of the polynomi;)l Q {x), and a,
A. are natural numbers {root multiplicities), then decomposition of (1) i~t~
partial fracltons is justified:
P (x) A, A2 Aa
Q {x)"""x=a+ (x....:.:u) 2 + · · '+ (x-a)"+ · ·'
· L1 L2 L,.
... +x-t+(x-1)2+ ... +(x t)1.· <2>
To calculate the undetermined coefficients A 1 , A 2 , ••• , both sides of the
identity (2) are reduced to an integral form, and then the coefficients of
like powers of the variable x are equated (tlrst method). These coeffi.
cients may likewise be determined by putting [in equation {2) or 111 an equi·
valent equation) x equal to suitably chosen numbers (second method).
122 Indefinite Integrals [Ch. 4

Example t. Find
xdx
S(x-1) (x+ 1) 2 I.
Solution. We have:
X A 81 82
(x-1) (x + 1) 2 = x-1 + x + I + (x + 1) 2 '

Whence
x= A (x-f- 1) 2 -f-8 1 (x-1) (x+ ))-f-8z(x-1). (3)

a) First method of determtntnR the coeffictents. We rewrite identity (3) in


the form x=(A-f-8 1 )x 2 -f-(2A-j-8 2 )x+(A-B,-8 2 ) Equating the coeffici-
ents of identical powers of x, we get:
O=A-f-8 1 ; 1=2A-j-82 ; O=A-8,-8 2 •
Whence
I
A=-;r:

b) Second method of determtning the coeffictents. Putting x= 1 in identity


(3), we will have:
I=A .A, i.e., A= 1 /,.
Putting x=-1, we get:
1
-1=-8 2 ·2, i.e., 82 = / 2•

Further, putttng x=O, we will have:

Example 2. Find
dx =I

Solution. We have:
Sx -2x -j-x
3 2 ·

I A B C
x 3 -2x 2 -f-xx-.,-(x-1'"")2 =x+ x-1 + (x-1) 2

'8nd
I =A (x-1) 2 -f- 8x (x-1)-j-Cx. (4)
When solving this example it is advisable to combine the two methods
of determining coefficients. Applying the second method, we put x=O in
identity (4). We get I =A. Then, putting x= 1, we get I =C. Further, app-
lying the first method, we equate the coefficients of x 1 in identity (4), and
get·
O=A-f-8, i.e., 8=-1.
Hence,
11=1. 8-=-1, and C=1.
Sec. 5] Integration of Rational Functions 123"

Consequently,

I=
5 5
dx
--;-
dx
x- 1 +
5 dx
(x- 1)"=lnJxl-lnlx-ll-x-I'C.
I ,

If the polynom1al Q (x) has complex roots a ± 1b of mu lhphcity k, then


partial fractions of the form
A,x+B, + + Akx -t-Rk (5)
.1.•-t-px+q · (x 2 -t-px-f- <d
will enter 1nto the expansion (2). llere,
x2 + px+ q = (x-(a -f-tb)) [x-(a--zb)]
and A,, 8 10 •• , Ak, Bk are undetermined coeflic1ents wh1ch arc determmed
by the methods given above For k = 1, the frart10n (5) IS integrated dir(:ct-
ly; for k >I, usc is made of the reductzon method; here, 1t is first advi-
sable to represent the quat\rahc trinO!l1ta\

+( q-~) and mahe the substitution


X

x-t-%=z.
2
+ px + q in the fOr Ill ( X+~ rr

Example 3. F111d
x+ I
5(~ -l-4x 2 f 5)~ dx:.o/,
Solution. Smce
~
2
-1 ·h l 5 -- (x + 2) 2 + I ,
then, pulling .r-1-2--cZ, W\' i,!d
(' z-1 .
I= j (z• t- 1)2 az =
5(l:·- i l)i- j (z2 +T)2
;_dz ('(1-t-z')-z'
dz-=

I
=---~(z 2 ·i () ___ J
(' d.' 1
i"-i-=-T 'J czd I_-2(z'+l)
I I----2(z I-t-1}- 2

I z Z-/- I
--.trc tan z-- -- ----1--- arc tan z=- - -2 - - -
2(.Z2-I- I) 2 2(z -f- I)
_ _!_Jtl tanz.LC
2
1 =- 2
1
+ 3 _ _!_ate lau'x-J-'')-j-C
2(x -f--!r+5) 2 '- •
2°. The Ostrograd~ky method. If Q (.1) has multiple roots, then

\'P(x)rlr~o:.\:.£1'L+('Y(x) dx, (6)


oJ Q (.1) (~, (.\) J QJ (x)

where Q, (x) is the greatest common dtvtsor of the polynomial Q (x) and tis
derivative Q' (x);
Q1 (x) = Q (x): Q1 (x);

X (x) and Y (x) are polynomials wt!h undetermined coefficients, whose degrees
arc, respectively, less by umty than those of Q 1 (x) and Q2 (x).
The undetermined coefficients of the polynonuals X (x) and Y (x) are
computed by difrerentiating the identtty (6).
Example 4. Find
r dx
J (x' -1)2.
124 __________________ __l_nt_e~gr_a_ls_________________ [Ch. 4
In_d_e~~-nt_·te

Solution.
~=Ax"+Bx+C +SDx"+Ex+F dx
S (x -1) x•-t x3 - l
3 2

Differentiating this identity, we get


I (2Ax+B) (x3 -l)-3x2 (Ax"+ Bx+ C)+ Dx'+ Ex+F
(x1 -1) 2 (x 3 - l ) 2 x•-1
()r
I= (2Ax+ B) (x 3 -l)-3x2 (Ax"+ Bx+G) + (Dx"+Ex+F) (x -l).
3

Equating the coefficients of the respective degrees of x, we will have:


D=O; E-A=O; F-28=0; D+3C=O; E+2A=0; B+F=-1;
whence
1 2
A=O; B= -
3 ; C=O; D=O; E=O; F= - 3
and, consequently,
dx 1 x dx 25
S
(x 3 --l) 2 =-3 x•-1-3 x•-1 (7)

To compute the integral on the right of (7), we decompose the frachon


...--!--
x- 1
into partial fractions:

that is,
1 = L (x'+x+ 1) + Mx (x-1) +N (x-1). (8)
I
Putting x= 1, we get L=
3 .
Equating the coefficients of identical degrees of x on the right and leU
,of (8), we find
L+M=O; L-N=l,
,or
I 2
M- ---
3 ·' N=-- .
3

and
dx x J x2 + x + I 2 2x + I
S (x'-l)"=-3(x•-1)+91n (x-1)" +3 Jl3arctan l-,-3 +C.
Ftnd the loJiowwg integrals:

1280. s(x+a~~x+b) • 1282


dx
l)(x+2)(x+~) ·
S(x+2x"+41x-91

1281 sx·-s~+ 9
dx. 1283• (x-l)(x+3)lx-4)dx •
2
.\ -5xt-6 S
Sec. 6] lntewatinf!. Certain Irrational Functions 125

1284.
s 5x -j-2
1

x•- 5 -~.•+ 4 xdx. 1293. s (..1. 2 -4x+ 3) (x 2


dx
+ 4x-t- 5) •
1285. s x (xdx+ 1) 2 • 1294. s dr

1286. s x•-}
4-X .- - dx. 1295.
x3 -j- 1'
sx-:n· dx

1287.
s X

x -6x -j- 12r2-j-6


4 3

x•-6-~."!-12x-8 dx. 1296 . ..,\ dx


..~.•-j-.~:2+ 1.
5 5r -j-6x-j-92

1288. (x- ) (x+ ) dX. 1297. 5(1 +x


dx 2
32 12
s
2
)
1289.
2
x -8x+7
(..1. 2- 3x_ 10 )2 d'C. 1298. s 3x-j-5
(x• + 2x + 2) 2 dx.

1290.
s 2x-3
(x•- 3x+ 2 l.dx. 1299. s dx
(x + I) (x 2 + x + 1)3
1291. s x'+x+ 1
x(x 2 + I) dx. 1300. 5(x -4x-j-5) dx.
2
x"+ l
2

1292. S)(•x< l dx.

Applying Ostrogradsky's method, find the following integrals:

1301. 5(x+ 1/(:•+ 1)•. 1303. .)r (x 2 dt


-t- l )4 •

1302. 5(\.~ 1)2. 1304.


" x4 -2\ 2 -j-2
J(..1.•-2x+212dx.
Applying different procedures, find the integrals:

1305. S(..1.• -t-llt~x"-1-S)dx. 1310*. 5qx~xl 1).


x 1
+,.t 3
d\
1306.
5x• x -\-j-14
2_

2
+ dx.
2 1 13 I I.
SX(":\ --:tl) dx
5 2 •

1307. S (x- 1 )• (x- 2 ) dx. 1312 ·


S(x +-2~ t2) (x +2x-j-5)'
2 2

1308. 5)(• (x~~- 1) 2


1313. 5(t-Il'"'
x"d\

1309. 5x•-4x2d~-5r-2. 1314. 5x•~..~.G·


Sec. 6. lnhgrating Certain Irrational Functions
t•. Integrals of the Lrm
p,

SR [_x. (ax+b)ii;
cx+d ' (1)

where R is a rat1onal functton and p 1 , q 1 , p 1 , q 1 are whole numbers.


12S Indefinite Integrals [Ch. 4

Integrals of form (1) are found by the substitution


ax+b n
--=Z
cx+d '
where n is the least common multiple of the numbers q., q 2 ,
Example I. Find
.~ dx
J\--------
V2x-!- V2x-J·

Solution. The ~ubstitution 2.\-l =z4 leads to an tntegr3l of the form

Find the integrals:


r ./ dx.
3
1315. - 1321.
.J r x-1
I.(2-- ,\)
' x dx _dr~
1316. --- 1322.
•\ t/ax+ b · v ~r I - \
.. J(.r---=I
J'-v-~·+, ~xV_<--;+ 1)3. 1323. 11 ..
--- ·'·
1 l17.
• \ \
. \+I
, d~ r---
13ts. \ v ..
\ T v·~ X . 1324.
I
,
·v\ : __ :_:__l_t~x.
\ -I
••r-
"'' t --1
n X -1 3
Jl X-.r--=--
JaHL ~---=-dx. 13~5. dx.
•\ v ~+I r 2x ~- :.l
2

1320. (' _(-~ +2 dx.


J (x+ 1)
2
- Vx+ I

2°. Integrals of the form

(2)

where Pn (x) is a polynomta l of degree n


Put

J .r 2
P,. (x)
r ax + bx+c
dx= Q 11 _ 1 (x) r ax1 +bx+c+A.
.r sV dx
- ,
a.\ 2 -!--bx+c
(3)

where Q,._dx) is a polynomial of degree (n-1) with undetermined coeffl.


cients and A. is a number.
The coefficients of the polynomial Q11 _ 1 (x) and the number f.. are fuund
by differentiating identity (3).
Sec. 6} Integrating Certatn Irrational Functions 127

S x•
Example 2.
_
V x• + 4 dx = JVx4-j-4x
x 2 -j-4
2

dx =

=(Ax'+Bx'+Cx+D> . r-
r x 2 -j-4 +"- \ . dx2 .
v Jl x +4
Whence
.\•+4x' =(3Ax"+2Bx+C) Vx•+4+(Ax'+Bx"+Cx-j-D)x + A .
v x•-j-4 }r.l. 2 +4 V.l. 2 -j-4

Multiplying by Vx 2 +4 and equating the coefficients of identical degrees of


x, we obtain

Hence,
sx•JI x2 -J-4 dx= xs ~ x V x•-j-4 -2ln (x+ Vx• +4) +C.
2

3°, Integrals of the form

J (x-a)n ::x• + bx +c. {4)

They are reduced to tnll'grals of the form (2) by the substitution:


_1_=1 .
.\-a

Find the integrals:


1326. J' y,,•-
x•d~
~+I
• 1329.

Jl ,r;:_=-=-.dX.
1:327. 1330.
r I-.\"
I :328. r ,r Ix•·i- x• dx.
.l 1
1331 .

4°. Integrals of the binomial differentials


~ xm (a+ br")P dx, (5)

where m, n and p are rational numbers.


Cheby~hev's conditions. The integral (5) can be expressed in terms of a
finite combination of elementary functions only in the following three cases:
I) 1f p i~ a whole number;
1
2) if + is a whole number. Here, we make the substitution a+ bxn =
111
n
= z8 , where s is the denominator of lhe fraction p;
1
3) if
111
+
n
+ p is a whole number. Here, use is made of the substitution
ax-n+b=z , 3
128 Indefinite Integrals [Ch. 4

Example 3. Find

1 1 1 m+ 1 --}+t
Solution. Here, m= - 2 ; n=4; P=3; n 1
=2. Hence,
4
we have here Case 2 integrability.
The substitution

I +x 4 =z'
yields x=(z 8 -1) 4 ; dx=12z 2 (z 1 -1) 1 dz Therefore,

where z=VI+ t/x.


Find the integrals:

1332. ~ X
3
(1 + 2x 2
)-
s
2 dx. 1335. ,x·
\
'
v dx
lt-x•

1333. svdx . 1 -f-.x•


1336. ,. dx

Jx2 (2-f-x 3 ) 3
5
,

1334.
S.x 4
.r
I'
dx
1 -f-x 2
. 1337. ~

J V.x•
v + Vxf .dx
1

Sec. 7. Integrating Trigonometric Functions


1°. Integrals of the form

~ sinmxcosnxdx==lm, n• (1)

where m and n are integers.


l) If m= 2k + 1 is an odd positive number, then we put

I m, n = - ~ sin 2k x cosn xd (cos x) = - ~ ( 1-cos2 x) 1l cosn xd (cos x).


We do the same if n is an odd positive number.
Example t.
~ sln10 x cos1x dx = ~ sh110 x ( 1- sln 2 x) d (sin x) =
sin 11 x sin 13 x
= -11-- ---r3 +c.
Sec. 7) Integrating Trigonometric Functions 129

2) If m and n are even positive numbers, then the integrand (1) is trans-
formed by means of the formulas
I I
sln2 x= (I-eos 2x), cos• x= (I+ cos 2x),
2 2
1
sin x cos x=
2 sin 2x.
Example 2. ~ cos• 3x sin 4 3x dx = ~ (cos 3x sin 3x) 2 sin 2 3x dx =

- 4- 6x 1-cos
= sln
S 2
2
6x dx= I
8
s (sin 2 6x-sln 2 6x cos 6x) dx=

= ~ S(l-c~s 12x sin 2 6x cos 6x) dx=

= _!_ ( ~ -
8 2
sin 12x- _!_ sins 6x )
24 18
+ C°
3) If m = - ~t and n = - v are integral negative numbers of identical
parity, then

Im
'
11 = S
Sill
f'- dx
X COS X
, =Scosec~'-xsec•-•xd(tanx)=
~ ~+v
- V-2 --1
2

S(
2
= I+ 1- 1-- )" (l+tan 2 x) - 2 d(tanx)= s(l+tan
t .•
x)
an• x
d (tan x}.
\ an 2 x

In particular, the following integrals reduce to this case:

Example 3.
S...!!:_
cos• x
=S sec• xd (tan x) = S(I+ tan 2
x) d (tan x) =

=lan x+ 3I tan 1 x+C.


Example 4.
SSin~dt x = ¥1 ~ sin 1
-
xdx;·
cos 1 -
x BI stan -a X s X d
2 sec 2 x =
2 2

=..!_
8 t an 3 X
2
r
~ (I +tan• i sec 2 ..::_dx=~~ [tan-a ~+-2 -+ 2 8 2 t an X
2

~ ;] +C.
131 2

+tan ~] d( tan f)=! [- - -x +2ln j tan ~ j +


1
2tan•
2
5-1900
130 Indefinite Integrals [Ch. 4

4) Integrals of the form ) tanm x dx (or ) cotm x dx), where m is an In·


fegral positive number, are evaluated by the formula
tan 1 x=sec1 x-1
(or, respectively, cot• x = cosec• x- 1).
Example 5. 5 5
tan• x dx = tan• x (sec• x-1) dx = ta~• x- 5tan• x dx =
- 3-x-
-= tan• s - -x- tan x + x +C.
(sec• x-1) dx= tan•
3
5) In the general case, integrals I m n of the form (I) are evaluated by
means of reduction formulas that are usually derived by integration by parts.
2
E s dx s sln x+ cos• x d
xamp 1e 6· cos' x = cos' x x=
=S sin x
slnx·--dx+ dx .
s --=smx·----- 1 cos x
1 s --dx+ s -dx- =
cos' x cos x 2 cos• x 2 cos• x cos x
sin x I
o : : : :2 - +- 1njtanx+secxl+C.
2 COS X 2

Find the integrals:


1338. ~ cos• xdx. 1352. ) X
dx
X
sin- cos1 -
1339. ) sin'xdx. 2 2
) sin ( x+ ~ )
1340. ~ sin• x cos' x dx. 1353. sin x cos x dx.
· · x cos ,xdx.
1341. ssm 2 2 1354. 5
dx

1342. s
cos' x d
sin' x X.
sin' x ·
1355. ~sec' 4x dx.
1343. ~ ~in' xdx. 1356. ~ tan• 5xdx.
1344. ) sin~ x cos• x dx. 1357. ~ cot• xdx.
1345. ~ sin! x cos• x dx. 1358. ~ cot• x dx.
1346. ~ cos6 3x dx. 1359. 5\ tan• 3 + tan• 4
1
X X) dx.
dx4
1347. s sin 1360. ) x sin 1 x• dx.
x'
dx
1348. s cos• x·
1361. 5
cos• x
stn• x dx.

1349. 5
cos• x d
sin 8 x X. 1362. ) sin' X Vcosxdx.

1350. s sln1 xdxcos• x • 1363. sY dx


sin x cos• x •
1351. s dx
sin1 x cos• x •
1364. s dx
Ytanx'
Sec. 7] I ntegrattng T rigonometrlc Funct tons 131

2°. Integrals of the form ~ sin mx cos nx dx, ~ sin mx sin nx dx and
~cos mx cos nx dx. In these cases the following formulas arc usedl

l) sin mxcos nx= {[sin (m+ n) x+sin (m-n) x];


I
2) sin mx sin nx = [cos (m -n) x-cos (m + n) x);
2
I
3) cos mxcos nx= [cos (m-n) x +cos (m + n) x].
2
Example 7. Ssin9xsinxdx=S ~ [cos8x-cosl0x]dx=

=~sin8x-~sin lOx+C.

Find the integrals:


1365. ~sin 3x cos5xdx. 1369. ~ ros(ax ~ b)cos(ax-b)dx.
1366. ~sin lOx sin 15xdx. 1370. ~sin rot sin (rot +cp) dt.
1367. s cos 2X cos 3X dx. 1371. ~cos x cos• 3x dx.

1368. s· .2Xd
Sin 3X Sin 3 X. 1372. ~ sin x sin 2x stn 3x dx.
3°, Integrals of the form

~ R (sin x, cos x) dx, (2)

where R ts a rational fttnction.


I) By means of substitution
X
tan =t,
2
whence
. 21 l-1 2 2dt
stnx=l+t•• cost'=l+t•• dx=l+t~'

integrals of form (2) are reduced to Integrals of rational functions by the


new variable t.
Example 8. Find
dx
S
l+sin x+cos x
1
'

Solution. Putting tan ~ =t, we will have


2dt
l +t•

s•
I=
S 2t l-t 2
l+ 1 +ta+ 1 +t 1
132 Indefinite Integrals [Ch. 4

2) If we have the identity


R (-sin x, -cos x) eaR (sin x, cos x),
lhen we can use the substitution tan x = t to reduce the integral (2) to a
rational form.
Here,
t l
slnx= ,r , cosx= , r -
r l + t• r I+ t•
and
dt
x=arc tant, dx=l+t',
Example 9. Find
dx
Solution. Putting
S1+sin" x 1
·
(3)

t• dt
tanx=t, sln'x= 1 +t•, dx=1+t•'
we will have
I= r dt
J (l +tl) ( 1+ 1 ~ t•)
s~=-'-S d(ty'2)
1 + 2t• V2 1 + (t y--2 )I
1 .r- 1 .r-
= Y2" arc tan (t r 2)+C= Y"2 arc tan ( r 2 tan x)+C.

We note that the integral (3) is evaluated faster if the numerator and
denominator of the fraction are first divided by cos 2 x.
In individual cases, it is useful to apply artificial procedures (see, for
example, 1379).

Find the integrals:

1373. s3+~:osx' 1382* · S3 sin x +·,---s=-c-o--;;s•:-x


dx2 ·
1374. Ssin x+cos
dx
x'
1383*

Ssin -'+3sinxcosx-cos
2
dx
x' 2

1375.
cosx
Sl+cosx
d
X. 1384*

Ssin 2
dx
x-5sin x cos x ·

1376.
sin x
Sl-sin x
d
X.
1385. S(I-eos
sin x
x)'
dx
·
1377. s dx
8-4sinx+7cosx' 1386. S
sin 2x
1 +sin• x dx.
1378. Scos x + dx2 sin x + 3 · 1387• cos 2x d
Scos x+sin x X.
4 4

1379**, s2smx+3cos
3 S~n
x+2
x
d COS X
X.
1388 '
Ssin x-6sinx+5
1
COSX d
X.

1380. s+l tanx dx


1-tan x '
1389*. S dx
(2-sin x) (3-sin x) ·
1390*. sl-s~nx+cosx dx
1381*. 5l+:~os•x' 1+smx-cos x ·
Sec. 8) Integration of Hyperbolic Functions 133

Sec. 8. Integration of Hyperbolic Functions


Integration of hyperbolic functions is completely analogous to the inte•
gration of trigonometric functions.
The following basic formulas should be remembered:
1) cosh 2 x-sinh 2 X= 1;
2) sinh 2 x = ~ (cosh 2x-1);
l
3) cosh 2 x=
2 (cosh 2x+ 1);
4) sinh x cosh x =-} sinh 2x.

Example t. Find
~ cosh 1 x dx.
Solution. We have

5 cosh 2 x dx= 5~(cosh 2x+ l)dx= ~sinh 2x+ ~ x+C.

Example 2. Find
~ cosh 1 x dx.
Solution. We have
~ cosh 1 xdx= ~ cosh 2 xd (sinh x)= ~ (l+sinh 2 x)d(sinhx)=
. sinh x
1
=stnh x+- 3- +C.
Find the integrals:
1391. ~sinh' x dx. 1397. ~tanh 1 xdx.
1392. ~ cosh 4 x dx. 1398. ~ coth 4 x dx.
1393. ~sinh' xcosh xdx. 1399. 5sinh x +dxcosh x •
2 2

1394. ~ sinh• xcosh 2 xdx. 1400 · 52 sinh x ~ 3 cosh x '

1395 · 5smhxd:osh 2 x' 1401*. 5 tan::-1 •

1396· 5 sinh 2 :~osh 2 x • 1402 • s sinh x dx •


Ycosh 2x

Sec. 9. Using Trigonometric and Hyperbolio Substitutions for Finding


Integrals of the Form

~R(x, Yax 1
+bx+c)dx, (1)

where R ls a rational function.


134 Indefinite Integrals [Ch. 4

Transforming the quadratic trinomial ax 1 + bx+c into a sum or difference


of squares, the integral (1) becomes reducible to one of the following types
of integrals:
1) ~ R (z, Ym 2
-z 2) dz;

2) j R (z, Ym 1
+z 2 ) dz;

3) ~ R (z, Y Z2 m 2) dz.

The latter integrals are, respectively, taken by means of substitutions:


1) z=msint or z=m tanht,
2) z=m tan t or z=m sinh t,
3) z = m sec t or z = m cosh t.
Example 1. Find

Solution. We have
x2 + 2x+ 2= (x+ 1)•+ 1.
Putting x+l=tanz, we then have dx=sec 2 zdz and

I=
r dx
J (x+l)'Y(x+l)'+l=
2
s sec2 zdz
tan zsecz
s~dz=
2
sin z
= __1_ + C= Y x + 2x + 2 +
2
C
sin z x+ I '
Example 2. Find

Solution. We have
2
x2 +x+1= ( x+ 21 ) 3
+-;r·
Putting
t V3 . ¥3
and dx=--coshtdt,
2-smht
x+'2=-
2
we get

I= S( -V3. 2
- smh t-
2
t) -V32- cosh t. -V32- cosh t dt=
=3JF3s.
--
8
smh t cosh 2 t dt-
8
3s cosh 1 t dt =

V3
=3-
cosh 1 -
t -
3 ( 1 . 1 )
+ C•
8- - -3 8 -2 2 t
smh t cosh t-j--
Since

and
sinh t= ~ 3 ( x+ ; ) , cosh t= r, Yx 2 +x+ 1

t=ln (x+ ~+Yx 1 +x+t)+ln ;


3
,
Sec. 11] Using Reduction Formulas 135

we finally have
I

/=3(x
1 1 +x+l) -1 ~41 ( x+2I)
Yx 1 +x+1-
-~ In (x+ ~ + Yx 2 +x +1) +C.
Find the integrals:
1403. ~ V3-2x-x• dx. 1409. ~Vx•-6x-7dx.

1404. ~V2+x 2 dx.


" x•
1405. I
J
Y dx.
9+ x 2

1407. ~ V x• -4 dx.

1408. ~ Vx• -t-x dx.

Sec. 10. Integration of Various Transcendental Functions


Find the integrals:
1415. ~ (X2 + 1) 1
e2 x dx. 1421. 5
dx
e•-"+e-"- 2 •
1416. ~ x• cos• 3x dx. 1422. sVe•-" +e-" +I
dx •

1417. ~ x sin x cos 2x dx. 1423. 5x•ln : +;dx.


1418. ~ ezx sin' x dx. 1424. ~ ln 1 (x + V1 +X 1
) dx.
1419. ~ ex sin x sin 3x dx. 1425. ~ x arccos (5x-2) dx.
1420. ~ xex COS X dx. 1426. ~ sin x sinh x dx.

Sec. 11. Using Reduction Formulas


Derive the reduction formulas for the following integrals:
1427. In= 5(x•!xa•)n; find I 1 and I,.
1428. In=~ sinn xdx; find I, and I,.
136 Indefinite Integrals [Ch. 4

1429. In= Sco~~ x ; find I 1 and I 4 •


1430. In=~ xne-xdx; find 110 •

Sec. 12. Miscellaneous Examples on Integration


1431. 1448. s+ Y(I
xdx
x2) I - x4

1432. Sxt~;:_ + 2 dx. 5


1449. sYl- xdx
2x 2 -x4
.

1433. r x' I dx. 1450. f x+ I~ dx.


J x2+x+2 J (xa+ I) •
1434. s (x~~
x 5) . 1451*. S(x +4x)dxY4-x
2 1

1435. s + 2)~:x +
(x 3)" • 1452. ~ VXC9 dx.
1453. ~Vx-4x" dx.
1436. s(x+ l)~~x•+ I).

1437. S(x2 ~ 21 •.
1454. r
J x Yx +x+ 1 2
dx .

1438. Sx _::a + 1 • 1455. ~ x V x• + 2x + 2 dx.

1439.
s(xa-x +
4

xdx
1)3 .
1456. s /Xx4 x• - I
,
1440. s· 3
(I-2Y x)"
-
4
x dx. 1457. \
v x
dx
y I -x• .
S Vx~/ I)• dx. " dx
1441. < 1458 . jy .j/
I +x•

1442. sY + x+ x2
dx
I
. 1459. JV +x I
5
x 4
dx.

1443. ~ ~-x2:; dx. 1460. ~ cos 4 x dx.


1461. Scos xdxsin 5 x ·
1444. J(V x•+ V x ) .
Jr +smy-corx
dx 2
1462. 1 d
1445. sY 2x+ I dx
1463. J
x x.
sin' x dx.
2

1446. s (4x 2 -2x+ l)a

Vs-x+ Y5-x ·
dx
·
tfcos• x

1464. ~ cosec 5 5x dx.


1447. JJl xz
(x 1 -1)3
dx. 1465. s sinz x d
cos 8 x x.
Sec. 12] Miscellaneous Examples on Integration 137

1466. Ssin ( ~ -x )sin(~+ x) dx. 1484. ~ sinh x cosh x dx.

1467. Stan• ( ; + ~ ) dx. 1485. 5 sinh Vt-x dx.


Yi-x
1468. s2sinx+3cosx-5'
dx 1486. ssinh
sinh x cosh x d
x+cosh x x.
2 1

1469. s2+3dxcos x • 2 1487. ~ +.dx.


, Stn X

1470. s 2
dx 1 1488. sezx_2ex.
dx
cos x+2sinxcosx+2sin x •
1471. ssin xdxsin 2x · 1489. sezx- 6ex
ex

1472. s(2+cos x)dx(3+cos x) · 1490. I + 13 dx.


e•x 2. dx.

1473. J secz x
'Vtan 2 x+4 tan x+ I
dx.
s 2x
(ex+l)'

1474. 5 cos ax d
Yu 2 + sln 1 ax X.
1491. 1-4x dx.
1492. ~ (X 2 -1) w-zX dX,
1475. scos
x dx
3x · 2
1493. ~ llex t 1 dx.
1476. ~ x sin x dx. 2
1494. s arc tan
I
x d X.
X
1477. ~ 2
et' dx.
X
1495. Sx• arc sin ~ dx.
1478. ~ xezx dx.
1496. ~ cos (1n x) dx.
1479. ~ x 2 ln V 1-x dx.
1497. ~ (x'-3x) sin 5x dx.
J
1480. x arc tan x dx.
Yl +x' 1498. ~ x arc tan(2x+3)dx.
1481. Slll· x
s · 3x
2 COS <jdX. 1499. ~ arc sin Vx dx.
1482. s (sin x+
dx
cos x) 1 ' 1500. ~ /xldx.
1483. s dx
(tan x+ 1) sin 1 x •
Chapter V

DEFINITE INTEGRALS

Sec. 1. The Definite Integral as the Limit of a Sum


1°. Integral sum. Let a function f (x) be defined on an interval aos;;;;x.;;;;b,
and a=x0 < x 1 < ... < Xn=b is an arbitrary partition of this interval into
n subintervals (Fig. 37). A sum of the form
n-t
Sn = ~ f (s;) flx;, (I)
l=o
where
x;o;;;;;s;o;;;;;x;+ 1 ; flx;=x;+ 1 -x;;
i=O, 1, 2, ... (n-1),
is called the integral sum of the function f (x) on [a, b]. Geometrically, Sn
is the algebraic area of a step-like figure (see Fig. 37).

y
y

Fig. 37 Fig. 38

2°. The definite integral. The limit of the sum Sm provided that the
number of subdivisions n tends to infinity, and the largest of them, tl.xi,
to zero, is called the definite integral of the function f (x) within the limits
from x=a to x=b; that is,
n-1 b
lim ~ f (;j) tl.Xj = r f (x) dx, (2)
max Ax1 -. o 1 = 0 ~
Sec. 1) The Definite Integral as the Limit of a Sum 139

If the function f (x) Is continuous on [a, b), it is integrable on (a, b); i.e.,
the limit of (2) exists and is independent of the mode of partition of the
interval of integration (a, b) into subintervals and is independent of the
choice of points ~~ in these subintervals. Geometrically, the definite integral
(2) is the algebraic sum of the areas of the figures that make up the curvilin-
ear trapezoid aABb, in which the areas of the parts located above the x-axis
are plus, those below the x-axis, minus (Fig. 37).
The definitiens of integral sum and definite integral are naturally gen·
eralized to the case of an interval [a, b). where a> b.
Example t. Form the integral sum Sn for the function
f (x) = l +x
on the interval [1,10] by dividing the interval into n equal parts and chaos·
ing points ~i that coincide with the left end-points of the subintervals
(x{. Xj+.l· Wnat is the lim sn equal to?
n-+"'
Solution. Here, ~x,=I0-!=.2_
n n
and ~i=x1 =x0 +i~x1 =1+ 9ni. Whence
9i 9i .
(~j)=l+l +-=2+-. Hence (Ftg. 38),
n n
n-1 n-1
~ "\..... ( 9i ) 9 18 81
8-n = ~f(~i) ~xi=~ 2+n n-=-nn+ n• (O+ 1 + ... +n-1)=
l=o l=o
= 18 + 81 n (n-1) = 18 + 81 (t-.!..) =5B .!__ 81
n• 2 2 n 2 2n'
lim
n-+"'
Sn=58.!..2 ·
Example 2. Find the area bounded by an arc of the parabola y = xz, the
x-axis, and the ordinates x=O, and x=a (a> 0).
Solution. Partition the base a into n equal y
parts ~x=~. Choosing the value of the func-
n
tion at the beginning of each subinterval, we will

r
have

Yl = 0; Y2 = ( ~ Ya = [ 2 ( ~ YJ
:r.
t ••••

un= [<n-1>
The areas of the rectangles are obtained by mul-
tiplying each Yk by the base ~x=~ (Fig. 39).
n
Summing, we get the area of the step-like figure

r
Fig. 39

Sn= ~ (~ 1
[I +2 +31 + ..• +(n-1)
1
).

Using the formula for the sum of the squares of integers,


n
n (n+ 1) (2n+ 1)
6
140 Definite Integrals [Ch. 5

we find
a'n (n-1) (2n-1)
6n 1

and, passing lo the limit, we obtain


1
S= lim Sn= lim a (n-l)n(2n-l) _a•
n ..,. co n ..,. co 6n• - 3 •

Evaluate the following definite integrals, regarding them as the


limits of appropriate integral sums:
b
1501. ) dx.
a -2
T 10

1502. ) (v 0 +gt) dt, 1504. ) 2x dx.


0
s
V0 and g are constant. 1505*. ) x' dx.

1506*. Find the area of a curvilinear trapezoid bounded by


the hyperbola
1
Y=x•
by two ordinates: x =a and x = b (0 <a< b), and the x-axis.
1507*. Find
X

f (x) = ) sin t dt.


0

Sec. 2. Evaluating Definite Integrals by Means of Indefinite Integrals


1°. A definite integral with variable upper limit. If a function f (t) is
continuous on an interval [a, b), then the function
X

F(x) = ) f (t) dt
a
Is the anliderlvative of the function f (x); that is,
F' (x) = f (x) for a .;;;;;; x ...:;; b.
2°. The Newton-Leibniz formula. If F' (x) = f (x), then
/)

~ f (x) dx=F (b)-F (a).


a
Sec. 2] Evaluating Definite Integrals by Indefinite Integrals 141

The antiderivative F (x) is computed by finding the indefinite integral


~ f (x) dx = F (x) + C.
Example t. Find the integral
a
~ x4 dx.
-I

Solution. Sx'dx=x'
-I
5
I' =
-J
31
5
_(-l)' =48~.
5 5

1508. Let
b

I=S 1 ~: (b>a>l).
Find a

I) ~~; 2) ~~.
Find the derivatives of the following functions:
X ...
1509. F (x) = ~ ln t dt (x>O). 151 t. F (x) = ~ e- 12 dt.
X
0 vx-
1510. F (x) = ~ V1+ r dt. 1512. I = ~ cos W) dt (x > 0).
X 1
X
1513. Find the points of the extremum of the function
X

y= -sin1- t dt 111
· th e reg1on
· x> 0.
S
0

Applying the Newton-Leibniz formula, find the integrais:

1514. sl+x'
1

dx 1516. ~ e1 dt.
X

0 -x

1515. s
-1 X

dx
x• • 1517. ~:cost dt.
-2 0

Using definite integrals, find the limits of the sums:


1518**. lim
n .... ..,
(J.n + n + ... + n-;n
2
2
1
).

1 1
lim ( n +I I+ n + 2+ ... +-+
1519**. ,,.,..., n n) •
1520. Jim lP+ 2P+;;. +nP {p >0),
II-+ Gil nJ'
142 Definite Integrals [Ch. 6

Evaluate the integrals:


2 a
dx
1521. S (x•-2x+3 ) dx. 1534. SY5+4x-x• '
1 I
8 1
y•dy
1522. S <V2x+ Vx)dx.
0
1535. Sy'"y• +4.
0
:rt
&

1536. S cos• ada.


0
• n
1524. SVx-2dx. I

I
-s
1537. S sin' cp dcp.
1525. sy' 25+
dx
3x ·
e•
dx
-a
0 1538. Sxlnx·
1526. s x•-1
dx
e
e
·
-2 1539. s sin ~n x) dx.
1527. sx•+3x+2
I
1
xdx n
• &
0
1 1540. S tan x dx.
1528. ~ y' dy
y+2"
n
&
-I

s
I

1529. x•+d:x +5. 1541. Scot• cp dcp.


0 n

1530. 5x~-~:+2
I

1542. S1
I

:eu dx.
0

1531. S28 ~ 1 dz. 1543.


1

S cosh x dx.
0
0
n

·reo:;.
&

1532. S sec• ada. 1544. X.


n In 2


VI J1
P.
dx 1545. S sinh 1 x dx.
1533.
S Yl-x•'
0
0
Sec. 3] Improper Integrals 14~

Sec. 3. Improper Integrals


1°. Integrals of unbounded functions. If a function f (x) is not bounded
in any neighbourhood of a point c of an interval (a, b) and is continuous
for a~ x < c and c < x <; b, then by definition we put
b c-e b
r f (x) dx =
J e~oJ
r f (x) dx + e-+oJ
lim lim r f (x) dx. (1)
a a c+e

If the limits· on the right side of (I) exist and are finite, the improper inte-
gral is called conuergent, otherwise it is diuergent. When c=a or c=b, th~
definition is correspondingly simplified.
If there is a continuous function F (x) on [a, b) such that F' (x) = f (x)
when x rf: c (generalized antideriuatiue), then
b
~ f(x)dx=F(b)-F(a). (2)
a
b
If If (x) I..,;;;;; F (x) when a..,;;;;; x oe;;;b and ~ F (x) dx converges, then the in-
a
tegral (I) also converges (comparison test).
f(x)lc-xjm~~Ai:oo,A::pO,i.e.,f(x)-
iff(x);::,O and lim
x-+c
when x-.. c, then I) for m < 1 the integral (I) converges, 2) for
1 e-x
A Ira
m;::,: I the
integral (I) diverges.
2°. Integrals with Infinite limits. If the function f (x) is continuous when
a~ x < oo, then we assume

"' b
r f (x) dx =
J
Jim
b ..... "'
r f (x) dx
J
(3)
a a

and depending on whether there is a finite limit or not on the right of (3),
the respective integral is called conuergent or diuergent.
Similarly,
b b CIO b
~ f (x) dx =a,!!~"' ~ f (x) dx and ~ f (x) dx= a!!~'"'~ f (x) dx.
-ao a -ao b-++ooa

CJ>

If 1/ (x) 1< F (x) and the integral ~ F (x) dx converges, then the lnfe-
a
gral (3) converges as well.
A
If f (x)-;;;.: 0 and lim f (x) xm =A =F
=F 0, I. e., f (x) -Iii whenoo, A
X~CJ> X
x- oo, then 1) for m > l the integral (3) converges, 2) for m ~ l the inte-
gral (3) diverges.
144 Definite Integrals [Ch. 5

Example 1.

5
1
tix
X
z= lim
e-+o
-se -z+
dx
X
lim
1!-+0
5 1
dx
-z= bm
X
1
. ( --1
1!-+0 B
1 - 1) =co
) + 1'tm ( -
e-+0 B
-1 -1 I!

and the integral diverges.


Example 2.
GO b

S dx =
1+x1
lim
b-+GO
S1+dx X
2 = b-+<1>
lim (arc tan b-are tan 0) =% .
0 0

Example 3, Test the convergence of the probabtlity integral

(4)

Solution. We put
GO 1 GO

~ e-x• dx= ~e-xt dt:+ ~e-xt dx.


0 0 1

The first of the two integrals on the right is not an improper integral, while
lhe second one converges, since e-xt,..;;; e-x when x;;:,: I and
GO b
re-X dX= lim re-X dX= lim (-e-b+e- 1 )=e- 1;
J
1
b-+<1> J
1
b-+ao

hence, the integral (4) converges.


Example 4. Test the following integral for convergence:

(5)

Solution. When· x--+ + co, we have


I

Since the integral

converges, our integral (5) likewise converges.


Example 5, Test for convergence the el\iptic integral
1
dx
SYt-x•'
0
(6)
Sec. 3) Improper Integrals I45

Solution. The point of discontinuity of the integrand is x= 1. Applying


the Lagrange formula we get
I I
Yl-x' =~f(t-x)·4x 1 = !... · - , '
V 1
(1-x)' 2x:
where x < x1 < I. Hence, for x-+- I we have

Since the integral

converges, the given integral (6) converges as well.

Evaluate the improper integrals (or establish their divergence):


5 dx
1 IX>
dx
1546.
0
vx-· 1554.
-oo
5 l+x 2

2 IX>
dx
1547.
-I
5d:. 1555.
-co
5x'+4x+9 •
I
"'
1548. 5~
xP • 1556. ~ sin x dx.
0
1
a J

1549. dx
5 (x -1) 1 '
1557. 5xf:x •
0

I
I
dx
1550.
5 Yt-x•'
0
1558. 5 ~~~X
0
X •

CD a>

1551. 5 dxX ' 1559. 5xf:x (a> 1).


1 a
CD CD

1552. 5 dx
x• • 1580. 5 ~~~X X
(a> 1).
1 a
n

sxP'
CD I

1553. dx 1561. ~ cotxdx.


1 0
146 Definite Integrals [CII. I

... ...
1562. ~ e-kx dx
0
(k > 0). 1565. 5x'~ 1 •
0

"' I

1563. 5 3
~1 ~nl x dx. 1566.
5
dx
~5
X- X

0 Q
...
1564. 5(x•~I)•.
a

Test the convergence of the following integrals!


100 1

$v v . 5v=·
dx dx
1567. 1571.
X +2 X +x'
0
l-x4
I
dx
1572.
5.., TriX'
1573. 5 s~. x dx.
:rt
I

"'
1570. 5Yx'+
xdx
I ·
0

1574*. Prove that the Euler integral of the fi1st kind (beta-
function)
1

B (p, q) =) xP- 1
(1-x)q- 1 dx
0

converges when p > 0 and q > 0.


1575*. Prove that the Euler integral of the second kind (gam-
ma-function)
..,
r (p) = ~ Xp-!8-x dX
0

converges for p > 0.

Sec. 4. Change of Variable in a Definite Integral


If a function f (x) is continuous over a...;xc;;;b and x-cp (t) Ia a function
continuous together with its derivative <p' (t) over a...;t.-;;;6 1 where a=-cp(a)
and b=<p (p), and f [q> (t)J is defined and continuous on the Interval a...;tc;;~.
Sec. 4) Change of Variable in a Definite Integral 147

then
b II
~ f (x) dx = ~ f [q> (t)) q>' (t) dt.
a a
Example t. Find
a
~ x• V a1 -x2 dx (a> 0).
0
Solution. We put
x=asint;
dx = a cos t dt.
Then t=arcsin~ and, consequently, we can take a=arcsin0=0,
a
~=arc sin I= ~ . Therefore, we shall have
lt
a 2

S x• Va 2
x"dx= ~ a 2 sin 2 t }"a•-a•sin•tacostdt=
0 0
lt lt lt
2 • •

=a•
Ssln
2 2
t cos t dt = a•s a4s
4 sln 2t dt = B (I-eos 4!) dt =
2

0 0 0

=a
a
4
( I sin4t
t -4 )Ina'
=w.
0

1576. Can the substitution x =cost be made in the integral

Transform the following definite integrals by means of the


indicated substitutions:
I lt

1577. ~Vx+ldx, x=2t-l. •


1580. S f (x) dx, x =arc tan t.
I

1578. S~d=x=-
Yt-x•'
x= sin t.
1581. For the integral
I
b
•4
a
Sf (x) dx (b >a)
a
dx
1579. X= sinh t.
SYx'+l'

'
148 Definite Integrals (Ch. 5

indicate an integral linear substitution


x=at + ~.
as a result of which the limits of integration would be 0 and 1,
respectively.
Applying the indicated substitutions, evaluate the following
integrals:
4

S1+ v ,
dx
1582.
X
0
19 .,
(x-2) '
1583. S •
(x-2) 1• 3
dx, + x-2=z'.
I
lna
1584. SVex-1 dx, ex-1 =Z1 •
0
:It
dt t
1585. tan 2 = z.
S3+2 cost.
0
:It

tan x= t.
Evaluate the following integrals by means of appropriate
substitutions:
1
Ins
1587. S ~· dx. 1589.
S
ex Jle'i=T d
fx+3 X.
Yl X
0
I

15S8. srx;=t
1
dx. 1590.
I

S2x+ Y3x+
0
dx
1•

Evaluate the integrals:


a a
1591. Sx V x dx+5x+ 1 •
2
1593. )Vax-x• dx.
I 0

I 1:1'1

1592. S(1 !xx•)•.


-I
1594.
dx
S'5 - 3 cosx •
0

1595. Prove that if f (x) is an even function, then


a a
) {(x) dx = 2 ~ f(x) dx.
-a o
Sec. 5] Integration by Parts 149

But if f (x) is an odd function, then


a
~ f(x)dx=O.
-a
1596. Show that
«J «J «J

-
5e-x"dx=2 5e-x"dx= 5e;;dx.
ct> 0 0

1597. Show that


1

5 -dx
- - -sin
X
arc cos x-
xd5l

x ·
0 0

1598. Show that


lt .:!
l I

~ f(slnx)dx= ~ f(cosx)dx.
0 0

Sec. 5. Integration by Parts


If the functions u (x) and v (x) are continuously differentiable on the
interval [a, b), then
b b b

~ u (x) v' (x) dx=u (x) v (x)


a
J- ~
a a
u (x) u' (x) dx. (l)

Applying the formula for integration by parts, evaluate the


following integrals:
2 «J

1599. ~ xcosxdx. 1603. ~ xe-xdx.


e
"'
1600. ~In xdx. 1604. ~ e-ax cos bxdx (a> 0).
1
1
"'
1601. ~ x'eudx. 1605. ~ e-axsm bxdx (a> 0).
0
lt

1602. ~ e" sin xdx.


0
150 Definite Integrals [Ch. 5

1606**. Show that for the gamma-function (see Example 1575)


the following reduction formula holds true:
r(p+ 1)=pr(p) (p>O).
From this derive that r (n + 1) = nl, if n is a natural number.
1607. Show that for the integral

• •
In=~ sinnxdx = ~ cosn xdx
0 0

the reduction formula

holds true.
Find I n• if n is a natural number. Using the formula obtained,
.eva! uate I, and 110 •
1608. Applying repeated integration by parts, evaluate the
integral (see Example 1574)
1

B (p, q) = SxP- 1
(1- x)q- 1 dx,
0

where p and q are positive integers.


1609*. Express the following integral in terms of B (beta-
function):


In.m = ~ sinm X cosn X dx,

if m and n are nonnegative integers.

Sec. 6. Mean-Value Theorem


1°. Evaluation of integrals. If f(x)-s;;.F(x) for a-s;;.x-s;;.b, then
b b
Sf (x) dx,.;;;. ~ F (x) dx. (I)
a a
If f (x) and q> (x) are continuous for a,.;;;. x,.;;;. b and, besides, q> (x) :;;;z, 0, then
b b b
m ~ q> (x) dx ,.;;;. ~ f (x) q> (x) dx o;;;;;; M ~ q> (x) dx, (2)
a a a

where m is the smallest and M is the largest value of the function f (x) on
the interval [a, b}.
Sec. 6] Mean-Value Theorem 151

In particular, if q> (x)"""" I, then


b
m (b-a) o;;;; ~ f (x) dx o;;;; M (b-a). (3}
a

fhe inequalities (2) and (3) may be replaced, respectively, by their equiva·
,ent equalities:
b b
~ f (x) q> (x) dx = f (c) ~ q> (x) dx
a a
md
b
~ f(x)dx=f(~)(b-a),
a

where c and ~ are certain numbers lying between a and b.


Example 1. Evaluate the integral

Solution. Since 0 <; sin 1 x.;;;; I, we have

n
2< /<2
n l/32'
that is,
1.57 <I< 1.91.

2°. The mean value of a function. The number

s
b

11 = b 1 a f (x) dx
a

Is called the mean value of the function f(x) on the interval a<xo;;;;b.

1610*. Determine the signs of the integrals without evaluating


them:
I U'l

a) ~ x• dx; c) Ssi~ x dx.


-I 0
11

b) ~ X COS X dx;
0
152 Definite lntegra/s [Ch. b

161 t. Determine (without evaluating) which of the following


integrals is greater:
I I

a) SVt +x• dx
0
or s
0
dx;
I I

b) Sx• sin • x dx or Sx sin 1 xdx;


0 0
z a
c) S ex"dx or Sexdx.

Find the mean values of the functions on the indicated inter-


vals:
1612. f(x)=x\ O~x~ 1.
1613. f(x)=a+b cosx, -n~x~n.
1614. f(x)= sin 1 x, O~x~n.
1615. f (x) =sin' x, O~x~n.
I

1616. Prove that Sy dx


2+x-~
lies between ; ~ 0.67 and .)._ ~
r2
0

~ 0. 70. Find the exact value of this integral.


Evaluate ihe integrals:
I .
1617. SV4-j-x dx. 2
1620*. SxVtanx.
~
+I 2

1618. Ss!x•.
-I
1621. s~dx
X
n

zn
'
1619. sIO+~cosx
0

1622. Integrating by parts, prove that


zoon
0< s~d
x
lOOn
X <-1
-
lOOn'
Sec. 7] The Areas of Plane Figures 153

Sec. 7. The Areas of Plane Figures


1°. Area in rectangular coordinates. If a continuous curve is defined in
rectangular coordinates by the equation y=f (x) If (x) :;;;;.0], the area of the
curvilinear trapezoid bounded by this curve, by two vertical lines at the

0 a X J X

Fig. 40 Fig. 41

points x=a and x=b and by a segment of the x-axis a..;;;xo;;;;;b (Fig. 40),
is given by the formula b

S= 5f
<I
(x) dx. (I)

x2
Example 1. Compute the area bounded by the parabola y = - , the
straight lines X=l and x=3, and the X·axis (Fig. 41). 2

y
r

X)

0 b X
-2 g=f,(:t)
Fig. 42 Fig. 43

Solution. The sought-for area is expressed by the integral


•x 2
1
S= -dx-4-
S
1
2 - 3.
154 Definite Integrals [Ch. 5

Example 2. Evaluate the area bounded by the curve x=2-y-y1 and


1he y-axis (Fig. 42).
Solution. Here, the roles of the coordinate axes are changed and so the
sought-for area is expressed by the integral
I

S= S<2-y-y 1 )dy=4..},
-z

where the limits of Integration y 1 = -2 and Yz =I are found as the ordinates


of the points of intersection of the curve with the y-axis.

--

Fig. 44 Fig. 45

In the more general case, if the area S is bounded by two continuo us


curves y={1 (x) andy={ 2 (x) and by two verticallinesx=a and x=b, where
fdx).,;;;. f 2 (x) when a...:;; x...:;; b (fig. 43), we will then have:
b
S= S[{ 2 (x)-{ 1(x)) dx. (2)
a

Example 3. Evaluate the area S contained between the curves


y=2-x2 and y 1 =x 1 (3)
(Fig. 44).
Solution. Solving the set of equations (3) simultaneously, we find the
limits of integration: X1 =-l and x1 = I. By virtue of formula (2), we obtain
I
X1
I

S= I 2
3
(2-x -x / •)dx= ( 2x----x a)l 2
=2-.
S
-I
3 5 -1 15

If the curve Is defined by equations In parametric form X=<p(t), Y='\[l(t)


lhen the area of the curvilinear trapezoid bounded by this curve, by two
~S~ec~·~7~]______________T~h_e__
A_r_ea_s_o~f__
P_la_n_e_F_i~g_u_re_s_________________155

vertical lines (x=a and x=b), and by a segment of the x-axis is expressed
by the integral
t.
S= ~ ¢(1) cp' (I) dt,
t,

where 11 and 11 are determined from the equations


a=cp (1 1) and b=cp(t 2) [¢(1) ~0 on the interval [1 1 , 12 ]).
Example 4. Find the area of the ellipse (Fig. 45) by using its parametric
equations
x =a cost,
{ y = b sin I.

Solution. Due to the symmetry, it is sufficient to compute the area of a


quadrant and then multiply the result by four. If in the equation x=a cost
we first put x=O and then x=a, we get the limits of integration 11 = ~and
12 = 0. Therefore,
0 2

! S= 5
~ bsina(-sinl)dt=ab sin 2 1dt=~b
n o
2

and, hence, S=Mb.


2°, The area in polar coordinates. If a curve is defined in polar coordi-
nates b~' the equation r ~= f (cp), then the area of the sector AOB (Fig. 46),
bounded by an arc of the curve, and by two rad1us vectors OA and OB,

y /

Fig. 46 Fig. 47

which correspond to the values cp 1 =a and cp 1 = ~. is expressed by the


hitegral
f3
s ={ 5If (cp)]l dcp.
a

Example 5. Find the area contained inside Bernoulli's lemniscate


r 2 =a 2 cos2cp (Fig. 47).
156 Definite Integrals [Ch. 5

Solution. By virtue of the symmetry of the curve we determine first one


quadrant of the sought-for area:

1623. Compute the area bounded by the parabola y=4x-x 2


and the x-axis.
1624. Compute the area bounded by the curve y =In x, the
x-axis and the straight line x =e.
1625*. Find the area bounded by the curve y=x (x-1) (x-2)
and the x-axis.
1626. Find the area bounded by the curve y' = x, the straight
line y=l and the vertical line x=8.
1627. Compute the area bounded by a single half-wave of the
sinusoidal curve y = 5in x and the x-axis.
1628. Compute the area contained between the curve y =tan x,
the x-axis and the straight line x=-i .
1629. Find the area contained between the hyperbola xy = m\
the vertical lines x=a and x=3a (a>O) and the x-axis.
1630. Find the area contained between the witch of Agnesi
a•
y = xs +as and the x-axis.
1631. Compute the area of the figure bounded by the curve
y=x 8 , the straight line y=8 and they-axis.
1632. Find the area bounded by the parabolas Y1 =2px and
2
X = 2py. •
1633. Evaluate the area bounded by the parabola y=2x-x•
and the straight line y = - x.
1634. Compute the area of a segment cut off by the straight
line y=3-2x from the parabola y=x 2 •
1635. Compute the area contained between the parabolas Y"""'x•,
y = ~ and the straight line y = 2x.
1636. Compute the area contained between the parabolas
x2 2 a
y= 3 and y=4- 3 x.
1637. Compute the area contained between the witch of
1 x2
Agnesi y= l+x 2 and the parabola Y= 2 .
1638. Compute the area bounded by the curves y- e", v- s-x
and the straight line X= 1.
Sec. 71 The Areas of Plane Figures 157

1639. Find the area of the figure bounded by the hyperbola


xt yt
iii- b• = 1 and the straight line x = 2a.
1640*. Find the entire area bounded by the astroid

x• +Y' =a•.
1641. Find the area between the catenary
X
y=a cosh-,
a

the y-axis and the straight line y = ;e +


(e1 1).
1642. Find the area bounded by the curve a•y• = x• (a 1 - X1 ).
1643. Compute the area contained within the curve

1644. Find the area between the equilateral hyperbola x1 -y2 =


= 9, the x-axis and the diameter passing through the point (5,4).
1645. Find the area between the curve y=~. X
the x-axis,
and the ordinate x=I (x>l).
1646*. Find the area bounded by the cissoid y• = 2ax'
-x
and its asymptote x = 2a (a> 0).
x (x a) 2
1647*. Find the area between the strophoid y•= - and
2a-x
its asymptote (a> 0).
1648. Compute the area of the two parts into which the
circle X2 + !/ = 8 is divided by the parabola y2 = 2x.
1649. Compute the area contained between the circle x• + y• = 16
and the parabola X 1 =12(y-l).
1650. Find the area contained within the astroid
X= a cos' t; y= b sin' t.
1651. Find the area bounded by the x-axis and one arc of
the cycloid
x =a (t- sin t),
{ y = a ( 1 -cos t).
1652. Find the area bounded by one branch of the trochoid
x=at-b sint, (O<bo;;;::. )
{ y = a- b cos t """"a

and a tangent to it at its lower points.


158 Definite Integrals [Ch. 5

1653. Find the area bounded by the cardioid


x =a (2 cost-cos 2!),
{ y =a (2 sin t- sin 2t).

1654*. Find the area of the loop of the folium of Descartes


3at 3at 2
X=l+t'; Y=l+t''

1655*. Find the entire area of the cardioid r=a (1 +coscp).


1656*. Find the area contained between the first and second
turns of Archimedes' spiral, r = acp
(Fig. 48).
1657. Find the area of one of the
leaves of the curve r =a cos 2cp.
1658. Find the entire area bound-
ed by the curve r 2 = a 2 sin 4(p,
1659*. Find the area bounded by
the curve r =a sin 3q>.
1660. Find the area bounded by
Pascal's Jimac;on
Fig. 48 r = 2 +cos q>.

1661. Find the area bounded by the parabola r =a sec 1 ~


and the two half-lines q> = ~ and cp = ;.
1662. Find the area of the ellipse r= I+:coscp (e< 1).
1663. Find the area bounded by the curve r = 2a cos 3cp and
lying outside the circle r=a.
1664*. Find the area bounded by the curve x 4 + y 4 = x1 + y•.

Sec. 8. The Arc Length of a Curve


1°. The arc length in rectangular coordinates. The arc length s of a curve
Y=f(x) contained between two points with abscissas X=a and X=b is
b

S= ~ VI +y'2dx.
a

+
Example 1. Find the length of the astroid x•l• y•l• =a 211 (Fig. 49).
Solution. Differentiating the equation of the astroid, we get
' y'l.
y =--,-,.
X a
Sec. 8) The Arc Length of a Curve 159

For this reason, we have for th:! arc length of a quarter of tlie astroid:

Whence s=6a.
2°. The arc length of a curve represented parametrically. If a curve is
represented bv equations in parametric form, x = cp (t) and y = 1jl (t), then the
arc length s of the curve is
t,
s= ~ 2
Yx'
+y' 2 dt,
t,
where t 1 and t 2 are values of the parameter that correspond to the extremities
of the arc.
y

y
s=F?a
X

Fig 49 Fig. 50

Example 2. Find the length of one arc of the cycloid (Fig. 50)
x=a(t-slnt),
{ y=a (I-eos t).
dx dy
Solution. We have dt =a (I- cost) and dt =a sin t. Therefore,
2n zn
s= S Ya (J-cost) +a sln tdt=2a Ssin: dt=Ba.
2 2 2 1

0 0

The limits of integration 11 =0 and 12 =2n: correspond to the extreme points


of the arc of the cycloid.
If a curve is defined by the equation r=f (cp) In polar coordinates, then
the arc length s is
13
s=~ Yr2+r' 1 dcp,
a

where a and ~ are the values of the polar angle at the extreme points of
the arc.
160 Definite Integrals (Ch. 5

Example 3. Find the length of the entire curve r=asin 1 ~ (Fig. 51).
The entire curve is described by a point as <p ranges from 0 to 3:rt.

Fig. 51

Solution. We have r' =a sin 2 %cos %, therefore the entire arc length of
!he curve is
m rn

s= S .v. / a 2
sin' .5!:. + a
3
2
sin' .5!:. cos 2 .5!:. d<p =a
3 3
Ssin .5!:.3 drp = 3rca2 '
2

0 0

1665. Compute the arc length of the semicubical parabola


y• = x' from the coordinate origin to the point x = 4.
1666*. .Find the length of the catenary y =a cosh~ a
from the
vertex A (O,a) to the point B (b,h).
1667. Compute the arc length of the parabola y = 2 Vx from
X =0 to X= 1.
1668. Find the arc length of the curve y = e" lying between
the points (0,1) and (l,e).
1669. Find the arc length of the curve y =In x from x = V3
to X= V8.
1670. Find the arc length of the curve y =arc sin (e-") from
X=O to X= 1.
1671. Compute the arc length of the curve x =In sec y, lying
between y=O and y= ~ .
1672. Find the arc length of the curve X=~ y 2 - ~ lny from
u= 1 to u=e.
Sec, 9) Volumes of Solids 161

1673. Find the length of the right branch of the tractrix

x=Va 2 -y 2 +alnla+Y~~ from y=a to y=b(O<b<a).

1674. Find the length of the closed part of the· curve 9ay•=
=x(x-3a?.
1675. Find the length of the curve y= In ( coth ~) from x=a
to x = b (0 <a< b).
1676*. Find the arc length of the involute of the circle

x =a (c?s 1 + 1 sin t), } from t = 0 to t = T


y =a (sm t-t co-:,t) ·

1677. Find the length of the evolute of the ellipse

1678. Find the length of the curve


x =-a (2 cost- cos 2t}, }
y =a (2 sin t- sin 21).

1679. Find the length of the first turn of Archimedes' spiral


r = aq,.
1680. Find the entire length of the cardioid r =a (1 +cos <p).
1681. Find the arc length of that part of the parabola
r=asec•; which is cut off by a vertical line passing through
the pole.
1682. Find the length of the hyperbolic spiral rc:p = 1 from the
point (2, 1 / 2 ) to the point ('/ 1 ,2).
1683. Find the arc length of the logarithmic spiral r = aem'P,
lying inside the circle r =a.
1684. Find the arc length of the curve <p = ; (r +
~ from )
r= 1 to r= 3.

Sec. 9. Volumes of Solids


1°. The volume of a solid of revolution. The volumes of soll~s formed by
the revolution of a curvilinear trapezoid (bounded by the curve yt;;f (x), 'the
x-axis and two vertical lines x=a and x=bl about the x- and 'y-axes are

6 - 1900
162 Definite Integrals (Ch. 5

expressed, respectively, by the formulas:


b b
I) Vx=:t ~ y1 dx; 2) Vy=2n ~ xydx*).
a a

Example t. Compute the volumes of solids formed by the revolution of a


figure bounded by a single lobe of the sinusoidal curve y =sin x and by the
segment 0 ~ x ~ n of the x-axis about: a) the x-axis and b) the y-axls.
Solution.

n
b) Vy=2n ~ x sin x dx= 2n ( -x cos x+ sin x)~ = 2n1 .
0

The volume of a solid formed by revolution about the y-axis of a figure


bounded by the curve x =g (y), the y-axis and by two parallel lines y = c and
y=d, may be determined from the formula
d
Vy=n ~ x 2 dy,
c
obtained from formula (1), given above, by interchanging the coordinates
x and y.
If the curve Is defined in a different form (parametrically, in polar coor-
dinates, etc.), then in the foregoing formulas we must change the variable of
Integration in appropriate fashion.
In the more general case, the volumes of solids formed by the revolution
about the x- andy-axes of a figure bounded by the curves y, = f1 (x) and y2 = f 2 (x)
(where f 1 (x)~f.(x)), and the straight lines x=a and x=b are, respectively,
equal to
b

Vx=n ~ (y:-y~) dx
a
and
b
Vy=2n ~ x (y 1 -y1) dx.
a

Example 2. Find the volume of a torus formed by the rotation of the


circle x 2 +(Y-W=a1 (b:;;..a) about the x-axis (Fig. 52).
"') The solid is formed by the revolution, about the y-axis, of a curvilinear
trapezoid bounded by the curve y=f(x) and the straight lines x=a, x=b,
and y=O. For a volume element we take the volume of that part of the solid
formed by revolving about the y-axis a rectangle with sides y and dx at a
distance x from the y-axis. Then the volume element dVy = 2n xy dx, whence
b
Vy=2n ~ xydx.
G
Sec. 9] Volumes of Solids 163'

Solution. We have
y1 =b- Va•-x• and y 2 =b+ Ya 2 -x2 •
Therefore,
" .
Vx=n ~ [(b+ Ya -x
2 1 2
) -(b- Ya -x
2 2 2
) ] dx=
-a

= 4nb
"
~ Jfa 1 -x2 dx=2n 2 a1b
-u

(the latter integral is taken by the substitutiOn x =a sin t).

-a 0 x a X

Fig 52 Fig. 53

The volume of a solid obtained by the rotation, about the polar axis, of a
sector formed by an arc of the curve r = F (<p) and by two radius vectors
'P ~=a., 'P = f\ may be computed from the formula
13
Vp =; n Sr• sin <pd <p.
a
Thts same formula is conveniently used when scel<ing the volume obtained
by the rotation, about the polar axis, of some closed curve defined in nolar
coordinates.
Example 3. Determine the volume formed by the rotation of the curve
r =a sin 2<p about the polar axis.
Solution.
a a
V p=2·; n Sr• sin <p dcp =! na• Ssiu 2<p sin cp dcp=
0 0
1

ll
a
= ~3 na• Ssin4 cp cos• 1p dcp = 64 na•.
105
0
164 Definite lnte~rrals (Ch. 5

2°, Computing the volumes of solids from known cross-sections. If S = S (x)


Is the cross-~rctional arE'a cut oil by a plane pPrpendicular to some straight
line (which we lake to be the x-axis) at a point with abscissa x, then the
volume of the solid is
.t'.
V =: ~ S (x) dx,
x,

where K 1 and x1 are the abscissas of th~ extreme cross-sections of th~ solid.
Example 4. D~termine the volume of a wedge cut oil a circular cylinder
by a plane passmg through the diamrter of the basl' and inclinrd to the base
at an angle a. The radius of the base is R (Fig. 53).
Solution. For the x-axis W? take th~ diam~ter of the base along which
the cutting plane intersects the base, and for the y-axis Wi! take the diameter
of the base perpendicular to it. The equation of the circumference of the base
+
Is x• y2 = R2 •
Thi! area of the s~>ction ABC at a distance x from the origin 0 is
1 1 y2
S (x) =area {). ABC= AB·BC= yy tan a =2' tan a. Therefore, the sought-
2 2
for volume of the wedge is
R R
V=2· ~ Sy•tanadx=tana S (R -x )dx=; tanaR
1 1 1

0 0

1685. Find the volume of a solid formed by rotation, about


the x-ax is, of an area bounded by the x-axis and the parabola
y= ax-x' (a>O).
1686. Find the volume of an ellipsoid formed by the rotation
x• y•
of the ellipse iii'+ 'b• = 1 about the x-ax is.
1687. Find the volume of a solid formed by the rotation, about
the x-axis', of an area bounded by the catenary y =a cosh~, a
the
x-axis, and the straight lines x= ±a.
1688. Find the volume of a solid formed by the rotation, about
the x-axis, of the curve y = sin 2 x in the interval between x = 0
and x=n.
1689. Find the volume of a solid formed by the rotation, about
the x-axis, of an area bounded by the semicubical parabola y• =X 1 ,
the x-axis, and the straight line x = J.
1690. Find the volume of a solid formed by the rotation of
the same area (as in Problem 1689) about the y-axis.
1691. Find. the volumes of the solids formed by the rotation
of an area bounded by the lines y=e", x=O, y=O about: a) the
x-axis and b) the y-axis.
1692. Find the volume of a solid formed by the rotation, about
the y-axis, of that part of the parabola y 1 = 4ax which is cut ofT
by the straight line x =a.
Sec. 9] Volumes of Solids 165

1693. Find the volume of a solid formed by the rotation, about


the straight line x=a, of that part of the parabola !:J:a =4ax which
is cut ofl by this line.
1694. Find the volume of a solid formed by the rotation, about
the straight line y = - p, of a figure bounded by the parabola
y'=2px and the straight line X=~.
1695. Find the volume of a solid formed by the rotation, about
the x-axis, of the area contained between the parabolas y = X2
andy= V"t.
1696. Find the volume of a solid formed by the rotation,
about the x-ax!s, of a loop of the curve (x- 4a) !I =ax (x- 3.l)
(a>O).
1697. Find the volume of a solid generated by the rotation
1
of the cyssoid y1 =A - about its asymptote x=2a.
2a-x
1698. Find the volume of a paraboloid of revolution whose
base has radius R and whose altitude is H.
1699. A right parabolic ~egment whose base is 2a and altitude h
Is in rotation about the base. De:ermine the volume of the result-
ing solid of revolution (Cwalieri's "lemon").
1700. Show that the volume of a part cut by the plane x=2a
off a solid formed by the rotation of the equilateral hyperbola
x 2 - y2 = a 1 about the x-axis is equal to the volume of a sphere
of radius a.
1701. Find the volume of a solid formed by the rotation of a
figure bounded by one arc of the cycloid x=-a (t- sin t),
y=a(l--cost) and the x-axis, about: a) the x-axis, b) they-axis,
and c) the axis of symmetry of the figure.
1702. Find the volume of a solid formed by the rotation of
the astroid x =a cos 1 t, y = b sin 1 t a bout the y-axis.
1703. Find the volume of a solid obtained by rotating the
cardioid r =a (1 +cos <p) about the polar axis.
1704. Find the volume of a solid formed by rotation of the
curve r =a cos 2 q> about the polar axis.
1705. Find the volume of an obelic;k whose p~rallel bases are
rectangles with sides A, 8 and a, b, and the altitude is h.
1706. Find the volume of a right elliptic cone whose base is
an ellipse with semi-axes a and b, and altitude h.
1707. On the chords of the astroid x''•+y''•=a'l., which ar~
parallel to the x-axis, are constructed squares whose sides are
equal to the lengths of the chords and whose planes are perpen-
dicular to the xy-plane. Find tlie volume of the solid formed by
these squares.
~16_6__________________D~e~fi~n~~·te~ln~t~~~ra~l~s______________ (Ch. 5

1708. A circle undergoing deformation is moving so that one


of the points of its circumference lies on the y-axis, the centre
describes an ellipse ;: +~ = 1, and the plane of the circle is
perpendicular to the xy-plane. Find the volume of the solid
generated by the circle.
1709. The plane of a moving triangle remains perpendicular
to the stationary diameter of a circle of radius a. The base of
the triangle is a chord of the circle, while its vertex slides along
a straight line parallel to the stationary diameter at a distance h
from the plane of the circle. Find the volume of the solid (called
a conoid) formed by the motion of this triangle from one end of
the diameter to the other.
1710. Find the volume of the solid bounded by the cylinders
x2 -t-z'=a2 and Y2 -t-Z'=a'.
171 l. Find the volume of the segment cut off from the ellip-
y2 22
tic paraboloid 2p+2q=X by the plane x=a.
1712. Find the volume of the solid bounded by the hyperbo-
x2 y' z2
loid of one sheet a2 +b 2 - c2 =1 and the planes Z=O and z=h.
x2 y2 22
1713. Find the volume of the ellipsoid 2a b2
c = l. + +•
Sec. 10. The Area of a Surface of Revolution
The area of a surface formed by the rotation, about the x-axis, of an
arc of the curve y=f(x) between thE' points x=a and x=b, is expressE'd by
the formula b b

Sx=2n SYd;xdx=2n Sy Vl+y' 2


dx (I)
a

(ds is the differE'ntial ol the arc of the curve).

y
.Y
A
2a

X
-f---l.--- --'------------1---
7Ca ZlCa ,(
Fig. 54 Ftg. 55

If the equation of the curve is represented differently, the area of the


surface Sx is cbtained from formula (1) by an appropriate change of variables.
Sec. 10] The Area of a Surface of Revolution 167
Example 1. Find the area of a surface formed by rotation, about the
x-axis, of a loop of the curve 9y 2 =x (3-x) 1 (Fig. 54).
Solution. For the upper part of the curve, when 0EO;;xE0;;3, we have
y=! (3- x) Whence the differential of the arc ds= ;~ dx. From for-
Vx.
mula (1) the area of the surface

Example 2. Find the area of a surface formed by the rotation of one arc
of the cycloid x=a (t-sm t); y=a (I-eos t) about its axis of symmetry
(Fig. 55).
Solution. The desired surface is formed by rotation of the arc OA about
the straight line AB, the equation of which is x=na. Taking y as the inde-
pendent variable and noting that the axis of rotation
AB is displaced relative to the y-ax1s a distance na, we
will have y
A
za
• ds
S=2n
5
0
(na-x) dy · dy.

Passing to the variable t, we obtain


n
Ba
S=2n j' (na-at+as!nt) y(~~Y+(~~rdt= X
0
n
=2n 5 (na-at+aslnt)2asin ~ dt=
8
F1g. 56

1714. The dimensions of a parabolic mirror AOB are indicated


in Fig. 56. It is required to find the area of its surface.
1715. Find the area of the surface of a spindle obtained by
rotation of a lobe of the sinusoidal curve y= sinx about the
x-axis.
1716. Find the area of the surface formed by the rotation of
a part of the tangential curve y =tan x from x = 0 to x = ~ ,
about the x-axis.
1717. Find the area of the surface formed by rotation, about
the x-axis, of an arc of the curve y ==e-x, from x = 0 to x = +oo.
168 Definite Integrals (Ch. 6

1718. Find the area of the surface (called a catenoid) formed


by the rotation of a catenary y=acosh~ about the x-axis from
a
x=O to x=a.
1719. Find the area of the surface of rotation of the astroid
x•J• -1 y•t• = a•t• about the y-axis.
1720. Find the area of the surface of rotation of the curve
x={ y 2 - ~ lny about the x-axis from y= 1 to y=e.
1721 *. Find the surrace of a torus formed by rotation of the
circle x 2 +(y- b) 2 =a 2 about the x-axis fb>a).
1722. Find the area of the surface formed by rotation of the
xz •
ellipse 01 + /ii11 = 1 about: 1) the x-axis, 2) the y-axis (a> b).
1723. Find the area of the surface formed by rotation of one
arc of the cycloid x =a (I- sin t) and !J =a (1- co~ t) about: a) the
x-axis, b) the y-axis, c) the tangent to the cycloid at its highest
point.
1724. Find the area of the surface formed by rotation, about
the x-axis, of the cardioid
x =a (2 cost-cos 2t), }
y =a (2 sin t- sin 21).

1725. Determine the area of the surface formed by the rotation


of the lemniscate r= =a= cos 2<p about the polar axis.
1726. Determine the area of the surface formed by the rotation
of the cardioid r = 2a (1 +co~ <p) about the polar axis.

Sec. 11. Moments. Centres of Grav:ty. Guldin's Theorems


1°. Static moment. The static moment relative to the /-axis of a material
point A having mass m and at a distance d from th:! /-axis is the quantity
Mz=md.
The static moment relative to the /-axis or a system of n material roints
with masses m1 , m2 , ••• , mn lying in the plane of the axis and at distances
d 1 , d,, ••. , dn is the sum
n
Mt= ~midi, (1)
i=l

where the distancE's of points lying on one side of the l-axis have the plus
sign, thosE' on thE' othl"r side have the minus sign. In a similar manner we
define the stattc moment of a system of potnts rt>lative to a rlane.
If the massPs conllnuously fill the line or figure of the xy-plane, then the
11tatic moments Mx and My about the x- and y-axes are exoressed \respective-
ly) a!l intl"grals and not as the sums ll). For the cases of geometric figures,
lbe density is considered equal to unity.
Sec. 11] Moments. Centrts of Gravity. Guldin's Theorems 169

In particular: I) for the curve x=x (s); y = y (s), wh~re the parameter s
Is the arc length, we have
L L
Mx= ~ y(s)ds; My=~ x(s)ds (2)

(ds= Y(dx) 1 +(dy)1 is the differential of the arc);

v
b

l X
a X t--- b ----'1'
Fig. 57 Fig. 58

2) for a plane figure bounded by th~ curve y=y (x), th.! x-axis and two
verhcal hnes x=a and y=b, we obtam
b b
Mx=; 5ylyldx; My= Sxlyldx. (3)
a a
Example 1. Find the stat1c mom~nts about the x- and y-axe> of a triangle
bounded by th~ straight lines: : +-)i-=1, x=O, y=O (F1g. 57)

Solution. Here, y=b ( 1- : ) . Applyir.g formula (3), we obtain

and

2°. Moment of Inertia. The moment of 111ertta, about an /-axis, of a m1fc·


rial point of ma!>s m at a d1stance d from th~ l-ax•~. •~ the number 1 1 ~md • 2

Tht> moment of werlia, about an /-axis, of a system of n matenal pomts


with masses m1, m:., ••• , m,1 1s the sum
11

It'-=-~ mid:.
i-"1
170 Definite Integrals (Ch. 5

where d1 , d1 ••• , dn are the distances of the points from the t-axis. In the
case of a continuous mass, we get an appropriate integral In place of a sum.
Example 2. Find the moment of inertia of a triangle with base b and
altitude h about Its base.
Solution. For the base of the triangle we take the x-axis, for its altitude,
the y-axis (Fig 58).
Divide the triangle into intlnitely narrow horizontal strips of width dy,
which play the role of elementary masses dm. Utilizing the similarity of
triangles, we obtain
h-y
dm=b -h-dy
and

Whence

3°. Centre of gravity. The coordinates of the centre of gravity of a plane


figure (arc or area) of mass M are computed from the formulas
- My - Mx
X=M. y=M.

where Mx and My are the static moments of the mass. In the case of geomet·
ric figures, the mass M is numerically equal to the corresponding arc or area.
For the coordinates of the centre of gravity (x, if) of an arc of the plane
curve y = f (x) (a~ x ~b), connecting the points A [a, f (a)] and B [ b, f (b)),
we have
B h B b
~xd~ ~xYI+(y')•dx ~ yds ~Y Yl +(y') 2
dx
- A a
X= -.-s-= :;.b _ _ _ _ __
- A a
y=---
s b
~ Yl + (y') 2
dx ~ Yl + (y') 2
dx
a a

The coordinates of the centre of gravity ('X; if) of the curvilinear trapezoid
a.;;;;;x.;;;;;b, O.,;;;;;yos;;;;,f(x) may be computed from the formulas
b b

Sxydx ~ Sy"dx
a y=
x=-s-· _a::..__
s
b
where S = ~ y dx is the area of the figure.
a
There are similar formulas for the coordinates of the centre of gravity of
a volume.
Example 3. Find the centre of gravity of an arc of the semicircle
x•+u"=a•; (y~O) (Fig. 59).
Sec 111 Moments. Centres of Gravity Guldin's Theorems 171

Solution. We have
y = v-a~---X-2; y' = --o::=X=-c-c-
y a 2 x•
and

Whence
a u
My=
J'
-a
xds=
'
v
-a
Vaax-x
2 2
d~:--cO,

u "
Mx= .~ Va•-r•~-
J yds·.. .\. . Va•-x• ··2a2 ,
-a -a
u

M- .r
-a
:rra.

Hence,
2
x=O; y=Ta.

4°. Guldin's theorems.


Theorem 1. The area of a surface obtamed by the rotation of an arc of
a plane curve about some axis lying in the same plane as the curve and not
intersecting it is equal to the product of the length of the curve by the
circumference of the circle described by the centre of gravity of the arc of
the curve.
Theorem 2. The volume of a solid obtained by rotation of a plane figure
about some axis lying in the plane of the figure and not intersecting it is
equal to the product of the area of this fi~ure by the circumference of the
circle described by the centre of gravity of the figure.

)(

Fig. 59

1727. Find the static moments about the coordinate axes of


a segment of the straight line
~+1.=1
a b •
lying between the axes.
172 Definite Integrals [Ch. 6

1728. Find the static moments of a rectangle, with sides a and b,


about its sides.
1729. Find the static momen~s. about the x- and y-axes, and
the coordinates of the cen~re of gravity of a triangle bounded by
the straight lines x+y=a, x=O, and y=O.
1730. Find the static moments, about the x- and y-axes, and
the coordinates of the centre of gravity of an arc of the astroid
t I I

x• -tY' =a',
lying in the first quadrant.
1731. Find the static moment of the circle
r =2asin <p
about the polar axis.
1732. Find the coordinates of the centre of gravity of an arc
of the catenary
X
y=acosh- a
from X= -a to x=a.
1733. Find the centre of gravity of an arc of a circle of radius a
subtending an angle 2a.
1734. Find the coordinates of the centre of gravity of the arc
of on~ arch of the cycloid
x=a (t- sin t); y =a (1-cos 1).
1735. Find the coordinates of the centre of gravity of an area
b!l2 = 1 and the coordinate axes (x ;;;:.0,
2
f)ounded by the ellipse az+
X

y;;;;. 0) . .
1736. Find the coordinates of the centre of gravity of an area
bounded by the curves
Y=X,I Y=vx.
l.r-

1737. Find the coordinates of the centre of gravity of an area


bounded by the first arch oi the cyclOid
x=a(t-sint), y=a(l-cost)
and the x-axis.
1738**. Find the centre of gravity of a hemisphere of radius a
lying ahove the xy-plane with centre at the ongin.
1739**. Find the centre of gravity of a homogeneous right
circular cone with base radius r and altitude h.
1740**. Find the centre of gravity of a homo~eneous hemi-
sphere of radius a lying above the xy-plane with centre at the
origin.
Sec. 121 ApplyinR Definite lniel(rals to Solution of Phqsical Problems 173

1741. Find the moment of inertia of a circle of radius a about


its diame~er.
1742. Find the moments of inertia of a rectangle with sides
a and b about its sides.
1743. Find the moment of inertia of a right parabolic segment
with base 2b nnd altitude h about its axis of symmetry.
1744. Find the moments of inertia of the area of the ellipse
2 2
~~ + ~. = 1 about its principal axes.
1745**. Fmd the polar moment of inertia of a circular rim~
with radii R, and R, (R,<R 1 ). that is, the moment of inertia
about the axis passing through the centre of the ring and perpen-
dicular to its plane.
1746**. Find the moment of inertia of a homogeneous right
circular cone with base rad1us R and altitude H about its axis.
174 7**. Fmd the moment of inertia of a homo~?,eneous sphere
of radius a and of mass M about its diameter.
1748. Find the surlace and volume of a torus obtained by
rotating a c1rcle of .-ad1us a about an axts lying in its plane
and at a distance b (b >a) from its centre.
1749. a) Determine the po!>ition of the centre of gravity of
2 2 •

an arc of the astroid x3 + y> =a> lying in the first quadrant.


b) Find the Cl'n:rc of gravity of an area bounded by the curves
y' = '2px and x' = 2py.
17fiO**. a) Find the centre of gravity of a sen:icircle usin~~
Guldin's theorem.
b) Prove by Guldin's theorem that the centre of gravity uf
a triangle is distant from its base by one thtrd of its altitude

Sec. 12. Applying Definite Integrals to the Solution of Physical Problems


1°. The path traver~ed by a point. II a point IS m motion ~lon~ some
curve ar:d the ~hsolute value of the velocity u=t (t) is ~ known fumuon of
the time t, then lhe path traversed by the po1nt in an mterval ol time
(t 1 , t 1 ) is
t,
S= ~ f (t) dt.
I,

Example t. The velocity of a point is


o=0.11 1 m/~ec.

F'ind the path s covE>red by the point m the interval of lime T= to ~ec follow-
Ing the comme1rement ol mot10n. What is the mean velocity d mouon
durmg this interval?
174 Definite Integrals [Ch. 5

Solution. We have:

s= S0.1t dt = 0.1 !•1: = 250 metres


1
0

and
s
Vmean = y= 25 mjsec.
2". The work of a force. If a variable force X= f (x) acts in the direction
of the x-axis, then the work of this force over an interval [x 1 , x,] is
x.
A=~ f (x) dx.
x,
Example 2. What work has to be performed to stretch a spring 6 cm, if
a force of 1 kgf stretches it by 1 em?
Solution, According to Hook's law the force X kgf stretching the spring
by xm is equal to X= kx, where k is a proportionality constant.
Putting x=O.Ol m and X= 1 kgf, we get k= 100 and, hence, X= IOOr.
Whence the sought-for work is
0,06 o.oo
A= ~ 100xdx=50x2 i =0.18 kgm

3". Kinetic energy. The kinetic energy of a material point of mass m and
velocity v is defined as

The ktnettc energy of a system of n material points with masses


m 1 , m2 , ••• , mn having respective velocities v 1 , v 2 , ••• , Vn• is equal to
n I

K= L m~v i. (I)
{. =t

To compute the kinetic energy of a solid, the latter is appropriately parti-


tioned into elementary particles (which play the part of material points); then
by summing the kinetic energies of these particles we get, in the limit, an
integral in place of the sum (1).
Example 3. Find the kinetic energy of a homogeneous circular cylinder
of density 6 with base radius R and altitude h rotating about its axis with
angular velocity ro.
Solution. For the elementary mass dm we take the mass of a hollow
cylinder of altitude h with inner radius r and wall thickness dr (Fig. 60).
We have:
dm= 2nr ·Mdr.
Since the linear velocity of the mass dm Is equal to v = rro, the elementary
kinetic energy is
Sec. 12] Applying Definite Integrals to Solution of Physical Problems 175

Whence
R
nw 2{)R 4 h
K= 2
M r'dr = .
1t(J)
S
0
4

4°. Pressure of a liquid. To compute the force of liquid pressure we use


Pascal's law, which states that the force of pressure of a liquid on an area S
at a depth of immersion h is
p=yhS,

where y is the specific weight of the liquid.

X
dr r

y
Fig. 60 Fi!.( 61

Example 4. Find the force of pressure expertenced by a semictrcle of


radiu> r submerged vertically in water so that it> diameter is flush with the
water surface (Ftg 61).
Solution, We partition the area of the semicircle into elements-strips
parallel to the surface of the water. The area of one such element (ignoring
higher-order infinitesimals) located at a distance h from the surface Is

ds:..: 2xdh =2 Vr"-h 2 dh.

The pressure experienced by this element is

dP=yhds=2yh Vr 2 -h 2 dh,

where y is the specific weight of the water equal to unity.


Whence the entire pressure is

1751. The velocity of a body thrown vertically upwards with


initial velocity v0 (air resistance neglected), is given by the
171 [Ch. 5

formula
v = V 0 -gt,
where t is the time that elapses and g is the acceleration of grav-
ity. At what distance from the initial position will the body
be in t seconds from the time it is thrown?
1752. The velocity of a body thrown vertically upwards with
initial velocity V 0 (air resistance allowed for) is given by the
formula
V=C·tan (-! t+arctan ~o),
where t is the time, g is the acceleration of gravity, and c is
a constant. Find the altitude reached by the body.
1753. A point on the x-axis performs harmonic oscillations
about the coordinate origin; its velocity is given by the formula
v= V0 cos u>t,
where t is the time and V 0 , ro are constants.
Find the law of oscillation of a point if when t = 0 it had
an abscissa x = 0. What is the mean value of the absolute magni-
tude of the velocity of the point during one cycle?
1754. The velocity of motion of a point is v=te-o.ott m/sec.
Find the path covered by the point from the commencement of
motion to full stop.
1755. A rocket rises vertically upwards. Considering that when
the rocket thrust is constant, the acceleration due to decreasing
weight of the rocket increases by the law 1 = a~bt (a- bt > 0),
find the velocity at any instant of time t, if the initial velocity
is zero. Find the altitude reached at tim~ t = t 1 •
1756*. Calculate the work that has to be done to pump the
water out of a vertical cylindrical barrel with base radius R and
a !tit ude H.
1757. Calculate the work that has to be done in order to pump
the water out of a conical vessel with verlex downwards, the
radius of the base of which is R and the altitude H.
1758. Calculate the work to be done in order to pump wateJ
out of a semispherical boiler of radius R = 10 m.
1759. Calculate the w;)rk needed to pum;J oil out of a tank
through an up,Jer opening (the tank has the shape of a cylinder
with horizontal axis) if the specific weight of the oil is y, the
length of the tank H and the radius of the base R.
1760**. What work has to be done to raise a body of mass m
from the earth's surface (radius R) to an altitude h? What is
the work if the body is removed to infinity? -
Sec. 12\ Applqin[! Definite lnte~rals to Solution of Phq~ical Problems 1n
1761**. Two electric charges e0 = 100 CGSE·and e1 =200 CGSE
tie on the x-axis at points x0 = 0 and x, = J em, respectively.
What work wi II be done if the second charge is moved to point
X 2 = 10 em?
1762**. A cylinder with a movable piston of diameter D=20 em
and length l = 80 em is filled with steam at a pressure
p = 10 kgf cm 2 • What work must be done to halve the volume of
the sleam with temperature kept constant (isothermic process)?
1763**. De:ermine the work performed in the adiabatic expan·
~ion of air (having initial volume V 0 = 1 m' and pressure
p,-= 1 kgfjcm 2 ) to volume v, = 10 m'?
1764**. A vertical shaft of weight P and i
radius a rests on a bearing AB (Fig. 62). fp
The frictional force between a small part a
of the base of the shaft and the surf ace of
the support in contact with it is F = JlPO,
where p =canst is the pressure of the shaft
on the surface of the support referred to
unit area of the support, while ll is the coef-
ficient of friction. Find the work done by the
frictional force during one revolution of the
shaft.
1765**. Calculate the kinetic energy of a Fig. 62
disk of ma-ss M and radius R rotating with
angular velocity <1> about. an axis that passes through its centre
perpendicular to its plane.
1766. Calculate lbe kinetic energy of a ri~ht circular cone of
mass M rotating with angular velocity <1l about its axis, if the
radius of the base of the cone is R and the altitude is H.
1767*. What work has to be don~ to stop an iron sphere of
radius R = 2 me:rcs rotating with angular velocity ol = 1,000 rpm
about its diameter? (Specific weight of iron, v= 7.8 g~cm' .)
1768. A vertical trian1le With base b and altitude h is sub·
merged vertex cbwnwards in water so that its base is on the
surface of the wa!er. Find the pressure of the water.
1769. A vertical dam has the sh 1pz of a trapezoid. Calculate
the water pressure on the dam if we kn::>w that the upper ba~e
a= 70 m. the lower ba:;e b = 5() m, and the height h = ~0 m.
1770. Ftnd the pressure of a liquid, whose sp~cific weight is y,
on a vertical ellipse (wtlh axes 2a and 2b) whose centre is sub-
merged in the liquid to a distance h, while the ma]ur axis 2a
of the ellipse is parallel to the level of lhe liquid (h;;;: b).
1771. Find the water pressure on a vrrlical Circular cone
with radius of base R and altitude H submzrged in wa~e1 verlex:
downwards so that its base is on the surface of the waler.
178 Definite Integrals [Ch. 5

Miscellaneous Problems
1772. Find the mass of a rod of length l = 100 em if the linear
density of the rod at a distance x em from one of its ends is
<'>=2+0.001 x• gjcm.
1773. According to empirical data the specific thermal capacity
of water at a temperature t° C (0 .s;;; t .s;;; 100°) is
C= 0.9983-5.184 X I0- t
5
+ 6.912 X IO-' / 1 ,
What quantity of heat has to be expended to heat I g of water
from 0° C to 100° C?
1774. The wind exerts a uniform pressure p g/cm• on a door
of width b em and height h em. Find the moment of the pressure
of the wind striving to turn the door on its hinges.
1775. What is the force of attraction of a material rod of
length l and mass M on a material point of mass m lying on
a straight line with the rod at a distance a from one of its ends?
1776**. In the case of steady-state laminar flow of a liquid
through a pipe of circular cross-section of radius a, the velocity
of flow v at a point distant r from the axis of the pipe is given
by the formula

where p is the pressure difference at the ends of the pipe, f.l. is


the coefficient of viscosity, and l is the length of the pipe.
Determine the discharge of liquid Q (that is, the quantity of
liquid flow,ing through a cross-section of the pipe in unit time).
1777*. The conditions are the same as in Problem 1776, but
the pipe has a rectangular cross-section, and the base a is great
compared with the altitude 2b. Here the rate of flow v at a point
M (x,y) is defined by the formula
v= 2:/ [b2- (b- y)2).
Determine the discharge of liquid Q.
1778**. In studies of the dynamic qualities of an automobile,
use is frequently made of special types of diagrams: the veloci-
ties v are laid off on the x-axis, and the reciprocals of correspond-
ing accelerations a, on the y-axis. Show that the area S bounded
by an arc of this graph, by two ordinates v = t/ 1 and v = V2 , and
by the x-axis is numerically equal to the time needed to increase
the velocity of motion of the automobile from v 1 to V1 ( accelera-
tion time).
Sec. 12) Applying Definite Integrals to Solution of Physical Problems 179

1779. A horizontal beam of length l is in- equilibrium due to


a downward vertical load uniformly distributed over the length
of the beam, and of support reactions A and B (A= B = ~ ) ,
directed vertically upwards. Find the bending moment Mx in
a cross-section x, that is, the moment about the point P with
abscissa x of all forces acting on the portion of the beam AP.
1780. A horizontal beam of length I is in equilibrium due to
support reactions A and B and a load distributed along the
length of the beam with intensity q = kx, where x is the distance
from the left support and k is a constant factor. Find the bend-
ing moment Mx in cross-section x.
Note. The intensity of load distribution is the load (force) referred to
unit length.
1781 *. Find the quantity of heat released by an alternating
sinusoidal current
I= 10 sin(:; t-<p)
during a cycle T in a conductor with resistance R.
Ch2pter VI

FUNCTIONS OF SEVERAL VARIABLES

Sec. 1. Fa~ic Notions


1°. The concept of a function of stveral variables. Functional notation.
A variable quantity z is called a single-valued function of two variables x,
y, if to each set of their values (x, y) in a givPn ron(!e there corre~ponds a
unique value of z The variables x and y are C'alled arguments or independent
variubles. The functional relation is denoted by

z=f (x, y).

Similarly, we define fundions of three or more arguments.


fxample t. Express the volume of a cone V as a function of its gcn·
eratri x x ancl of its base ra d 1us y
· Solution. From geometry we l<now that the volume of a cone is

where h is the altitude of the cone. But h = V x•- y 2


• Hence,

z ·~
~
This is the desired functional relation.
The value of the !unction z=f(x.y) at a
potnl P (a.b\. that is, when x= a and y=b,
lz is drnoted by I (a. b) or f (P) Genrrall y speak-
inl(. the (.'eornetnc rerres€ntation of a !unc-
I tion like z=-f (x,y) in a rectangular coordi·
I Y nate system X, Y, Z is a surface (Fig. 63).
! Pf:r:,yJ Example 2. Find I (2, -3) and t ( 1, f) if

f (x,y) = x +r/
2
-.,-.
Fig. 63 ..xy

Solut;on. Substitutinq r=2 and y= -3, we find

:Z'+t-3) 2 13
/(2, -J)= !l·2·(-3) -f2·
Sec. 1} Basic Notions 181

Putting x= 1 and replacing y by }!_, we will have


X

y )2
I 1L)= I+ ( x =xz+yz
2. I ( 7)
(
f 'x '2xy '

that is, f ( 1 . }!._ \


' \ X)
= f (x, y).
2°. Domain of definition of a function. By the domain of definition of a
function l = f (x, y) WP under,! and a set of point~ (x, y) in an xy-plane in
which the given function is defint>d (that is to say, in w:1ich 11 takes on def-
Inite real value~) In the simplest cases, the rlomain of definition of a func-
tion h a finite or infinite part of the xy-plane bounded by one or several
curves (the boundan1 of the domain).
Smlllilrly. for a funr-tion of three variablE's u=f (x, y, z) the domain of
definition of the funchon IS a volume in Ayz-space.
Example 3. Find the domain of definition ol the function
1

Solution. The fum·tion ha~ real values if4-x 2 -y2 >0orx 2 +y 2 <4.
The latter 1nrquality is sati~fierl hy tlw cr.ordinates ol points lying inside a
circle of radiot' 2 with centre ill th.;> coordmalp orwm. The domain of defi·
nition ol the function is the 1nlenor of the circle (F1g 64).

y
y

-2

F1g. 64 F1g 65

Example 4. Find the domain of definition of the function

z =arc sin
X
2 + V-AY-
X
Solution. The first term of the function 1s defined for -I<;
2 .;;;;; l or
-2.;;;;x.o;;;2. The second term has n•al values if xy>-0. i.e., m two cases:
when { ~~Z· or when { ~~~8·. The doma1n of definition of the entire
function is shown in fig. 65 and includes the boundaries of the domain.
182 Functions of Several Variables [Ch. 6

3°. Level lines and level surfaces of a function. The level line of a func-
tion z = f (x, y) is a line f (x, y) = C (in an xy-plane) at the points of which
the function takes on one and the same value z=C (usually labelled in
drawings).
The level surface of a function of three arguments u = f (x, y, z) is a sur-
lace f (x, y, z) = C, at the points of which the function takes on a constant
value u=C.
y Example 5. Construct the level Jines of
the function z=x2 y.
Solution. The equation of the level lines
c
has the form x2 y = C or y = 2 .
X
Putting C=O, ±I, ± 2, ... ,we get a family
of level Jines (Fig. 66).

X 1782. Express the volume V of a


regular tetragonal pyramid as a func-
tion of its altitude x and lateral edge y.
1783. Express the lateral surface S
o! a regular hexagonal truncated pyra-
Fig. 66 mid as a function of the sides x and y
of the bases and the altitude z.
1784. Find f(l/2,3), f(l, -1), if
f(x, y)=xy+~.
y

1785 Find
xz-yz
f (y,x), f(-x, -y),
!)' if
f(x, y)=2Xy.
1786. Find the values assumed by the function
f(x, Y)= 1 + x-y
at points of the parabola y = r, and construct the graph of the
function
F (x) = f (x, X2 ).
1787. Find the value of the function
z = x4 + 2xzyz + y4
~'---:!"---'-:'"­
l-x2-gl
+
at points of the circle X 2 y 2 = R 2 •
1788*. Determine f (x), if

f ( ~ ) = y~ (y > 0).
1789*. Find f (x, y) if
{(x+y, x--y)=xy+y'.
Sec. 1] Basic Notions 183

1790*. Let z=Vu+f(Vx-1). Determine ihe functions f and


z if z=x when y= l.
1791 **. Let z = xf ( ~) . Determine the functions f and z if
Z= Vf+Yi when X= 1.
1792. Find and sketch the domains of definition of the fol-
lowing functions:
a) Z= Vl-x•-y•; i) z = V y sin x;
b) z = 1 + V -(x-y)•; j) z = ln (x 2 + y);
c) z=ln(x+y); x-y .
+ 2 .,
k) z=arctang-
1 xy
d) z = x +arc cosy;
I
e) z = V 1- x• + V 1- y 2 ; I) Z=--
x• + y• '
0

f) Z=arcsin1L; I
X m) z= ·
Vy-Y·x'
g) z = VX2=4 + V4=iji; I I
h) z = V (r + y• -a•) (2a 2 -x2 -y•) n) Z= x-1 +-y;
(a> 0); o) zV sin (X 2 + y•).
1793. Find the domains of the following functions of three
arguments:
a) u = Vx+ V!/ + V'z; c) u =arc sin x+ arc sin Y+ arc sin z;
b) u=ln(xyz): d) u=Vl-x•-y•-z•.
1794. Construct the level Jines of the given functions and de-
termine the character of the surfaces depicted by these functions:
a) Z=x+y; d) z=Vxy; g) Z= ~ ;
b) Z=x'+y•; e) Z=(l +x+y)•; h) Z=-y-;
yX
2
1-I X 1-1 y I;
1
c) Z= X - y•; f) Z= i) Z=
X2
+x y1 •

1795. Find the level lines of the following functions:


a) z=ln(x'+y); d) Z={(y-ax);
b) z =arc sin xy; e) z = f (f) .
c) z = f (V x• + y•);
1796. Find the level surfaces of the functions of three inde-
pendent variables:
a) u = x+ Y·t- z;
b) u = x• + y• + z•;
c) u=x•-t y•-z•.
184 Functions of Several Variables (Ch. 6

Sec. 2. Continuity
1°. The limit of a function. A number A is called the limit of a function
Z=f (x, y) as the point P' (x. y) approaches the point P (a, h). if for any
B > 0 there is a l> > 0 such that when 0 < Q < 6, when' Q = V (x-a) 2 + (y- b)1
is the distance between P and P', we have the inequality
lf(x, y)-AI<e.
In this case we write
lim f(x, y)=A.
X-+a
y-+b

2°. Continuity and points of discontinuity. A function z=f (x, y) is called


continuous at a pomt P (a, b) if
lim f (x, y) = f (a, b).
x-+a
IJ-+b

A function that is continuous at all points of a given range is called


contin!lo:Js over this range
A function f (x, y) may cease to be continuous either at separate points
(isolated pot~tl of discontmutly) or at points that form one or several lines
(lmes of dtscontinuity) or (at times) more complex geometric objects.
Example 1. find the discontinuities of the function
xy+ l
Z=>.z-y·

Solution. Tile function will be m~aningless if the denomin1tor becomes


zero. But )( 2 -y=O or y=x 2 i~ the equation of a parabola. Hence, the given
function has for its discontmuity the parabola y=x 2 •

1797*. Find the following limits of functions:


a) Ii;n (x~ -r y~) sin..!_ · e) lin +x ;
J( .... o xy' X· .. OX y
lj-+.} !/-> 0
·. x+y . . x•-!lz
b) 11.11 z-+2 . d) li.n ( 1 +E..)"; f) It 11 .......--+3 •
X->"' X y X--toOO Xj x ~o>. Y
l/ + 00 y->k lJ .....,.o

1798. Test the following function for continuity:


{(x, y) = { V1- X
2
- y
2
when X
2
+ !l .,;;;; 1,
0 when X
2
+y• > 1.
1799. Find points of discontinuity of the functions:
I
a) z =In Vx• +y 2
; c) z-
- l-x2 -ya ·•
I I
b) Z = ,-- ;
,..t-y 12
d) Z =COS-.
>.y
Sec. 31 Partial Derit~atiues 185

1800*. Show that the function

Z= { X
2xy
z-+ Y 2 when xI + yI =I= 0,
0 when x=y=O
is continuous with respect to each of the variables x and y sepa-
rately, but is not continuous at the point (0, 0) with respect to
these variables together.

Sec. 3. Partial Derivatives


1°. Definition of a partial derivative. If z = f (x, y), then assuming, for
example, y constant, we get the derivative
f!!= . f(x+!J.t,y)-f(x,y)= f'(x )
-" 1tm " x , Y,
ux .o.~ .... o u.X

which is called the partial deriuatiue of the function z with respect to the-
vartable x. In similar fa~hion we define and denote the partial derivative of
the function z w!th respect to the variable y It is obvious that to find partial
derivative-;, one can use the ordinary formulas of differentiation.
Example 1. Find the partial derivatives of the function

z=lntan~.
y
Solution. Regarding y as constant, we get
iJz I I I 2
OX =t---x -~-X
an- cos -
• y = ~·
y
ysm-
y y
Similarly, holding x constant, we will have
i}z I 1 (
a,=--x-·--x
x)
---z = - 2x
. 2x"
Y tan- cos2 - y y stn-
1
y y u
Example 2. Find the partial derivatives of the following function of three
arguments:
u =x 1y2z 2x-3y z 5. + ++
iJu
Solution. ax=3x 2y2 z+2,
iJu
oy=2x 1yz-3,
au
oz =x•y•+t.

2". Euler's theorem. A function f (x, y) fs called a homo_;eneous function of


degree n if for every real factor k we have the equality
f (kx, ky) =; knf (x, y)
186 Functions of Several Variables [Ch. 6

A rational integral function will be homogeneous if all its terms are of one
and the same degree.
The following relationship holds for a homogeneous differentiable function
.of degree n (Euler's theorem):
xf~ (x, y) + yf~ (x, y) = nf (x, y).

Find the partial derivatives of the following functions:


1801. Z=X 1 +y 1 -3axy. 1808. Z=XY.
1802. Z=x-y. sin .!!...
x+y 1809, Z=e X
1803. Z= fL. . , /x•-y•
X 1810. z =arcsin J1 x"+Y".
--
1804. Z= Vx•-y•.
1811. z=lnsin;J;.
1805, Z= X ,
Y x"+ y• 1812. u=(xyy.
1806. z=ln(x+Vr+y 1
).
1813, U = zXY,
1807. z=arctanlL.
X
, '
1814. Find fx(2, I) and fu(2, I) if f(x, Y)=
, , ,
Yrxy-f--.
--x
y
1815. Find fx(l, 2, 0), / 11 (I, 2, 0), fz(l, 2, 0) if
f (x, y, z) =In (xy + z).
Verify Euler's theorem on homogeneous functions in Exam-
ples I8I6 to 18I9:
1816. f(x,y)=Ax"+2Bxy-Cy 2 • 1818. t(x,y)=v~·
, ~+Y

1817. Z= "+x 2 • 1819. f(x,y)=ln.lt.,


X ~ X
1820. Find ax(~· where r=Vx•+u•+z 1 •

1821. Calculate /It


iJr
ii ,
iJip
if x=rcoscp and y=rsincp.
az dz ,
1822. Sh ow t hat xa;+Y ay=2, If z=ln(x•+xy+y•).
iJz iJz . JL
1823. Sh ow t hat xa;+Y ay=xy+z, If z=xy+xex·
4 Sh ow tat
182. h au au au .
ax+ay+a 2 =0, If u=(x-y)(y-z)(z-x).
1825. Show that ~+aau+~=I, if u=x+x-y
QX y QZ y-z'
1826. Find z = z (x, y), it 0- =~.
y x +Y
°2
Sec. 4) Total DiUerential of a Function 187

1827. Find z=z(x, y) knowing that


iJz xz+y2
- =--and
0X X
z(x, y)=siny when x= 1.

1828. Through the point M(l,2,6) of a surface z=2x•+y1


are drawn planes parallel to the coordinate surfaces XOZ and
YOZ. Determine the angles formed with the coordinate axes by
the tangent lines (to the resulting cross-sections) drawn at their
common point M.
1829. The area of a trapezoid with bases a and b and alti-
as as as
tude his equal to S= 1 / 2 (a-j-b)h. Find au, ab, iJh and, using
the drawing, determine their geometrical meaning.
1830*. Show that the function

_I ~+2
2xy
,
·r
I X
2
+Y =I= 0 ,
2
f( X, y ) ---. X y
. \ ' 0 ' if X=tj=O

has partial derivatives f~ (x, y) and f~ (x, y) at the point (0, 0),
although it is discontinuous at this point. Construct the geomet-
ric image of this function near the point (0, 0).

Sec. 4. Total Differential of a Function


\
0
, Total increment of a function. The total increment of a function
z= f (x, y) is the difference
~Z--"i'lf(x, !1)-=f(x-j-~x. y+t\y)-f(x, y).
2°, The total dift'erential of a function. The total (or exact) dtUerential of
a function z = f (x, y) is the principal part of the total increment ~z. which
is linear with respect to the increments in the arguments ~x and ~y.
The difference between the total increment and the total differential of
the function is an inlinitesimal of higher order compared with Q= Y ~x 2 ~y 2 • +
A function definitely has a total differential if its partial derivatives are
continuous. If a function has a total differential, then it is called differen-
tiable. The differentials of independent variables coincide with their incre·
ments, that is, dx= ~x and dy= ~y. The total differential of the function
z = f (x, y) is computed by the formula
az iJz
dz = ihdx-f- ay dy.

Similarly, the total differential of a function of three arguments u = f tx, y, z)


is computed from the formula
au au au
du = iJxdx+ ay dy+ oz dz.
Example 1. For the function
f (x, y)=x1 -t-xy-y 2
find the total increment and the total differential.
188 Functions of Several Variables (Ch. I

Solution. f (JC+ l:!.x, y + l:!.y) = (x+ llx) 2 + (x + l:!.x) (y+ l:!.y)-(y + lly) 2 ;
+ +
l:!.f (x, y) = {( t'+ l\x) 2 + (x + l:!.x) (y + 1\y)-(y lly) 2 )-(x 2 xy-y") =
+ +
= 2X•I:!.X I:!.A 2 +X•I:!.y lj•l:!.x+ l\X•I:!.y·-2!J·I:!.lj-!J.y'=
= [(2x+ y) l:!.x + (x-2y) l:!.y) + (llx 2 + llx·lly- 6.!/).
Here, the expression df = (2x + y) l:!.x + (x-2y) l:!.y is the total differential of
the function. while (.!\x' + b.x·l:!.y-l:!.y 2 ) is an Infinitesimal of higher order
compared with V ll-t 2 + i\y•.
Example 2. Find the total differential of the function
Z= Vx 2
+y2•

Solution.
oz
-=
x
;
oz Y
ax vx• + y• ay= v..~.•+y~·
xdt"+ydy
dz
vl•+u'.
3°. Applying the to!al dilfcrential of a function to approximate calculations,
For suffirim'ly small 1.!\xl and I b.yl and, hence, for ~ufficiently small
Q= y' .!\. 2 + b.y 2 , we have for a differentiable function z=ftx, y) the approx·
imate equality .!\z:::::: dz or
az az
ax +ay
l:!.z:::::: Ax b.y.

Example 3. The altitude of a· cone is H = 30 em, the radius of tht> base


R = 10 em. How will the volume of the cone change, if we increase H by
3 mm and diminish R by I mm?
Solutlo11. The volume of the cone is V = ~ nR'H. The change in volume
we replace approximately by the differential
1
AV:::::: dV=an (2RH dR+R 2 dH)=
= 31 n (-2·10·30·0.1 + 100·0.3)= -lO.n ::::::-31.4 em•.

Example 4. Compute 1.02'·01 approximately.


Solution. We consider the functiOn z == xY. The desired numbE.'r may be
considered the increasE.'d value of this function when x= l, y=3, l\x=0.02,
lly=O.Ol. The initial value of the function z=l'=l,
llz:::::: dz= yxY-J b.x+xY In x b.y= 3·1·0.02+ l·ln 1·0.01 =0.06.
Hence, 1.02a.o 1 :::::: 1 + 0.06= 1.06.

1831. For the function f(x, y)=x•y find the total increment
and the total dillerential at the point (1, 2); compare them if
a) 11x = 1, 11y = 2; b) L\x = 0.1, 11y = 0.2.
1832. Show that for the functions u and v of several (for
example, two) variables the ordinary rules of differentiation hol<h

c) d( *
a) d(u+v)=du+dv;
)=vdu;udo.
b) d(uv)=udv+vdu;
Sec. 4) Total Differential of a Functton 189

Find the total differentials of the following functions:


1833. z=x'+y'-3x y. 1841. z=lntanE...
I 834. z = X 2y'. X
x•- 11• 1842. Find df (1, 1), if
f83 5. Z =A 2 + ~~ ,
1836. z=sin 2 x+cos•y.
f(x, y) =;.
1837. z = yxY. 1843. u = xyz.
1838. z =In (x' + y'). 1844. u = V-;x2:--\--y-;":-+---,z'.

1839. f(x,y)=ln (1 +~)· 1845. U= ( xy+ f r


xy
1840. z =arctan Y
X
+ 1846. u =arc tan 3 .
z
1847. Find df (3, 4, 5) if
+arc tan~.
f (x, y, z) = y •z • •
+Y A

1848. One side of a rectangle is a= 10 em, the other b = 24 em.


How will a diagonal l of the rectangle change if the side a is
increased by 4 mm and b is shortened by 1 mm? Approximate
the change and compare it with the exact value.
1849. A closed box with outer d imcnsions 10 em, 8 em,
and 6 em is made of 2-mm-thick plywood. Approximate the
volume of material used in making the box.
1850*. The central angle of a circular se~tor is 80°; it is dt:>sired
to reduce it by 1°. By how much should the radius of the sector
be increased so that the area will remain unchanged, if theorig-
inal leng:h of the radius is 20 em?
1851. Approximate:
a) (1.02)'· (0.97) 2 ; b) V(4.05) 1 + (2.93) 1 ;
c) sin 32°·Cos 59° (when converting degrees into radius and
calculating sin 60° take three significant figures; round off the
last digit).
1852. Show that the relative error of a product is approxima-
tely equal to the sum of the relative errors of the factors.
1853. Measurements of a triangle ABC yielded the following
data: side a= 100m±2m. side b = 200m±3 m, angle
C=60°±l 0 • To what degree of accuracy can we compute the
side c?
1854. The oscillation period T of a pendulum is computed
from the formula
r---y
T=2ttY - -
g ,
190 Functions of Several Variables [Ch. 6

where l is the length of the pendulum and g is the acceleration


of gravity. Find the error, when determining T, obtained as a
result of small errors Ill= a and tlg = ~ in measuring l and g.
1855. The distance between the points P0 (x 0 , Yo' and P (x, y)
is equal to Q, while the angle formed by the vector P 0 P with
the x-axis is a. By how much will the angle a change if the
point P (P 0 is fixed) moves to P1 (x +dx, y+dy)?

Sec. 5. Differentiation of Composite Functions


1°. The case of one independent variable. If z=f(x, y) is a differentiable
function of the arguments x and y, which in turn are differentiable function~
of an independent variable t,
X=q> (/), y='ljl (/),

then the derivative of the composite function z= f [q> (t), 'ljl (t)] may be com-
puted from the formula
(I)

In particular, if t coincides with one of the arguments, for instance x,


then the "total" derivative of the function z with respect to x will be:

(2)

Example t. Find ::, if

z=e1"+ 2Y, where x=cost, Y=l 2 •


Solution. From formula (I) we have:
dz ' acost+2l'
dt=e1"+ 2Y·3(-sint)+e1 "+ 2Y·2·2t =e'"+ 2Y(4t-3sin t)=e (4t-3~int).

Example 2. Find the partial derivative aaz and the total derivative dz II
x dx'
z=e"Y, where g=q> (x).
Solution. ~=ye"Y.
From formula (2) we obtain
dz
dx = ye"Y + xeXY q>' (x).
2°. The case of several independent variables. If z is a composite function of
several independent vari~bles, for insta':'ce, z = f (x,y), where x= q> (u,u),
y='ljl (u, v) (u and v are mdependent vanables). then the partial derivatives z
with respect to u and v are expressed as
az=~~+~ay (3)
au dxilu ilyilu
Sec. 5] Differentiation of Composite Functions 191

and
az az ax az ay
au
av =ax + ay av 0 (4)

In all the cases considered the following formula holds:


az
dz= ax dx+ azay dy
(tile invariance property of a total dtfferential).
Example 3. Find ~~ and :~ , if
u
z= f (x, y), where X=UV, Y=-v.

Solution. Applying formulas t3) and (4), we get:


az • • 1
au=
f X (X, y)' V + f y (X, !f) V
and
iJz _ • F' u
(fv -fx (x, y) u-, Y (x, y) V2 .
Example 4. Show that the function z = cp (x1 + y 2) satisfies the equation
az az
urrx-x ay=O.
Solution. The function cp depends on x and y via the intermediate argu-
ment x2 + y2 = t, therefore,
~- dt
ax- ~ CJx-Cj)
at - I (
X
I+ Z) 2
y X

and

a~=~ aat =cp' (xz+ yz) 2y.


y dt y
Substituting the partial derivatives into the left-hand side of the equa-
tion, we get
az az
y (fx - X ay = ycp'(x1 +y 1 ) 2x-xcp' (x1 +y 2 ) 2y = 2xycp' (x1 +y 2 )-2xy cp'(x1 + y 2 )oa0,
that is, the function z satisfies the given equation.

1856. Find :: if
z=f. where x=e', y=lnt.

1857. Find :~ if
. X
u= l n sm VY, where X= 3tZ, y= Vf+l.

1858. Find :~ if
u =xyz, where X= t• + 1, y= In t, z = tant.
192 Functions of Several Variables [Ch. 6

1859. Find du if
dt
u= V xz+yz
z , where x = R cost, y = R sin t, z =H.

1860. Find :: if

z=u'O, where u=sinx, v=cosx.


1861. Find :: and :: if

z =arc tan.!L
X
and y = x•.
1862. Find ~ and ~ if
z = xY, where y = cp (x).
1863. Find :: and ~ if
z=f(u, v), where u=x•-y•, v=exY,

1864. Find ~ and ~ if


z=arctan~.
y
where x=usinv, y=ucosu.

1865. Find :: and ~ if


z= f (u), where u = xy +Y • X

1866. Show that if


u =<I> (x'+ y• + z•), where x = R cos <p cos "~'•
y = R cos <p sin 'IJ', z = R sin <p,
then

1867. Find ~ if
u = f (x, y, z), where y = cp (x), z ='I' {x, y).
1868. Show that if
z=f (x+ay),
where f is a differentiable function, then
dz az
-ay=a ax·
Sec. 6] Derivative in a Given Direction 193

1869. Show that the function


w= f (u, v),
where u=x+at, v=y+bt satisfy the equation
aw- iJw+b iJw
at -a ax ay .
1870. Show tl-tat the function
z =y cp (x 2 -Y 2 )
. f"1es th e equa t.10n -a-+-a-=~
sa t 1s I iJz I iJz z
·
X X y y y
1871. Show that the function
z = xy + xcp ( ~ )

· f"1es t he equa t ·ton x-


sa t 1s az iJz
0X -1-Y uy
-:;-=Xy + z.
1872. Show that the function

z = eY q' (ye 2~')


satisfies the equation (x 2 -y 2 ) ~+xy~=xyz.
1873. The side of a rectangle x """"~ 20 m increases at the rate
of 5 mjsec, the other side y = 30 m decreases at 4 m/sec. What
is the rate of change of the perimeter and the area of the rect-
angle?
1874. The equations of motion of a material point are
X= t, y =-= t•, Z = t•.
What is the rate of recession of this point from the coordinate
origin?
1875. Two boats start out from A at one time; one moves
northwards, the other in a northeasterly direction. Their veloci-
ties are respectively 20 km/hr and 40 kmjhr. At what rate does
the distance between them increase?

Sec. 6. Derivative in a Given Direction and the Gradient of a Functiort


_..
to. The derivative of a function in a given direction. The derivative of a
function z=f(x, y) itt a given direction l=PP 1 is
iJz = lim f (Pd-f (P) ,
iJl P,P -.o P 1P

]_ 1900
194 Functions of Several Variables {Ch. 6

where f(P) and f (P 1) are values of the function at the points P and P,
If the function z is differentiable, then the following formula holds:
iJz iJz iJz .
iJl =ax COS a + ay Sill a, (1)

where a Is the angle formed by the vector l with the x-axis (Fig. 67).

0 X
Fig. 67

In similar fashion we define the derivative in a given direction l for a


function of three arguments u = f (x, y, z). In this case
du iJu iJu iJu
ai =ax cos a +ay cos ~ + az cos)', (2)

where a, ~. '\' are the angles between the direction l and the corresponding
coordinate axes. The directional derivative characterises the rate of change
of the function in the given direction.
Example I. Find the derivative of the function z=2x 2 -3y 2 at the point
P (1, 0) in a direction that makes a 120° angle with the x-axis.
Solution. Find the partial derivatives of the given function and their
values at th~ point P:

~=4x; (~)P =4;


iJz
ay= -6y; au
(oz) P =0.

Here,
cos a= cos 120° = - { ,

sin a= sin 120° = -2-. ¥3


Applying formula (1), we get
1
oz
-=4 I ) +O· •r 3
( --
iJl 2 2 = - 2.
The minus sign indicates that the function diminishes at the given point and
in the given direction.
2°. The gradient of a function. The gradient of a function z = f (x, y) 1s
.a vector whose projections on the coordinate axes are the corresponding par-
Sec. 6) Derh•ative in a Given Direction 195

tia\ derivatives of the given function:


iJz iJz
grad z =OX l + ayl· (3)

The derivative of the given function in the direction l is connected with


the gradient of the function by the following formula:
iJz
Cfi =pro jl grad z.

That is, the derivative in a given direction is equal to the projection of the
gradient of the function on the direction of differentiation.
The grad1ent of a function at each point is directed along the normal to
the corresponding level line of the function. The direction of the gradient of
the function at a given point is the direction of the maximum rate of increase
of the function at this point, th!t is, when l=grad z the derivative~~ takes
on its greatest value, equal to

In similar fashion we define the gradient of a function of three variables,


u~f(x, y, z):
iJu iJu
grad u =a-X I + a-y j
au k. + ;;-
vZ
(4)

The grad1ent of a functwn of three variables at each point is directed along


the norma 1 to the level :>urface passing through this point.
Example 2. Find and construct the gradient of the function Z=x 2 y at
the point P (I, 1).

y
2
1
-V:
-
PI
-- -~
1

0 2 X
Fig. 68

Solution. Compute the partial derivatives and their values at the polnf P.

~=2xy; (~)P=2;

Hence, grad z=2l+J (Fig. 68).


7*
196 Functions of Several Variables {Ch. 6

1876. Find the derivative of the function z = X 2 -xy-2yJ


at the point P (1, 2) in the direction that produces an angle
"Of 60° with the x-axis.
1877. Find the derivative of the function z=x'-2x 2 y+xy + 1
2

.at the point M (1, 2) in the direction from this point to the
point N (4, 6).
1878. Find the derivative of the function z=lnVx 2 -f-Y 2 at
the point P (1, 1) in the direction of the bisector of the first
quadrantal angle.
1879. Find the derivative of the function u=x 2 -3yz+5 at
the point M (1, 2, -1) in the direction that iorms identical
angles with all the coordinate axes.
•sso. Find the derivative of the function u=xy+yz +zx at
the point M (2, 1, 3) in the direction from this point to the
point N (5, 5, 15).
1881. Find the derivative of the function u =In (ex+ e>' + e~)
at the origin in the direction which forms with the coordinate
axes x, y, z the angles a, ~. y, respectively.
1882. The point at which the derivative of a function in any
direction is zero is called the stationary point of this function.
Find the stationary points of the following functions:
+
a) z = X 2 xy + yz -4x-2y;
b) Z=X 3 +Y 3 -3xy;
c) u=2y 1 +z 1 -xy-yz+2x.
1883. Show that the derivative of the function z=LX
taken
at any point of the ellipse 2x 2 2 2
+
y = C along the normal to the
et1ipse is equal to zero.
1884. Find grad z at the point (2, 1) if
z= X
3
+ y'- 3xy.
1885. Find grad z at the point (5, 3) if
Z= V x~-y~.
1886. Find grad u at the point (1, 2, 3), if u =xyz.
1887. Find the magnitude and direction of grad u at the
point (2, -2, 1) if

1888. Find the angle between the gradients of the function


Z= In ~ at the points A (1/2, 1/4) and B (1, 1).
Sec. 7) Higher-Order Derivatives and DiOerential~ 197

1889. Find the steepest slope of the surface


Z =X
1
4y
2
+
at the point (2, 1, 8).
1890. Construct a vector field of the gradient of the following
functions:
a) z=x+y:
b) z=xy;

Sec. 7. Higher-Order Derivatives and Differentials


1°. Higher-order partial derivatives. The second partwl derivatives of a
function z= f (x, y) are the partial derivatives of its first partial derivatives.
For second derivatives we use the notations

~ (i) = ~:~ = r:x (x, y);

a (iJz) a•z "


if!; iJx = axay = fx 11 (x, y) and w forth.

Derivatives of order higher than second are similarly defined and denoted.
If the partial derivatives to be evaluated are continuous, then the result
of repeated dtUerentwtwn is independent of the order in which the difjerentta-
lton ts performed.
Example 1. Ftnd the second partial denvatives of the func!ton

z= arc tan~
y •

Solution. f1r~t lind the first partial dertvatives:


az I I y
ax=---xz. -y= .\ 2 +Y 1 '
1+--yz
(}z l ( X) X
ay = 1+ x 1 - y• = - x• + y 2 •

y•
Now differentiate a second time:
a•z - !!.._(-y-)-- (x 2xy
ox• - iJx x• + y• - +y 2 2) 2 '

a•z iJ ( X ) 2xy
cJy• = ay - x• + y• = (x" + y•)• •
cJ2z a( y ) +
I· (x 2 y 2)-2Y·1/ x•-y•
ax ay = iJY x• + y• = +
(xl y 1) 2 (x 2 +y 2) 1 •

We note that the so-called "mixed" partial derivative may be found in a


<lifferent way, namely:
a•z iJ2 z iJ ( X ) J•(X2 + y )-2X•X
2
X
2
-y"
ax ay = iJy ax =ax - x•+ yi =- (x + y
1 2) 2 (x"+ y2 ) 2 •
198 Functions of Several Variables [Ch. 6

2°. Higher-order differentials. The second differential of a function


z=f (x, y) is the, differential of the differential (first-order) of this function:
d 2 z=d (dz)
We similarly define the differentials of a function z of order higher than
two, for instance:

and, generally,

If z = f (x, y), where x and y are independent variables, then the second
differential of the function z is computed from the formula
iJ2z iJ!z
d
2
=iJiz + 2 iJx iJy dx dy +iJyt dy
z iJx2 dx
2 2
• (I)

Generally, the following symbolic formula holds true:

dnz= ( dx ~ +dy ~ rz;


it is formally expanded by the binomial law.
If z = f (x, y), where the arguments x and y are functions of one or sev-
eral independent variables, then
iJZz d 2 iJ2z d . d i}tz d iJz d2 iJz d2 (2)
d zz=iJx2 X + 2 iJxiJy X y + iJyZ y + iJX
2
x+ay y,
If x and y are independent variables, then d 2x=0, d 2 y=O, and formula (2)
becomes identical with formula (I).
Example 2. Find the total differentials of the first and second orders of
the function
z = 2x'-3xy-y2.
Solution. First method. We have
az iJz
ax=4x-3y, iJy=-3x-2y.
Therefore,
iJz iJz
iJz=ax dx+ au
dy= (4x-3y) dx-(3x+2y) dy.
Further we have

whence it follows that


iJ2z
2 iJ2z 2 + 2 iJ-iJ iJ2z 1
d z = iJ-
X2
dx X y
dx dy + iJ- 2
y 2 dy = 4dx - 6 dx dy- 2 dy '
2

Second method. Differentiating we fmd


dz=4x dx-3 (ydx+xdy)-2ydy=(4x-3y) dx-(3x+2y) dy.
Differentiating again and remembering that dx and dy are not dependent on
x and y, we get
d 2 z= (4dx-3dy) dx-(3dx+2dy) dy=4dx 2 -6dxdy-2dy 2 ,
Sec. 7) Higher-Order Derivatives and Differentials 199

if
-./ x•
Z=C v 7+/Ji'y• •
. a•z iJ z a•z .
1

1892. Ftnd ax2' axay, ay• tf


z= In (x1 + y).
1893. Find a•z if
axay

1894. Find a~·;Y if


z=arctan ,x-+y.
-xy
1895. Find ~;. if

1896. Find all second partial derivatives of the function


u=xy+yz -t zx.
. d
1897 . F tn a•u t'f
ax ay az

z =sin (xy).
1899. Find f:x (0, 0), f:,, (0, 0), /;~,1 (0, 0) if

f(x, y)=(l + X)m(l +Y)n.


a•z a•z .
a-a =;;---a
1900. Show that X y ulj X
tf

z=arc sin V-x-


_Y.
X

1901. Show that axay


a•z a•z .
-= ax tf
ay-
Z=XY.

1902*. Show that for the function


200 Functions of Several Variables [Ch. 6

[provided that f (0, 0) = 0] we have


f; y (0, 0) = -1, f;,x
·'
(0, 0) = + 1.
2
1903. Find iJ'z2 iJ'z a z2 if
iJx ' axay' iJy
z=f(u, v),
where u=x•+y\ v=xy.
1904. Find ~:~if u=f(x, y, z),
where z = IJl (x, y).
. iJ'z iJ'z iJ 2z .
1905. Fmd a:Xz, axay, ayt tf
z=f(u, v), where U=«p(x, y), V=lj>(x, y).
1906. Show that the function
u =arc tan J..X
satisfies the Laplace equation
2 2
iJ u iJ u
-+-=0
iJx2 ayt ·
1907. Show that the function
l
u=ln-,
r

where r=V(x-a) 2 +(y-b) 2 , satisfies the Laplace equation


iJ2u iJ'u
ax•+ iJy2 = 0.
1908. Show that the function
u (x, t) =A sin (a'At + <p) sin AX
satisfies the equation of oscillations of a string
iJ'u _ 2 iJ'u
iJt2- a axz.
1909. Show that the function
(x-xo) 2 +(y- Y0 ) 2 +(2 -zo)'
u(x, y, z, t)= V1 e
(2a rrl) 3

(where X 0 , !J 0 , Z0 , a are constants) satisfies the equation of heat


conduction
Sec. 7] Higher-Order Derivatives and DiOerentials ~01

1910. Show that the function


u = cp (x-at) + 'ljJ (x +at),
where cp and 'ljJ are arbitrary twice differentiable functions, satis-
fies the equation of osci !lations of a string
a•u 2 a•u .
at•=a ax•
1911. Show that the function
Z = Xfj! ( ~) } 'ljJ ( ~)
satisfies the equation
X -
2
2 iJ z
ox2
+ 2X - +l-
a•z
y iJxoy
a•z- 0
y oy•- .
2

1912. Show that the function


u = cp (xy) + l/ xy\jl ( ~)
satisfies the equation
2 a•u
X ;;-•-
2
y iJ---"2
a•u = 0
.
ux y

1913. Show that the function z=f(x-1-cp(y)] satisfies the equa-


lion
i)z a•z az a•z
ax ox ay = Jii ax• .
1914. Find u-=u(x, y) if

1915. Determine the form of the function u = u (x, y), which


satisfies the equation

1916. Find d 2Z if
z=e"Y.
1917. Find d2 U if
u= xyz.
1918. Find d 1 z if
Z=cp(t), where t=x 2 +y1 •
1919. Find dz and d 2 z if
X
z= uv where u =-,
y
v =xy.
202 Functions of Several Variables (Ch. 6

1920. Find d 2 z if
z = f (u, v), where u =ax, v =by

1921. Find d z if
1

z = f (u, v}, where u = xeY, v = ye".


1922. Find d1z if
Z= e" cosy.
1923. Find the third differential of the function
z = x cosy+ y sin x.
Determine all third partial derivatives.
1924. Find df(l, 2) and d•f(l, 2) if
f(x, y)=X 2 -f-xy+y 2 -4lnx-10lny.
1925. Find d 2/ (0, 0, 0) if
f (x, y, z) = x• + 2y• + 3z 2 -2xy + 4xz + 2yz.

Sec. 8. Integration of Total Differentials


\ The condition for a total dift'erential. For an expression P (x, y) dx +
0
,

+ Q (x, y) dy, where the functions P (x, y) and Q (x, y) are continuous in a
simply connected region D together with their first partial derivatives, to be
(in D) the total differential of some function u (x, y), it Is necessary and suf-
ficient that
aQ ap
ax= iJy.
Example t. Make sure that the expression
(2x+ y) dx+ (x+ 2y) dy
is a total differenllal of some function, and find that function.
Solution. In the given case, P=2x+y, Q=x+2y. Therefore,~=~:=
=I, and, hence,
au au
(2x+y)dx+(x+2y)dy=du= axdx+ ay dy,
where u is the desired function.
au
It is given that ax= +
2x y; therefore,

u= ~ (2x+y)dx=x•+xy+~p(y).
ou
Buf on the other hand oy=x+<p'(y)=x+2y,whence<p'(y)=2y,<p(y)=y 2 +C
and
u=x 2 +xy+y1 +C.
Finally we have
(2x+y) dx+(x+ 2y) dy=d (x 1 +xy+ y 1 -!-C).
Sec. 8] Integration of Total Dtf}erentials 203

2°. The case of three variables. Similarly, the expression


P(x, y, z)dx+Q(x, y, z)dy+R(x, y, z)dz,
where P (x, y, z), Q (x, y, z), R (x, y, z) are, together with their first p~rtial
derivatives, continuous functions of the variables x, y and z, is the total
differential of some function u (x, y, z) if and only if the following conditions
are fulfilled:

Example 2. Be sure that the expression


(3x 2 +3y-1} dx+(z 2 -j-3x) dy+(2yz +I) dz

is the total differential of some function, and find that function.


Solution. Here, P=3x 2 +3y-l, Q =z 2 +3x, R=211z+ 1. We establish
the fact that

and, hence,
au au iJu
(3x 2 -j-3y-l)dx+(z 2 +3x) dy+(2yz+ I) dz=du =-a dx+a- dy+-a dz,
X !J Z
where tt is the sought-for function.
We have
au
ax =3x 2 + 3tj-l,
hence,
u= ~ (3x 2 +3y-l)dx=r+3xy-x+cp(y, z).

On the other hand,


au acp 2

a-=3x+a-=z
y y
+3x,
au aiJl
-=-=2yz-j-l,
iJz iJz

whence ~IJl =z 2 and iJaiJlZ =2yz+ l. The problem reduces to finding fhe function
vy
of two variables qJ (y, z) whose partial derivatives are known and the condi·
tion for total differential is fulfilled_
We lind cp:
cp (y, z) = ~ z1dy = yz 2 + ljJ (z),
~: =2yz-j-ljl' (z) =2yz-j- I,
'Jl' (z)= l, 'Jl (z)=z+C,

that is, cp (y, r)=yz2 +z+C. And finally,


u = x' + 3xy -x + yz 2 + z +C.
204 Functions of Several Variables [Ch. 6
~--------------~----~--------------------~-
Having convinced yourself that the expressions given below
are total differentials of certain functions, find these functions.
1926. y dx+x dy.
1927. (cos x+ 3x 2 y) dx+ (x 3 -y 1 ) dy.
(x+2y)dx+ydy
1928 . (x+ y)2
x+2y . 2x-y
1929. -r--+
X lj 2
dx-~+
X lj
dy. 1

1930. _!_ dx--=- dy.


!I y•
1931. y x•+ya
x dx+ y" Y
xz+ya
dy.
1932. Determine the constants a and b in such a manner that
the expression
(ax 2 + 2xy -1- y 2) dx-(x 2 + 2xy + by 2 ) dy
(x• + y•)~
should be a total differential of some function z, and find that
function.
Convince yourself that the expressions given below are total
differentials of some functions and find these functions.
+
1933. (2x+ y+ z) dx (x + 2y+z) dy+ (x + y -1- 2z) dz.
1934. (3x 2 +2y• + 3z) dx+ (4xy+2y-z) dy + (3x-y-2) dz.
1935. (2xyz-3y 1 z-f- 8xy• 2) dx+ +
+
(x 1 z-6xyz+ 8x1 y -1- 1) dy -i- (x 2 y-3xy 2 -1- 3) dz.
1936. (>-;.)dx 1-(!-;~)dy+(!-;.)dz.
193 7. x dx + y dy + z dz
Yx2+y•+z2
1938*. Given the projections of a force on the coordinate axes
X--Y_2 Y-~2
-(x+y) ' -(x+y) '

where A is a constant. What must the coefficient A be for the force


to have a potential?
1939. What condition must the function f (x, y) satisfy for the
expression
f (x, y) (dx + dy)
to be a total differential?
1940. Find the Function u if
du = f (xy) (y dx + x dy).
Sec. 9] Differentiation of Implicit Functions 205

Sec. 9. Differentiation of Implicit Functions


1°, The case of one independent variable. If the equation f (x, y) =0, where
I (x,y) is a differentiable function of the variables x and y, defines y as a
function of x, then the derivative of this implicitly defined function, provided
that /~ (x, y) f= 0, may be found from the formula
dy ~~ (x, y)
(I)
dx = - 1~ (x, y) ·
Higher-order derivatives are found by successive differentiation of formula
(l)
dy d2 y
Example I. Fwd iiX and dx• If
0

(x 2 + y 2 ) 3 - 3 (x 2 + y2 ) + 1 = 0.
Solution. Denoting the left-hand side of this equation by f (x, y), we find
the partial denvatives
f~(x, y) =3 (x 2 + y 2 ) 2 ·2x-3-2x=6x [(x 2 + y2 ) 2 -l],
(Jt, y)=3(x 2 +y 2 ) 2 ·2y-3·2y=6y [(x 2 +y 2 ) 2 -l).
Whencl', applyin~ formula (I), we get
dy t:(x,y) 6x((x 2 +y 2 ) 2 -1) x
dx=---f'(
!I x, y
) 6y((x 2 +y 2 ) 2 -l)=-!/ ·

To find the second derivative, differentiate with respect to x the first deriva-
tive which we have found, taking into consideration the fact that y is a func-
ttun of x·

d 2 tl_!}_(·-!..)--l·y-x~ y-x(-f) Y!+x•


dx 2 - dx y - y2 y• = - ---ys--
20. The case of several independent variables. Sirmlarly, if the equatioll
F (x, y, z) =-=0, where F (x, y, z) is a differentiable function of the variables
x, y and z, defines z as a function of the independent variables x and y and
F~ (x, y, z) f= 0, then the partial derivatives of this implicitly represented
function can, generally speaking, be found from the formulas
i)z F~(x, 1/. z) i}z F;,(x, y, z)
(2)
lh=- F; (x, y, z) ' ily F; (x, y, z) •
Here is another way of finding the derivatives of the function z: differenti-
atin~ the equation F (x, y, z) =0, we find
iJF iJF iJF
-dx+-:;-dY+a-dz=O.
X uy a Z

Whence it is possible to determine dz, and, therefore,


iJz iJz
ax and ay.
206 Functions of Several Variables [Ch. 6

Example 2. Find :~ and ~~ if


x2 -2y2 + 3z2 -yz + y = 0.
Solution. First method. Denoting the left side of this equation by F (x, y, z),
we find the partial derivatives
F;(x, y, z)=2x, F~(x, y, z)=-4y-z+l, F;(x, y, z)==6z-y.
Applying formulas (2), we get

az F~ (x, y, z) 2x az F~ (x, !f, z) l-4y-z


ax= ' (
Fz X, y, Z ) = - 6z-y ; ay- - p' (
z X, y, Z
) 6z-y

Second method. Differentiating the given equation, we obtain


2x dx-4y dy+6zdz-y dz-z dy+dy=O.
Whence we determine dz, that is, the total differential of the implicit func-
tion:
dz = 2x dx+(J-4y-z) dy
y-6z

Comparing with the formula dz=~;dx+~dy, we see that

az 2x az l-4y-z
ax= y-6z ' ay y-6z
3°. A system of implicit functions. If a system of two equa lions
F(x, y, u, v)=O,
{ G (x, y, u, v)=O
defines u and v as functions of the variables x and y and the Jacobian

D (F, G) au av
D (u, v) ao ao # 0•
au av
then the differentials of these functions (and hence their partial derivatives
as well) may be found from the following set of equations
dF aF aF aF
-- dx +ay- dy + - du + - dv = 0 '
ax au av
ao (3)
{ ao ao ao
0- X dx + -iJy dy +-aU du -l-' 0- V dv =
0.

Example 3. The equations


u+v=x+y, xu+yv=l
. ~
define u and v as funchons of x and y; find ax , iJy , ax and
~ ~ ~
ay .
Sec. 91 Differentiation of Implicit Functions 207

Solution. first method. Differentiating both equations with respect to x.


we obtain

whence

Similarly we find
iJu v + y iJv v + x
dy = -X- y ' iJy = X - y

Second method. By differentiation we find two equations that connect the


differentials of all four variables:
du+dv=dx+dy,
xdu +u dx+y dv+ v dy=-0.
Solvin~ this system for the dtfferentials du and dv, we obtain
du = _ (u + y) dx + (v + y) dy , dv=(u+x)dx+(v+x)dy.
x-y x-y
Whence
iJu u + y iJu v +y
ax=-x-y' Jij=-x-y'
uu u+x iJv v+x
ax=x--y' ay=x-y.

4°, Parametric representation of a function. If a function z of the varia-


bles x and y is represented parametrically by the equations
x=x(u, v), y=-y(u, v), z=z(tl, v)
and
ax ax
D (x, y) auay iJuay :f. o,
D (u, v)
au av

then the differential of this function may be found from the following system
of equations
ax iJx
dx- au
du + av dv'
ay ay
dy =au du + av dv. (4)
{ iJz iJz
dz= a-du-t-
ll
;;;- dv.
vV

Knowing the differential dz = p d~ +q dy, we find the partial derivatives


iJz oz
ax=P and ay-"q,
208 Functions of Several Variables [Ch. 6

Example 4. The function z of the arguments x and y is defined by the


equations

. az az
Fmd ax and ay.
Solution. first method. By differentiation we find three equations that
connect the differentials of all five variables:
dx=du+dv,
{ dy=2u2du+2v 2dv,
dz= 3u du + 3v dv.
From the first two equations we determine du and dv:
du __ 2v dx-dy dv ___ dy-2u dx
- 2(o-u) ' - 2(v-u) ·
Substituting into the third equation the values of du and dv just found, we
nave:
dz = 3u2 2v dx-dy + 302 dy- 2u dx =
2 (v-u) 2 (o-u)
dy = _ 3110 dx +~ (u + v) dy.
2 2
= 6uv (u-v) dx+3 (v -u )
2(v-u) 2
Whence
az az 3
ax= -3uv, ay =2 (u + o).

Second method. From the third given equation we can find


az - 3 2 ~ + 3 2 av . az = 3 2 au + 3 2 av (5)
ax - u ax v ax • ay u ay v ay .
Differentiate the first two equations first with respect to x and then with
respect to y:
1 =au+ iJv
ax ax·
0 =au+~
ay ay •
au
{ 0=2u- i)v au av
ax +2v-
ax. l=2u -+2v-.
ay ay
From the first system we find
au v iJv u
ax=v-u. ax=u-v
From the second system we find
au 1 av
ay 2(u-v)' ay 2(v-u)'

Substituting the expressions ~and ~; into formula (5), we obtain

az v u
-ax =3u2 --+3v
v-u
2
--=
u-v
-3uv,
az _ 2 1
ay- 3u 2(u-v)
+ 3v 2
'2(v-u)
1 3 (
2 u
+ v).
Sec. 9] Differentiation of Impltcit Functions 209

1941. Let y be a function of x defined by the equation


x• y•
az+b2=L
. dy d 2y d3 y
Fmd dx' dx" and dxS •
1942. y is a function defined by the equation
x"+y"+2axy=0 (a>l).
Show that ~~ = 0 and explain the result obtained.
1943. Find it if y = 1+ yx.
dy d2y
1944. Find dx and dx" if y=x+lny.

1945. Find (it:).=, and (~~)x=, if


x"-2xy +y"+x+y-2 =0.
Taking advantage of the results obtained, show approximately
the portions of the given curve in the neighbourhood of the point
X= l.
1946. The function y is defined by the equation
In Vx• + y• =a arc tan .JL (a'i=O).X
2
. dy dy
Ftnd dX and dx" .

1947. Find ~~ and


1 +xy-ln (exy +e -xy) = 0.
1948. The function z of the variables x and y is defined by
the equation
X
3
+ 2y + Z3 3
- 3xyz-- 2y+ 3 =0.
. iJz iJz
Fmd ax and ay.
1949. Find ~ and az if
ax ay
X COS y -!- y COS Z -1- Z COS X = 1.
1950. The function z is defined by the equation
x"+ y•- z•-xy =0.

Fllld ax
i)z
and
az
au for the system of values X=-1, y=O, Z=l.
210 Functions of Several Variables [Ch. 6

ax ay az
1952. f(x, y, z)=O. Show that ay---=-1.
az ax
1953. z = cp (x, y), where y is a function of x defined by the
equation 'ljl{x, y)=O. Find::.
1954. Find dz and d 2 z, if
x• + y• + z• = a•.
1955. z is a function of the variables x and y defined by the
equation
2x
2
+ 2y + z
2 2
-- 8xz- z + 8 = 0 •
Find dz and d z for the values x = 2, y = 0, z = l.
2

1956. Find dz and d 2 z, if In z =x+y +z-1. What are the


first- and second-order derivatives of the function z?
19:>7. Let the function z be defined by the equation
x 2 + y 2 + Z2 = cp (ax+ by + cz),
where cp is an arbitrary differentiable function and a, b, c are
constants. Show that
az az
(cy- bz) ax+ (az-cx) Jii = bx-ay.
1958. Show that the function z defined by the equation
F(x-az, y-bz)=O,
where F is an arbitrary differentiable function of two arguments,
satisfies the equation
a az+b~-I
ax ay- .

1959. F (;, ; ) =0. Show that x~: +u~=z.


1960. Show that the function 2 defined by the equation
y = xcp (z) + 'ljl (z) satisfies the equation
a z (az)•- 2 ~ az ~+ a z(az)a _0
2 2

(1x2 ay ax iJy ax iJy ay 2 ax - .


1961. The functions y and z of the independent variable x are
defined by a system of equations x• + y 2 -z• =~ 0, x• + 2y 2 + 3z 2 = 4.
. dy dz d1 y d 2z
Fmd iiX, dx, dx•, dx• for X= 1, y=O, 2= 1.
Sec. 10] Change of Variables 211

1962. The functions y and z of the independent variable x are


defined by the following system of equations:
xyz=a, x+y+z= b.
2 2
Find dy, dz, d !J, d z.
1963. The functions u and v of the independent variables x and y
arc defined implicitly by the system of equations
U=X-t-y, UV=!J.
Calculate
au au a2u iJ 2 u a2u av av a2v a2v a2v
~·~·~2·~~·ay2•~·8y·a~u~ay•~2

for x=O, y=l.


1964. The functions u and v of the independent variables x
and y are defined implicitly by the system of equations
u-t-v=x, u-yv=O.
Find du, dv, d2 u, d 2 v.
1965. The functions u and v of the variables x and y are
defined implicitly by the system of equations
x = <p (u, v), y = 'ljJ (u, v).
p· d au au av av
In iJX ' Fy ' ax ' iJy •
1966. a) Find :: and ~, if x = u cos v, y = u sin v, z = cv.
~ and adzy , if x = u + v, y = u -v, z = uv.
b) Find vX
c) Find dz, if X=e"+v, y=e"-v, Z=UV.
1967. z = F (r, <p) where r and <pare functions of the variables
x and y defined by the system of equations
x = r cos <p, y = r sin <p.
. az az
Ftnd ax and ay.
1968. Regarding z as a function of x andy, find~ and ~, if
X=aCOS<pCOS'IjJ, !J=b sin<pCOS'\jJ, Z=CSin'ljJ.

Sec. 10. Change of Variables


When changing variables in differential expre$sions, the derivatives in
them should be expressed in terms of other derivatives by the rules of differ·
entiation of a composite function.
212 Functions of Several Variables [Ch. 6

1°. Change of variables In expressions containing ordinary derivatives.


Example 1. Transform the equation
2 d2y dy az -
x -dX
z+2xd-+zY-0
X X
1
putting x=T.
Solution. Express the derivatives of y with respect to x in terms of the
denvatives of y with respect to t. We have
dy dy
dy = dt = dt = -t2 dy
dx dx 1 dt '
dt -12

Substituting the expressions of the derivatives just found into the given
e:juation and replacing x by+, we get
2
1 1•( 2 dy
72' dt+t
d y)
dt 2 + 2 'TI ( - t•dy)
dt +a•t2 y~-O
or

Example 2. Transform the equation


2
d y (dy)' dy
x dx 2 + .dx -ax=O,
taking y for the argument and x for the function.
Solution. Express the derivatives of y with respect to x in tenns of the
derivatives of x with respect to y.
dy 1
ax= dx;
dy
d2x d 2x
2 2 2

ax=- (:;r. :; = - (:;r.


d y d ( I ) d ( I ) dy dy I dy
dx
2
=ax ~ = dy :;

l
Substituting these expressions of the derivatives into the given equation, we
will have

xl
::~ +(dx)s-
I -(t!!.)s I I
~ =O,
L dy dy dy
Sec. 10] Change of Variables 213

or, finally,

X
d'x
dy2- 1 + dy
(dx)2 = 0.

Example 3. Transform the equation


dy x+y
dx x-y'
by passing to the polar coor<iinates
x= rcos <p, y= rsin <p. (1)

Solution. Considering r as a function of <p, from formula (1) we have

dx =cos <p dr -r sin <p d<p, dy "=sin <p dr + r cos rp d<p,


whence
. dr
<p-d -1 r cos <p
sin <p dr + r cos <p d<p
Sill
dy tp
dx =cos <p dr- r Sill (jl drp dr .
cos <p d<p- r su1 ljl

Putting into the given equation the expresswns for x, y, and~, we will have
sin <p ~ + r cos <p r cos <p + r sin <p
dr . r cos (jl- r sm <p '
cos rp- - - r sm <p
dcp
or, after sJmpliHcations,
dr
dcp = r.

2° Change of variables in expressions containing partial derivatives.


Example 4. Take the equation of oscillations of a string

(a f: 0)

and change it to the new independent variables a. and ~. where a.= x-at,
~=x+at.
Solution. Let us express the partial derivatives of tt with respect tox and t
in terms of the partial derivatives of u with respect to a. and ~- Applying
the formulas for differentiating a compos1te function
au aa aa au a~ au au aa au a~
at = aa at +a~ at ' Ji: = aa. ax+ a~ ax ·
Wl' get
214 Functions of Several Variables [Ch. 6

Example 5. Transform the equation x 2 i)az


X
+u aazy =z
2 2
, takin~ ll=X, 'l.!=

=_!_ _ _!_for the new independent variables, and w = _!_ _ _!_ for the new
II X Z X
functton.
02 ·
. l d enva
S o Iution. L e t us express tl1e par t 1a ·
· t 1ves ax
iJz anu" ay 111 t erms o f th e

part1al deri,vatives ~: and ~~. To do this, differentiate the given relation-


slups between the old and new variables:
dx dz
du=dx, dw
- xz -
- - -za.

On the other hand,


aw aw
dw =au du + iJv dv.
Therefore,
aw aw dx dz
-du+-dv=---
iJu QV X2 zZ
or

Whence
2
I -ow
dz=z 2 ( - I iJw) z iJw
dx+--dy
2x
--- 2 -
iJu y 2 iJv
x iJv
and, consequently,
Sec. 101 Change of Variables 21[)

and
az z1 iJw
ay=!f av ·
Substituting these expressions into the given equation, we get
x2zZ (..!.- aw-..!. iJw) +z2 av
x• au x av 1
aw = zl
or
aw
au =0.
1969. Transform the equation
x 2 d•y1 +2x~ +y=O
dx dx '
putting x = e1•
1970. Transform the equation
2
( l-x2 )dd>.y-xr!JI.=o
2
dx '
putting x =cost.

X
Ft~ 69

1971 Transform the following equations, taking y as the ar-


gument:
a) :~+2Y (~r =0,
b)~ d3y-
3
3
dx dx
(d•y)z
1
dx
= 0.

1972. The tangent of the angle 1.1. formed by the tangent line
MT and the radius vector OM of the point of tangency (Fig. 69)
is expressed as follows:
y--xy
I

tan t.t=---
1 + 1Ly'
X
216 Functions of Several Variables [Ch. 6

Transform this expression by passing to polar coordinates:


x=rcosq>, y=rsinq>.
1973. Express, in the polar coordinates x=rcosq>, y=rsinq>,
the formula of the curvature of the curve
y"
K= [I+ (y')2J':, ·
1974. Transform the following equation to new independent
variables u and v:
iJz iJz
y--x-=0
iJx iJy '
if U=X, V=X 1 +y2 •
t 975. Transform the following equation to new independent
variables u and v:
i)z iJz
x-+y--z=O
iJx iJy '
·r
l ll=X, V=x· Y
1976. Transform the Laplace equation
iJ2u iJ2a
iJx2 + iJy2 = 0
to the polar coordinates
x = r cos q>, y = r sin q>.
1977. Transform the equation
I i)Zz 2 i)Zz
X axz-Y i)y2 =0,
X
putting u =XY and v =-. y
t 978. Transform the equation
iJz i)z
y iJx -xay = (y-x) z,
by introducing new independent variables
I I
U=X 1 +y 2 , V=-+-
x y
and the new function W= In z-(x+y).
1979. Transform the equation
a•z iJ 2z i) 2z
iJx2-2iJx iJy + ayz =0,
taking u=x+y, v=.!L for the new independent variables and
Z X
W= X: for the new function.
Sec. 11] The Tangent Plane and the Normal to a Surface 217

t 980. Transform the equation


i)Zz
iJXZ
+ 2 iJX iJy -f- iJy 2 -
i)Zz i)Zz - 0
1

putting u=x+y, v=x-y, w=xy-z, where w=w(u,v).

Sec. 11. The Tangent Plane and the Normal to a Surface


I o. The equations of a tangent plane and a normal for the case of explic-
It representation of a surface. The tangent plane to a surface at a point M
(point of tangency) is a plane in which lie all the tangents at the point M to
various curves drawn on the surface through this point.
The normal to the surfacP is the perpendicular to the tangent plane at the
point of tangency
If the equation of a surface, in a rectangular coordinate system, IS gi\<en
in explicit form, z=f (x, y), wht>re f (x, y) is a differentiable function, then
the equation of the tangent plane at the point M (x 0 , y0 , z0 ) of the surface is

Z- Z0 = t.: (X 0, Yo) (X- X0) + f;, (Xo, Yo) (Y- Yo)· (I)

Here, z0 =f (x 0 , y0 ) and X, Y, Z are tht> current coordinates of the point of


the tangent plane.
The equations of the normal are of the form
X-x 0 Y -Yo l-Z0
- ,- - - = - - 1 . (2)
f.~ (Xo, Yo) fy (Xo, Yo) -

where X, Y, Z are the current coordinates of the point of the normal.


Example I. Write the equations of the tangent plane and the normal to
x•
thl' surface z= -y• at the point M (2, -1,1).
2
Solution. Let us lind the partial derivatives of the g1ven function and
their values at the point M
iJz
iJx = x,
az
-=-')11
cJy -.'

Whence, applying formulas (I) and (2), we will have z-1 = 2 (x-2) + 2 (y +I)
x-2
or 2x-j-2y-z-l =0 which is the equation of the tangent plane and - -=
2
=y t 1
=z /, which is the equation of the normal.
2°. Equations of the tangent plane and the normal for the case of implic-
it representation of a surface. When the equation of a surface is represented
implicitly,
F (x, y, z)=O,
and F (x 0 , y0 , z0) = 0, the corresponding equations will have the form

F~ (Xo, Yo• Zo) (X -Xo) + F; (Xo, Yo• Zo) (Y -y0 ) + F~ (xo, Yo• Zo) (l -lo) = 0 (3)
218 Functions of Several Variables (Ch. 6

which is the equation of the tangent plane, and


X-~ Y-~ Z-~
(4)
F~ (x 0 , y0 , z0 ) F~ (x 0 , y0 , Z0 ) F~ (x 0 , y0 , Z0)

which are the equations of the normal.


Example 2. Write the equations of the tangent plane and the normal to
the surface 3xyz-z1 =a1 at a point for which x=O, y=a.
Solution. Find the z-coordinate of the point of tangency, putting X=O,
u=a into the equation of the surface: -z1 =a1 , whence z= -a. Thus, the
point of tangency is M (0, a, -a).
Denoting by F (x, y, z) the left-hand side of the equation, we find the
partial derivatives and their values at the point M:
F~=3yz, (F~)M= -3a1 ,
F;= 3xz, (f~)M=O,
f~=3xy-3z 1 , (i~)M = -3a1 •
Applying formulas (3) and (4), we get
-3a 2 (x-0) + 0 (y-a)-3a 2 (z +a) =0
or x+z+a=O, which is the equation of thP tangent plane,
x-0 y-a z+a
-3a 2 = -o-= -3a 2
or -f=y~a=z;a. which are the equations of the normal.

1981. Write the equation of the tangent plane and the equa·
tions of the normal to the following surfaces at the indicated
points:
a)to·the paraboloid of revolution z=x 2 +y' at the point
(1. -2,5);
x2 y2 z2
b) to the cone 16 +-g- 8 =0 at the point (4, 3, 4);
c) to the sphere x• + y• + z• = 2Rz at the point (R cos a,
R sin a, R).
1982. At what point of the ellipsoid
x2 u2 z2
a2 + 71 +Ci = 1
does the normal to it form equal angles with the coordinate axes?
1983. Planes perpendicular to the x- and y-axes are drawn
through the point M (3, 4, 12) of the sphere r + !/ + z• = 169.
Write the equation of the plane passing through the tangents to
the obtained sections at their common point M.
1984. Show that the equation of the tangent plane to the
central surface (of order two)
ax• +by2 +cz2 =k
Sec. 1/] The Tangent Plane and the Normal to a Surface 219

at the point M (x 0 , y 0 , Z 0 ) has the form


ax 0 x+ by 0 y + cz 0 z = k.
1985. Draw to the surface x• + 2!/ + 3z• = 21 tangent planes
parallel to the plane x + 4y + 6z = 0.
x• y• z2
1986. Draw to the ellipsoid a•+ b!+ c• = l a tangent plane
which cuts off equal segments on the coordinate axes.
1987. On the surface x• + y• --z•- 2x = 0 find points at which
the tangent planes are parallel to the coordinate planes.
1988. Prove that the tangent planes to the surface xyz = m1
form a tetrahedron of constant volume with the planes of the
coordinates.
1989. Show that the tangent planes to the surface Vx + ~~y +
-1 V z= Va cut off, on the coordinate axes, segments whose sum
is constant.
x• y2 z1
1990. Show that the cone 2a +-bz=--z c
and the sphere

are tangent at the points (0, + b, c).


1991. The angle between the tangent planes drawn to given
surfaces at a point under consideration is called the angle between
two surfaces at the point of their intersection.
At what angle does the cylinder x• + y• = R2 and the sphere
(x-R) 2 :·t/-\-Z2 =--=R 2 intersect at the point M(~. R ~3, o)?
1992. Surfaces are called orthogonal if they intersect at right
angles at each point of the line of their intersection.
Show that the surfaces x• + !l + z• = r• (sphere), y = x tan rp
(plane), and z• =--= (x1 + !/) tan 2 'Ill (cone), which are the coordinate
surfaces of the spherical coordinates r, <p, 'llJ, are mutually ortho-
gonal.
1993. Show that all the planes tangent 1o the conical surface
z "'~ xf ~I 7
11 ) •
at the potnt M (x 0 , y 0 , Z 0 ), where X 0 'F 0, pass through
the coordinate origin.
1994*. Find the projections of the ellipsoid
x1 + y• + z•-xy-l =0
on the coordinate planes.
1995. Prove that the normal at any point of the surface of
revolution z=f<V x1 -f-Y1 } (f' +0} intersect the axis of rotation.
220 Functions of Several Variables [Ch. 6

Sec. 12. Taylor's Formula for a Function of Several Variables


Let a function f (x, y) have continuous partial derivatives of all orders
up to the (n+ 1) th inclusive in the neighbourhood of a point (a, b). Then
Taylor's formula will hold in the neighbourhood under consideration:
l , ,
j(x, y)=f(a, b)+TI lfx(a, b)(x-a)+fy(a, b)(y-b)]-t

+ 211 lfxx(a,
II 2 ,
b)(x-a) +2fxl/(a, b)(x-a)(y-b)+fuu<a,b)(y-b) ]-!- ...
II 2

... +iii
1 [ i)
(x-a>ax+<u-b>ay
i)]n {(a, b)+R,.(x, y), (I)
where
Rn (x, y)= (n ~I)! [ (x-a) :x + (y-b) :U] 'l+' f [a+ 0 (x-a), b-t 0 (y-b)J
(0 <0< 1).
In other notation,
, l , I •
f (x-+ h, Y-+ k)=f (x, y)+11 [hfx (x, y)-tkf/1 (x, y)]-t
21
[h 2 fu (x, y) -1-
,, 2 •
+2hkfxy (x, y)+k fyy (x, y)]
1 [ a a] 11 f (x, Yl-1-
+ ... +Til
h iJx k iJy +
+(n~l)! [h:x+ka~r+' t<x-1-I:Jh; y-+Ok), (2)
or
llf (x, y)= :, df (x, y)-t ~ d"f (x, y) + ...
.. . + :! dnf (x, y) + (n; I) I dn Hf (x-1- Oh; y -f- Ok) (:))

The particular case of formula (1), when a=b=O, is called Maclaunn's


formula.
Similar formulas hold for functions of three and a larger number of
variables.
Example.' Find the increment obtained by the funct10n f(x, y)=A'-
-2y'+3xy when passing from the valuesx=l, y=2 to the valuesx,-d-1-h,
y,=2+k.
Solution. The desired increment may be found by applying formula (2).
First calculate the successive partial derivatives and their values at the
given point (1, 2):
f~ (x, y) = 3x2 + 3y, r:o. 2)=3·1+3·2=9.
f~ (x, Y)= -6y 2 +3x, f~O. 2)=-6·4-1-3·1=-21,
r:x (x, y) = 6x, t:A (I ' 2) = 6. 1 = 6,
r:y (x, y) = 3, t:uo. 2) = 3,
f'uu (x y) = -12y, f~y(l, 2)= -12·2= -2"1,
t;;;,, (x, y) = 6, ~~~x(l, 2)=6,
fxxy (x, y) = 0, t;:~(l. 2)=0,
f~~Y (x, y)=O, t:;;u< I, 2) = 0,
t~;Y (x, y) = - 12, f~~y(l' 2) = -12.
Sec. 12] Taylor's Formula for a Function of Several Vartables 221

All subsequent derivatives are identically zero. Putting these results


into formula 12), we obtain: ·
llf(x, y)=f(l+h. 2+k)-f(l, 2)= ;, [h·9+k{-21)]+

+_!_
2!
W·6 + 2hk·3 + k 2 { -24)] + _!__
3!
W·G +3h 2 k·O+ 3hk 2 ·0+ k 3 ( -12)] =
=9h-2lk + 3h +3hk-12k +h"-2k 1 •
2 2

1996. Expand f (x + h, y + k) in a series of positive integral


powers of h and k if
f (x, y) = ax• -1- 2bxy + cy•.
the function f(x,y)=-x 2 +2xy+3y 2 -6x-
1997. Expand
- 2y-4 by Taylor's formula in the neighbourhood of the point
(-- 2, 1).
1998. Find the increment received by the function f (x, y) =
=-x"y when passing from the values x=1, y=1 to
x, ~""""' 1-1 lz, y, = 1 +k.
1999. Expand the function f(x, y, z)=x"-t-y•-t-z"+2xy-yz-
-- 4x- 3y-z + 4 by Taylor's formula in the neighbourhood of
the point (1, 1, 1).
2000. Expand f(x i h, y-J k, z-1 /) in a series of positive in-
tegral powers of h, k, and /, if
f (x, y, z) ~-= x• ; y• -1- z• -2xy-2xz-2yz.
2001. Expand the following function in a Maclaurin's series
up to terms of the thml order inclusive:
f (x, y) = e" sin y.
2002. Expand the following function in a Maclaurin's series
up to terms of order four inclusive:
f (x, y) =cos x cosy.

2003. Expand the following function in a Taylor's series in


the neighbourhood of the point (l, 1) up to terms of order two
inc Iusi ve:
f (x, y) = y·~.

2004. Expand the following function in a Taylor's senes in


the neighbourhood of the point (1, -1) up to terms of order
three inclusive:
f (x, y) = e"+Y,
222 Functions of Several Variables (Ch. 6

2005. Derive approximate formulas (accurate to second-order


terms in a and ~) for the expressions
l+u. b) ,/(l+a.)m+(l+~)''
a) arc t an I-~· V 2 ,

if 1a 1and I~ I are small compared with unity.


2006*. Using Taylor's formulas up to second-order terms,
approximate
a) Vl.o3; Vo.9s; b) (0.95)2.01.
2007. z is an implicit function of x and y defined by the
equation z'-2xz+y=0, which takes on the value Z= 1 for X= 1
and y = 1. Write sever a I terms of the expansion of the function
z in increasing powers of the differences x--1 and y-1.

Sec. 13. The Extremum of a Function of Several Variables


0
\ • Definition of an extremum of a function. We say that a function
f (x,
y) has a maximum (mint mum) f (a, b) at the point P (a, b), !I for a II
points P' (x, y) different from P in a sufflci('ntly small neighbourhood of P
the inequality f(a, b)> f(x, y) (or, accordingly, {(a, b)< f(x, y)] is fulfilled.
The generic term for maximum and minimum of a function is extremum.
In similar fashion we define the extremum of a function of three or more
vanables.
2°. Necessary conditions for an extremum. The points at which a chffe-
rentiable function f (x, y) may attain an extremum (so-called stattonary points)
are found by solving the following system of equat10ns:
I~ (x, y) =0, 1;, (x, y) --0 (I)

(necessary ccmditions for an extremum). System (I) is equivalent to a s1ngle


equation, df (x, y) = 0. In the general case, at the point of the extremum
P (a, b). the function f (x, y), or df (a, b)= 0, or df (a, b) does not exist.
3°. Sufficient conditions for an extremum. Let P (a, b) be a stationary
point of the function f (x, y), that is, df (a, b)-~ 0. Then: a) if d2f (a, b) < 0
+
for dx 2 dy 2 > 0, then f (a, b) is the maximum of the function f (x, y); b) if
+
d 21 (a, b) > 0 for dx 2 dy 2 > 0, then f (a, b) is the rnintmum of the functiOn
f (x, y); c) if d 21(a, b) changes sign, then f (a, b) 1s not an extremum of I (X", y).
The foregoing conditions are equivalent to the following: let f~ (a, b),~
=I~ (a, b)= 0 and A= f:x (a, b), 8 =--= 1:y (a, b), C =c f~y (a, b). We form the
dtscrimlllant

Then: 1) if ~ > 0, then the function has an extremum at the point


P (a, b), namely a maximum, if A< 0 (or C < 0), and a minimum, 1f A> 0
(or C > 0); 2) if ~ < 0, then there is no extremum at P (a, b); 3) if ~ == 0,
then the question of an extremum of the function at P (a, b) remains open
(which is to say, it requires further investigation).
4°. The case of a function of many variables. For a function of three or
more variables, the necessary conditions for the existence of an extremum
Sec. 13] The Extremum of a Function of Several Variables 223

are similar to conditions (1), while the sufficient conditions are analogous to
the conditions a), b), and c) 3°.
Example 1. Test the following function for an extremum:
z=x'+ 3xy1 -l5x-12y.
Solution. find the partial derivatives and form a system of equations (1):
az 3x + 3y
ax= 2 2
-
i)z
15 = 0; dy = 6xy- 12 = 0
or
x2 -t-y2 -5=0,
{ xy-2=0.
Solving the system we get four stationary points:
P 1 (1, 2); P 1 (2, I); P1 (-l,-2); P 4 (-2,-l).
Let us find t;Je second derivatives
iPz
axz=6x,
and form the discriminant ft.= AC- 8 2 for each stationary point.
A=(~:~)P,=6, 8=( 0~ ;Y)P,=12, C=(~~)P,=
2

1) For the po111t P 1:


=6, ft.=AC-8 2
=36-144 < 0. Thus, there is no extremum at the pomt P 1•
2) For the point P 2 : A-= 12, B =6, C=
12; ft.= 144-36 > 0, A> 0. At P 1
thP function has a minimum. Th1s minimum 1s equal to the valut> of the
function for A--2, Y"'' 1·
Zmin -o8 +G-30-12= -28.

3) For the point P,: A ~c -6, B = -12, C ~= -6; ft.= 36-144 < 0. There
i'6 no extremum.
4) For the point P,: A-~ -12, B= -6, C = -12; ft.= 144-36 > 0, A< U.
At the point P 4 the function has a maximum equal to Zmax = -8-6 + 30 +
+ 12-~28
s~. Conditional extremum. In the simplest case, the conditio11al extremum
of a function f(-l, y) IS a maximum or minimum of this function which IS
attained on the condition that its ar!-!uments are related by the equation
'P (x, y> = 0 (coupllllf! equal ton). To find the conditiOnal extr£-mum of a func-
tion (x, y), given the relationship cr (x, y) =(I we form the SO·Called Lagra,zge
fu net ion
F (x, y) ~ f (x, y) +A.·cp (x, y),
where A. is an undetermmed multiplkr, and we seek the ordinary extremum
of this auxiliary function. The neces~ary conditions for the extremum reduce
to a system of three equal ions:
iJF =::; iJf +A. iJcp =O
iJx ax ax •
iJF--= ~+A. iJljl =O (2)
{ dy -- iJy iJy •
cp (x, y) =0
with three unknowns x, y, J.., from which it is, generally speaking, possible
to determine these unknowns.
224 Functions of Several Variables [Ch. 6

The question of the existence and character of a conditional extremum is


solved on the basis of a study of the sign of the second differential of the
Lagrange function:
2
aap 1 azp arp 2
d F(x, y)= axz dx -t-2axaydxdy+ ayr dy

for the given system of values of x, y, A. obtained from (2) or the condition
that dx and dy are related by the equation

~: dx-j- ~: dy=O (dx 2 -t-dy 2 =F 0).

Namely, the funchon f (x, y) has a conditional maximum, if d'F < 0, and a
conditional minimum, if d 2F > 0. As a particular case, if the discriminant t1
of the function F (x, y) at a stationary point is positive, then at this point
there is a conditional maximum of the function f (x, y), if A < 0 (or C < 0),
and a conditional minimum, if A> 0 (or C > 0)
In similar fashion we find the conditional extremum of a function of
three or more variables provided there is one or several coupling equations
(the number of which, however, must be less than the number of the variables)
Here, we have to introduce into the Lagrange function as many undetermmed
multipliers factors as there are coupling equations.
Example 2. Find the extremum of the function
z=6-4x-3y
provided the variables x and y satisfy the equation
x2-j-y2= l

Solution. Geometrically, the problem reduces to finding the greatest and


least values of the z-coordinate of the plane z=6-4x-3y for points of its
intersection with the cylinder .t 2 +y 2 =l
We form the Lagrange function
F (x, y) = 6- 4x-3y +A. (x 2 + y2 -l).

We have:.~ =-4+21.x, ~~=-3+2f.y. The necessary conditions yield the


following system of rquatwns:

Solving this system we find


5 3
"-,=2· y,=5·
and

Since

It follows tha~
Sec. 13] The Extremum of a Function of Ser•eral Variables 225

5
If t..= 2 , x=s4 and
3
y=s, then d2 F > 0, and, consequenHy, the function

has a conditional minimum at this point. If!..=- i, x=-: andy=-~,


then d"F < 0, and, consequently, the function at this point has a conditional
maximum.
Thus,

6'. Greatest and smallest values of a function. A function that is diffe·


rentiable in a limited closed region attains
its greatest (smallest) value either at a sta-
tionary point or at a point of the boundary
of the reg10n.
Example 3. Determine the greatest and
smallest values of the function
z = x• + y 2 - xy + x + y
in the reg10n
x,;;;;O, y ,;;;;0, x+y;::::- 3
Solution. The indicated region is a tri·
angle (Fig. 70).
1) Let us find the stationary po111ts:

f z't""" 2x-y 1· I =0,


\ z~ = 2y-x+ I =0;
whence x-=- 1, II·=- I; and we get the point M (-1, -1)
At M the value of the function z111 =-l It is not absolutely necessary
to test for an extremum
:2) Let us inves!tgate the function on the boundaries of the region.
+
When x- 0 we have z = y 2 y, and the problem reduces to seeking the
greatest and smallest values of thts function of one argument on thr interval
-3.-;;;;;y,;:;;O. Investigating, we lind that (Zgr)x=o=6 at the point (0, -3);
(z8 m)~=·=-+ at the po111t (0, - 1
/ 2)
2
When y::..O we get Z=X -!-X. Similarly, we find that (Zgrlv=o=6 at the
point (-3, 0); (Zsmly=o=- i at the point (- 1/ 2 , 0)
When x-f-y=-3 or y=-3-x we will have Z=3x 2 +9x+6. Similarly
3
we find that (lsmlx+v=-•=-4 a t th e potnt
. 3
(\ -2· :~ ) ; (Z~tr>x+v=-•=6
-2
metres coincides with (Zgrlx=• and (lgr)~=·· On the straight line x y = - 3 +
we could test the function for a coni:litional extremum without reducing to
a function of one argument.
3) Correlating all the values obtained of the function z, we conclude
that z11 r=6 at the points (0, -3) and (-3, 0); Zsm=-1 at the stationary
point M.

8-1900
226 Functions of Several Variables [Ch. 6

Test for maximum and minimum the following functions of


two variables:
2008. Z= (x-})1 +2y1 •
2009. z = (x-1} 1 -2y'.
2010. z=x1 +xy+y'-2x-y.
2011. z=x'y 2 (6-x-y)(x>0, y>O).
2012. z = X 4 + y4 -2x• +4xy-2y' .
.. / x• y•
2013. Z=XY V 1-(jf"-1)2 •
2014. Z= 1-(x1 -f-y2 )'ia.
2015. Z=(x'+y•)e-cx'+Y•J.
2016. z = 1+x-y .
Yl +x1 +Y2
Find the extrema of the following functions of three variables:
2017. U=x•+y• +z•-xy+x-2z.
y2 z2 2
2018. u=x+-
4X
+-+-(x>O,
y Z
y>O, z>O).
Find the extrema of the following implicitly represented func-
tions:
2019*. x• + y• + z'-2x+ 4y-6z-11 = 0.
2020. x'-y 1 --3x+ 4y + z• + z-8=0.
Determine the conditional extrema of the following functions:
2021. z=xy for x+y= 1.
2022. Z-= x +2y for x• + y• = 5.
2023. z = x• + y• for i + ~ = 1.
2024. z=cos•x+cos•y for y-x=: .
2025. u=x-2y+2z for x• + y• + z• = 9.
x• y• z2
2026. u = x• + y• + z• for £i2-l-fi2+cz= 1 (a>b >c>O).
2027. U=xy'z 8
for x+y+z= 12(x>0,y>0, z>O).
2028. U=XYZ provided x+y+z=5, xy+yz+zx=B.
2029. Prove the inequality
x+y+z
3 ;;;;:.
v--
xyz,
if x;;;;;:.O, y~O, z ~0.
Hint: Seek the maximum of the function u=xyz provided x+u+z=S.
Sec. 141 Finding the Greatest and Smallest Values of Functions 227

2030. Determine the greatest value of the function z = 1 +x+ 2y


in the regions: a) x~O. y~O. x+y~ 1; b) x~O. y~O.
x-y~ 1.
2031. Determine the greatest and smallest values of the func-
tions a) Z=X 1Y and b) Z=X 1 -Y1 in the region x'+y ~1. 1

2032. Determine the greatest and smallest values of the func-


tion z=sinx+siny+sin(x+y) in the region O~x~;.
:rt
O~y~2.
2033. Determine the greatest and smallest values of the func-
tion z=x'+y 1 -3xy in the region O~x~2. -1~y~2.

Sec. 14. Finding the Greatest and Smallest Values of Functions


Example 1. It is required to break up a positive number a into three
nonnegative numbers so that their product should be the greatest possible.
Solution. Let the desired numbers be x, y, a-x-y. We seek the maxi-
mum of the function f (x, y) =xy (a-x-y).
According to the problem, the function f (x, y) is considered inside a
closed triangle x~O. y~O. x+y~a (Fig. 71).

Fig. 71

Solving the system of equations

f~(x, y)==y(a-2x-y)=0,
{ fu (x, y)=ax(a-x-2y)=0,

we will have the unique stationary point ( ~ , ~) for the intel'lor of the
triangle. Let us test the sufficiency conditions. We have

fn(x, Y)=-2y, t:y(x, Y)=a-2x-2y, t;y(x, y)=-2x.


a•
228 Functions of Several Variables [Ch. 6

Consequently,
A= t:x ( ~ , ~ ) =- { a,

B = t:u (~ , i-) = - ~ a,
C=f~v(;, ~)=-fa and
£\ = AC- 8 2
> 0, A < 0.
And so at ( ~ . ~ ) the function reaches a maximum. Since f (x, y) = 0 on
the contour of the triangle, this maximum will be the greatest Vdlue, which
is to say that the product will be greatest, if x=y=a-x-y=!!_ , and the
aa 3
greatest value is equal to 'if.
Note The ploblem ran also be solved by the methods of a conditional
extremum, by seeking the maximum of the function u=xyz on the condition
that x+y+z=a.

2034. From among all rectangular parallelepipeds with a


given volume V, find the one whose total surface is the least.
2035. For what dimensions does an open rectangular bathtub
of a given capacity V have the smallest surface?
2036. Of all triangles of a given perimeter 2p, find the one
that has the greatest area.
2037. Find a rectangular parallelepiped of a given surface S
with greatest volume.
2038. Represent a positive number a in the form of a product of
four positive factors which have the least possible sum.
2039. Fi11d a point M (x, y), on an xy-plane, the sum of
the squares of the distances of which from three straight lines
(x=O, y=O, x-y+ 1 =0) is the least possible.
2040. Find a triangle of a given perimeter 2p, which, upon
being revolved about one of its sides, generates a solid of
greatest volume.
2041. Given in a plane are three material points P 1 (x" !/ 1 ),
P 2 (X 2 , !/ 2 ), Pa (X 3 , !/ 3 ) with masses mt> m 2 , m 3 • For what position
of the point P (x, y) will the quadratic moment (the moment nf
inertia) of the given system of points relative to the point P
(i.e., the sum m 1 P 1 P 2 + m 2 P 2 P 2 + m 3 P 3 P 2 ) be the least?
2042. Draw a plane through the point M (a, b, c) to form
a tetrahedron of least volume with the planes of the coordinates.
2043. Inscribe in an ellipsoid a rectangular parallelepiped of
greatest volume.
2044. Determine the outer dimensions of an open box with a
given wall thickness 6 and capacity (internal) V so that the
smallest quantity of material is used to make it.
Sec. 141 Finding the Greattst and Smallest Values of Functions 229

2045. At what point of the ellipse


xz y2
7+v= 1
does the tangent line to it form with the coordinate axes a tri-
angle of smallest area?
2046*. Find the axes of the ellipse
5x 1 + Bxy+ 5y 2
= 9.
2047. Inscribe in a given sphere a cylinder having the
greatest total surface.
2048. The beds of two rivers (in a certain region) approxi-
mately represent a parabola y=X 2 and a straight line x-y-2=0.
It is required to connect these rivers by a straight canal of least
length. Through what points will it pass?
2049. Find the shortest distance from the point M (l, 2, 3)
to the straight line
X y Z
T= -3=2 ·
2050*. The points A and B are situated in different optical
medta separated by a straight line (Fig. Z2). The \"elocity of
A

I c
/3 I
Fu~. 7'2 Fig. i3

light in the first medium is ua> in the second, U 2 • Applying the


Fermat principle, according to which a light ray is propagated
along a line AMB which requires the least time to cover, derive
the law of refraction of light rays.
2051. Using the fermat principle, derive the law of reflection
of a light ray from a plane in a homogeneous medium (Fig. 73).
2052*. If a current I llows in an electric circuit containin~ a
resistance R, then the quantity of heat released in unit time is
proportional to J"R. Determine how to divide the current I into
236 Functions of Several Variables (Ch. 6

currents I 1 , I 2 , I • by means of three wires, whose resistances are


R1 , R1 , R 1 , so that the generation of heat would be the least
possible?

Sec. 15. Singular Points of Plane Curves


1°. Definition of a singular point. A point M (x~, y 0) of a plane curve
f(x, y)=O is called a singular point if its coordinates satisfy three equations
at once:
f (xo, Yo)= 0, f~ (xo, Yo)= 0, f~ (X0 , Yo)= 0.
2°. Basic types of singular points. At a singular point M (x 0 , y 0), let the
second derivatives
A= f:x (xo, Yo),
B = t:y (xo, Yo),
C = f;y (xo, Yo)

be not all equal to zero and

then:
a) if 6. > 0, then M is an isolated point (Fig. 74);
b) if 6. < 0, then M is a node (double point) (Ftg. 75);
c) if 6.=0, then M is either a cusp of the first kind (Fig. 76) or of the
second kind (Fig. 77), or an isolated point, or a tacnode (Fig, 78).

Fig. 74 Fig. 75

When solving the problems of this section it is always necessary to draw


the curves.
Example 1. Show that the curve y 1 = ax 2 + x• has a node if a > 0; an
isolated point if a < 0; a cusp of the first kind if a= 0.
Solution. Here, f(x, y)=ax 2 +x'-y 1 • Let us find the partial derivati·
ves and equate them to zero:
f~ (x, y) =2ax +3x1 =0,
f~ (x, y)=- 2y=0.
Sec. M) Singular Points of Plane Curves 231

This system has two solutions: 0 (0, 0) and N ( - : a, 0) : but the


coordinates of the point N do not satisfy th~ equation of the given curve.
Hence, there is a unique singular point 0 (0, 0).
y

Fig. 76 Fig. 77 Fig. 78

Let us find the second derivatives and their values at the pomt 0:

t:x (x, y) = 2a + 6x, A= 2a,


t:u (x, y) = 0, 8 = 0,
t;,,(x, y)=-2, C=-2,
.1=AC-82 =- 4a.

Fig. 79 F1g. 80 Fig. 81

Hence,
if a> 0, then .1 < 0 and the point 0 is a node (Fig. 79);
if a< 0, then A> 0 and 0 is an isolated point (Fig. 80);
if a~-= 0, then .1 =0. The equation of the curve in this case will be
y 2 =x• or y= ± y7; y=exists only when x~O; the curve is symmetric
about the x-ax is, which is a tangent. Hence, the point M is a cusp of the
first kind (Fig. 81 ).
232 Functions of Several Variables (Ch. 6

Determine the character of the singular points of the follo-


wing curves:
2053. y• =-x• +x\
2054. (y-x 1 ) 2 = x•.
205li. a'y2 = a• x'- x 8 •
2056. x•y• -x•-y• =0.
2057. x 8 +y'-3axy=O (folium of Descartes).
2058. y• (a-x) = X3 ( cissoid).
+
2059. (x 2 y 1 ) 1 = a2 {x 2 -y 2 } (lemniscate).
2060. (a+ x) y• =(a-x) x• ( strophoid ).
+
2061. (x• y•) (x-a)• = b•x• (a> 0, b > 0) (conchoid).
Consider three cases:
1) a>b, 2) a=b, 3) a<b.
2062. Determine the change in character of the singular point
of the curve Y2 =(x-a)(x-b)(x-c) depending on the values of
a, b, c(a.s;;;b~c are real).

Sec. 16. Envelope


1°. Definition of an envelope. The envelope of a family of plane curve~
is a curve (or a set of several curves) which is tangent to all linrs of the
given family, and at each point is tangent to some line o! the given family.
2°. Equations of an envelope. If a family of curves

f (x, y, a.) =0

dependent on a single variable parameter a. has an envelope, then till' J,ara-


metric equations of the latter are found from the system of equations
f_(x, y, a)= 0,
(I)
{ fa (x, y, a.)= 0.

Eliminating the parameter a. from the system {1), we get an equation of


the form
0 (x, y) =0. (2)

It should be pointed out that the formally obtained curve (2) (the ~a­
called "discriminant curve") may contain, in addition to an envelope (if
there is one), a locus of singular points of the given family, wh1ch locus 1s
not part of the envelope of this family.
WhPn solving the problems of this section it is adv1sable to make
drawings.
Example. Find the envelope of the family of curves
xcos a+ ys!n a-p =0 (p =const, p > 0).
Sec. 16] Envelope 233

Solution. Thl' given family of curves depends on the parameter a.. Form
the system of equations (1):
xcosa+yslna.-p=O,
{ - x sin a.+ y cos a= 0.
Solving the system for x and y, we obtain parametric equations of the
envelope
x = p cos a, y = p sin a.
Squarmg both equations and adding, we eliminate the parameter a:
xz+ 11 z =p'.
y

l·Jg. 82

Thus, the envetore of this family of straight lines Is a circle of radius p


with centre at the origin. This particular family of straight lines is a family
of tangent lines to th1s circle (Fig. 82).

2063. Find the envelope of the family of circles


I 2 aZ
(x-a) +y =T.
2064. Find the envelope of the family of straight lines
y=kx+fk
(k is a variable parameter).
2065. Find the envelope of a family of circles of the same
radius R whose centres lie on the x-axis.
2066. Find a curve which forms an envelope of a section
of length l when its end-points slide along the coordinate axes.
2067. Find the envelope of a family of straight lines that
form with the coordinate axes a triangle of constant area S.
2068. Find the envelope of ellipses of constant areaS whose
axes of symmetry coincide.
234 Functions of Several Variables (Ch. 6

2069. Investigate the character of the "discriminant curves"


of families of the following lines (C is a constant parameter):
a) cubic parabolas y= (x-C) 1 ;
b) semicubical parabolas if= (x-c)•;
c) Neile parabolas y• = (x-C) 1 ;
d) strophoids (a+x) (y-C) 1 =X2 (a-x).

Fig. 83

2070. The equation of the trajectory of a shell fired from a


point 0 with initial velocity V0 at an angle a to the horizon
(air resistance disregarded) is

Taking the angle a as the parameter, find the envelope of all


trajectories of the shell located in one and the same vertical
plane ("safety parabola") (Fig. 83).

Sec. 17. Arc Length of a Space Curve


Th~ diflerenttal of an arc of a space curve in rectangular Cartesian coor-
dinah•s is equal to

where x, y, z are the current coordinates of a roint of the curve.


If
X=x(t), y=y(t), Z=Z(t)

are parametric equations of the space curve, then the arc length of a section
of it from t=t 1 to t=t 2 is
1
dx )2 ( dy )• ( dz )
2
s= J[
t,
v
, ;(
Cit + ([[ + df dt.
SP.c. 18] The Vector Function of a Scalar Argument 235

In Problems 2071-2076 find the arc length of the curve:


1
21
2071. X= t, y= t,1 Z=3 from 1=0 to 1=2.
3
2072. x = 2 cos t, y = 2 sin t, z =n- t from t=O to i=n.
2073. x = et cos t, y = et sin t, z = et from t = 0 to arbitrary t.
2074. y= 2 ,
x1 x•
z= 6 from x=O to x=6.
2075. x• = 3y, 2xy = 9z from the point 0 (0, 0, 0) toM (3, 3, 2).
2076. y=aarcsin~' Z= a4 In a+x from the point 0(0, 0, 0)
a a-x
to the point M (X 0 , Yo• Z0 ).
2077. The position of a point for any time t(t>O) is defined
by the equations
x=2t, y=lnt, z=t 1 •
Find the mean velocity of motion between times t = 1 and t = 10.

Sec. 18. The Vector Function of a Scalar Argument


) , The derivative of the vector function of a scalar argument. The uector
0

function a= a (t) may be defined by specifying three scalar functions ax (t),


ay (t) and az (t), which are its projections on the coordinate axes:
a=ax (t) I +ay (t)J+az (t) k.
The derivative of the vector function a= a (t) with respect to the scalar
argument t is a new vector function defined by the equality
~ _ lim a (t + ht)-a(t) dax (t) i +day (t) j +daz (t) k
di-!J.t-H i\t dt dt dt •
The modulus of the derivative of the vector function is

I I= ,v
~
2 1
f(dax)• (da_v) daz )
dt dt + dt + (
dt •
The end-point of the variable of the radius vector r=r(t) describes in space
the curve
r=x (t) l+y (t)J+z(t)k,
which is called the hodograph of the vector r.
The derivative !~ is a vector, tangent to the hodograph at the corre-

Idf I=dt"·
sponding point; here,
dr ds

where s is the arc length of the hodograph reckoned from some lnlttal point,
For example, I:~I= 1.
236 Functions of Several Variables (Ch. 6

If the parameter t is the time, then ~= 'll is the velocity vector of the

extremity of the vector r, and ~;~= ~~ ='W is the acceleration vector of the
extremity of the vector r.
2°. Basic rules for differentiating the vector function of a scalar argument.
d da db de
1) dt (a+b-c)=dt+dt-dt;
2) d~ (ma)=m ~~ , where m is a constant scalar;
d dqJ da
3) dt (cpa)=dta+q>df, where q>(l) is a scalar function oft;
d da db
4) (if (ab)="iitb+adf;
d da db
5) Tt (axb)=Ttxb+axdt;
d da dq> .
6) dfa(q>(t)]="'it(p· lit'
7) a~~ =0, if lal=const.

Example 1. The radius vector of a moving point is at any instant of


1ime defined by the equation
r= i-4t'J+3t 2k. (I)

Determine the trajectory of motion, the velocity and acceleration.


Solution. From (I) we have:
X= I, y=-41 2 , Z=31 2 •

Eliminating the time t, we find that the trajectory of motion is a straight


line:
x-l y z
-0-- -4=3·

From equation (I), differentiating, we find the velocity

!!:!_=-8tJ+6tk
dt
and the acceleration

The magnitude of the velocity is

I~~ I= V<- 81)"+ (61) = 10 It/.


2

We note that the acceleration is constant and 1~

J~:~l= V<- 8)'+ 6 2


= to.
Sec. _1--'8]:..__ _ ___:_T_he:..._:__Ve_c_to_r....;.F_;.u....;.n;_ct....:io....;.n....:o:!..f....;.a:....S.:..:c:..:..a.:..:.la;_r....;.A.:..:r.2g.::.:um.:..:.e:.:..:n;_t_ _ _ _--=.23:..:.7

2078. Show that the vector equation r~r 1 = (r 1 - r 1 ) t,


where r 1 and r. are radius vectors of two given points, is the
equation of a straight line.
2079. Determine which lines are hodographs of the following
vector functions:
a) r =at+ c; c) r =a cos t b sin t; +
b) r = at + bt;
1
d) r =a cosh t b sinh t, +
where a, b, and c are constant vectors; the vectors a and b
are perpendicular to each other.
2080. Find the derivative vector-function of the function
a(t)=a(t)a 0 (t), where a(t) is a scalar function, while a 0 (t)
is a unit vector, for cases when the vector a (t) varies: 1) in
length only, 2) in direction only, 3) in length and in direction
(general case). Interpret geometrically the results obtained.
2081. Using the rules of differentiating a vector funct bn with
respect to a scalar argument, derive a formula for differentiating
a mixed product of three vector functions a, b, and c.
2082. Find the derivative, with respect to the parameter t,
of the volume of a parallelepiped constructed on three vectors:
a=i+tj+t•k;
b = 2ti- j+ t'k;
c = - t•i + t'j+ k.
2083. The equation of motion is
roc-= 3i LOS f + 4j sin t,
where t is the time. Determine the trajectory of motion, the
velocity and the acceleration. Construct the trajectory of motion
and the vectors of velocity and acceleration for times, t = 0,
I,_, Jt anu-1 t = Jt .
1 2
2084. The equation of motion is
r=2i cost +2jsin t +3kt.
Determine the trajectory of motion, the velocity and the accel-
eration. What are the magnitudes of velocity and acceleration
and what directions have they for time t = 0 and t = ~ ?
2085. The equation of motion is
r = i cos a cos rot+ j sin a cos rot+ k sin rot,
where a and ol are constants and t is the time. Determine the
trajectory of motion and the magnitudes and directions of the
velocity and the acceleration.
238 Functions of Several Variables (Ch. 6

2086. The equation of motion of a shell (neglecting air re-


sistance) is

where V 0 {vox• voy• V 0 z} is the initial velocity. Find the velocity


and the acceleration at any instant of time.
2087. Prove that if a point is in motion along the parabola
y=~. z=O in such a manner that the projection of velocity
a
on the x-axis remains constant ( ~; = const), then the accelera-
tion remains constant as well.
2088. A point lying on the thread of a screw being screwed
into a beam describes the spiral

x=acose, y=asine, z=h6,

where e is the turning angle of the screw, a is the radius of the


screw, and h is the height of rise in a rotation of one radian.
Determine the velocity of the point.
2089. Find the velocity of a point on the circumference of a
wheel of radius a rotating with constant angular velocity ro so
that its centre moves in a straight line with constant velocity v0 •

Sec. 19. The Natural Trihedron of a Space Curve


At any nonsingular point M (x, y, z) of a space curve r=r(t) it is pos-
sible to construct a natural trihedron consisting of three mutually perpen-
dicular planes (Fig. 84):
1) osculating plane MM,M 2 , containing the vectors ~~ and :;~;
2) normal plane MM:M 1 , which is perpendicular to the vector ~ and
3) rectifying plane MM,M 1 , which is perpendicular to the first two planes.
At the intersection we obtain three straight lines;
I) the tangent MM,; 2) the principal normal MM 2 ; 3) the bwormul MM 1 ,
all of which are defined by the appropriate vectors:

I) T= ~~ (the vector of the tangent line);


dr d 2r .
2) B= dt X dtz (the vector of the b1normal);
3) N = BX T (the vector of the principal normal);
The corresponding unit vectors
T B
1'=m; ~=m;
Sec. 19] The Natural Trihedron of a Space Curve 239

may be computed from the formulas


d'f
dr dS
"C=dS;
V=l :: l; f'="CXV.

If X, Y, Z are the current coordinates of the point of the tangent, then


the equations of the tangent have the form
X-x Y-y Z-z {l)
T~ =r;=---r;-•

Rectlfging
plane

dr
T=-
dt

Fig. lH

wlme Tx-= :; ; T_~~= :'{--, Tz= :: ; from the condition of perpendicularity


of the !me and the plane we get an equation of the normal plane:
Tx(X-x)+T_~~(Y-y)+Tz(Z-z)=O. (2)
If in equations (I) and (2), ,,.e replace T x• T_v, T. by Bx, B)" 8 z and Nx,
Ny, Nr, we get the equations of the binormal and the principal normal and,
respectively, the osculating plane and the rectifying plane.
Example 1. Find the basic unit vectors 'f, v and Jl of the curve
X= t, 1J=i 2, Z=t 3
at the point t =I.
Write the equations of the tangent, the principal normal and the binor·
mal at this point.
Solution. We have
r= tl+ tlj t'k +
and
:~ =l+2tJ+3t k, 1

d 1r
dt•""' 2J +6tk,
240 Functions of Several Variables [Ch. 6

Whence, when t = 1, we get


dr
T=dt =l+2J+3k;
d~ i1 12
B= dd~ xdt~= I kI
3 =6i-6J+2k;

N=BxT=I~ Jo 1~~-221-IGJ+
1 2 3
tBk.

Con seq uentl y,


i+2J+3k A __ 3i-3j+k -lll-BJ+9k
"C VT4 ... - Vt9 V= Y266 •
Since for t=I we have x=l, y=l, z=l, it follows that
x-1 y-1 2-l
-~-= -2- = -3-

are the equations of the tangent,


x-1 y-1 2-l
-3-= -3 =-1
are the equations of the binormal and
x-1 y-J z-J
-11 = -8 =~g-

are the equations of the principal normal.


If a space curve is represented as an intersection of two surfaces
F(x, y, 2)=0, G(x, y, 2)=0,
. · dr d2 r
then tn place of the vectors dt and dtz we can take the vectors dr {dx, dy, d2}
and d 2 r {d2 x, d2 y, d 2 z}; and one of the variables x, y, z may be considered
independent and we can put its second differential ~qual to zero.
Example 2. Write the equation of the osculatin~ plane of the circle
x2 +y 2 +z 2 =6, x+y+zc~O (3)
at 1ts pomt M (J, 1, -2).
Solution. Differentiating the system (3) and considering x an independent
vanable, we will have
xdx+ y dy+ zdz=O,
dx+dy+dz=O
and
dx 1 +dy 1 + y d 2y+dz 2 + 2 d 2z-= 0,
d2 y+d 2Z=0.
Putting x= I, y= 1, z,=-2, we get
dy=-dx;
d 2y=-~3 dx 2 •I
Sec. 19] The Natural Trihedron of a Space Curve 241

Hence, the osculating plane is defined by the vectors

{dx, -dx, 0} and {o. -; dx 2


, j-dx1 }
or
{1, -1, 0} and {0, -1, 1}.
Whence the normal vector of the osculating plane is

8=1~ 0 -1
_{ ~1=-l-j-k
1
and, therefore, its equation is
- I (x-1)-(y-l)-(z+2)=0,
that is,
x+y+z=O,
as it should be, since our curve is located in this plane.

2090. Find the basic unit vectors T, v, Ji of the curve


x=:=l-cost, y=sint, Z=f

at the point t = ~ .
2091. Find the unit vectors of the tangent and the principal
normal of the conic spiral
r = et (l cost +j sin t l- k)
at an arbitrary point. Determine the angles that these lines make
with the z-axis.
2092. Find the basic unit vectors T, v, Ji of the curve
y=x•, z=2x
at the point x = 2.
2093. For the screw line
x=acost, y=asint, z=bt
write the equations of the straight lines that form a natural
trihedron at an arbitrary point of the line. Determine the direc-
tion cosines of the tangent line and the principal normal.
2094. Write the equations of the planes that form the natural
trihedron of the curve
x• + y• + z• = 6, x•- y• + z• = 4
at one of its points M (1, 1, 2).
2095. Form the equations ot the tangent line, the normal
plane and the osculating plane of the curve x = t, y = t'. z = t•
at the point M (2, 4, 8).
242 Functions of Several Variables (Ch. 6

2096. Form the equations of the tangent, principal normal,


and binormal at an arbitrary point of the curve
t' t• t•
x=-;r· Y=a· z=2·
Find the points at which the tangent to this curve is parallel
to the plane x+3y+2z-10=0.
2097. Form equations of the tangent, the osculating plane,
the principal normal and the binormal of the curve
t•
X=t, y=-f, Z=2
at the point t = 2. Compute the direction cosines of the binormal
at this point.
2098. Write the equations of the tangent and the normal
plane to the following curves:
a) x=Rcos•t, y=R sintcost, z=R sint for i= ~;
b) z =x'+y•, x=y at the point (1, I, 2);
c) x•+y•+z 2 =25, x+z=5 at the point (2, 2V3, 3).
2099 Find the equation of the normal plane to the curve
z=x•-ya, y=x at the coordinate origin.
2100. Find the equation of the osculating plane to the curve
x=e1 , y=e- 1, z=tV2 at the point i=O.
2101. Find the equations of the osculating plane to the curves:
a) x•+y~+Z =9, X2 -Y 2 =3 at the point (2, I, 2);
2

b) x• = 4y, x• = 24z at the point (6, 9, 9);


c) x• + z• = a•, y• t z" = b 2 at any point of the curve (x 0 , Yo• Z 0 ).
2102. Form the equations of the osculating plane, the principal
normal and the binormal to the curve
y•=x, X 2 =Z at the point {1, I, 1).
2103. Form the equations of the osculating plane, the princi-
pal normal and the binormal to the conical screw-line X= t cost,
y= t sin t, z = bt at the origin. Find the unit vectors of the
tangent, the principal normal, and the binormal at the origin.

Sec. 20. Curvature and Torsion of a Space Curve


1°. Curvature. By the curvature of a curve at a point M we mean the
number
K=-= 1 I'1m-,
q>
R ~s-+o lls
Sec. 20) Curuature and Torsion of a Space Curue 243

where <p is the angle or turn of the tangent line (angle of contingence) on a
segment or the curve MN, As is the arc length or this segment or the curve.
R is called the radius of curuature. If a curve is defined by the equation
r= r (s), where s is the arc length, then

~=\:~\·
For the case or a general parametric representation or the curve we have
dr d'rl
1 Idixiit• (l)
R= 1:~r
2°. Torsion. By torsion (second curt•ature) or a curve at a point M we
mean the number
T=-= 1 1'1 m0-
Q as-+o As '
where 0 is the angle or turn or the binormal (angle of contingence of the
second kind) on the segment or the curve Kf.v. The quantity Q is called the
radws of torsion or the radius of second curt•ature. lf r=r(s), then
drd'rd 1r
J__=
Q
±I ~l=ifiiJSidSi
ds (d•r)• ' ds 2

where lhe minus sign is taken when the vectors ~~ and 'Y have the same
direction, and the plus sign, when not the same.
If r=r(t), where t is an arbitrary parameter, then
dr d 2r d3r
dt dti 'ift'i
Q (dr d
dt X dt 1
2
r)• · (2)

Example I. Find the curvature and the torsion of the screw-line


r=lacost+Jasint+kbt (a>O).
Solution. We have
dr
dt = - l a sin t +j a cos t +kb,
tFT
dt 2 = - l a cos t - j a sin t,
d3r
dt•= - l a sin t -ja cost.
Whenct'

dt
1
I
r
dr X ddt•=
1
-a sin t
-a cost -a sin t 0
I
aJcost kb =lab sin t-jab cos t+a 1k
244 Functions of Several Variables [Ch. 6

and
dr d 2r d• r ~-a sin t a cost
. b
dtdt 2 dt•= -ac~st -asmt 0 =a b.
2
I
a sm t -a cost 0
Hence, on the basis of formulas (1) and (2), we get
1 a yaz~ a
R= (aa+bz)•J. = a2+b2
and
1 a 2b b
Q= az (az+ba) = a2+b!.
Thus, for a screw-line, the curvature and torsion are constants.
3° frenet formulas:

2104. Prove that if the curvature at all points of a line is


zero, then the line is a straight line.
2105. Prove that if the torsion at all points of a curve is zero,
then the curve is a plane curve.
2106. Prove that the curve
X=1+3t+2t\ y=2-2t-j-5i 2 , Z=l-/ 1
is a plane curve; find the plane in which it I ies.
2107. Compute the curvature of the following curves:
a) x=cost, y=sint, z=cosh tat the point i=O;
b) X2 -i-\-Z 2 =l, y 2 -2x+z=0 at the point (1, 1, 1).
2108. Compute the curvature and torsion at any point of the
curves:
a) x = e1 cos t, y = e1 sin t, z = e1 ;
b) x=a cosh t, y=a sinh t, Z=at (hyperbolic screw-line).
2109. Find the radii of curvature and torsion at an arbitrary
point (x, y, z) of the curves:
a) X 2 = 2ay, X 8 = 6a 2 z;
b) x• = 3p 2 y, 2xz = p 2 •
21 t 0. Prove that the tangential and normal components of
acceleration w are expressed by the formulas
dv v2
'Uil' = dt l', Wv= R V,
where v is the velocity, R is the radius of curvature of the
trajectory, '\' and v are unit vectors of the tangent and principal
normal to the curve.
Sec. 20) Curvature and Torsion of a Space Curve 245.

2111. A point is in uniform motion along a screw-line r =


= ia cost+ ja sin t + btk with velocity v. Compute its accelera-
tion w.
2112. The equation of motion is
r= ti + f 1 j+ t'k.
DetermiRe, at times f=O and t=l: 1) the curvature of the-
trajectory and 2) the tangential and normal components of the.
acceleration.
Chapter VII

MULTIPLE AND LINE INTEGRALS

Sec. 1. The Double Integral in RectanguiP.r Coordinates


1°. Direct computation of double integrals. The double integral of a con-
tinuous function f (x, y) over a bounded closed region S is the limit of the
corresponding two-dimensional integral sum

~ f (x, y) dx dy= lim ~~~(xi, Yk) Mt !lyk, (I)


(S) max ax1 ~ o i k
max ay,, ~ 0

where !lxi=xi+s-Xi, !lyk=Yk+ 1 -Yk and the sum is extended over tho~e
values of i and k for which the points (x1, Yk) belong to S.
2°. Setting up the limits of integration in a double integral. We dis-
tinguish two basic types of region of integration.

y
y
B D
c Yz
y
A
B

!It A c
0 :r, :X Xz X a X
Fig. 85 Fig. 86

1) The region of integration S (Fig. 85) is bounded on the left and right
by the straight lines x=x1 and x=x1 (x 2 > x1), from below and from abovE
by the continuous curves y = q>dx)(AB) and y = q> 1 (x)(CD)(q>1 (x) ;;;;.q> 1 (x)l.
each of which intersects the vertical x =X (x1 ...;;; X<; x1 ) at only one point (seE
Fig. 85). In the region S, the variable x varies from x to x1 , while the va·
riable y (for x constant) varies from y1 = q> 1 (x) to y1 = q> 1 (x). The integral (1) rna~
Sec. 1) The Double Integral in Rectangular Coordinates 247

be computed by reducing to an iterated integral by the formula


X2 Cl'o (X)

Hf (x,
(S)
Y) dx dy = ~ dx ~ f (x,
x1 cp 1 (X)
y) dy,

Cl'2 (X)
where x is held constant when calculating ~ f (x, y) dy.
IP1 (X)
2) The region of integration S is bounded from below and from above
by the straight linE's y=y 1 and y=y1 (y 1 > y 1), and from the left and the
right by the continuous curves x = 1j) 1 (y) (AB) and x = 11' 1 (y) (CD) (1j) 1 (y) ?-11'1 (y) ),
each of which intersects the parallel y = Y (y 1 .,.;;; Y,.;;; y 1) at only one poinU
(Fig. 86).
As before, we have
u. ,. (Y)

H f(x, y)dxdy= ~ dy ~ f(x, y)dx,


(S) '" ,p, (1/)
"'· (y)
herl', in the integral ~ f (x. y) dx we consider y constant.
111. (y)
If the region of integration doe~ not belong to any of the above-discussed
types, then an attempt is made to break it up into parts, each of which doe!'.
belong to one of these two types.
Example l. Evaluate the integral
I 1

I=~ dx ~ (x+ y) dy.


0 y

Fig. 87
Solution.

Example 2. Determine the limits of integration of the integral

Hf (x, y) dx dy
(Sl
'248 Multiple and Line Integrals (Ch. 7

if the region of integration S (Fig. 87) is bounded by the hyperbola y 2 -x2 =1


and by two straight lines x=2 and x=- 2 (we have in view the region con-
taining the coordinate origin).
Solution. The region of integration ABCD (Fig. 87) is bounded by the
straight lines x-= -2 and x=2 and by two branches of the hyperbola
y= Yi +x 2
and y=- Yi +x•;
that is, it belongs to the first type. We have:
2 V1+x•
~ ~ f (x, y) dx dy = ~ dx ~ f (x, y) dy.
(S) - 2 - VJ+x•

Evaluate the following iterated integrals:


2 I a 1

2113. ~ dy ) (x 2 + 2y) dx. 2117. ~ dy ) (x+2y)dx.


-a Y'-4
4 •
llt a
2114 Sdx S(x~y 11 • 2118. ~ dc:p ~ r dr.
a sin q>

l sl x•dy 2 a cos q>


2115. S dx 1 +u•. 2119. ~ dc:p ~ r• sin 2 <pdr.
0 n
2
1 lr1-x2

2120. ~ dx ~ V'l-x•-y• dy.


X
Write the equations of curves bounding regions over which the
following dduble integrals are extended, and draw these regions:
2 2-y
• 2X

2121. ) dy ~ f (x, y) dx. 2124. ~ dx ~f(x, y)dy.


-G u•
---1 ~
4 a
a x+• a V2"5"=X•
2122. ~ dx ~ f(x, y)dy. ~ dx
I x•
2125.
0
~0
f (x, y) dy.

2123.
"~ dy Jo-y
~ f (x, y) dx.
2

2126. ~ dx ~ f(x, y)dy.


X+2

0 y -I x•
Set up the limits of integration in one order and then in the
other in the double integral
Hf (x, y) dx dy
(S)

for the indicated regions S.


Sec. I] The Double Integral in Rectangular Coordinates 249-

2127. S is a rectangle with vertices 0 (0, 0), A .(2, 0), B (2, 1),
c (0, 1).
2128. S is a triangle with vertices 0 (0, 0), A (1, 0), B (l, 1).
2129. S is a trapezoid with vertices 0 (0, 0), A (2, 0), B (1, 1),
c (0, 1).
2130. S is a parallelogram with vertices A (l, 2), B (2, 4),
C (2, 7), D (1, 5).
2131. S is a circular sector OAB with centre at the point
0 (0, 0), whose arc end-points are A (1, 1) and B ( --·1, 1) (Fig. 88).

y~

B(-1,1) A(t, f)

0 X 0 X

21 a2. S is a nght parabolic segment AOB bounded by the


parabola BOA and a segment of the straight line BA connecting
the points B ( -.. 1, 2) and A (I, 2) (Fig. 89).
2133. S is a circular ring bounded by circles with radii r = 1
and R == 2 and with common centre 0 (0, 0).
2134. S is bounded by the hyperbola y 2 -K = 1 and the circle
X 1 t/ = 9 (the region containing the origin is meant}.
2

2135. Set up the limits of integration in the double integral

~ ~ f (x, y) dx dy
(Sl

if the region S is defined by the inequalities


a) x>O; y?=O; x-1-y-s;;;. 1; d) y-;.~x; x~-1; y~ 1;
b) x 2 -f-Y 2 ~a ; 2
e) y~x-s;;y-t 2a;
+
c) x 2 !/ -s;; x; 0 -s;; !I-s;; a.
Change the order of integration in the following double integrals:
• l2x I X

2136. ~dx ~ f(x, y)dy. 2137. ~ dx ~ /(x, y)dy.


o 3x• 0 2.11
250 Multiple and Line Integrals (Ch. 7

a V a•-x• I 1-y

2138. ~ dx ~ f (x, y) dy. 2141. ~ dy ~ f(x, y)dx.


a2-x2 -V1-Y'
za
a Vzax-x• 1 Va-u•
2139. ~ clx ~ f (x, y) dy. 2142. ~ dy ~ f(x, y)dx.
a o o u•
2 a
za Vm
2140. ~ dx ~ f (x, y) dy.
v~

RVI
2 X R VR•-x•
2143. ~ dx ~ f (x, y)dy+ ~ dx
~ f (x, y) dy.
0 0 RV2 0

2
1t sin x
2144. ~ dx ~ f(x, y)dy.

Evaluate the following double integrals:


2145. ~ ~ xdxdy, where S is a triangle with vertices 0 (0, 0),
(S)
A (1, 1), and B (0, 1).
y
y
B(0,2)

C(O, 1)

A(2,0)X 0 X

Fig. 90 Fig. 91

2146. ~ ~ x dx dy, where the region of integration Sis bounded


(S)
by the straight line passing through the points A (2, 0), B (0, 2)
and by the arc of a circle with centre at the point C (0, 1), and
radius 1 (Fig. 90).
Sec. 1] The Double Integral in Rectangular Coordinates 251

2147. SSr
(S)
dxdy
a1 -x1 -y1
, where S is a part of a circle of radius

a with centre at 0 (0, 0) lying in the first quadrant.


2148. HV
(Sl
X
2
- y 2 dx dy, where S is a triangle with vertices
0(0, 0), A(1, -1), and 8(1, 1).
2149. ~ ~ V xy- y• dx dy, where S is a triangle with vertices
(S)
0 (0, 0), A (10, 1), and B (1, 1).
X

2150. ~~eli dxdy, where Sis a curvilinear triangle OAB bound-


lSI
ed by the parabola y• =X and the straight lines x=O, y= 1
(Fig. 91).
2151. ss::~~~. where sis a parabolic segment bounded by
(S)

the para bola y = i


and the straight line y = x.
2152. Compute the integrals and draw the regions over which they
extend:
It I +COS X fl

a) ~ dx ~ cosy
if sin xdy;
2 1

0 c) ~ dy ~ 2
X sin 1 ydx.
It It 0
2 1 2

b) ~ dx ~ y dy;
4

o rosx

When solving Problems 2153 to 2157 it is abvisable to make


the drawings first.
2153. Evaluate the double integral

~ ~ xy 1 dxdy,
(S)

if S is a region bounded by the parabola y 2 = 2px and the straight


line x=p.
2154*. Evaluate the double integral
Hxydxdy,
(Sl

extended over the region S, which is bounded by the x-axis


and an upper semicircle (x- 2) 1 + y2 = 1.
252 Multiple and Line Integ ra/s [Ch. 7

2155. Evaluate the double integral


dxdy
Ss
(S)
Y2a-x'

where S is the area of a circle of radius a, which circle is tan-


gent to the coordinate axes and lies in the first quadrant.
2156*. Evaluate the double integral
~ ~ ydxdy,
(S)

-where the region S is bounded by the axis of abscissas and an


.arc of the cycloid
x=R (t -sint),
y=R (1- cost).
2157. Evaluate the double integral
~~xydxdy,
(S)

in which the region of integration S is bounded by the coordi-


nate axes and an arc of the astroid
x=Rcos 3 t, y=Rsin 3 t(o~t~ ~)·
2158. Find the mean value of the function f (x, y) = xy 2
in the
region S{O~x~ 1, O~y~ 1}.
Hint. The .mean value of a funclton f (x, y) in the re~wn S is the number

f=; 55 f (x, y) dx dy.


(S)

2159. Find the mean value of the square of the distance of


+
a point M (x, y) of the circle (x -a) 2 y 2 ~ W from the coordi-
nate origin.

Sec. 2. Change of Variables in a Double Integral


1°. Double integral in polar coordinates. In a double integral, when passing
from rectangular coordinates (x, y) to polar coordmates (r, cp), which are
connected with rectangular coordinates by the relations
X=f COS cp, y=r sin c.p,
we have the formula
Hf (x, y) dx dy = ~~ (r cos qJ, r sin cp) r dr dcp, (I)
(S) !S)
Sec. 2] Change of Variables in a Double Integral 253

If the region of integration (S) is bounded by the half-lines r =a and


r =~(a.<~) and the curves r = r 1 (q>) and r = r 2 (q>), where r 1 (q>) and
r 2 1q>)(r 1 (q>)~r 2 (cp)) are single-valued functions on the interval a~rp~~.
then the double integral may be evaluated by the formula
f3 rl ('IJ)
~ ~ F (q>, r) r dr drp = ~ drp ~ F (rp, r) r dr,
(S) a r 1 (<P)
r, (q>l

where F(rp, r)=f(rcosrp, rsinrp). In evaluating the integral ~ F(rp, r)rdr


r, (cp)
we hold the quantity rp constant.
If the regton of integration does not belong to one of the kinds that has
been examined, it is broken up into parts, each of whtch is a region of a
given type.
2°. Double integral in curvilinear coordinates. In the more general case,
if in the double integral

Hf (x, y) dx dy
(~I

it is required to pass from the variables x, y to the variables u, v, which


are connected with x, y by the continuous and ddTerentiable relationshtns
x=cr (u, v). !J--=¢(u, v)

that establtsh a one-to-one (and, in both directtons, continuou'>) correspondence


between the points of the region S of the xy-plane and the points of some
region S' of the UV-plane, and if the Jacobwn
ax a11
I=D(x, y)= JUJU
D (u, v) ux uy
dvifv
retams a constant sign in the region S, then the forlllula

~~f(x, y)d.\dy~=~~f[cr(u, v), ¢(u, v)lllldudv


(;) <•')
holds true
The limits of the new integral are determined from general rules on the
basis of the type of region S'
Example I. In pas~111g to polar coordtnates, evaluate

~ ~ V1-x 2 -y2 dx dy,


(S)

where the region S is a circle of radtus R =I with centre at the coordinate


ongin (Fig 92).
Solution. Putting x-= r .:os rp, y '""' sinrp, we obtain:
y·J-x 2 - y 2 = VI - (r cos cp) 2 - (r sin q>) 2 = V 1- r 1
254 Multiple and Line Integrals [Ch. 7

Since the coordinate r in the region S varies from 0 to 1 for any IJl• and IJl
varies from 0 to 2n, it follows that
211 1

SS Yl-x -y dxdy= SdiJl Sr Yl-r dr={- n.


2 2 2

(S) o o

Pass to polar coordinates r and <p and set up the limits of


integration with respect to the new variables in the following
integrals:
1 1 2 X

2160. ~ dx ~ f (x, y) dy. 2161. ~ dx ~ f (Vx 2 +y•)dy.


0

2162. Ht(x, y)dxdy,


(S)
where S is a triangle bounded by the straight lines y=x, y=-x,
y=I.
1 1

2163. Sdx Sf ( ~) dy.


-1 x2

2164. Hf (x, y) dxdy, where S is bounded by the lemniscate


«S>

X
a
Fig. 92 Fig. 93

2165. Passing to polar coordinates, calculate the double inte


gral
~ ~ ydxdy,
(S)

where S is a semicircle of diameter a with centre at the poin


c (i-
I 0) (Fig. 93).
Sec. 2] Change of Variables in a Double Integral 255

2166. Passing to polar coordinates, evaluate the double inte·


gral
~ ~ (x1 + y 1 ) dx dy,
(S)

extended over a region bounded by the circle x• y• = 2ax. +


2167. Passing to polar coordinates, evaluate the double in·
tegral
~~ Va 2 - X 2 -y 2 dxdy,
(S)

where the region of integration S is a semicircle of radius a with


centre at the coordinate origin and lying above the x-axis.
2168. Evaluate the double integral of a function f (r, rp) = r
over a region bounded by the cardioid r=a(l +cosrp) and the
circle r =a. (This is a region that does not contain a pole.)
2169. Passing to polar coordinates, evaluate
a V a•-x•
~ dx ~ V x• + y• dy.
0

2170. Passing to polar coordinates, evaluate


~ ~ Va•-x•-y•dxdy,
(S)

where the region S is a loop of the lemniscate


(x 2 + y 2 ) 1 = a• (x 1 - y 1 ) (x;:;;:;. 0).
2171*. Evaluate the double integral

f f y1-~-~dxdy,
'csi
x• y•
extended over the regionS bounded by the ellipse a•+bi=l by
passing to generalized polar coordinates:
X y •
a= r cos rp, 7i = r sm qJ.
2172**· Transform
c jlx

~ dx ~ f (x, y) dy
o ax

(0 <a<~ and c > 0) by introducing new variables u =x+y,


uv =Y·
256 Multiple and Line Integrals [Ch. 7

2173*. Change the variables u=x+y, v=x-y in the integral


1 1

~ dx ~ f (x, y) dy.
0 0

2174**· Evaluate the double integral


~ ~ dxdy,
(S)

where S is a region bounded by the curve


x2
( a2 + Y2)2 _ ~-i
b2 - 112 k2 •
Hint. Make the substitution
x=ar cos <p, y=br sin <p.

Sec. 3. Computing Areas


}
0
• Area in rectangular coordinates. The area of a plane region S is
S=~~dxdy.
(S)

If the regionS is defined by the inequalities a.;;;;.x.;;;;;b, <p(x)oe;;;;;y.;;;;;'ljl(x),


then
b ljl (X)

S= ~ dx ~ dy.
a cp (x)

2°. Area in polar coordinates. If a region S in polar coordinates r and <p


is defined by the inequalities a.;;;;.cp.;;;;;~. {(cp).;;;;.r.;;;;;F(cp), then
(J F (cp)

S =~~ r dcp dr = ~ dcp ~ r dr.


(S) u f (cp)
2175. Construct regions whose areas are expressed by the in-
tegrals
2 X+2 a Va•-y•
a) ~ dx ~ dy; b) ~ dy ~ dx.
-1 x2 a-u
Evaluate these areas and change the order of integration.
2176. Construct regions whose areas are expressed dy the in-
tegrals
:rt
arc tan 2 a sec cp 2 a (1 +co~ qJ)

a) ~ dcp ~ rdr; b) ~ dcp ~ r dr.


:rt a
4

Compute these areas.


Sec 8] Computin~ Areas 257

2177. Compute the area bounded by the straight lines x = y,


x=2y, x+y=a, x+3y=a(a>0).
2178. Compute the area lying above the x-axis and bounded
by this axis, the parabola !J 2 =4ax, and the straight line x+y=3a.
2179*. Compute the area bounded by the ellipse

(y-x)• + K = 1.
2180. Find the area bounded by the parabolas

!J 2 =10x+25 and !J 2 =-6x-j-9.

2181. Passing to polar coordinates, find the area bounded by


the lines
x•-ty"=2x, x"+y"=4x, y=x, y=O.

2182. Find the area bounded by the straight line r cos rp = 1


and the circle r = 2. (The area is not to contain a pole.)
2183. Find the area bounded by the curves

r =a (1 +cos (r) and r =a cos (j• (a> 0).

2184. Find the arPa bounded by the line

2185*. Find the area bounded by the ellipse

(x-2y J-3) 2 + (3x-j-4y-1)"=100.


2186. Find the area of a curvilinear quadrangle bounded by
the arcs of the parabolas x• = ay, x• =by, y• =ax, y• = ~x (0 <
<a<b, O<a<~).
Hint. Introduce the new variables u and u, and put
x•= uy, y 2 = ux.

2187. Find the area of a curvilinear quadrangle bounded by


the arcs of the curves !l =ax, y• = bx, xy =a, xy = ~ (0 <a< b,
O<a<~).
Hint. Introduce the new variables u and u, and put
xy=u, y 2 =ux.
9 -1900
258 Multiple and Line Integrals [Ch. 7

Sec. 4. Ccmputing Volumes


ThP volume V of a cylmdrotd bounded above by a continuous surface
z = f (x,
y), b£ low by the pIt ne z = 0, and on the sid£s by a right cylindrical
surface, which cuts out of the xy-plane a region S (Fig. 94), is equal to

V= ~ ~ I (x, y) dx dy.
tS)

2188. Use a double integral to express the volume of a pyra·


mid wilh vertices 0(0, 0 0), A (I, 0, 0), B(l, 1, 0) and C(O, 0, 1)
(Fig. 95). Set up the limits ol integration.

z z
c(o,o, 1)

Fig. 94 Fig. 95

In Problems 2189 to 2192 sketch the solid whose volume is


expressed by the given double integral:
I 1-X 2 v 1- ,,
2189. ~ dx ~ (1-x- y) dy. 2191. ~ dx ~ (1-x)dy.
0 0
2 2-X 2 2

2190. ~ dx ~ (4 - x - y) dy. 2192. ~ dx ~ (4 -x- y) dy.


o n o 2-x
2193. Sketch the solid whose volume is expressed by the in-
a l
1
U2:-:fi
tegral ~ dx ~ Va 2
- r - y 2 dy; reason geometrically to find the
0 0
value of this integral.
2194. Find the volume of a solid bounded by the elliptical
paraboloid z=2X 2 +Y 2 +1. the plane x+y=l, and the coordi·
nate planes.
2H5. A solid is bounded by a hyperbolic paraboloid z = x•- t1
and 1he planes y=O, z=O, x= 1. Compute its volume.
Sec. 5) Com•mlinz th~ Area~ of Surfaces 259

2196. A solid is bounded by the cylinder x• t-z~ =a• and the


planes y=O, z=O, y=x. C>mpute its volume.
Find the volum~s bounded by the followinJ surfaces:
2197. az = y•, x• + y• = ,.•, z = 0.
2198. Y = vx-. u= 2Jlx, x + z =6, z =o.
2199. Z=X 2 -f-y 2 , Y=X 2 , !J= l, Z=0.
2200. x+y+z=a, 3x+y=a, {x+-y=a, y=O, z=O.
x• z• b
2201. -.+a=
a c
l, y=-X,
a
y=O, z=O.
2202. x• + y• = 2ax, z =ax, z = ~x (a>~).
In Problems 2203 to 2211 use polar and generalized polar
coordinates.
2203. Find the entire volume enclosed between the cylinder
x• + y• = a• and the hyperboloid x• + !l- z• = - a•.
2204. Find the entire volume contained between the cone
2(x" +if)-z"=O and the hyperboloid x• +y"-z' =-a•.
2205. Find the volume bounded by the suriaces 2az =x" + y\
x• f- !l -z• = a•, z = 0.
2206. Determine the volume of the ellipsoid
x• y' z2
u.-+IJ2+c-=L
2207. Find the volume of a solid bounded by the paraboloid
2az = x• + y• and the sphere x• + y• + z" = 3a 2 • (The volume lying
inside the par,tboloid is meant.)
2208. Compute the volume of a solid bounded by the xy-plane,
the cylinder x• ..._ y• = 2a~. and tl:e cone x• + y• = z•.
2209. Compute thP volume of a solid bounded by the xy-plane,
the surface z =-- ae- tx•q•>, and the cyl i·nder x• + y• = R•.
2210. Compute the volume of a solid bounded by the xy-plane,
• ~• ,1• • x• y2 x
the parabolotd Z=-.+' ,, and the cylmder 2 +-.=2-.
a 6 a 6 a
2211. In what ratio does the hyperboloid x"+y 2 - Z 2 =a1
divide the volume of the sphere x• + y• + z".::;:: 3a"?
2212*. Find the volume of a solid bounded by the surfaces
Z=X-j-y, xy= l, xy=2, Y=X, y=2X, z=O(x>O, y>O).

Sec. 5. Computing the Areas of Surfaces


The area a of a smooth single-valued surface z=f (x, y), whose projection
on the xy-plane is thl' region S, is t>qual to

9*
260 Multiple and Line Integrals [Ch. 7

2213. Find the area of that part of the plane ~+ f+ f = 1


which lies between the coordinate planes.
2214. Find the area of that part of the surface of the cylin-
der X 2 + y2 = R 2 (z;;;: 0) which 1ics between the planes z = mx and
Z= nx(m >n > 0).
2215*. Compute the area of that part of the surface of the
cone x2 - y 2 = Z2 which is situated in the first octant and is
bounded by the plane y + z =a.
2216. Compute the area of that part of the surface of the
cylinder X 2 +if=ax which is cut out of it by the sphere
+
xz yz + zz =- az.
2217. Compute the area of that part of the surface of the
xz
sphere X 2 + y 2 + Z 2 = a 2 cut out by the surface U2 + b2 = 1.
1/2

2218. Compute the area of that part of the surface of the


paraboloid y 2 + Z2 = 2ax which lies between the cylinder Y2 =ax
and the plane x =a.
2219. Compute the area of that part of the surface of the
cylinder X 2 + y 2 = 2ax which lies between the xy-plane and the
cone x 2 + Y2 = Z 2 •
2220*. Compute the area of that part of the surface ol the
cone X2 -!/=Z 2 which lies inside the cylinder X2 -f-Y 2 =2ax.
2221 *. Prove that the areas of the parts of the surfaces of the
paraboloids x 2 +y 2 ='2az and x 2 -Y 2 =2az cut out by the cylin-
der X2 + y 2 = R 2 are of equivalent size.
2222*. A sphere of radius a is cut by two circular cylinders
whose base diameters are equal to the radius of the sphere and
which are tangent to each other along one of the diameters of the
sphere. Find the volume and the area of the surface of the re-
maining part of the sphere.
2223* An opening with square base whose side is equal
to a 1s cut out of a sphere of radius a. The axis of the opening
coincides with the diameter of the sphere. Find the area of the
surface of the sphere cut out by the opening.
2224*. Compute the area of that part of the helicoid
z=carctan JLx which lies in the first octant between the cylin-
2
ders X -l-Y2 =a 2 and X
2
-f-Y 2 =b 2 •

Sec. 6. Applications of the Double Integral in Mechanics


1°. The mass and static moments ot a lamina. If S 1s a region in an
xy-plane occup1ed by a lamina, and Q (x, y) 1s the surface density of the
lamina at the point (x, y), then the mass M of the lamina and its static
Sec. 6] Applications of the Double Integral in Mechanics 261

moments Mx and My relative to the x- and y-axes are 'expressed by the


double integrals
M= ~ ~ Q (x, y) dx dy, Mx= ~ ~ YQ (x, y) dx dy,
(S) (.'>)

My=~ ~ XQ (x, y) dx Jy. (l)


(S)

If the lamina is homogeneous, then Q (x, y) = const.


2°. The coordinates of the centre of gravity of a lamina. lf C (X, Y) is the
centre of grav1ty of a lamina, then
- My - Mx
X=M' Y=Af·

where M is the mass of the lamina and Mx. My are its static moments rela-
tive to the coordinate axes (see 1°). If the lamina is homogeneous, then in
formulas (I) we can put Q = I.
3°. The moments of inertia of a lamina. The moments of inertJa ol a
lamina relative to the x- and !f-axes arc, respectively, equal to
lx= ~ ~ y 2 Q(x, y)dxdy, ly= ~ ~ x 2 Q(x, y)dxdy. (2)
(S) (S)

The moment of inertia of a lamina relative to the ongin is

10 = ~ ~ (x 2 -t-y 2)Q(X, y)dxdy=lx+ly. (3)


(,~)

Putting Q (x, !I)-= 1 in formulas (2) and (:3), we get the geometric moments of
mcrtJa of a plane figure.

2225. Find the mass of a circular lamina of radius R if the


density is proportional to the distance of a point from the centre
and is equal to b at the edge of the lamina.
2226. A lamina has the shape of a right triangle with legs
OB =a and OA = b, and its density at any point is equal to the
distance of the point from the leg OA. Find the static moments
of the lamina relattve to the legs OA and OB.
2227. Compute the coordinates of the centre of gravity of the
area OmAnO (Fig. 96), which is bounded by the curvl' y = sin x
and the straight line OA that passes through the coordinate origin
and the vertex A ( ~ . 1)
of a sine curve.
2228. Find the coordinates of the centre of gravity of an area
bounded by the cardtoid r=a(l -t-cos<p).
2229. Find the coordinates of the centre of gravity of a cir-
cular sector of radius a with angle at the vertex 2a (Fig. 97).
2230. Compute the coordinates of the centre of gravity of an
area bounded by the parabolas t/ = 4x ~- 4 and y• =- 2x +4.
2231. Compute the moment of inertia of a triangle boun;led
by the straight linesx+y=2, x=2, y=2 relative to the x-axi~;.
262 Multiple and Line lnfe[!rals [Ch. 7

2232. Find the moment of inertia of an annulus with diame-


ters d and D (d <D): a) relative to its centre, and b) relative to
its diameter.
2233. Cvmpute the moment of inertia of a square with side a
relative to the axi5 passing through its vertex perpendicularly to
the pl<me of the square.
2234il. C Jmpute the moment of inertia of a segment cut off
the parabola y• =ax by the straight line x=a relative to the
straight line y =-a.
y
y
•'

A(; , t}
X
0 X
Fig. 96 Fig. 97

2235*. Compute the moment of inertia of an area bounded by


the hyperbola xy=4 and the straight line x-t-y=5 relative to
the straight line x=y.
2236". In a square la11i111 with side a, the density is propor-
tional to the distance from one of its vertices. C)mpute the mo-
ment of inertia of the lamina relative to the side that passes
through t6is vertex.
2237. Find the moment of in-:rtia of the cardioid r=a(l +coscp)
relative to the pole.
2238. Compute the moment of inertia of the area of the lem-
niscate r"=-o2a"cos2<p relative to the axis perpendicular to its
plane 10 the pole.
2239*. C::>mpute the moment of inertia of a homogeneous lamina
bounded by one arc of the cycloid x=a(t-sint), y=a(l-cost)
and the x-ax1s, relative to the x-axis.

Sec. 7. Triple Integrals


1°. Triple integrals in rectangular coordinates. The triple integral of the
function f (x, y, ;) extended over the region V is the limit of the corre-
sponding threefold iterated sum:
~ ~ ~I (x, !J, z) dx dy dz =lim ~~~~(X£, Yj• z11 ) AX£ tJ.y1 t!z,.
V m"x \x; -+ o i i 11
maxtl.'';-+O
max ll.zk-+ o
~ec. 7] Triple lntewafs 2t:3
----~----------------------~----~------------------------

Evaluation of a tri pie mtegral reduces to th~ succes>1ve compuh t10n of tht!
three ordinary (on~fold iterated) integrals or to the computation of one
double and one sinde integral.
Example 1. Compute
I= ~ ~ ~ x3y2z dx dy dz,
v
where the region V is defined by the in!'qualilies
O~x~ I, O~y~x. O~z~xy.

Solution. We have
J X XII 1 X XII

I= Sdx Sdy JxVz dz = Sdx Sx y 3 2


fI dy =
0 0 0 0 0
1 X 1 X 1

= Sdx Jx'~~ dy= S~ t Idx= j';; d~= 1~0 ·


0 0 0 0 0
Example 2. Evaluate
~ ~~ 2
x dx dy dz,
(\')
. . x 2 t/ z2
l'Xtended over the volume of the ellipsoid -+~,+-=I.
a• b- c•
Solution.
a a
~ ~ ~ 2
x dx dy dz = ~ x dx
2
~~ dydz= ~ 2
x Syzdx,
(\') -a <SuJ -a
11 2 z2 x2
\\hNe SJ'Z IS fhe area of the ellipse -b'2 +-.=l--.,X=COnst,andisequa}lo
c a

Syz =Jlb
V l--;;·C
~
a· V-.\. ( !--=nbc
a•
1-- . x')
u•
We there! ore fi na II y gel
a

SSSx• d x dy dz ='nbc Sx• ( I - ~) dx = ~ nu"bc.


~~ -a
2•. Change of variables in a triple integral. If in the triple inle~ral

~ ~ ~ f (x, y, z) dx dy dz
(\')

it is required to pass from th~ variables x, 11, z to the variables u, v, w,


which an•connect•:dwithx,!J. zbvther,•lation' x=<p(u, v. w), y=¢(u,v.w),
z=x(u, 11, w), where the functiOn~ <p. ¢.X ar~:
I) continuous together with their pa1tial fir'>t derivatives;
2) in one-to-one ( 1nd, 111 both directions. contitlliJU>) corre,pondence be-
tween the poiPts of the region of integrdtwn V ol xyz-spdce and the potnts of
some region V' of UVW-spacc;
264 Multtple and Line Integrals [Ciz. 7

3) the functional determinant (Jacobian) of these functions


ax iJx iJx
lfU av ow
I= D (x, y, !)__= oy ay iJy
D (u, v, w) au av ow
az oz az
au iJv iJw

retains a constant sign in the regwn V, then we can make use of the for·
mula

~ ~ ~ f (x, y, z) dx dy dz =
(V)

=~ ~ ~f[rp(u,v,w), ljJ(u,v,w), X(u,v,w)}jljdudvdw.


(V')

z z

X
Fig. 98 rtg. 99

In parttcular,
I) for cylindrical coordinates r, cp, h (ftg. 98), where
x=rcosrp, y=rsinrp, z=-h,
we get I= r;
2) for spherical coordinates cp, ~~. r (cp is the longitude, 1p the latitude,
r the radius vector) (Ftg. 99), where
x=r cosljJcos cp, Y=r cos¢ sin cp, z=r sin ~'•

2
we have I=-= r cos¢.
Example 3. Passing to spherical coordinates, compute
~ ~ ~ vx•+y 2 +z 2 dxdydz,
(V)

where V is a sphere of radius R.


Solution. For a sphere, the ranges of the spherical coordinates rp (longi-
tude), 1jJ (latitude), and r (radius vector) will be
SPC 71 Triple Integrals 265

We therefore have
:rc
z:rc 2 R.
~ ~ ~ Yx 2 -l-!l-l-z 2 dxdydz=~dcp ~ dljJ~rr 2 cosljldr=nR'.
(\') 0 :rc 0
2

3°, Applications of triple integrals. The volume of a region of three-dimen-


siOJ1al xyz-space is
V= ~ ~ ~ dxdydz.
(\')

The mass of a solid occupying the region V is


M,-~ ~ ~ ~ )'(\, y, z) dxdydz,
(V)

where y (x, y, z) is the density of the body at the po1nt (x, y, z).
The static moments of the body relative to the coordmate planes are
Mxyo= ~ ~ ~ y(x, y, z)zdxdydz;
(V)

M rz = ~ ~ ~ y (x, y, z) .1: dx dy dz;


(\')

Mzx= ~ ~~ y(x, y, z) ydxdydz.


(V)

The coordinates of tire cetrtre of gravtly are


- Myz - Mzx - MXl'
X =A:r !/ ~~""""Kf' Z =f\1·

If the solid is homogeneous, then we can put y (x, y, z) = 1 in the for-


mulag for the coordmates of the centre of grav1ty.
The moments of inertia relative to the coordinate axes are
lx= ~ ~ ~ (y 2 -j-z 1 )y(x, y, z)dxdydz;
(V)

fy,=~ ~ ~(z 2 -j-X 2 )y(x,y,z)dxdydz;


(V)

lz= ~ ~ ~ (x 2 +y 2)y(x, y, z)dxdydz.


(I')

Putting y (x, y, z) =I In these formulas, we get the geometric mom en{s


of inertia of the body.
A. Evaluating triple integrals
Set up the limits of integration in the triple integral
~ ~ ~ ,'(x, y, z)dxdydz
(Vl
for the indicated n·gions V.
266 Multinle a'ld Lire Tntegrals [Ch. 7

2240. V is a tetrahedron bounded by the planes


X+ y + Z = 1, X= 0, y = 0, Z = 0.
2241. V is a cylinder bounded by the surfaces
x• + y• = RZ, z = 0, z =H.
2242*. V is a cone bounded by the surfaces
x2 •1 2 z2
-~
( + b• = cz ' z =c.
2243. V is a volume bounded by the surfaces
z = 1- x•- y•, z = 0.
Compute the following integrals:
I I I
dz
2244.
S S SVx+y+z+l ,
d X dy

2 zJ/x {4X;'''
2245. ~ dx
~
0
dy
0
~ xdz.

a J/~ Vu• x• u•
2246. J dx .I dy
s Ya 2
dz
-x2 -y 2 -z2 '

I 1-X 1-X-l/

2247. ~ dx ~ dy ~ xyzdz.
p
2248. Evaluate
d'(dydz
Sss IV I
(x + 11 + z +I ) 3
'

where V is the region ot mtegration bounded by the coordinate


planes and the plane x+y-+ z= l.
2249. Evaluate
~ ~ ~ (x+ y+z)"dxdydz,
lVI

where V (the region of integration} is the common part of the


pa,abolo1d 2al~x•ty• and the sphere x"+y"+z·~3a".
2250. Evaluate

where V (region <i. integration) rs the common part of the


spheres x• + y' 1- z• ~ R" and x• + y• + Z2 ~ 2Rz
Sec. 7] Triple Integrals 267

2251. Evaluate
E~ zdxdydz,
(\')

where V is a volume bounded by the plane z = 0 and the upp:!r


x• z2
half of the ellipsoid - 2 + -'-b• +-.
1/
c = 1. (I

2252. Evaluate

5Jrs (7x + (j2y• + czZ )


2 2
dx du dz,
(VI
x2
+ 2zc
2 2
y
where 11 is the interior of the ellipsoid -a2 +-b. = 1.
2253. Evaluate
~ ~ ~ z dx dy dz,
(I')

where V (the region of integration) is bounded by the cone


2
!z2 2 2
Z = R" (x and the plane z =h.
f-- !J )

2254. Passing to cylindrical coordinates, evaluate


~ ~ ~ dx dydz,
(\')

where V is a region bounded by the surfaces X


2
-l-!/ +Z 2
=2Rz,
x• -1 !J 2 z• and containing the point (0,0, R).
-o=c

2~55. Evaluate
2 1~ 2 t - xJ a

~ tlx ~ dy ~ z V x• + y• dz,
0 0 0

first transforming it to cylindrical coordinates.


2256. Evaluate
2T l,.2fX-A."' 1 4.f~-rJ-y-a

~ tlx ~ dy ~ dz,
o -V~

first transforming it to cylindrical coordinates.


2257. Evaluate
R VR•-x• V R•-x•-u•
~ dx ~ dy ~ (x
1
+y 1
) dz,
-R -JIR•-x•
first transforming it to spherical coordinates.
268 Multiple and Line Integrals [Ch. ~

2258. Passing to spherical coordinates, evaluate the integral


~ ~ ~ V x• + !J2 + z2 dx dy dz,
(V)

where V is the interior of the sphere K + y• + z2


~ x.

B. Computing uolumes by means of triple integrals


2259. Use a triple integral to compute the volume of a solid
bounded by the surfaces
y• =4a" -3ax, y• =ax, z =±h.
2260**. Compute the volume of that part of the cylinder
X + y• = 2ax which is contained between the paraboloid K + !J2 = 2az
2

and the xy-plane.


2261*. Compute the volume of a solid bounded by the sphere
x 2 +y2 -J-Z 2 =a 2 and the cone z·~-=x•+y= (external to the cone).
2262*. Compute the volume of a solid bounded by the sphere
X -I-!J +Z =4 and the paraboloid x +!J =3z (internal to the
2 2 2 2 2

paraboloid).
2263. Compute the volume of a solid bounded by the xy-planc,
the cylinder x•-t--y•=ax and the sphere x•+y• -t-z•=a• (internal
to the cylinder).
2264. Compute the volume of a solid bounded by the paraboloid
y• z' .x
z -;;a+cz-=2aandtheplanex=a.

c C. Applications of triple integrals


to mechanics and physics
2265. Find the mass M of a rec-
tangular parallelepiped O~x~a.
v 0 ~ y ~ b, 0 ~ z ~ c, if the den-
sity at the point (x, !J, z) is
(;!(X, y, z)=x+y-1-z.
2266. Out of an octant of the
sphere x• + !J 2 + z• ~ cZ, x ::-::! 0,
Fig. 100 y~O. z~O cut a solid OABC
bounded by the coordinate planes
and the plane = r
+ = 1 (a~ c, b ~c) (Fig. 100). Find the mass
of this body if the density at each point (x, y, z) is equal to
the z-coordinate of the point.
2267*. In a solid which has the shape of a hemisphere
X +y
2 2 2
+
Z ~ a , z ~ 0, the density varies in proportion to the
2
Sec. 8] Improper Integrals Dependent on a Parameter 26!)

distance of the point from the centre. Find the centre of gravity
of the solid.
2268. Find the centre of gravity of a solid bounded by the
paraboloid y 2 t2Z 2 =4x and the plane x=2.
2269*. Find the moment of inertia of a circular cylinder,
\vhosc altitude is h and the radius of the base is a, relative to
the axis which serves as the diameter of the base of the cylinder.
2270*. Find the moment of inertia of a circular con~
(altitude, h, radius of base, a, and density Q) relative to
the diameter of the base.
2271**. find the force of attraction exerted by a homogeneous
cone of altitude h and vertex angle (t (in axial cross-section) on
a matenal point containing unit mas' and located at its vertex.
2272**. Show that the Ioree of attraction exerted by a homo-
geneous sphere on an external material point does not change if
the entire mass of the sphere is concentrated at its centre.

Sec. 8. Improper Integrals Dependent on a Parameter.


Improper Multiple Integrals
1°. Differentiation with respect to a parameter. In the case of certain
ri'slrtc!IOns imposed on tlw funct10ns I (x. a), f~ (x, a) and on the correspond-
Ill!,( improper mtegrals we have the Letbntz rule

d c
"'
r,
00

da j f (x, a) dx=.) fa (x, a) dx.


l1 ll

Example 1. By difft>rentiatin~ with respt>ct to a parameter, evaluate

Solution. Let

Then

s
<Xl

aF (a, fl) = - xe-•x• dx= _!_ e-•x'l


~ ~
00

= _ _!.
~
0 0

Whrnce F (a, ~) ==- ~ In a+ C (~). To find C (B). we put a= B in the latter


equation. We have 0=-~ ln~+C(~).
I
Whence C (~) = In~. Hence,
2
~
F (a, B) = -
I 1 1
2 In a+ 2 In B= 2 In a.
270 Multiple and Line Integrals (Ch. 7
------------------~----------~~--------------~---
2°, Improper double and triple integrals.
a) An infinite region. If a function f (x, y) is continuous in an unbounded
region S, then we put

Hf(x, y)dxdy= ~m Hf(x, y)dxdy,


(Sl CJ S(O)
(I)

where a is a finite region lying entirely within S, where a-+ S signifies that
we expand the region a by an arb1trary law so that any roint of S should
enter it and remain in it. If there is a limit on th~> right and if it does not
<lepend on the cho1ce of the rP~Cion a, then the corresponding improper inte-
gral IS called WtlVi!rRellt, otherwise it is d1uergent.
II the inh·gr;l!lcl f(x, y) i5 nonnegative lf(x, y)~O), then for thP con-
vergPnce of an IIIIJ'IOPer integral it 1s nrcNsary and sufficient for the limit
on the right of (I) to exist at least for one system of regions a that exhaust
the region S.
b) A di\continuous function. If a function f (x, y) is everywhere contin-
uous in a bounded closed region S, excPpt the point P (a, b), then we put

~~ f (x, y) dx dy = lim ~ ~ f (x, y) dx dy, (2)


(S) e ... o (S )
E

where s. is a region obtained from S by elimmating a small region of d1a


mPtH e that contams the roint P. If (2) has a limit that dol's not depend
on the lYre of small re~<ions rlimirated from S, the imrroper integral under
consideration is called wnuer[:.enl, olhc TWISP it is dtuergent.
If f (x, y):;:,. 0, tht n thr l1mit on the right of (2) is not dependent on the
type of regions eliminated from S; for instance, such reg:ons may be c1rcles
of radius f with centre at P.
The concept of improper double integrals is readily extended to the case
of trirle integrals.
Example 2. Test for convergence
rr dxdy
(3)
J J (I + x2 + y2)P '
(S)

where S is the entire xy-plane.


Solution. Let a be a circle of radius Q with centre at the coordinate
origin. Passing to polar coordinates lor p :f. I, we have
Ul Q

I (a)= ss(I+~~
(CJ)
y2)P =
0
s s dcp
0
(I :d;2)P
2n
= s ~ i _::~-pI:
0
(I dcp= I~ p [(1 + ()2)1-P-1).

If p <I, then lim I (a)= lim I (a)= oo and the integral diverges. But if p >I,
a-..s 0 .... a:

then lim I (a)=~~ and the integral converges. For p =I we have


Q-+"' p-
Sec. 8] Improper Integrals Dependent on a Parameter 271

l (a)=
zSn dq> SQ - rdr
- =rt In (I +Q2); lim I (a)= oo, that is, the integral
1 +r 2 II._....,
0 0
diverges.
Thus,· the integral (3) converges for p > I.

2273. Find f' (x), if


"'
f (x) = ~ e-xu• dy (x > 0).
X

2274. Prove that the function

-oo

satisfies the Laplace equation


iJ"u iFu
OX 2-+ - - 0.
dt!"-
2275. The La place transformatiOn F (p) for the function f (t)
is defined by the formula
"'
F (p) =~e-Mf {I) dt.
u

Find F (p), if: a) f (t) = 1; b) f (t) =ear~ c) f (t) =sin ~t;


d) f (I)= cos ~t.
2276. Taking advantage of the formula
I

dX = _!_ > 0),


Sxn- (n
1
tl
u
compute the integral
I

~ xn-l}nxdx.
t)

2277*. Using the formu Ia

s"' e-PI dt = i (p > 0),


evaluate the integral
272 Multiple and Line Integrals [Ch. 7

Applying di!Terentiation with respect to a parameter, evaluate


the following integrals:
rt:J

2278. se-·x~-e-;ixdx (u>O, ~>0).

"'
2279. S e-•x -:;e-;;x sin mx dx (a > 0, ~ > 0).
0

"'
2280 . JC arc tan ax d
x(I+x2 ) X.

2281. sIn I

(1-a"x") dx
x• Yl-x 2
(/a/< 1).
0
00

2282. ~e-axsin}xdx (u~O).


0

Evaluate the following improper integrals:


"' "-
2283. ~ dx ~ e-(H!il dy.
I y> X

2284. ~ dy ~eli dx.


0

2285. ~S >-~~:~., where S is a region defined by the inequali-


(S>
ties x~ I, y~x·.

2286* . .) dx (' s
00 "'

dr
(x•+y"·J·a•)• (a>O).
0 0

2287. The Euler-Poisson integral defined by the formula


00 00

f =~e-x' dx may also be written in the form 1 = ~ f!-Y' dy. Eval-


o 0

uate 1 by multiplying these formulas and then passing to polar


coordinates.
2288. Evaluate
00 "' rt:J

s0
dx ~ dy ~ (x• + y• ~ z• + !)•.
0 0
Sea. 9] Line Integrals 273

Test for convergence the improper double integrals:


2289**. ~~lnVx•-t-y"dxdy, where Sis a circle x•+y 1 ~l.
(S)

2290. 55 (x~~:.)~, where S is a region defined by the ine-


IS)
quality x• -1 y• ~ 1 ("exterior" of the circle).
• dxdtt
2291*.
SJ V<x~y)•'
(S)
where Sis a square jx!~l. !YI~1.

2292. 555 (x• ~;Y:~.)'l , where V is a region defined by the


(I')

inequality x• + y• -1- z" ~ 1 ("exterior" of a sphere).

Sec. 9. Line Integrals


1°. Line integrals of the first type. Let f (x, y) be a continuous function
<Jnd y = cp (x) [a~\~ b) be the equatiOn of some smooth curve C.
Let us construct a system of points M 1 (x;. y,) (i = 0, I, 2, ... , n) that
break up the curve C into elementary arcs A1, :=-;M; = 8s; and let us form the
,
integral sum S, = ~ f (x;, y,) i.\s 1 • The limit of this sum, when n ~ oo and
t-1
max i.\s;-+ 0, 1s called a !tne tntegral of the ftrst type
I!

lim ~ f(x;, Y;) l'l.s;= ~ f (x, y) ds


11 ..... ~ i=l c
(ds 1s the arc differential) and is evaluated from the formula
b
~ f(x, y) ds = ~ f(x, Cf (x)) Y I+ (qJ' (x)) 2
dx.
c <l

In the case of parametric representation of the curve C: x = cp (I),


Y-= ¢(I) [a~ t ~ we have n
~
~ f (x, y) ds = ~ f (cp (t), ~,(I)) Y qJ' 2 (I)+ ¢' 2 (t) dt.
C a

Also considered are line integrals of the first type of functions of three
variables f (x, y, :z) taken along a space curve. These integrals are evaluated
in li'<e fashion A line integral of the lirsl type does not depend on the direc-
tioJ! of the path of inlef!ralton; if the integrand f is interpreted as a linear
density of the curve of integration C, then this integral represents the mass
of the curve C.
274 Multiple and Line Integrals [Ch. 7

Example I. Evaluate the line integral


~ (x+y> ds,
c
where C is the contour of the triangle ABO with vertices A (1, 0), 8 (0, 1),
and 0 (0, 0) (Fig 101).
Solution. Here, the equation AB is y= 1-x, the equation 08 is x=O,
and the equat1on OA IS y = 0. We therefore have

~~+~~=~~+0~+~~+~~+~~+~~=
C AB 00 OA
I I I

= ~ Y2dx+ ~ ydy+ ~ xdx= Y2 +I.


0 0 0

2°. Line integrals of the second type. If P (x, y) and Q (x, y) are contin·
uous functions and y=cp (.1) 1s a smooth curve C that runs from a to b as
y

A X
Fig. 101

x varies. then the corresponding ltne tntegral of the second type is expressed
as follows:
b
~ P (x, If) dx + Q (x, y) dy = ~ I P (x, q> (x)) + cp' (x) Q (x, ql (x)) I dx.
C a
In the more general case when the curve C is represented parametrically:
x=q> (t), y=¢ (t), where t varies from a to ~. we have
~
~ P (x, y) dx + Q (x, y) dy + ~ [P (<p (t), ¢(I)) cp' (t) + Q (<p (t), ¢ (t)) 'ljl' (t)] tit.
c u

Similar formulas hold for a line integral of the second type taken over a
space curve.
A line mtegral of the second type chanaes stgn when the dtrectton of the
path of 111tegratton ts reversed. Th1s integral may he interpreted mechanically
as the work of an appropriate variable Ioree { P (x, y), Q (x, y)} along the
curve of integrdtlon C
Example 2. Evaluate the line integral

~ y 2 dx+x 2 dy,
c
Sec. 9) 275

where C is the upper half of the ellipse x=a cost, y=b sin t traversed
clockwise.
Solution. We have
0

~ y• dx + x• dy = ~ {b2 sin" t • (-a sin t) + a• cos 2


t . b cos t] dt =
c 11 0 0

= -ab 2 Ssin 1
ab 2 •
t dt + a b Scos• t dt = ~
2

n n
3°. The c:ase of a total dilferentia!. If the inte~rand of a line integral
of the second type is a total differential of some smgle-valued function
Uo-=U(x, y), that is, P(x, y)dx+Q(x, y)dy~dU(x, y), then thts line integral
ts not dependent on the path of integration and we have the Newton-Leibn iz
formula
(x,. y,)
~ P(x, y)dx+Q(x, y)dy=U(x 2 , y,)-U(x 10 y 1), (l)
(x,. u,)
where (x 1 , y,) 1s the initial and (x •• Y:l is the termtnal point of the path
In particular, if the contour of inte~ration C ts closed, then

~P (x, y)dx-1-Q (x, y)dy=O (2)


c
If I) the contour of inter.:rntion C i~ cont:Jined entirely within some
simnlv-conneclt•d regio.1 ') and 2) the functions P (~. 1/) and Q (x, y) to!..'ether
wllh tlkir parltal denV<•tives of the first orrler are continuous in S, then a
neces~arv and sufficie'lt co Hhtio 1 fo1 th~ existence of the function U is the
i.lenticai fullilment (In S) of the equality
aQ aP (3)
ax= iJy
(see integration of total di!Terentials) If conditions one and two are not ful-
filled, !hr presence of rondilton (3) does not guarantee the existence of a
sllH!le-valued lunct10n U, and formulas (I) and (2l may prove wron!..' (sre
Problem 2.t 2) We give a method of linding a funclton U (x, p) from its
total dirlcrenlial based on the use of line integrals (which is yet ar:olher
method of inle~rating a total diiTerenlial). For the contour of mtegra\1on C
let us take n brokt•n l111e P,,P,M (F1g 102), where P 0 (x 0 , y0 ) is a fixrd ! oint
and M (x, y) 1s a variable point. Then along P 0 P 1 we have Y=Yo and dy=O,
nnd along P ,M we have dx = 0 We ,:;et:
(X, U)

U(x, y)-U(x0 , y 0 )= ~ P(x, y)dx+Q(x, y)dy=


(X 0 , Yo) x U

= ~ P (x, y0 ) dx+ ~ Q (x, y) dg.


Uo

Simtlarly, integrating with respect to P 0 PaM, we have


U X
U (x, y)-U (.t0 , Yo)=~ Q (x0 , y) dy+ ~ P (x, y) dx.
Uo Xo
276 Multiple and Line Integrals [Ch. 7

Example 3. (4x+2y) dx+(2x-6y) dy=dU. Find U.


Solution. Let X0 =0, y 0 =0. Then
X 1/

U (x, y) = ~ 4x dx+ ~ (2x-6y) dy+C=2x 2 +2xy-3y 2 +C


0 0
or
y X

U (x, IJ) = ~ -6y dy+ ~ (4x + 2y) dx + C= -3y 2+ 2x 2


+3xy +C,

where C = U (0, 0) is an arbitrary constant.

!:1

0 x0 X X
Fig. 102

4°, Green's formula for a plane. If C is the boundary of a region S and


the functions P (x, y) and Q (x, y) are contmuous together with their first-
order partial derivatives in the closed region S + C, then Green's formula holds:

pP dx+Q dy= 55(~~- a;;) dxdy,


C (S)

here t'•e circulation about the contour Cis chosen so that the regionS should
remain to the left.
5°, Applications of line integrals.') An area bounded by the closed contour Cis

S= -§ ydx= pxdy
c c
(the direction of circulation of the contour is chosen counterclockwise).
The fo lowing formula for area is more convenient for application:

S= ~ § (x dy-y dx) = ! § x 2
d ( ~) .
c c
2) The work of a force, having projections X= X (x, y, 2), Y = Y (x, y, z),
Z=Z(x, y, 2) {or, accordingly, the work of a force field), along a path Cis
Sec. 9] Line Integrals 2i7

expressed by the Integral


A= ~ X dx +Y dy + Z dz.
c
If the force has a potential, i.e., if there exists a function U = U (x, y, z)
(a potential funct1on or a force function) such that
uu au au
-=X -~Y, -=-=Z,
iJx • oy dz
then the worl\, irrespective of the shape of the path C, is equal to
(x,, !h z,) (x,, 11,. z,)
A= ~ Xdx-j-Ydy-j-Zdz= ~ dU=U(x 2 , y2 , z2)-U(x 1 ,y, z-,),
(r 1 , y 1 , z,) (r 1 , 11 1, z 1 )

where (r 1 , y 1 , z1) is the Initial ant! (x 2 , y 2 , z 2 ) is th~ termmal point of the path.

A. Line Integrals of the First Type


Evaluate the following line integrals:
2293. ~ xy ds, where C is the contour of the square I xI +I !J I=a
c
(a> 0).
2294. \ .r _ ds ::--:--, where C is a segment of the straight line
(; r x• 1- y t· ·I
connecting the points 0 (0, 0) and A (1, 2).
2295. Jr xyds, where C is a quarter of the ellipse £i2 ~
+ 1;2~ = 1
c
I ying in the first quadrant.
2296. ~ y"ds, where Cis the first arc of the cycloid x =a (t- sin t).
c
!J= a (1-CO'i f).
2297. ~ v-x.;;-+-u-=-· ds, where c is an arc of the involute of the
c
circle xo·o·a(eost 1-lsint), !J=a(smt-tcost) l0~t~2Jtj.
2298. ~ (x" +!/ 2 2
) ds, where C is an arc of the logarithmic spi-
c
ral r = ae 111 'P (m > 0) from the point A (0, a) to the point 0 (- oo, 0).
2299. ~ (x +y) ds, where C is the right-hand loop of the lem-
c
niscate r• = a• cos 2cp.
2300. ~ (x 1- y) ds, where C is an arc of the curve x = t.
c
y = ~;, z = t' (0 ~ t ~ I].
278 Multiple and Line Integrals [Ch. 7

2301. r~X +d!y + Z


2 2 , where C is the first turn of the screw-line
x = a cos t, y = a sin t, z = bt.
2302. ~V2y 2 -\-Z 2 ds, where Cis the circle x•+y•+z•=a 1 ,
c
X=lj.
2303*. Find the area of the lateral surface of the parabolic
cylinder y = f x• bounded by the p Janes z = 0, x = 0, z = x, y = 6.

2304. Find the arc length or the co111c screw- I inc C x = ae cost,
1

y = ae 1 sin t. z = ae 1 from
the point 0 (0. 0, 0) to the point A (a, 0, a).
2305. Determine the mass of the contour of the ellipse
:: + ~~ = 1, if the linear density ol it at each point M (.t, y) is
-equal to I y I·
2306. find the mass of the first turn of the screw-line x =a cost,
y=asint,z=!Jt, if the density at each point is equal to the
radius vector of this point.
2307. Determine the coordinates of the centre of gravity of
a half-arc of the cycloid
x=a(t-sint), y=a(1-cost) [O~t~nl.
2308. Find the moment of inertia, about the z-axis, of the
first turn of the screw-line x=acost, y=asint, z=bt.
2309. With what force will a mass M distributed with uni-
form density over the circle x• -1- y'=a 2 , z=O, act on a massm
located at the point A (0, 0, b)?

B. Line Integrals of the Second Type


Evaluate the following line integrals:
2310. ~ (x 2 -2xy)dx-l- (2xy-~-y 2 )dy, where AB is an arc of the
AB
parabola y=x• from the point A(1, 1) to the point 8(2, 4).
2311. ~ (2a-y)dx l-xdy, where C is an arc of the first
c
arch of the cycloid
x=a(t-sint), y=a(1-cost)
which arc runs in the direction of increasing parameter t.
2312. ~ 2xy dx-x• dy taken along different paths emanating
UA
from the coordinate origin 0 (0, 0) and terminating at the point
A (2, 1) (Fig .•103):
a) the straight line OmA;
Sec. 9] Lint! lnte~ral~ 2i9
--~----------------------~---------------------

b) the parabola OnA, the axis of symmetry of wh1ch is the


y-axis;
c) the parabola OpA, the axis of symmetry of wh1ch 1s the
x-axis;
d) the broken line OBA;
e) the broken line OCA.

2313. ~ 2xy dx t X
1
dy as in Problem 2312.
OA
231 4*. J; (x+ u) dx-(x-q) dy
:Y x2+ yz
taken alom! the circle x 2
~
+y =a•2

counterclockwise.
y

X
B(2,0)
Fig. 103

2315. ~ y 2 dx+x 2 dy, where C is the upper half of the ellipse


r:
x =a cost, y = b sin t traced clockwise.
2316. ~ cosy dx- sin x dy taken along the segment AB of the
AH
bi~ector of the second quadrantal angle, if the absc1ssa ol the
point A is 2 and the ordinate of B 1s 2.
2317. fx!l(ll¥~~~; where Cis the right-hand loop ot lhe
dt!),
2
lemniscate r• =- o cos 2cp traced counterclockwise.
2318. Evaluate the line integrals with respect to expressions.
wh1ch are total di!Terentials:
(2. 3) (3, 4) (1. I)

a) ~ x dy + y dx, b) ~ x dx + y dy, c) ~ (x + y) (dx + dy),


I - 1. tl (0, I) (0, 0)

(I. I)

d) S 11 dx;t x dy {along a path that does not intersect the


(1. I)
x-axis),
!280 Multiple and Line Integrals [Ch. 7

(X, y)

e) dx+dy (along a path that does not intersect the


S x+y
( 2..
1 '
2....)
a
straight line x + y = 0),
(x,. y,J
f) ~ cp (x) dx + 1jJ (y) dy.
(x,. y,)

2319. Find the antiderivative functions of the integrands and


.evaluate the integrals:
(3, 0)

a) ~ (x' + 4xy 8 ) dx-+ (6x 1y 2 -5y') dy,


(-2, -1)
(1, 0)

b) S x~~=~)~x (the integration path does not intersect the


(0, - I )
straight line y = x),
(1, 1)

c) S
(x+2y)dx+ydy (th integration path does not intersect
(x+y)z e
(1, 1)
the straight line y = - x),
(1, I)

d) 5 (v
(o, o)
x
xz+yz +u)dx+(vxz+yz
Y +x) dy.
2320. Co~pute

x• y•
taken clockwise along the quarter of the ellipse Q2+ IJ!= 1 that
lies in the first quadrant.
2321. Show that if f (u) is a continuous function and C is a
-closed piecewise-smooth contour, then
p
c
f (x + y
2 2
) (x dx + y dy) = 0.

2322. Find the antiderivative function U if:


a) du = (2x+3y) dx+(3x--4y) dy;
b) du = (3x 2 -2xy+ y 2 ) dx-(x 2 -2xy+3y 2 ) dy;
c) du=ex-y [(1 + x+y}dx+(l-x-y)dy];
d) du = .!!.:.__ ..!!L
x+y+x+y'
Sec 9) Line Integrals 281

Evaluate the line integrals taken along the following space


curves:
2323. ~ (y--z)dx+(z-x)dy+(x-y)dz, where C is a turn
c
of the screw-line x= a cost,
y=a sin t, f
lz = bt'
corresponding to the variation of the parameter t from 0 to 2n.
2324.
c
p
y dx + z dy + x dz, where C is the circle
x = R cos a cos t,
f
y = R cos a sin t,
l
z = R sin a (a= const),
traced in the direction of increasing parameter.
2325. ~ xydx-+-yzdy+zxdz, where OA is an arc of the
OA
circle
x• + y• + z• = 2Rx, z = x,
situated on the side of the xz-plane where !J > 0.
2326. Evaluate the line integrals of the total differentials:
(6, .•• 8)

a) ~ xdx+ydy-zdz,
(1, "· -•)
(a, !J. c)

b) ~ yzdx-t-zxdy-1-xydz,

s
(1, 1, 1)
(3, •• 5)
C) Xcf\ 1_-_IJd!f+zdz
+
Y~.\. -1- u• z• '
(o, o, o)

(
X, 1/, _.!._)
XI/

d) 5 yzdx+ztdtJ-f-xydz (the integration path is situnted


xyz
(1, I, 1)
in the first octant).
C. Green's Formula
2327. Using Green's formula, transform the line integral
l=:fVx•-1 y'dx+y[xy+ln(x-t- Vx +!J
2 2
)] dy,
c
where the contour C bounds the region S.
282 Multiple and Line /ntearals [Ch. 7

2328. Applying Green's formula, evaluate

I= fc 2 (r -t- y") dx + (x +y) dy, 2

where C is the contour of a triangle (traced in the positive direc-


tion) with vertices at the points A (I, 1), B(2, 2) and C(1, 3).
Verify the re~ult ob1ained by computin~ the inteeral directly.
2329. Applying Green's formula, evaluate the inccgral
p-x y dx + xy dy,
c
2 2

where C is the circle X 2 + y 2 = R2 traced counterclockwise.


2330. A parabola AmB. whose ax1s is the y-ax1s and whose
chord is An8, is drawn through the points A (1, 0) and 8 (2, 3).
Find p (x + y) dx-(x-y) dy directly and by applying Green's
AmBnA
formula.
2331. Find ~ exy [i dx !- (1 + xy) dy], if the points A and B
AmB
lie on the x-axis, while the a1ea, bounded by the integration
path AmB and the segrr:ent A8, is equal to S.
2332*. Evaluate A; x dt~-y 2d!. Consider two cases:
:;; ,\ +!!
a) when the origin is outside the contour C,
b) when the contour enC'ircks the origin n times.
2333**. Show that If C is a closed curve, then
p
c
cos (X, n) ds = 0,

-where s is the arc length and n is the outer normal.


2334. Applying Green's formula, find the value of the integral
I= p[x cos (X, n) -t-y sin (X,
c
n)] ds,

where ds is the differential of the arc and n is the outer normal to


the contour C.
2335*. Evaluate the integral
):, dx-dy
:f
c x+y '
taken alcmg the contour of a square with vertices at the points
A (1, 0). 8 (0, I), C (- 1, 0) and D (0, -1), provided the contour
is trat:ea countercln~kwi~e.
Sec. 9) Line lntearals 28~

D. Applications of the Line I nil'gral


Evaluate the areas of figures bounded by the following curves:
2336. The ellipse x =a ws t, y = b sin t.
2337. The astroid x =a cos• t, y =a sin' t.
2338. The cardioid x=a (2cost--cos2t), y=a (2sint-
sin2t).
2339*. A loop of the folium of Descartes x•+y'-3:zxy=0
(a> 0).
2340. The curve (x + y)• = axy.
2341 *. A circle of radius r is rolling without sliding along a
fixed circle of radius R and outside it. Assuming that ~ is an
integer, find the area bounded by the curve (epicycloid) described
by some point of the nwving circle. Analyze the particular case
of r = R (cardioid).
2342*. A circle of radius r is rolling without slidin~ along
a fixed circle of radius R and inside tt. Assuming that !i.r is an
integer, find the area bounded by the curve (hypocycloid) de-
scribed by some point of the moving circle. Analyze the particular
case when r= ~(astroid).
2343. A field is generated by a force of constant magnitude F
in the positive x-diredion Find tlw work that the held does
when a material point traces clockwise a quarter of the circle
x• + y• = W lying in the first quadrant.
2344. Find the work done by the force of rravity when
a material point of mass m is moved irom position A (x" y" z.)-
to position B (x., y •• z.) (the z-axis is directed 'vertically up-
wards).
2345. Find the work done by an elastic force dtrccted towards
the coordinate origin tf the magnitude of the force is prop::>rtion-
al to the distance of the point f10111 the orig111 and if the point
of application of the force traces counterclockwise a quarter of
the ellipse~+~~= 1 !yin~ in the first quadrant.
2346. Find t:1e p)tential fun~tion of a for~c R {X, Y, Z}
and determine the work don~ by the force over a given path if:
a) X=O, Y=O. Z=-mg (force of gravity) and the mate-
rial point is moved from pvsition A (x,, y., z.) to position
B (x •• Y.. z.);
b) X =-~t:, Y =-J.I~, Z =-tt:-. where f.l=Const and
r r '
r = V x• + y• + z" (Newton attractive force) and the material point
-;--.....-~

moves from position A (a, b, c) to infinity;


284 Multiple and Line Integrals [Ch. 7

c) X=-k 2 x, Y=-k 2 y, Z=-k 2 z, where k=const (elastic


force), and the initial point of the path is located on the sphere
x• +y 2 + z 2 = R 2 , while the terminal point is located on the sphere
X +y
2 2
+
z• = r (R > r).
2

Sec. tO. Surface Integrals


1°. Surface integral of the first type. Let f (x, y, z) be a continuous
function and z=~c:p (x, y) a smooth surfaceS.
The surface integral of the ftrst type is the limit of the integral sum
n
) ) f (x, y, z) dS = lim .~ f (x;, Yi· z,) M;,
S n-+oot=l

where I:!S; is the area of the Jth element of the surface S, the point (x;, y.,
z;) belongs to this element, and the maximum diameter of elements of par-
ti lion tends to zero.
The value of this integral is not dependent on the choice of side of the
surface S over which the integration is performed.
If a projectton a of the surface S on the xy-plane i~ single-valued, that
is, every strdight line parallel to the z-axis intersects the surface S at only
one point, then the appropriate surface integral of the first type may be
calculated from the formula

~ ~ f (x, y, z) dS =- ~ ~ f (x, y, c:p (x, y)] V 1+rr: (x, y) +cp; (x, y) dx dy.
s (a)

Example t. Compute the surface integral

~~ (x+y+z)dS,
s
where S is the surface of the cube 0.,;;;; x.,;;;; I, 0.,;;;; y.,;;;; I, 0.,;;;; z.,;;;; I.
Let us computP the sum of the surface integrals over the upper edge of
the cube (z = I) and over the lower edge uf the cuue (z = 0):
I I I I I I

H(x+y+ 1)
00
dx dy+ ~~
00
(x-t-y) dxdy= H(2x+2y+ 1)
00
dxdy=3.

The desired surface integral is obviously three times greater and equal to

~~ (x + y + z) dS = !),
s
2°. Surface integral of the second type. If P = P (x, y, z), Q = Q (x, !J, z),
R = R (x, y, z) are continuous functions and s+ is a side of the smooth sur-
lace S characterized by the direchon of the normal n {cos u, cos~. cosy}, fhen
ihe corresponding surface tntegral of the second type is expressed a~ follows:

) ~ P dy dz + Q dz dx + R dx dy = H (P cos a +Q ws ~ + R cos \') dS.


S+ S
Sec. 10] Surface Integrals 285

When we pass to the other side, s-,


of the surface, this integral re-
verses sign.
If the surfaceS is represented implicitly, F{x, y, z)=O, then the direc-
tion cosines of the normal of this surface are determ!ned from the formulas
I iJF I oF I oF
cos a = l5 ax , cos ~ = 0 ay , cos y = 0 az ,
where

and the choice of sign before the radical should be brought into agreement
with the side of the surf ace S.
3°. Stokes' formula. If the functions P = P (x, y, z), Q = Q (x, !J, z),
R = R (x, y, z) are continuously differentiable and Cis a closed contour bound-
ing a two-sided surface S, we then have the Stokes' formula
p
c
P dx + Q dy + N dz =
=55 [(oR_iJQ)cosa+(oP _aR)cos~+(aQ_aP) cosvl dS
a!/ az oz a~ ox cJy '
s
where cos a, cos~. cosy are the direction cosinl's of the normal to the sur-
face S, and the directwn of the normal is defined so that on the side of the
normal thl' contour S IS traced counterclockwise (m a rigiJI·handed coorclmate
t>) stem).
Evaluate the following surface integrals of the first type:
2347. ~~(x"+u")d.S', where Sis the sphere x 2 +!J 2
-/-Z 2 =a 2 •
~

2348. ~~ V x• + !/ dS where S is the lateral surface of the


2
s2 2
cone a2 r.2
.\

11
- -12 =0 [O~z<b].
7
Z

Evaluate the following surface integrals of the second type:


2349. H
s
yz dy dz + xz dz dx -1- xy dx dy, where S is the external
side of the surface of a tetrahedron bounded by the planes x=O,
V=0, Z=0, X-/-!J-/-Z~=a.
2350. H
s
zdxdy, where S is the external side of the ellipsoid
x• y• z•
aa--1- 112+ C2 = 1.

2351. H
s
x dy dz + !J dz dx + z• tlx dy,
1 2
where S is the externa I
side of the surface of the hemisphere x• !/ z• = a• (z:;:;;. 0). + +
2352. Find the mass ol the surface of the cube 0 ~ x ~ 1,
O~!J~ 1, O~z~ 1, if the surface density at the point M (x, !J, z)
ls equal to xyz.
286 Multiple and Line Integrals [Ch. 7

2353. Determine the coordinates of the centre of gravity of a


homogeneous parabolic envelope az = Jt t y 2 (0 ~ z ~a).
2354. Find the moment of inertia of a part of the lateral
surface of the cone z = V x 2 + y 2 [0 ~ z ~ h] about the z-axis.
2355. Applying Stokes' formula, transform the integrals:
a) 1 (X 2 - yz) dx + (y 2 - zx) dy + (z 2 - xy) dz;
c
b) py dx + z dy + x dz.
c
Applying Stokes' formula, find the given integrals and verify
the results by direct calculations:
2356.
c
p
(y -r· z) dx + (z + x) dy + (x + y) dz, where C is the circle
x2 -t-Y 1 -t- Z 2 =a 2 , x+y+z = 0.
2357. 1 (y-z) dx + (z-x) dy + (x-y) dz, where Cis the ellipse
c 2
X -j- lj
2
= l, X+ Z = l.
2358. 1xdx-t-(x+y)dy+(x+y+z)dz, where Cis the curve
c
x=a sin t, y=a cost, z =a (sin t+cos t) [0 ~ t ~ 2n).
2359 . .if) y 2 dx-t Z 2 dy-J-x 2 dz, where ABCA is the contour of
ABC4
6 ABC with vertices A (a, 0, 0), 8 (0, a, 0), C (0, 0, a).
2360. In what case is the line integral
I = 1 P dx + Q dy + R dz
c
over any closed contour C equal to zero?

Sec. 11. The Ostrogradsky-Gauss Formula


If S is a clmed smooth surface bounding the volume V, and P = P (x, y, z),
Q=Q(x, IJ, z), R=R(r, y, z) dre functions that are contmuoustogdherw1th
th~ir first partial denvahves in the closed region V, then we have the Ostro·
groJ,dsky-Gauss formula

ss
S
{P cos u + Q cos~+ R cosy) dS= 55 s(~: +~; +::)
(VI
dx dy dz,

where crs u, cos~. cosy are the direction cosines of the outer normal to the
surface S

Applying the Ostrogradsky-Gauss formula, transform the fol-


lowing surface wtegrals over the closed surfaces S bounding the
'
lee. 11) The Ostro{!radsky-Gauss Formula 287

rolume V(t·osa, cos~. cosy are dtrection cosines of the outer


1ormal to the surface S).
2361. ~ ~ xy dx dy + yz dy dz + zx dz dx.
s
2362. ~ ~ x• d y dz + y• dz dx + z• dx dy.
2363 _ s.,.
~

x crs a+ y c0s ~ + z cosy dS.


Vx +y +z
2 2 2
s
2364. 55s (~ cos a + ~~ cos ~ + ~cosy) dS.
Using the Ostrogradsky-Gauss formula, compute the following
surface integrals:
2~63. ~~x 2 dydz+y•dzdx+z 2 dxdy, where Sis the external
s
side of the surface of the cube O~x~a. O~y~c. O~z~a.
2366. ~ ~ x dy dz + y dz dx + z dx dy, where S is the externa I side
s
of a pyramid bounded by the surfaces x-+y+z=a, x=O, y=O,
z =- 0.
2367. ~~ x 3 dydz-!-y'dzdx=z'dxdy, where Sis the external
:-,
side of the sphere x• +y 2 -j-Z 2 =a 2 •
2368 ~ ~ (x" cos a -i y• cos~+ z• cosy) dS, where S is the exter-
s
na I total surface of the cone

2369. Prove that if S is a closed surface and is any fixed


direction, then
~ ~ cos (n, l) dS = 0,
s
where n is the outer normal to the surface S.
2370. Prove that the volume of the solid V bounded by the
surface S is equal to
V =-} 55 (x cos a+ y cos~+ z cosy) dS,
s
where (!Osc, cos~. cosy are the direction cosines of the outer
normal to the surface S.
28R Multiple and Line Integrals [Ch. 7

Sec. 12. Fundamentals of Field Theory


1°. Scalar and vector fields. A scalar field is defined by the scalar function
of the point u = f (P) = f (x, y, z), where P (x, y, z) is a point of space. The
surfaces f (x, y, :z) = C, where C =canst, are called level surfaces of the scalar
field.
A vector field is defined by the vector function of the point a= a (P) =
=a (r), where P is a point of space and r=xl+yJ+zk is the radius vector
of the potnt P. In coordinate form, a=axl+avJ+azk. where ax=llx (x, y, z),
uy = ay (x, y, :z), and az = az (x, y, z) are projections of the vector a on the
coord•nate axes. The vector lines (force lines, flow lines) of a vector field are
found from the following system of differential equations
dx dy dz
llx =a;= az.
A scalar or vector field that does not depend on the time t is called
stationary; if it depends on the time, it is called nonstationary.
2°, Gradient. The vector
au au . au
grad U (P) =ax l + ay J+ iJz k:== \lU,

where \l = i aa + la~ + k ;_ is the Hamil toni an operator (del, or nabla), is


X y uZ
called the gradtent of the field U = f (P) at the given point P (cl. Ch. VI, Sec. 6).
The gradient is in the directwn of the normal 11 to the level surface at the
roint P and in the directiOn of Increasing function U, and has length equal
to

~~ = vr (~~r + (~~r + (~~r


If the direction is given by the unit vector l {cos a, cos~. cosy}. then
au au au
ar=gradU·l=grad,U=ax cosa+dy cos~+
au cosy
02
(the derivative of the function U in the direction 1).
3°. Divergence and rotation. The divergence of a vector field a (P) =a>.{ 1-
+ ayJ+az
. k . tl
ts 1e sea I ar d'tv a= aux
iJx + DZ~ V a.
duy iJaz
+ay·
The rotation (curl) of a vector field a (P) =axl+ayJ+azk is the vector

rot a= (iJaz-
oy az
iJay) l + ( aa.-<- aaz) j + (aay- iJax) k
az ax ax iJy
-=s V X a.

4°, Flux of a vector. The flux of a vector field a (P) through a surfaceS
in a direction defined by the unit vector of the normal n {cos a, cos~. cosy}
to the surface S is the integral

~~ an dS = ~~ an dS = ~ ~ (ax cos a+ ay cos ~ + az cosy) dS.


s s s
If S is a closed surface bounding a volume V, and n is a unit vector of the
outer normal to the surface S, then the Ostrogradsky·Gaus~ formula holds,
Sec. 12.] Fundamentals of Field Theory 289

which in vector form is

cffi an dS = ~ Hdiva dx dy dz.


J_l (V)

5°, Circulation of a vector, the work of a fteld. The line mtegrai of the
vector a along the curve C is defined by th~: formula

~ adr= ~ a5 ds= ~ axdx+avdy+azdz (I)


c c c
and represents the work done by the field a along the curve C (a., is the
projection of the vector a on the tangent to C).
If C is closed, then the line integral (I) is called the ctrculuttOn of the
vector field a around the contour C.
If the closed curve C bounds a two-sided surfaceS, then Stokes' formula
holds, which in vector form has the form

p
c
a dr= ~ ~ n rot adS,
s
where n is the vector of the normal to the surface S; the direction of the
vt>ctor should be chosen so that for an observer looking in the direction of n
tht> circulation of the contour C should be counterclockwise in a right-handed
coordmate system.
6°. Potential and solenoidal llelds. The vector lield a (r) is called poten-
twl if
a=grad U,
where V=f(r) is a scalar function (the potellttu/ of the field).
For the potentiality of a field a, given in a simply-connected domain,
it is necessary and sufficient that it be noll rotational, that is, rot a=O. In
that case there exists a potential U defined by the equation
dU =ax dx+av dy+ az dz.

If the potential U is a single-valued function, then ~a dr = U (8)-U (A);


AB

in particular, the circulation of the vector a is equal to zero: pa


c
dr= 0.

A vector field a (r) is called solenoidal if at each point of the field div
a=O; in this case the flux of the vector through any closed surface is zero.
If the field is at the same time potential and solenoidal, then div (grad U) =0
and the potential function U is harmonic; that is, it satisfies the Laplace
a•u a•u a•u a• iJ2 a•
equation 0~· +ay•+az• =0, or t:.V=O. where t:.=v"=o~•+ay•+az•isthe
Laplacian opera"tor

2371. Determine the level surfaces of the scalar field V=f(r),


+
where r = Vx• y•-t-z•. What will the level surfaces be of a field
U = F (Q), where Q = Vx• y•? +
10-1900
290 Multiple and Line Integrals [Ch. 7

2372. Determine the level surfaces ot the scalar field


.
U =arc sm z
.r .
I' xz+yz

2373. Show that straight lines parallel to a vector c are the


vector lines of a vector field a (P) = c, where c is a constant
vector.
2374. Find the vector lilies of the field a=- royl-~ roxj, where ro
is a constant.
2375. Derive the formulas:
+ +
a) grad (Cp C 2 V) = C 1 grad U Ca grad V, where C 1 and C1
are constants;
b) grad (UV) = U grad V V grad U; +
c) grad (U 2 ) = 2U grad U;
!!__) _ V grad U -U grad V.
d) gra d ( V - ya ,
e) grad cp (U) = cp' (U) grad U.
2376. Find the magnitude and the direction of the gradient
of the field V=x'+y 3 +Z 3 -3xyz at the point A (2, l, I). Deter-
mine at what points the gradient of the field is perpendicular to
the z-axis and at what points it is equal to zero.
2377. Evaluate grad U, if U is equal, respectively, to: a) r,
b) r', c) +, d) f (r) (r = V X 2
+y + z
2 2
).

2378. Find the gradient of the scalar field U = cr, where c is


a constant vector. What will the level surfaces be of this field,
and what will their position be relative to the vector c?
2379. Find the derivative of the function U = ;_
a
Yb_;. ~ at a
c
+ +
given point p (x, y, z) in the direction of the radius vector r of
this point. 'In what case will this derivative be equal to the
magnitude of the gradient?
2380. Find the derivative of the function U =_!__ in the di-
rection of l {cos a, cos~. cosv}. ln what case will this derivative'
be equal to zero?
2381. Derive the formulas:
+ +
a) div (C 1 a 1 C 2 a 2 ) = C 1 div a 1 C 2 div a 2 , where C 1 and C 1 are
constants;
b) div (Uc) =grad U · c, where c is a constant vector;
c) div(Ua)=grad U·a+ Udiva.
2382. Evaluate div (f) .
2383. Find div a for the central vector field a (P) = f (r)!.... ,
where r = XV + +Z
2
Y
2 2
• '
Sec. 12] Fundamentals of Field Theory 291

2384. Derive the formulas:


a) rot (C,a, + c.a.) = C, rot a,+ c. rot a 1 , wher~ C, and c. are
constants;
b) rot(Uc)=gradU-c , where cis a constant vector;
c) rot(Ua)=gradU ·a+Urota.
2385. Evaluate the divergence and the rotation of the vector
a if a is, respectively, equal to: a) r; b) rc and c) f (r) c, where c
is a constant vector.
2386. Find the divergence and rotation of the field of linear
velocities of the points of a solid rotating counterclockwise with
constant angular velocity m about the z-axis.
2387: Evaluate the rotation of a field of I inear velocities
'V=ro·r of the pointsof a body rotating with constant angular
velocity ro about some axis passing through the coordinate origin.
2388. Evaluate the divergence and rotation of the gradient of
the scalar field U.
2389. Prove that d iv (rot a)= 0.
2390. Using the Ostrogradsky-Gaus s theorem, prove that the
flux of the vector a= r through a closed surface bounding an
arbitrary volume v is equal to three times the volume.
2391. Find the flux of the vector r through the total surface
of the cylinder x•+!l~R". O~z~H.
2392. Find the flux of the vector a= x'l + y'j+ z"k through:
a) the lateral surface of the cone x'+ y2 z2
~ ~ H•, 0 ~ z ~ H, b) the

total surface of the cone.


2393*. Evaluate the divergence and the !lux of an attractive
force F = - m:r
of a point of mass m, located at the coordinate
origin, through an arbitrary closed surface surrounding this point.
2394. Evaluate the line integral of a vector r around one
turn of the screw- I ine x = R cost; y = R sin t; z =hi from t = 0
to t = 2n.
2395. Using Stokes' theorem, evaluate the circulation of the
vector a= x•y•; + j+ zk along the circumference x• + y• = R 2 ; z = 0,
taking the hemisphere z = VW -x•- y• for the surface.
2396. Show that if a force F is central, that is, it is directed
towards a fixed point 0 and depends only on the distance r from
this point: F=f (r) r, where f (r) is a single-valued continuous
function, then the field is a potential field. Find the potential U
of the field.
2397. Find the potential U of a gravitational field generated
by a material point of mass m located at the origin of coordi-
nates: a=-~ r. Show that the potential U satisfies the Laplace
equation 11U = 0.
10*
292 Multiple and Line Integrals [Ch. 7

2398. Find out whether the given vector field has a potential U,
and find U if the potential exists: ·
a) a= (6x•y-4xy) i + (3x1 -2y)j;
+
b) a= yzi zx}+ xyk;
+ + +
c) a= (y z) i (x z) j + (x + y) k.
2399. Prove that the central space field a= f (r) r will be so-
lenoidal only when f (r) = r~, where k is constant.
2400. Will the vector field a= r (c x r) be solenoidal (where c
1!> a constant vector)?
Chapter V Ill

SERIES

Sec. 1. Number Series


1°. Fundamental concepts. A number series
..,
a1 + a 1 + ... +a,.+ ... = ~ an (1)
n=l
Is called convergent if its partial sum
Sn=a 1 +az+ ... +a,.
has a finite limit as n---+ oo. The quantity S= lim S,. is then called the sum
n-+ oo
of the series, while the number
R,.=S-S,.=an+t +a,.+z+ ...
is called the rematnder of the series. If the limit lim S,. does not exist (or Is
n-+ oo
infinite), the series is then called dtvergent.
If a series converges, then lim a,.= 0 (necessary condition for convergence).
n-> oo
The converse is not true.
For convl'rgence of the ~cries (I) it is necessary and sufficient that for
any positive number e it be possible to choose an N such that for n > N
and for any positive p the following inequality is fulfilled:
\a11+1+an+2+· · .+an+pl <e
(Cauchy's test).
The convergence or divergence of a series is not violated if we add or
subtract a lini te number of its terms.
2°. Tests of convergence and divergence of positive series.
a) Comparison test I. If 0-=:;;;; a11 ,.;:;;;; b,. after a certain n = n0, and the series
00

b, + b2 + ... + b,. + ... = ~ b,. (2)


n=t

converges, then the series (I) also converges. If the series ( I) diverges, then
(2) diverges as well.
It is convenient, for purposes of comparing series, to take a geometric
progression:
00

~ aq" {a 'f:. 0),


n=o
294 Series {Ch. 8

which converges for I q I< land diverges for I q I ;;o: l, and the harmonic sertes
"' I
L,n-·
11=1
which is a divergent series.
Example I. The series
I I I I
f:2 + 2·2 2 + 3·2 3 + ·" + ll·2n+ "'
converges, since here
I I
a"= n·2n < 2"'
while the geometric progression

11=1

whose ratio is q = ! , converges•


.t.
Example 2. The series

ln2+~+
2 3 .. . +~+
ll ...

d1. Vt'rges, smce


· 1't s genera I t erm -In n IS
· grea t er than the corresponding term
ll

.!_ o! the harmonic series (which diverges).


1l
0
b) Comparison test 11. If there exists a finite and nonzero limit Jim n
b11 n ""x
(in particular, if an' bn), then the ser1es (I) and (2) converge or d1verge at
the same time.
Example 3. The series
1+ 3I + 5I + " · 2n -1I
+ "' +
<11verges, smce
. ( -1- : -
IJm 1 ) = 1- =/: 0
n-+"' 2n-J n 2 '

whereas a series with general term _!._ diverges.


n
Example 4. The series
I I 1 1
2-1 +22 -2 +23 -3 + · · · +2n-n + "'
converges, since

}~mao(2"~n :;n)=l, i.e.,

I
while a series with general term 2" converges.
Sec. lJ Number Series 295

c) D'Alembert's test. Let a11 >0 (after a certain n) and let there be
a limit
lim an+'=q.
n-> oo an
Then the series (I) converges if q <I, and diverges if q > 1. If q=l, then
it is not known whether the series is convergent or not.
Example 5. Test the convergence of the series
1 3 5 2n-1
2+2 +2a + · · · +"2"+ · ·•
2

Solution. Here,
2n-l 2n+I
a11=---pr-• an+1= 2n+1
and
I+.!__
lim an+'= lim (2n + I) 2n lim ~-_!_
n ... , a, 11
n .. oo 2 j- 1 (2n- I) 2n-.oo _ _!_-2'
1
211
Hence, the ~iven series converges.
d) Cauchy's test. Let an>- 0 (after a certain n) and let there be a limif
lim n/{l -q
fl-+JJ v "-.
Then (I) converges if q <I, and diver~-:es if q >I. When q= 1, the question
of the convergence of the serirs remains open.
c) Cauchy's integral test. If a,= f (11), where the function f (x) is positive,
rnonotonJcally decreasing and continuous for x>-a>-1. the series (I) and the
integra\
(X)

~ f (x) dx
a
converge or diver~e at the same time.
By means of the integral test it may be proved that the Duichlet sertes
00

L. ~p
11=1
(3)

convcn.(es if p > I, and diverges if p ~I. The convergence of a large number


of sem•s may be tested by comparing with the corresponding Dirichlet
series (:~)
Example 6. Test the following series for convergence
I I I I
r:2 + 3·4+ 5·6+ · · · + (;-;:;2,-t----:-:1)--;;2~n + " '
Solution. We have
I I I I
a11 =~--:-:-~ 4n 2 - - 1 - 4n 2 •
(2n-l) 211
1--
2n
296 Series [Ch. 8

Since the Dirichlet series converges for p = 2, it follows that on the basis of
comparison test II we can say that the given series likewise converges.
3°. Tests for convergence of alternating series. If a series
Oa azl I I+ I +···+I I+ ... ,
an (4)
composed of the absolute values of the terms of the series (1), converges,
then (1) also converges and is called absolutely converl{ent. But if (I) con-
verges and (4) divPrges, then the series (1) is called conditionally (not abso-
lutely) convergent.
For invPstigating the absolute convergence of the series (1), we can make
use [for the series (4)] of the familiar convergence tests of positive series.
For instance, (1) converges absolutely if

lim
n-+ a:>
I "+
0
U11
1
/ < I or
n
lim
-)o (£1
i'/1 a,1 1< I.

In the general case, the divergence of (1) does not follow from the diver·
0
gence of (4). But if lim / n+a/ >I or lim j11an I> I, then not only does
n -t> co an n-+ e~:~
(4) diverge but the series (I) does also.
Lei bniz test If for the a \tern at i ng series
b 1 -b 2 +b 8 -b~+ ... (b,.;::.O) (5)
thefollowing conditions are fulfilled: l)b 1 ?-b 2 ~b 3 :;:: . . . ;2) lim b11 =0,
n-+""
then (5) converges.
1n this case, for the remainder of the series Rn the evaluatiOn
IR,.I~b,.+a
holds.
Example 7. Test for convergence the series

I- ( 32)z - (3)'
5 + (4)4 n )"
7 +- .. +(-!) n(ll-l)( 2n-l + ... 2

Solution. Let us form a series of the absolute values ot th~ terms of


this series:
2
1+' ( 32 ) + ( 53 )' + ( 74 )4 + ·· · + ( 2n-
n )"
I + .. ·
Since
lim
n -+ ..,
V(-
2n- 1
11
- ) " =lim
n -+ "'
_n_= lim _ _=.!_
2n - I n -+ oc _ ..!._ 2 '
1
2
n
the series converges absolutely.
Example 8. The series

1-~+i-- ... +(-1) +1,*+ ... 11

converges, since the conditions of the Leibniz test are iulfilled. This series
converges conditionally, since the series
I I I
1+2+3+ .. ·+n-+···
diverges (harmonic series).
Sec 11 Number Series 297

Note. For the convergence of an alternating series it is not sufficient that


1ts general term should tend to zero. The Leibniz test only states that an
alternating series converges if the absolute value of its general term tends
to zero monotonically. Thus, for example, the series
I I I I I I
l-5+2- 5 +3-''' +k-5k+··· 2

diverges despite the fact that its general term tends to zero (here, of course,
the monotonic variation of the absolute value of the general term has been
violated). Indeed, here, S 2k=S~+S~, where

· I • I I • (I I
Sk=l+2+3+ ... +k' Sk=- S+52+ ... +5kI ) '
and lim S~ = oo (S~ is a partial sum of the harmonic series), whereas the
k ... "'
limit lim S~ exists and is finite (S~ is a partial sum or the convergent geo-
k ... ,
metric progression), hence, lim s2k = 00.
k .... "'
On the other hand, the Leibniz test is not necessary for the convergence
of an alternating series: an altrrnating series may converge if the absolute
value of its general term tends to zero in nonmonotonic fashion
Thus, the sertes
I I I 1
J-22 + 3' -42 + · " + (2n - I ) 3

converges (and it converges absolutely), although the Leibniz test is not ful-
filled: though the absolute value of the general term of the series tends to
z~;>ro, it does not do so monotonically.
4°. Series with complex terms A series with the general term C11 =a11
fiJ
+
+ 1b11 W=- I) converges if, and only if, the series with real terms~ a 11
n=r
"' b11 converge at the same time; in this case
and ~
t1=1

"' 00 r1>

~ C11 = ~ a,.+i~b 11 • (6)


fl=l n=r
The series (6) definitely converges and is called absolutely convergent, if the
series

~ IC
11=1
11 I=
11=1
±V a~+ b~1 ,
whose trrms are the moduli of the terms of the series (6), converges.
5°. Operations on series.
a) A convergent series may be multiplied termwise by any number k;
that 1~, if

then
298 Series (Ch. 8

b) By the sum (difference) of two convergent series


a1 -/-a 2 -/- ••• +an+ ... =8 1, (7)
b1 -/-b 2 -/- ••• -/-bn+ ... =S 2 (8)
we mean a series
(at± bt)-!-(a2 ± b2)-/- ... -/-(an± bnH- ... =St ± S2.
c) The product of the series (7) and (8) is the serit>s
ct + c2 + ... + cn + ... ' (9)
where C11 =a 1 b11 -/-a~bn_ 1 -/- . . . +anb 1(n=l, 2, ... ).
If the series (7) and (8) converge absolutely, then the series (9) also con-
verges absolutely and has a sum equal to S 1S 2 •
d) If a st>ries converges absolutely, its sum remains unchanged whrn the
terms of the series are rearranged. This property is absent if the series con-
verges conditionally.

Write the simplest formula of the nth term of the series using
the indicated terms:
I I I I I
24ot. 1 -t- 3 -t- 5 + 7I + ... 2404. 1+4+9 +16+ ...
I I I I 3 4 5 6
2402. 2+-:r+6+ s+ ... 2405. 4 + g-+ 16 + 25 + ...
2 2 4 G 8
2403. 1 -t- 2 -t- 43 -t- 84 + ... 2406. s+s +rr +14 + ...
I I I I , I I
2407 · 2·+5+ 12+2o 1 30+42+ · · ·
1-3 1·3·5 1-3·5·7
2408 · 1 +J:4+1·4·7+1·4-7·10+ ...
2409. 1- I + 1 - I -t 1 - 1 + .. .
2410. I +i-+3 +++5+-i- + .. .
In Problems 2411-2415 it is required to write the first 4 or
5 terms of the series on the basis of the known general term a11 •
~-2 I
2411. a11 =n 2 +l. 2414. a 11 =l 3 +(-l)"j"'
2412. (-l)"n
~
( 2 +sin n
2
n) cos mt
2415. a,.= nl
2413 a = 2 -/-(-l)"
" n2
Test the following series for convergence by applying the com-
parison tests (or the necessary condition):
2416. 1-1 + 1-1 + ... +(-tt-l+ ...
2 8
0
1:;
417 · s2 +2 I ( 2)
5 +3I ( 2)
s + · .. + nI ( 2 )"
5 + .. · ·
s c. I) Numb::r Series 299
~--------------------------------------------

2 3 4 n+l
24 t 8· 3 + 5 + 7 + . . . + 211 1- l -t- ...
y'To- vm + Vto- ... t!+ vlO + ...
1
I I I . (-I)" t
2419. -j-

I I I I
2420. 2+4+6+ ... +21! + ...
h+ft +~1 + · · · +Ion +1 + · · ·
1
242 1.
1
2422. )-+
r1·2
)-+.)
r2·3 r3·4
+ ... +· .rrn(ll+l)
-+ ...
2 3
2 2 2"
2423. 2+ 2 + 3 + ... + 11 + ...
2424. l +- .J-+) + ... + .~--+· ...
r2 r3 rn
I I I I
52+ 82 + · · · + t:Jn _
I

2425. 22 1 1)2 + ···


2426.
, V2 V3
-+--+--+ Vii
2 :J V.2 4 V3 · · · . . (11L -I· -I)-Vn
--i-
··· 1

Using d'Aiembert's test, test the followin;~ ~enes for conver-


gence:
24 ~,, 7 . V!f
I L 3
T 2
5 + 1 2rz- I 1+
-2 ~,.-2 ..• T ( V2t I . . .
2 2·5 2·5·8 2·5·8.- .(311-l)
2428. T + r:5 + fT.ii + · · · + 1·5·9 ... (411-3) + · · ·
Test for convergence, using Cauchy's test:
,
2429. 2
T+ 3 (:3) + (4)• 2

5 + ... + (n+l)" 1- ••• 2n -1

,2430. 2I + (2)'
5 + (3)s
-8 + ... + (311-1
n
+ ... )211-•

Test for convergence the positive series:


I
2431. 1 + 2'• + -3,I + ... + -1I -j- ...
'. 11
I I I I ,
2432. 3-j-lf-t-15-t- ... +(n-t-1)2-IT···
2433 · 1~4+4~7+7.\o+ ···+(3n-2)\3n+l)+ .••
2
I 4 9 11
2434. :r+g-+ 19+ ... +2n2+1 + ...
r:
243 t>. I + 2 +3+ . n ,
2 5 10 ···Tfi +1-r- ..• 2

3 5 7 ~+I
24 36 . 22·33 + 3 -4 + 4 ·5 + ... + (11 + 1) 2
2 2 2 2 (11 + 2) 2 + ...
300 Serit>s (Ch. 8

!+(~).+( 1~)'+
11

2437. ... +(3}~1) + .. .

2438. ( i) ++ ; + (.~)++ ••• + (;~! !) ~ + .. .


2439. e +e•8 +e•27 + ... +n•e11 + ...
1

2 211 - 1
2440. 1 + 2• + 34, + ... + ~~,11 + .••
11 2! 3! n!
24 41. 2+1+2.+1 +2'+1 + ... +2
+1+ •••11

2 4 211-1
2442. 1 +rr + 21 + ... + (n- 1) 1 + .. .
1 1·3 1·3·5 1·3·5 ... (2n-1)
2443• 4+4·8+4·8·12+ " ' + 4·8·12 ... 4n + ...
(II)• (2!)• (3!)• (nt)•
2444. 21 +41 + m + •.. + (2n)! + ...
2445. 1000 + 100~:!002 + 1000·11~~-2; 1004 + .••
1000·1002·1004 ... (998+ 2n)
... + 1·4·7 ... (311-2)
+ •••
2 2·5·8 2·5·8... (6n -7)(6n-4)
2446. 1+ 1·5·9+ · •· + 1·5·9 ... (8n-11) (Sn-7) + · '·
2447 1 + 1·5 + 1·5 ... (4n-3) +
· 2 2·4·6+ · · · 2·4·6 ... (4n-4) (4n-2) ··•
2448 ..!__ -1- !_:_!_! + 1·11· 21 +
1-11 · 21. .. (!On -9) + +
• 1!3! 51 "· (2n-1)1 "•
1·4 1·4·9 1·4·9 ... 11 2
2449 . 1+1·3·5+ 1·3·5·7·9+ ···+1·3·5·7·9 ... (4n-3) +···
,., rt>

2450. ~ arc sm
. y--.
I ~-1_1
2455. .....,.n
11=1 n Inn:
11 ~a

00 00

2451. ~.
~n !~ 2 n •
1 2456.
sm n•.
11=1 11=2
00 00

2452. ~ ln ( 1 +~). 2457. L n·ln n·\n In n •


11=1 11=z

"' rt>

2453. ~ l n nt+l
n• • 2458. ~nzl n'
n=t 11 =z
00

2454. ~l:n • 2459. Lo"" Y n (nI +I) •


n=z 11 = 1
Sec. 1) Number Series 301

QO
"'
2460. L
n=J Vn(n+l)(n+2)•
' 2465. :E~
nn •
n=l
QO QO

246t. L.
n=an Inn+ Vln'n
' . 2466. L2nn1
n=l
nn •
QO ao
2462. ~.., 1
2467. L3nnl
~nt/n-vii· n" •
n=l
"" Vn ex,

2463. :E (2n -
n=1
v
1) ( 5 ~ n -1)
. 2468*. ~ennl
n=1
tln •

ao

2464. L. (1- cos ~ ) .


n=l
QO

'\...., ____!.___n ••
2469. Prove that the series ~nPlnq
n=2
1) converges for arbitrary q, if p>1, and forq>1, if p=l;
2) diverges for arbitrary q, if p < 1, and for q ~ 1, if p = 1.
Test for convergence the following alternating series. For con·
vergent series, test for absolute and conditional convergen4!:e.
I
2470. 1- 3 + 5I - ... ·f-
(-1)"- 1
2n-l + ...
I I (-1)"- 1
2471. 1-·;r+-~-
r 2 V3
... + Vn + ...
I I (-1)"- 1

2472. 1--;r+-g- ... + n• + ...


2 3 (-l)n-ln
2473 · 1 -7+13- ... + 6n-5 + ...
2474 3
· r:-2-2·3
5 + 3.4-
7 f-(
... · -
2n+l
1)n-J n(n+l) + ...

2475.

2476.

2477.

2478.

2479.
302 Series [Ch. 8

L"" (-1)n 1: n.
00

2481. 2482. L (-l)n-J tan n .~-.


n=l r n
n=t
2483. Convince yourself that the d' Alembert test for conver-
gence does not decide the question of the convergence of the
"" an, where
series ~
1!=1

whereas by means of the Cauchy test it is possible to establish


that this series converges.
2484*. Convince yourself that the Leibniz test cannot be
applied to the alternating series a) to d). Find out which of
these series diverge, which converge conditionally and which con-
verge absolutely:
I I I I I I
a) Y2--I-Y2+1+Y3-I-Y3+1+Y4- 1-Yf+l+ ...

I
( a2k-t =
I I
y k; I I
I- I ' a2k = - y k; I + I) ;

b) 1-3 + 2-33 + 22 - 35 + ...


1
( a2k-1 = 2k-• • a2k = - 3 2L1) ;
I I I I I
c) l-3+~-3 2 +5-33+ · · ·
( a2k-• = 2k I I' a2k=- ~k);
I I I I I
d) 3- 1 -1-y-s+rr-g-+ ...

(a2k-•=4kl I' a2k=-4kl3).


Test the following series with complex terms for convergence:
L n (2+Q~
00 00

2485. 2n • 2488. L.i~.


n=t n==l

2486 .. L n (2t-l)n
00

3" • 2489.
""
.Lv~ .
n=t n=• n+l
00

2487. Ln(3~i)n'
n=t
2490. I.""
n=• (n+i)
I
Yn ·
Sec. I) Number Series 303

~ ~

2
2491
'
~ 1
""-- [n + (2n -I) i) 2 •
2492.~[n(
.._.
-i)+ 1 J"
n (3-2i) -3i '
n=t n=t
I I
2493. Between the curves y = and y = 2X and to the ri~ht
8
X
of their point of intersection are constructed segments parallel
to the y-axis at an equal distance from each other. Will the sum
of the lengths of these segments be finite?
2494. Will the sum of the lengths of the segments mentioned
in Problem 2493 be finite if the curve y X is replaced by the =-;
I
curve y=-?
X

2495. Form the sum of the series L


~

n=t
1
t n and
rn
L (-l~:-n.
n===I
Does this sum converge?
«>

~ 211-l
2496. Form the difference of the divergent series £-
1
n=l
"'
and L 2~
Jl:;:;J
and test it for convergence.
2497. Does the series formed by subtracting the series
oc 00

~ 2rz -1
""-- I
~ -n1 converge?
from the series ""--
n=t 11=-t
2498. Choose two series such that their sum converges while
their difference diverges.
2499. f-orm the product of the series L"'
ll=l n
.~-- and
r 11
L"' 2}_
n=l
1 •

Does this product converge?


2500. Form the series ( 1
this series converge?
+; +
+ + ... + 2}_ 1 + ... Does r
2501. Given the series 1 + ~ 1 - ~~ + ... + + . . . Estimate <::>"
the error committed when replacing the sum of this series with
the sum of the first four terms, the sum of the first five terms.
What can you say about the signs of these errors?
2502*. Estimate the error due to replacing the sum of the
series
} + 2\ ( ~ ) + ~! ( ; ) + •••+ h(}) n + •••
2 1

by the sum of its first n terms.


304 Series [Ch. 8

2503. Estimate the error due to replacing the sum of the


5eries
1 1 1
1 +21+31+ ... +iii+·-·
by the sum of its first n terms. In particular, estimate the accu-
racy of such an approximation for n = 10.
2504**. Estimate the error due to replacing the sum of the
series
1 1 1
1 + 22 + 32 + ... + nz + ...
by the sum of its first n terms. In particular, estimate the accu-
racy of such an approximation for n =I ,000.
2505**. Estimate the error due to replacing the sum of the

+r
f.eries
1 +2 (! r
+ 3 ( + ... +n ( + ... +rn-2
by the sum of its first n terms.
(yJ

2506. How many terms of the series L <-~n-t does one have
n=t
to take to compute its sum to two decimal places? to three
decimals?
2507. How many terms of the series L (2n ~I) 5n
n~•
does one
have to take to compute its sum to two decimal places? to three?
to four?
I I I
2508*. Find the sum of the series r:-2+ 2 . 3 + 3 . 4 + .. · +
I
+n(n+l)+ · · •
2509. Find the sum of the series
Vx+<Vx-Vx)+ <Vx-Vx)+ ... +<-k+ Vx-zk- Vx)+ ...
Sec. 2. Functional Series
1°. ~egion of convergent'!. The set of values of the argument x for which
the functional series
(l)
converges is called the region of convergence of this series. The function
S (x) = lim Sn (x),
n .. ®

where Sn(x)=f 1 (x)+f 2 (x)+ ... +fn(x), and x belon~s to the region of con-
vergence, is called the sum of the series; Rn (x) = S (x) -Sn (x) is the remainder
of the series.
Sec. 21 Functional Series 305

In the simplest cases, it is sufficient, when determiping the region of


convergence of a series (1), to apply to this series certain convergence tests,
holding x constant.

Diverges Converges
,((('4{'-'9 Diverges
-J
j<<<<<<<<

-1 0 1
•X
F1g. lG4

Example 1. Determine the region of convergence of the series


2 3
x+l+(x+l) +(x+l) + +(x+l)n+ (2)
l ·2 2 • 22 3 · 23 • • • n •2n · ..
Solution. Denoting by u,. the general tE-rm of the series, we will have
lim llln+tl= lim lx+lln+l2nn _lx+ll
IU 11 1 n-..<X>2 11 + 1 (n+l)lxln- 2
n-+<X>
Using d' Alembert's test, we can assert that the series converges (and converges
absolutely), if I xt II< l, that is, if -3 < x<I; the series diverges, if

lxtll >I, that is, if -oo <x<-3 or l <x< oo (Fig. 104). When x=l
we get the harmonic series l + ~ +-}+ ... ,which diverges, and when X = - 3

we have the series -I+~--}+ ... , which (in accord with the Leibniz
test) converges (conditionally).
Thus, the senes converges when -3..;;;;x< l.
2°. Power series. For any power series
C0 -j-c 1 (.t-a) +c 2 (x-a)• + ... +c,. (x-a)n + ..• (3)

(en and a are real numbers) there exists an interval (the inlert•al of conver-
gence) I x-a 1 < R with centre at thP point x=a, with in which the series (3)
converges absolutely; for 1 x-a 1 > R the series diverges. In special cases, the
tadius of convergence R may also be equal to 0 and oo. At the end-points of
the interval of convergence x=a :1: R, the power series may either converge
or diverge. The interval of convergence is ordinarily determined with the
help of the d'Alembert or Cauchy tests, by applying them to a series, the
terms of which are the absolute values of the terms of the given series (3).
Applying to the series of absolute values
I Co I+ I C1 II x-a I+ ... +I c,. II x-a In+ ...
the convergence tests of d'Aiembert and Cauchy, we get, respectively, for the
radius of convergence of the power series (3), the formulas

R= l
lim ;YI C11 I
and R= nlim
-+oo
1.5!...
+
C11
j
1 •
11-+00

However, one must be very careful in using them because the limits on the
right frequently do not exist. For example, if an infinituce of coefficients c,.
306 Series (Ch. 8

vanishes [as a particular instance, this occurs if the series contains terms
with only even or only odd powers of (.t-a)), one cannot use these formulas.
It is then advisable, when determining the interval of convergence, to apply
the d' AI em bert or Cauchy tests directly, as was done when we investigated
the series (2), without resorting to general formulas for the radius of con·
vergence.
If z=x+tY is a complex variable, then for the power series
c0 +c1 (2-2 0 ) +c 2 (2-20) 2 + ... +en (z-z 0 )n +... (4)
(Cn=an+lbn, z0 =x0 +iy0 ) there exists a certain circle (circle of convergence)
12-20 1 < R with centre at the point z = z0 , inside which the series convcr~es
absolutely; for 1z-z 0 I> R the series diverges. At points lying on the cir·
cumft>rence of the circle of convergence, the series (4) may both com•er~e and
diverge. It is customary to determine the circle of convergence by means of
the d'Alembert or Cauchy tests applied to the series
leo I+ lcii·IZ-Zo 1+1 c2I·J z-z. !2+. ··+I Cn 1·1 z-zo In+ ... ,
whose terms are absolute values of the terms of the given series. Thu~. for
example, by means of the d' Alembert test it is easy to see that the ctrcle oi
convergence of the series
z+l+(z+l) 2+(z+l) 3 + +(z+W+
1·2 2·2 2 3-23 • • • n·2" ···
is determined by the inequality lz+ II< 2 [tt is sufficient to repeat the cal-
culations carried out on page 305 which served to determine the interval oi
convergence of the series (2), only here x is replaced by z]. The centre of
the circle of convergence lies at the point z =-I, while the radtus R of tltt~
circle (the radius of convergence) is equal to 2.
a•. Uniform convergence. The functional series (I) converges uniformly on
some interval if, no matter what e > 0, it is possible to lind an N such that
does not depend on x and that when n > N for all x of the given tntervdl
we have the inequality I Rn (x) I< e, where R11 (x) is the remainder of th"
given series.
If lfn(x)l~cn (n=l, 2, ... )when ae;;;;;;xe;;;;;;b and the number series
"'
~ en converges, then the functional series (I) converges on the tnterval
n=t
[a, b' absolutely and uniformly (Weierstrass' test).
The power series (3) converges absolutely and uniformly on any interval
lying within its interval of convergence. The power series (3) may be term-
wise differentiated and integrated within its interval of convergence (for
I x-a I < R); that is, if
C0 + C1 (x-a) + c2 (x-a) 2 + ... + Cn (x-a)n + ... = f (x), (5)
then for any x of the interval of convergence of the series (3), we have
C1 + 2c2 (x-a) + ... + ncn (x-a)"- 1 + ... =f' (x), (6)
X X X X

~ c0dx+) c1 (x-a) dx+ ~ c2 (x-a) 2 dx+ •.. +) c11 (x-a)" dx+ ... =
Xo X0 X0 x0
oo
""
=£.....en
1
(x-a)"+ (x0 -a)"+
n +1
1

=
5f
X

(x) dx (7)
n=o ~
Sec. 2] Functional Series 307

[the number x0 also belongs to the interval of convergence oL the series (3)].
Here, the series (6) and (7) have the same interval of convergence as the
series (3).

Find the region of convergence of the series:


00
"'
2510. L: ,:x. 2518. L:ln! x" ·
n=l 11=1
00 TJ

2511. L (-1)"+' nix •


11:1
2519. L (211~ I) x" •
11=1

"'
~..,.(-I)"+'
2512. ""'"""-~ _I_
nlnx. 2520.
TJ

v·n
I. (x--2)' 1•
11=1 ll-=1

2513. L"' sin(2n-l)x 2521. L


"I)

2n+l
ll=l
(211-1) 2 •
fl='J
(11 + 1) x'" ·
5

2514.
"'
L2". X
Stn.-;n. 2522.
L"' (-1)n-t
1!·3" (x-5) 11 •
11=0 " ll=l

"'
2515**. L c~;,-~.
n=o
~... ~
2523. -... n''.
11=1 X

"' 00

2516. L (-1)"t' e-""" .'·


n=.o
2524*. L (x" + 2"1x") •
11=-1
00
"'
2517. L:~
x".
11=1
2525. L
n= -1
x".
Find the interval of convergence of the power series and test
the convergence at the end-points of the interval of convergence:
L (n+2n 1)-1- x
Cf' 7)
5 211
2526. ")"""
.._.X.
II
2531 . I ·
n=o n=o
,
L. 11·2~~~
00

2527. 11 • 2532. L (-1)" (2n + 1)' x".


H=t 11=0

2528.
L"' 2tt-l • x<~~-t
2533.
00

L x"nl •
rz=t 11=1

2529.
L 00
2"-Jx~n-l
2534. L
00

ntx".
(4n-3) 1 •
11=1 /1"'1
00

L (-l):-
00
1
2530. x" • 2535. L:·~
n" •
r&=l n=t
308 Series [Ch. 8

... QO

2536. L. (2nn+ 1y~~-· x!'. 25510 .L


(x + 5)z11-1
:./no4 11 •
n=l 11=1
... ...
2537. L 3n'xn•. 2552 ~
• ~ (2n-1)211
(x- 2 )
11


n=o
... ...
2538o
~n~t(-ir· 2553. L (-1) 11 +I X

2539. L... n!xn


nil
X (2n-1) 211 (x-1 ) 11
(3n-2)zn •
11=1
0
...
... 2554. ~ n!(x+3)n
2540.
L xll-1 ~ nn
t1=2
n·3 11 ·lnn • ...
«> 2555. L= (n+l)!n
(x+ 1)11
(n+l) 1 0

2541. L.x••l. 11 1
n=l ...
QO 25560 L (x-3)zn
2542**. L n! xnlo 11
= (n+l)ln(n+ll ·
...
1
11=1

,., ~
QO

xn' 25570 L (-It+l X


2543 . ~ 2"-Jnn • 11=1
11=1 (x-2) 11
"' nn x(n-t-1)ln(n+l).
2544*. L. :~~ .
11=1

2545o ~t-l) 11 _ 1 (x-S)II

*)"'<x-lt.
TL=J

~ \ n·3 11 ...
&

11=1
2559*. 1+
.L"' <xn· 53r.
(
1
2546 .
rE=J
2560.
2547 "' (x-1)211
~

2548.
o ~ no9 11
11=1

L
n=l
(-l)"_J(x-2)2" •
2n

~561.
=
,; ( _ 1>"

X (x-2) 11
o
V~2
n+l X

...
L (3n-2) (x-3)n
QO

2549o ~ (x+ 3l
11
2562.
~ n2 •
11=1
n=o (n+l)22''+'
QO

2550. Ln 11
(x + 3t. 2563. L (-It 3 11
(x- )
n=l n=o t2n +1) Yn -t I .
Sec. 2] Functional Series 309

Determine the circle of convergence:


<tJ

2566. ~ (z- 2i)"


4... n·3n •
n=o ll=l
<tJ <tJ
'\..., ~ z•n
2565. 4... ( 1 + ni) 2 n. 2567. 4... 2 , •
n=o n=o
2568. (1 +2i)+(1 +2i)(3 +2i)z+ ... +
+( + +
1 2i) (3 2i) ... (2n + 1 + 2i) zn + ...
z z2
2569. 1+ 1_;+(1-i)(l- 2i) + ...
zn
' ' · + (l-i)(l-2i) ... (1-ni) + · · •
<tJ

2570. 1: Cn~ ~jr zn.


2

n=o
2571. Proceeding from the definition of uniform convergence.
prove that the series
1+x+x•+ ... +xn+ ...
does not converge uniformly in the interval (-1, 1), but con-
verges uniformly on any subinterval within this interval.
Solution. Using the formula for the sum of a geometric progression, we
get, for 1 x 1 < I,

Within the interval (-I, I) let us take a submterval [-I +a, 1-a], where
a is an arbitrarily small positive number. In this subinterval lxl:s;;; 1-a,
ll-x l ~ u and, consequently,
ll-a)"+'
I R" (x) I..; u
.
To prove the uniform convergence of the given series over the subintervat
1-l+a, 1-a]. it must be shown that for any e>O it is possible to choose
an N dependent only on e such that for any r1 > N we will have the ine·
quality I Rn (x) 1 < e for all x of the subinterval under consideration.
(1-a)"+'
Taking any e>O, let us require that <e; whence (1-a)''+'<ea,
a
. In (ea)
(n +I) In (1-a) <In (ea), that 1s, n + 1 >In (l-a) [since In (1-a) < 0] and
In (eu) . In (ea) .
n>ln( 1 -a)-l. Thus, puttmg N=ln(l-a)-1' we are convmced that
when n > N, 1 R, (x) I is indeed less than e for all x of the subinterval
(-1 +a, 1-a] and the uniform convergence of the given series on any sub·
interval within the mterval (-1, I) is thus proved.
As for the entire interval ( -1, 1), it contains points that are arbitrarily
. . x"+'
close to x= 1, and since hm Rn (x) = hm - - = oo, no matte• how large n is •
.t-+1 .t-+• 1 -x
310 Series [Ch. 8

points x will be found for which R,. (x) is greater than any arbitrarily large
number Hence, it is impossible to choose an N such that for n > N we
would have the inequality 1 R., (x) I < e at all points of the interval (-I, I),
and this means that the convergence of the series in the interval (-I, I)
is not uniform.

2572. Using the definition of uniform convergence, prove that:


a) the series
x x2 x"
1+1!+2!+ ... +/iT+ ...
converges uniformly in any finite interval;
b) the series
x2 x' ~· (-l)"- 1 x'"
T-2+;r-·· ·+ n +-"
converges uniformly throughout the interval of convergence
(-1, 1);
c) the series
I I I
1+¥+ 3x+ ... + n:x+ ...
converges uniformly in the interval (1 -1· cS, oo) where c5 is any
positive number;
d) the series
(x 2 -x 4 )+(x 4 - X6 )-t-(x 8 -x8 ) + ... +(x 211
-X
211
+
2
) + ...
converges not only within the interval (-1, 1), but at the extre-
mities of this i nterva I, however the convergence of the series in
(-1, 1) is nonuniform.
Prove the uniform convergence of the functional senes in the
indicated intervals:
.,
2573. Lx" on the interval [-1, 1].
n=l na
2574. L"' sinnx
2"
over the entire number scale.
.,
11=1

2575. ~ ( -1 )"- 1 Vnn on the interval lO, 11.


Applying termwise differentiation and integration, find the
sums of the series:
x2 x3 x"
2576. x+2+3+ 0 0 0 +n-t- 0 ••

~ ~ x"
2577. x- 2 + 3 - ... +(-1)"- 1 n-+ ...
x3 xs xcn-1
2578. x-t- 3 -t- 5 + ... + 2n-I + ...
Sec. 8] Taylor's Serzes 311
xa xs x2n-1
2579. x- 3 + 5 - ... +(-1t-l 2n-l + ...
2580. 1 +2x+3r + ... (n+ 1)xn+ ... +
2581. 1- 3x2 -1- 5x 4 - ••• + (- w - l (2n-1) X 211 - 2
+ ...
2582. 1· 2 + 2 · 3x +3 · 4x2 + ... + n (n + 1) x"- 1 + ...
Find the sums of the series:
I 2 11 3
2583. -+•+3+
X X X
... +n+.
X
··
xs x• x4n-s
2584. x+s+g+ ... + 4n-3+ ...
* I I I (-l)n-1
25 85 · 1-3·3+5.3 2 -7·3•+···+(2n-1)3"- 1 +··•
I 3 5 2n-l
2586. 2+22 +23+ ... +2,.--+ ...

Sec. 3. Taylor's Series


1". Expanding a function in a power series. If a function f (x) can be-
expanded, in some neighbourhood I x-a I< R of the point a, in a series of
powers of x-a, then this series (called Taylor's series) is of the form
.- (a)
f (x) -f (a)+ f (a)(x-a) + 2! ' 2 '(Ill (a)
(x-a) + ... + -
. r n
- (x-a) +... (I)
111

When a=--0 the Taylor series is also called a Maclaurin's series. Equation (I)
holds 1f when 1 x-a I< R the remainder term (or simply remainder) of the
Taylor series

R,.(x)=f(x)- [
II tk)
f(a)Lf k/a)(x-a) 1'
J ---+0
k=l
as oo.
11 _.....
To evaluate the remainder, one can make use of the formula
(x a)"+ 1
R,. (x) = (;t-l )I ft"+ 11 [a +0 (x-a)J, where 0 < 0 <I (2)

( Lagral1£!e's form).
Example l. Expand the function f (x) =cosh x in a series of powers of x.
Solution. We flnd the derivatives of the given function f (x) =cosh x,
f' (x) = smh X, f" (x) =CCSIJ X, f"' (X)=sin!Jx, ... ; genl'ra))y, fl 11 l (x)=COSIJX,
if n is even, anrl [l"l(x)=sinhx, if n IS odd. Putting a=O, we get f(O)=I,
{'(0)=0, f"(O)=I, /"'(0)=0, ... ; generally, [!" 1 (0)=1, if n is even, and
{t"l (0) = 0 if n is odd. Whence, from (I), we have:
x• x4 x:n
coshx=l +2r+4r+ ... + ( n)l + ... (3) 2
To determine the interval of convergence of the series (3) we apply tha-
d' Alembert test. We have
. I
hm x•n + • w
:A-
n-.oo (2n+2)1 (211)1
I= n-."'(2n+l)
.
hm x2
(2n+2)
0
312 Serie~ [Ch. 8

for any x. Hence, the series converges in the interval - oo < x < oo. The
remainder term, in accord with formula (2), has the form:
.tn+J
Rn(x)=(n+l)!cosh8x, if n is odd, and
.tn+J
Rn (x) = (n +I)! sinh eX, if n is even.

Since 0 > e >I, it follows that


e~" +e-~"
I cosh e X I= 2 ~ el X I •

lxln+t !..:_Ln
and therefore IRn(x)l~(n+l)lefxl. A series with the general term n!
converges for any x (this is made immediately evident with the help of
d' Alembcrt's test); therefore, in accord with the necessary condition for
convergence,
lim _x I __
ln+J =O,
n-> rr; (n+l)l

and consequently lim Rn (x) = 0 for any x. This signifies that the sum of the
n-+cr;
sen~s l3) for any x is indeed equal to cosh x.
2". Techniques employed for expanding in power series.
Making use of the principal expansions
x x2 xn
). r=l+n+2f+ ... +iiT+ ... (-oo<x<oo),
x .\a xs x2n+J
II. sinx=TI-3i+5f- ... +(-l)n (2n+l)l+ ... (-oo<x<oo),
x2 x~ x2n
lll. cosx=l-2i+4!- ... +(-l) 11 ( 2n)l+··· (-oo<x<oo),
IV I m m (m-1)
. ( +x) ·= I +nx+
m
21 x + ...
2

... +m(m-l) .. }m-n+l)xn+ ... (-I <x<l)*),


n.
~:2 xa xn
V ln(l+x)=x-1'+3'- ... +(-l)n-J n-+ ... (-1 <x~l),

and also the formula for the sum of a geometric progression, it is possible,
in many cases, simply to obtain the expansion of a given function in a po-
wrr series, without having to investigate the remainder term. It is sometimrs
advisable to make use of termwise differentiation or integration when expan-
ding a function in a series. When expanding rational functions in power
·!wries it is advisable to decompose these functions into partial fractions.

"') On the boundaries of the interval of convergence (i.e., when x= -I


and x = 1) the expansion IV behaves as follows: for m ~ 0 it converges abso-
lutely on both boundaries; for 0 > m > -1 it diverges when x =-I and
conditionally converges when x = 1; for m -=o;-1 it diverges on both boun-
daries.
Sec. 3] Taylor's Series 313

Example 2. Expand in powers of x *) the function


3
f(x) = (l-x) (I+ 2x) ·

Solution. Decomposing the function into partial fractions, we will have


1 2
f(x)=l-x+l+2x'
Since
(J)

1
- - = 1
1-x
+ x +x + ... =
2
'\...,
k..t,
x" (4)
n=o
and

1
~ 2 x=1-2x-f-(2x) 2 - ••• = L"' (-1)"2"x", (5)
n=o
it follows that we finally get

The geometric progressions (4) and (5) converge, respectively, when 1x 1 < 1
I I
and I x 1< ; hence, formula (6) holds for I xI < , i.e., when
2 2
I I
-2<x<2·
3°. Taylor's series for a function of two variables. Expanding a function
of two variables f (x, !/) into a Taylor's senes in the neighbourhood of a
point (a. b) has the form
1 [ (x-a>ax+<y-b)ay
{(x,y)={(a, b)+TI 11 a] . 1 [ (x-a>ax+
f(a, b)+ 21 a
+(y-b) ~ r {(a, b)+ ... -f- ~ [<x-a) :X +(y-b) fv]" {(a, b)+ .. , (7)

If a=b=O, the Taylor series 1s then called a Mac/U!utn's senes. Her(' the
notation is as follows:
a a] of (x, y) + iJf (X, !/)
[ (x-a> ax+ (y -b) ay t (a, b)= ax
(X-a) (y-b);
iJy
x=a x=a
y=b y=b
iJ J iJiJ2f (x, y)
(x....:....a)a;;+<y-b)ay {(a,b)= axz
2
(x-a) 2 -l-
[
x=a
IJ=b

(x-a)(y-b)+ iJ2f(x,
iJy2
y) (y-b)l and so forth.
x=a
y=b

•) Here and henceforward we mean "in positive integral powers".


a14 Series [Ch. 8

The expansion (7) occurs if the remainder term of the series

Rn(x, u>=f(x, u>-{t<a.bl+ ±. ~[<x-a)~+<u-b>~rf(a,


k=l
b)} -o
as n-+ oo. The remainder term may be represented in the form

Rn (x, y) + 1 [
(n + l)l (x-a)
a (y-b) ayaJn+l f (x, y)
ax+
x=a+fJ(x-a)
y=b+fJ(y-b)
where 0 <8< l.

Expand the indicated functions in positive integral powers


of x, find the intervals of convergence of the resulting series and
investigate the behaviour of their remainders:
2587. ax (a> 0). 2589. cos (x +a).
. (
2588 · sm x+4 ·
n) 2590. sin1 x.
2591*. Jn(2+x).
Making use of the principal expansions l-V and a geometric
progression, write the expansion, in powers of x, of the following
functions, and indicate the intervals of convergence of the series:
2x-3
2592. (x-l)". 2598. cos• x.
3x-5
2593. x•-4x + 3 • 2599. sin 3x + x cos 3x.
X
2594. 2600. 9 +x•.
1
2601 . --:--
¥4-.!.2.

2596. sinh x. 2602. In !+;.


2597. cos2x. 2603. In (1 +x-2x").
Applying differentiation, expand the following functions in
powers of x, and indicate the intervals in which these expansions
occur:
2604. (l+x)ln(l+x). 2606. arcsinx.
2605. arctanx 2607. ln(x+Vl+x2 ).
Applying various techniques, expand the given functions in
powers of x and indicate the intervals in which these expansions
occur:
2608. sin" xcos• x. x 2 -3x+ l
2612. x•- 5x+ 6 ,
2609. (1 +x) e-x.
+
2610. (1 ex)a. 2613. cosh' x.
I
2611. Vs+
x. 2614. 4 _x 4 •
Sec 3] Taylor's Series 315

2615. In (x 1 + 3x + 2). X

X 2618. 51n (I~ x) dx.


2616. 5si: x dx.
0
X

2617. ~ e-x• dx.


2619. sYl-x
X

dx
4

Write the first three nonzero terms of the expansion of the


following functions in powers of x:
2620. tan x. 2623. sec x.
2621. tanh x. 2624. In cos x.
2622, eLOH. 2625, eXsinx.
2626*. Show that for computing the length ot an ellipse it is
possible to make use of the approximate formula

s ~ 2na ( 1 - ~) ,

where e is the eccentricity and 2a is the major axis of the


ell ipse.
2627. A heavy stnng hangs, under its own weight, in a ca-
tenary line y =a cosh_.:_, where a=!!_ and H is the horizontal
t.l q
tension of the string, while q is the weight of unit length. Show
that for small x, to the order of x•, it may be taken that the
string hangs in a parabola y=a +·~.
a 2
2628. Expand the function x"-2x 2 -5x-2 in a series of
powers of x-I 4.
2629. f(x)=5x"-4x•-3x+2. Expand f(x+h) in a series of
powers of h
2630. Expand In x in a series of powers of x-1.
I
2631. Expand in a series of powers of x-I.
X

2632. Expand .!.


X
in a series of powers of x
I
+ l.
2633. Expand x•+gx+ 2 in a series of powers of x +4.
2634. Expand x•+~x+? in a series of powers of x +2.
2635. Expand ex in a series of pqwers of x 2. +
2636. Expand Vx
in a series of powers of x-4.
2637. Expand cosx in a series of powers of x-i.
2638. Expand cos• x in a series of powers of x- T.
2639*. Expand In x in a series of powers of : +;.
318 Series [Ch. 8

2640. Expand ~r-X


in a series of powers of -1 +
X
.
, !+x x
2641. What is the magnitude of the error if we put appro-
ximately
1 1 1
e1!:j 2 + 21 +3i+4i?

2642. To what degree of accuracy will we calculate the num-


ber ~ , if we make use of the series
xa
arc tanx=x- 3 + 5xs - .•. ,
by taking the sum of its first five terms when x = 1?
2643*. Calculate the number ~ to three decimals by expand-
ing the function arc sin x in a series of powers of x (see Exam-
ple 2606).
2644. How many terms do we have to take of the series
x2
COS X = 1- 2] + ... ,
in order to calculate cos 18° to three decimal places?
2645. How many terms do we have to take of the series
. xa
smx=x-3!+ ... ,
to calculate sin 15° to four decimal places?
2646. How many terms of the series
X X X2
e =I +li+2i+ ...
have to be taken to find the number e to four .decimal places?
2647. How many terms of the series
x2
In (1 +x)=x- 2 + ... ,
do we have to take to calculate In 2 to two decimals? to 3 de-
cimals?
2648. Calculate V7 to two decimals by expanding the func-
tion V 8 + x in a series of powers of x.
2649. Find out the origin of the approximate formula
V a + X1!:ja
2
+;a (a> 0), evaluate it by means of V 23, putting
a= 5, and estimate the error.
2650. Calculate V 19 to three decimals.
Sec. 3) Taylor's Series 317

2651. For what values of x does the approximate formula


x•
cosx~1-
2
yield an error not exceeding 0.01? 0.001? 0.0001?
2652. For what values of x does the approximate formula
sin x~x

yield an error that does not exceed 0.01? 0.001?


1/1

2653. Evaluate ssinx .


-x- dx to four decunals.
0
I

2654. Evaluate ~ e-xz dx to four decimals.


I

2655. Evaluate ~ Vxcosxdx to three decimals.


0
1
. I s.
Yxx dx to three dectma
2656. Evaluate .) sin
0
1/4

2657. Evaluate ~ V1 +x' dx to four decimals.


0
1/D

2658. Evaluate ~ Vx ex dx to three decimals.


0
2659. Expand the function cos (x- y) in a series of powers
of x and y, find the region of convergence of the resulting series
and investigate the remainder.
Wt ite the expansions, in powers of x and y, of the following
functions and indicate the regions of convergence of the series:
2660. sinx·siny. 2663*. ln(1-x-y+xy).
2661. sin (x +Y )·
2
~664*. arctan x+y .
1

1 + 1-~
2662*. -=!..__jf .
1-j-x-y
2665. f (x, y) = ax• + 2bxy + cy•. Expand f (x + h, y -f k) in po·
wers of h and k.
2666. f (x, y) = x 1 -2y 1 + 3xy. Find the increment of this
function when passing from the values x = 1, y = 2 to the values
X==1 +h, .IJ=2+k.
2667. Expand the function eX+Y in powers of x-2 and y 2. +
2668. Expand the function sin (x + y) in powers of x and
:rt
!I -2·
318 Series [Ch. 8

Write the first three or four terms of a power-series expansion


in x and y of the functions:
2669. ex cosy.
2670. (1 + xr+Y,

Sec. 4. Fourier Series


1°. Dirichlet's theorem. We say that a function f (x) satisfies the Dirich-
let conditions in an interval (a, b) if, in this interval, the function
I) is uniformly bounded; that is 1f (x) I~ M when a< x < b, where M
is constant;
2) has no more than a finite number of points of discontinuity and
all of them are of the first kind [i.e., at each discontinUIty ~
the function f (x) has a finite limit on the left f (~-0) = lim f (~-E) and a
e -+-O
finite limit on the right f (~ +
0) = lim f (~+E) (E > 0)];
e ~·
3) has no more than a finite number of points of strict extremum.
Dirichlet's theorem asserts that a funct10n f (x), which in the interval
(- n, n) satisfies the Dirichlet conditions at any point x of this interval at
which f (x) is continuous, may be expanded in a trigonometric Founer senes:

f(x) = ~ +a1 cos x+ b1 sin x+a 2 cos 2x-j- b2 sin 2x+ .•. +an cos nx+
0

+bnsinnx+ ... , (I)

where the Fourier coeffictents an and bn are calculated from the formulas

I
On=;t s
lt

-lt
I
f(x)cosnxdx(n=O, 1, 2, .•. ); bn=rr: s .
lt

-lt
f(x)smnxdx(n=l,2, ... ).

If xis a poinf of discontinuity, belonging to the interval (-n, J!), of a


function f (x), then the sum of the Fourier series S (x) is equal to the arithme-
tical mean of the left and right limits of the function:
I
S(x)="2 [f(x-O)+f(x+O)].

At the end-points of the interval x=-n and x=n,


I
S (- n) = S (n) =
2 [/ (- n + 0) + f (n-0)].
2°. Incomplete Fourier series. If a function f(x) is even [i.e., f(-x) =
=f(x)), then in formula (I)
bn=O (n= I, 2, ... )
e~nd
lt

an = } SI (x) cos nx dx (n = 0, I, 2, •.• ).


0
Sec. 41 Fourier Series 319

li a !unction f(x) is odd [i.e., f(-x)=-f(x)), then an=O (n=O, I, 2 •.• )


and n
bn = ~ 5
0
f (x) sin nx dx (n = 1, 2, ... ).

A function specified in an interval (0, :n:) may, at our discretion, be conti-


nued in the interval (- :n:, 0) either as an even or an odd function; hence,
it may be expanded in the interval (0, l't) tn an incomplete Fourier series
of sines or of cosines of multir-le arcs.
3°. Fourier series of a period 21. If a function f (x) satisfies the Dirichlet
conditions in some mterval (-1, l) of length 2/, then at the discontinuities
of the function belonging to this interval the following expansion holds:
a :n:x l'tx 2l'tx . 2:n:x
f(x)=- 0
1 +b2 Sln-1-+ ...
2 +a 1 cos y+b 1 sin T +a 2 cos-
n:n:x nnx
•.. +ancos-1-+bnsln- + ... •
1
where
l
1
an=y 5f nnx dx (n=O, I, 2, ... ),
(x) cos~-
-I
I (2)
I
bn=y 5{(x)sm-
. nnx1-dx (n=l, 2, ... ).
-I
At the points of discontinuity of the function f (x) and at the end-points
x= ±I of the interval, the stun of the Founer series is defined in a manner
simllar to that whtch we lta~e in the expansiOn in th<! interval (-l't, :n:).
In the case of an expansion of the functton f (x) in a Fourier series in
an arbttrary tnterval (a, a+2l) of length 21, the limits of integration in
formulas (2) should be replaced respectively by a and a-j-2/

Expand the following functions in a Fourier series in the


interval (- n, n), determine the sum of the series at the points
of discontinuity and at the end-points of the interval (x = - n,
x = n), construct the graph of the function itself and of the sum
of the corresponding series [outside the interval (-n, n) as well]:
C when -n<x~O.
2671 · f (x) = { C1 when 0 < x< n.
2
Consider the special case when c, = --1, C2 = l.
ax when --n<x~O.
2672 · f(x)= { bx when O~x<n.
Consider the special cases: a) a=b=1; b) a=-1, b=1;
c) a=O, b= 1; d) a= 1, b=O.
2673. f (x) = X 2 • 2676. f (x) =cos ax.
2674. f (x) = eax. 2677. f (x) =sinh ax.
2675. f (x) =sin ax. 2678. f (x) =cosh ax.
2679. Expand the f unction f (x) = -:rt-x F
- in a ourier series in
2
the interval (0, 2n).
320 Series (Ch. 8

2680. Expand the function ~ in sines of multiple arcs


f (x) =
in the interval (0, n). Use the expansion obtained to sum the
number series:
111 11111
a) 1-a+s-7+··· ; b) 1 +s-7-n+t3+17-''';
1 1 1 1
c) 1-s+7-TI+13- ...
Take the functions indicated below and expand them, in the
interval (0, n), into incomplete Fourier series: a) of sines of
multiple arcs, b) of cosines of multiple arcs. Sketch graphs of
the functions and graphs of the sums of the corresponding seiies
in their domains of definition.
2681. f (x) = x. Find the sum of the following series by means
of the expansion obtained:
1 1
1 + 32+ 52+ ...
2682. f (x) = X2 • Find the sums of the following number series
by means of the expansion obtained:

1) 1 + 221 + 32I + ... ; 2) 1- 2i


I I
+ 32- I
4zi" ...
2683. f (x) = eax.
:n:
1 when 0<x< 2 •
2684. f (x) =
{ 0 when 1- ~x<n.
2685. I <~l ~ { :n:
n-x when 2 <x<n.
Expand the following functions, in the interval (0, n), in
sines of multiple arcs:
x when 0 < x ~ ~ ,
2686. f (x) = :n:
{ 0 when <x<n.
2
2687. f(x)=x(n-x).
2688. f (x) = sin i.
Expand the following functions, in the interval (0, n), in co·
sines of multiple arcs:
2689 . f (x) = { 1 when 0 < x ~ h,
0 when h<x<n.
Sec. 4) Fourier Series :m

2690. f (x) = {
1 --=- when 0 < x ~ 2h,
2h
0 when 2h<x<n.
2691. f (x) = x sin x.
cos x when 0 < x ~ i- ,
2692. f (x) =
{ -cosx when ~ <x<n.

2693. Using the expansions of the functions x and x• in the


interval (0, n) in cosines of mult1ple arcs (see Problems 2681 and
2682), prove the equality
<¥>
2
'\. . . , ~= 2
3x -6nr+2n (O .- .c-
-= 1t
)
"'-' n• 1:2 -=X •
n=J

2694**. Prove that if the function f (X) is even and we have


x)
f ( .;- + = -f ( ~- x) ,
then 1ts fourier senes in the interval
(-n, n) represents an expansion in cosmes of odd multiple arcs,
and if the function f (x) is odd and f ( + =f (~ - then i x) x) ,
m the interval (-n, n) it is expanded in sines of odd mul-
tiple arcs.
Expand the following functions in Fourier series in the mdi-
cated intervals:
2695. f (x) = I x 1 ( - 1 < x < 1).
2696. f (x) = 2x (0 < x < 1).
2697. f(x)=e" (-l<x<l).
2698. f (x) = 10--x (5 < x< 15).
Expand the followin~ functiOns, in the indicated intervals,
in incomplete Founer series: a) m smes of multiple arcs, and
b) in cosines of multiple arcs:
2699. f (x) = 1 (0 < x < I).
2700. f (X) =X (0 < X < /).
2701. f (x) = x• (0 < x < 2n).
2702 f (x) = { 2-x x when 0 < x ~ 1,
' when 1 <x< 2.
2703. Expand the following function in cosines of multiple
arcs m the interval ( ~ , 3) :
3
f (x) = 1 when 2 < x E;;; 2,
\ 3-x when 2<x<3.
11-1900
Chapter IX
DlFFERENTIAL EQUATIONS

Sec. 1. Verifying Solutions. Forming Differential Equations of Families of


Curves. Initial Conditions
1°. Basic concepts. An equation of the type
F(x, y, y', ... , y)<nl=O, (I)
where y=y (x) is the sought-for functinn, is called a difJerential equatwn of
order n. The function Y=IJl(x), which converts equation (I) into an identity,
is called the solution of tht> equat10n, while th~ graph of this function IS
called an integral curue. If the solution is represented implicitly, (D (-t, y) =0,
then it is usually called an tntegral
Example I. Check that the function y=sinx is a solut10n of the equatton
y"+y=O.
Solution. We have:
y' =COS X, y" = - sinx
and, consequently,
y" +Y = - slnx+sin x~o.
The tntegral
<D (x, y, C,, ... , C11 ) = 0 (2)
of the differential equation (1), which conta1ns n independrnt arbitrary con-
stants C 1 , ••• , Cn and is equivalent (in the given region) to equatiOn (I), 1s
called the geneT'al mtegral of this equation (in the respective region). By asslf.!n-
ing definite values to the constants c, .... ' ell in (2), we get parltcLtlar
mtegrals.
Conversely, if we have a family of curvE's (2) and eliminate the raram-
eters cl, ... ' ell from the system of equations
d<D dn<D
<11=0, dx=O, .. ·• dx" =0,

we, generally speaking, get a differPntial equation of type (I) whose general
integral in the corresponding region is the relation (2).
Example 2. Find the differential equation of the family of parabolas
y = C 1 (x-C 2 ) 2• (3)
Solution. Differentiating equation (3) twice, we get:
y' =2C 1 (x-C 2) and y" =2C 1 • (4)
Eliminating the parameters C1 and C 2 from equations (3) and (4), we obtain
the desired differential equation
2yy" =u'".
Sec 11 Venfyina So/uctons 323

It ts easy to verify that th~ function (3) converts this equation into an
identity.
2°, Initial conditions. If for the desired particular solution y =y (x) or a
differential equation
(5)
the intltal conditions
y(Xo)~Yo. y'(xo)=y;, ••. , y(n-Jl(xo)=y~n-t)
are given and we know the general solution of equation (5)
Y=!p(X, C1 , ... ,Cn),
then the arbitrary constants C1 , ••• , Cn are determined (if this is possible)
from the system of equations
Yo= 'P (xo, C,, '• •' Cn),
y~ = 'P; (x0 , C., ... , Cn),

Example 3. Find the curve of the family


y= C,e"' + C2e- 2"', (6)
for which y (0) =I, y' (0) = - 2.
Solution. We have:
y' = C,e"' -2C 2e-ox
Putting x=O in formula~ (6) and (7), we obt~in (7)
l=C,+C 2 , -2~C,-2C.,
whence
and, hence,

Determine whether the indicated functions are solutions of the


given differential equations:
2704. xy' = 2y, y = 5x 2 •
I
2705 · Y •Z =X 2 +Y ! • Y=-;·
C2 x1
+
2706. (x 1- y) dx x dy = 0, y = :;: .
2707. y"+y=O, y=3sinx-4co~x.
2708. :;~ + (1) 1 X = 0, x = C, cos (1)/ +c. sin mt.
2709. y"- 2y' + y = 0; a) y = xe"', b) 11 = x•e".
2710. y"-(A. 1 t-A.,)y'+A.,A. 2y=0,
Y = C,eA,x -f C,eA,x.
Show that [or the given differential equations the indicated
relations are integrals:
27<1 t. (x-2y) y' = 2x-y, X 1 -xy f lj2 ==e.
11*
324 Dtflerent ial Equations tCh. 9

2712. (x-1/+l)y'=l, y=x+CeY.


2713. (xy-x) y" + xy'• + yy' -2y' = 0, Y= In (xy).
Form differential equations of the given families of curves
(C, c.. C2 , C, are arbitrary constants):
2714. y=Cx. 2721. In~=l+ay
2715. y = Cx1 • I!
a_ c (a JS a parameter).
2716 • y -· 2 X. 2 )I 2
2717. x•+y==cz. 27 2. (Y-Yo = px
x (y 0 , p are parameters).
2718. u= Ce. 2723 . Y -c
- 1e
ax-t-C -x
.e .
9 •.a C a a
271 . A. = (x - Y ). • 2724. y = C 1
cos 2x +c. sin 2x.
1/
2720. y•-t-l.=2+Ce-2 2725. y=(C.+Ckx)ex+C,.
X '
2726. Form the! differential equation of all straight lines in the
xy-plane. ; , ·
2727. Form the differential equation of all parabolas with
vertical axis in the xy-plan~.
2728. Form the differential equation of all circles in the
xy-plane.
For the given families of curves find the lines that sati~fy
the given initial conditions:
2729. x•-y• = C, y (0) = 5.
2730. Y=(C.+C.x)eu, y(O)=O, y'(O)=l.
2731. y=C,sin(x-C.), y(:rt)=l, y'(:rt)=O.
2732. y=C.e-x+c.ex+C,eu;
y (0) = 0, y' (0) = 1' y" (0) = - 2.

Sec. 2. First-Order Differential Equations


1°. Types of ftrst-order differentia] equations. A differential equation of
the first order in an unknown function Y, solved for the derivative y', is of
the form
y' =I (x, y), (I)
where I (x, y) is the given function. In certain cases it is convenient to
consider the variable x as the sought-for function, and to write (I) in the
form
x' =11 (x, y), ( l ')
1
where g (x, y) =I (x, y) •
Taking into account that y' =
(l) and (1') may be written in the
:!
and x' = :; , the differential equations
symmetric form
P (x, y) dx+ Q (x, y) dy= 0, (2)
where P (x, y) and Q (x, y) are knowrt functions.
By solutions to (2) we mean functions of the form y=cp (x) or x=\j1 (y)
that satisfy this equation. The general integral of equations (1) and (1 '), or
Sec. 21 First·Order DiOerential Equations 325

equation (2), is of the form


lll(x, y, C)=O,
where C is an arbitrary constant.
2°. Direction field. The set of directions
tan a= f (x, y)
is called a direction field of the differential equation (I) and is ordinarily
depicted by means of short lines or arrows inclined at an angle a.
Curves f (x, y) = k, at the points of which the inc! ination of the field
has a constant value, equal to k, are called isoclines. By constructing the
isoclinea and direction field, it is possibl~. in the simplest cases, to give a
y

rough sketch of the field oi int~gral curves, regarding the latter as curves
which at each point have the gtven direction of the field.
Example t. Usmg the method of isoclines, construct the field of integral
curves of the equation
y'=x.

Solution. By constructing the isoclines x=k (straight lines) and the di-
rection field, we obtain approximately the field of integral curves (Fig. 105).
The family of parabolas

is the ~~:eneral solution.


Using the method of isoclines, make approximate constructions of fields
of integral curves for the indicated differential equations:
2733. y' = - x.
2734. y' = - -yX .
2735. y' = 1 + y•.
2736. y' =x+y.
x-y
2737. y' =x•+y•.
326 Dtfferenttal Equations [Ch. 9

3°. Cauchy's theorem. If a function f (x, y) is continuous in some region


U {a< x <A, b < y < B} and in this region has a bounded derivative
/~ (x, y), then through each point (x0 , y 0 ) that belongs to U there passes one
and only one integral curve y=cp (x) of the equation (1) [cp (x0 )=y 0 ].
4°. Euler's broken-line method. For an approximate construction of the
integral curve of equation (I) passing through a given point M 0 (x0 , y 0), we
replace the curve by a broken line with vertices M; (x1, y 1), where

x1+ 1 =x;+l\x1, Yi+ 1 =Y;+l\u;.


llx 1= h (one step of the process),
fly;= hf (x;, Yt) (i = 0, I, 2, ..• ).

Example 2. Using Euler's method for the equation

find y (I), if y (0) =I (h = 0.1).


We construct the table:

1\ X;Y;
I
I Xj
I Y;
I IJt= 20

0 0 I 0
I 0.1 I 0 005
2 0.2 1.005 0.010
3 0.3 1 015 0 015
4 0 4 1.030 0 021
5 0.5 1.051 0 026
.6 0.6 1.077 0 032
7 0.7 1.109 0.039
8 0.3 1.148 0 046
9 0.9 1.194 0.054
10 1.0 1.248

Thus, y (l)= 1.248. For the sake of comparison, the exact value is

y(l)=e' ::::::1.284

Using Euler's method, find the particular solutions to the


given differential equations for the indicated values of x:
2738. y'=y, y(0)=1; find y(1) (h=0.1).
2739. y'=x-\-y, y(l)=1; find y(2), (h=0.1).
2740. y' = - 1 ~x, y (0) = 2; find y (1) (h = 0.1).
2741. y' = y-~, y (0) = 1; find y (1) (h = 0.2).
II
Sec. 3) Differential Equations w1th Variables Separable 327

Sec. 3. First-Order Differential Equations with Variables Separable.


Orthogonal Trajectories
ln. first-order eq11atlons with variables separable. An equation with variables
~eparable is a first-order equation of the type
y'=f(x)g(y) (l)
or
X (x) Y (y) dx+X 1 (x) Y, (y) dy=O (I')
Dividing both sides of equation (I) by g(yl and multiplying by dx, we ~et
dt(
- = f (x) dx Whence, by integrating, we get the general integral of equa-
g(y)
tion (I) in the form
dy =Sf(x)dx+C (2)
S g(y)
Similarly, dividing both sides of rqu<~l!on (I') by X, (x) Y (y) and integrating,
we get the general integral of (I') in thE' form
X ( r) dx + 5yY (y)
(y) dy = C (2')
5X,
1
(x)

II for some value y =Yo we have R (!!0 ) = 0, then the function y = llo is
also (as is directly evident) a solut10n of equation (I) Similarly, the straight
lines x=a and y=b will be the inte!.!ral curves of equation (1'), if a and b
are, respectively, the roots of the equat10ns X, (x) =0 and Y (y) =0, by the
left s!lle~ of which we had to divide the imtiai equation.
Example 1. Solve the equation
y ' =--'--.
II (3)
X

In particular, find the solutiOn !hat sathfies the initial conditions


y (I) =2
Solution. Equation (3) may be written in the torm
dy y
dx=-x
Whence, separating variables, we have
dy=-~
!/ X
and, consequently,
In IY 1=-ln I xJ +In C1 ,
where the arbitrary constant In C1 is taken in logarithmic form. After taking
antilogarithms we get the general solution
c
!J=x-· (4)

where C = ± C1 •
When d1V1d111g by y we could loo;e the solution !1=0, but the latter is
contamed in the formula (4) for C=O.
328 Differential Equations (Ch. 9

Utilizmg the ~tiven mitial conditions, we get C=2; and, hence, the de·
sired particular solutiOn is
2
Y=-;·
2° Certain dift'erential equatiom that reduce to equations with variables
separabh:. Differential equation~ of the form
y'=f(ax+by+c) (b f:; 0)

reduce to equations of the form (I) by means of the substitution u =ax+ by+ t',
where u 1s the new sought-for funct10n
3° Ortho~onal trafectoric:s are curves that intersect the lines of the given
family ll>(x, y, a\=0 1a 1s a paramd~'r) at a right angle. lf F(x, y,y')=O
is the diflerent1al equation of the family, then

f (X, y, - ~~) =0
Is the difrerential equation of the orthogonal trajectories.
Example 2. Find the orthoJonal trajectories of the family of ellips~s

x2 + '2tt" = a2• (15)

Solution DiffE>rentiatmg the equation (5). we find the d1,Jen:nt1al equa-


tion of the lam1ly
x-1- '2yy' =0.
y

Fig. 106

Wht'nce, replarlng y' by - -


y'. we get the differential equation ef the
orthogonal trajeclortes
2 2
x- Y =0 or q'= Y.
1/ X

Jntegrating, we have y = Cx1 llatmly of parabolas) (Fig. 106).


Sec. 31 DiOerential Equations with Variables Separable 329

4°. Forming dift'erential equations. When formmg differential equations tR


geometrical problems, we can fr£quently make use of the geometrical meaning
of the derivative as the tangent of an angle formed by the tangent hne to
the curve in the pos•live x-direction. In many cases thts makes it rossible
straightway to establish a relationship hetween the ordinate y of the desired
curve, its abscts•a x, and the taneent of the angle of the tangent line y',
that is to say, to obtain the diiTe•enttal equat10n. In other 10stances ~see
Problems 2783, 2890, 2895), u~e is made of the geometr.cal signtlicance of
the definite integral as the area of a curviltnear trapezotd or the length of
an arc. In !hi' ca,e, by hypoth('sts we have a simple mtegral equation
!since the destred function is under the 'ign of the mtegral); however, we
can readily pass to a differential equatton hy dtflerenliatmg both ~ides.
Example 3. Ftnd a curve passing through the pomt (3,2) for which the
segment of any tangent line contained between the coordinate axes is divid-
l'd in half at the potnt of tangency.
Solution. Let M (x,y) be the mid-pomt of the tangent line AB. which by
hypothl'sis is the point of tangency (the potnts A and B are points of mter-
~ection of the tangent line with the y- and x-axes). It is gtven that OA =2y
and 08 =2x. The slope of the tangent to the curve at M (x, y) is
dy OA !1
([X=- ua=---;;
This is the difierential equation of the sought-for curve. Transforming, we gel

d.\+~=0
X y
and, consequently,
lnx+lny ~lnCorxy=C.

Utilizing the inittal conditiOn, we determ111e C=3·2=6. Hence, the desired


curve is the hyperbola AY = 6.

Solve the differential equations:


2742. tan x sin• y dx + cos 2 x cot y dy = 0.
2743. xy' -- !I= !l.
2744. xyy' ~ 1-x•.
2745. y-xy' =a (I +x 2 y').
2746. 3t>x tan y dx +(l-ex) sec• y dy = 0.
2747. y' tan x = y.
Find the particular solutiOns of equations that satisfy the
indicated initial conditions:
2748. (l +ex) y y' =ex; !J = l when x = 0.
2749. (xy•+-x)dx+(x 2 y-y)dy=0; y=l when x=O.
2750. y' sin x =yIn y; y ~= l when x = ~ .
Solve the di!Terential equations by changing the variables:
3751. y' = (x + y) 2 •
2752. y = (8x + 2y + l )1 •
2753. (2x + 3y- 1) dx + (4x + 6y- 5) dy = 0.
2754. (2x-y)dx+(4x-2y+3)rly=0.
330 Dtfferential Equations [Ch. 9

In Examples 2755 and 2756, pass to polar coordinates:


2755. y' = ~-x
y
.
1 1
2756. (x +Y )dx-xydy=0.
2757*. Find a curve whose segment of the tangent is equal
to the distance of the point of tangency from the origin.
2758. Find the curve whose segment of the normal at any
point of a curve lying between the coordinate axes is divided in
two at this point.
2759. Find a curve whose subtangent is of constant length a.
2760. Find a curve which has a subtangent twice the abscissa
of the point of tangency.
2761 *. Find a curve whose abscissa of the centre of gravity
of an area bounded by the coordinate axes, by this curve and
the ordinate of any of its points is equal to 3/4 the abscissa of
this point.
2762. Find the equation of a curve that pa5ses through the
point (3, 1), for which the segment of the tangent between the
point of tangency and the x-axis is divided in half at the point
of intersection with the y-axis.
2763. Find the equation of a curve which passes through the
point (2,0), if the segment of the tangent to the curve between
the point of tangency and the y-axis is of constant length 2.
Find the orthogonal trajectories of the given families of cur·
ves (a is a parameter), construct the families and their orthogo-
nal trajectories.
2764. X 2 +if = a•. 2766. xy=a.
2765. y• =ax. 2767. (x-a) 1 r- y 2 = a 1 •

Sec. 4. First-Order Homogeneous Differential Equations


1°. Homogeneous equations. A differential equation
P (x, y) dx+ Q (x, y) dy=O (l)
is called homogeneous, if P (x, y) and Q (x, y) are homogeneous functions of
the same degree. Equation (l) may be reduced to the form

y'=f (f);
and by means of the substitution y=~xu, where u is a new unknown function,
it is transformed to an equatiOn wtth variables separable. We can also apply
the substitution x= yu.
Example l. Find the general solution to the equation
Sec. 4) First-Order Homogeneous Dif!erential Equations 331

Solution. Put u= ux; then u +xu'= eu +u or


e -ad u=-.
dx
X

Integrating, we get U=-ln ln.£, whence


X

y=-xlnln-.
c
X

2°. Equations that reduce to homogeneous equations.


If
(2)

and l\=1::::1;<':0,
then,puttinginto equation (2) X=U+a, y=v+~. Where
the constants a and ~ are found from the following system of equations,
a 1 a+b 1 ~+c 1 =0, a 2 a+b 2 ~ +c2 =0,

we get a homogeneous dilTerenhal equalion in the variables u and v. If


ll = 0, then, putttng in (2) a1x -t- b1y = u, we get an equation with variables
separable.
Integrate the difTerential equations:
2768. y 1 =1!.-l. 2770. (x-y)ydx-x•dy=O.
X

2769. y = -x+y
1
-.
X
2771. For the equation (x 2 +Y1 ) dx-2xydy=0 find the family
of integral curves, and also indicate the curves that pass through
the points (4,0) and (1,1), respectively.
2772. ydx+(2Vxy-x)dy=0.
2773. x dy- y dx = V X 2 -j- y2 dx.
2774. (4x 2 +3xy+y2 )dx+ (4y1 +3xy+x1 )dy=0.
2775. Find the particular solution of the equation (x 1 -3if) dx+
+ 2xy riy = 0, provided that y = l when x = 2.
Solve the equations:
2776. (2x- y + 4) dy + (x- 2y + 5) dx = 0.
2777 . Y 1 = l-3x-3y
1+ x + y
. 2778 . y 1 = x+2y+ 1 •
2x + 4y + 3
2779. Find the equation of a curve that passeg through the
point (1,0) and has the property that the segment ..:ut off b) the
tangent line on the y-axis is equal to the radius vector of the
point of tangency.
2780**. What shape should the reflector of a search light
have so that the rays from a point !:>OUrce of light are rellected
as a parallel beam?
832 DiOerential Equations [Ch. 9

2781. Find the equation of a curve whose subtangent is equal


to the arithmetic mean of the coordinates of the point of tan-
gency.
2782. Find the equation of a curve for which the segment
cut off on the y-axis by the normal at any point of the curve
is equal to the distance of this point from the origin.
2783*. Find the equation of a curve for which the area con-
tained between the x-axis, the curve and two ordinates, one of
which is a constant and the other a variable, is equal to the
ratio 'of the cube of the variable ordinate to the appropriate
abscissa.
2784. Find a curve for which the segment on the y-axis cut
off by any tangent I ine is equal to the abscissa of the point of
tangency.

Sec. 5. First-Order Linear Differential Equations.


Bernoulli's Equation
1°. Linear equations. A differential equation of the fQrm
y' +P (x)·y= Q (x) (I)
of degree one in y andy' is called linear.
If a function Q (x) = 0, then equation (I) takes the form
y' + P (x) · y = 0 (2)
and is called a homogeneous linear differential equation. In this case, the
variables may be separated, and we get the general solution of (2) in the
form

y=C·e
- JP(x) dx . (3)
To solve the inhomogeneous linear equation (1), we apply a method that
is called variation of parameters, which consists in first finding the general
solution of the respective homogeneous linear equation, that is, relation-
ship (3). Then, assuming here that C is a function of x, we seek the solution
of the inhomo~teneous equation (I) in the form of (3). To do this, we put into
(1) y and y' which are found from (3), and then from the differential equa·
tion thu!'. obtainerl we determine the function C (x). We thus get the general
solution of the inhomogeneous equation (1) in the form

y=C(x)·e
-J P (x) dx

Example l. Solve the equation
y' = tan x · y +cos x. (4)
Solution. The corresponding homogeneous equation is
y'- tanx·y=,O.
Solving 1t we get:
I
Y'"-C·--.
ccsx
Sec. 5) Bernoulli's Equation· 333

Considering C as a function of x, and differentiating, we fi'nd;


=-1- . dC sin x .c
y cos x dx + cos 2 x ·
Putting y and y' into (4). we get:
1 dC sin x C dC
- - • -+--·C=tanx·--+cosx or -=cos2 x,
cos x dx cos 2 x cos x ' dx
whence
C (x) = Scos xdx=} x+ {sin 2x+C
2
1•

Hence, the general solution of equation (4) has the form

y= ( ~ x +{ sin 2 ~ +C 1 ) • co~ x .

In solving the linear t>quation (1) we can also make use of the substitu-
lton
y= uv, (3)

where u and v are functions of x. Then equation (I) will have the form
[u' + P (x) u) v + t/u-= Q (x). (6)
If we requtre that
u' + P (x) u = 0, (7)
then from (7\ we find u, and from (6) we find v; hence, from (5) we find y.
2'. Bernoulli's equation. A first-order equatton of the form
y' + P (x) !/-= Q (x) y•,
where a :j: 0 and a :j: l, is called Berrwullt'~ equ11tion It is reduced to a li-
near equation by means of \Ill' substitution z=y 1 - • . It is also 1ossible to
apply diredly the substitutian y= uv, or the method of varia-
tion of parameters.
Example 2. Solve the equation
4
y'=-y+x r .'I·
.r-
X

Solution. This is Bernoullt's equation. Putting


y=U·V,
we t:et
u'v+v'u =!X uv -t-x y,;v or v (u' -~ u) + v'u=x yUU:
X
(8)

To determine the function u we require that the relation

u'-iu=O
X

be fulfilled, whence we have

Putting this expression into l8), we get


v·.~·=xJfux•,
334 Differential Equations [Ch. 9

whence we find u:
u=(; lnx+c Y.
and, consequently, the general sol uti on is obtained in the form

y=x4(~tnx+cr.
Find the general integrals of the equations:
2785. ddy _J!.. = x.
X X

2786. ~dyX + 2
X
Y = x•.

2787*. (1 -t y') dx = CV 1 + y• sin y-xy) dy.


2788. y•dx- (2xy +3) dy = 0.
Find the particular solutions that satisfy the indicated con-
ditions:
2789. xy'+y-e"=O; y=b when x=a.
2790. y ' 1-Y- -l-x=O; y=O when x=O.
-X 2
1
2791. y'-ytanx=--;COS X
y=O when X=O.
Find the general solutions of the equations:
2792. d'X!J!.+.l!..X = -Xy 1 •
2793. 2xy~-y 1 +x=0.
2794. y dx+ (x-;
x'y) dy=O.
2795. 3x dy = y (I+ x sin x-3y' sin x) dx.
2796. Given three particular solutions y, y., Y. of a linear
equation. Prove that the expression y.-y remains unchanged for
y-y.
any x. What is the geometrical significance of this result?
2797. Find the curves for which the area of a triangle formed
by the x-axis, a tangent line and the radius vector of the point
of tangency is constant.
2798. Find the equation of a curve, a segment of which, cut
off on the x-axis by a tangent I ine, is equal to the square of the
ordinate of the point of tangency.
2799. Find the equation of a curve, a segment of which, cut
off on the y-axis by a tangent line, is equal to the subnormal.
2800. Find the equation of a curve, a segment of which, cut
off on the y-axis by a tangent line, is proportional to the square
of the ordinate of the point of tangency.
Sec. 6] Exact Differential Equations. Integrating Factor 335

2801. Find the equation of the curve for which the segment
of the tangent is equal to the distance of the point of intersec-
tion of this tangent with the x-axis from the point M (O,a).

Sec. 6. Exact Differential Equations.


Integrating Factor
1°. Exact dift'erential equations. If for the differential equation
P (x, y) dx+ Q (x, y) dy=O (1)
the equality~:=~~ is fulfilled, then equation (I) may be written in the
form dU (x, y) = 0 and is then called an exact dtUerential equation. The gen-
eral integral of equation (I) is U (x, y) =C. The function U (x, y) is deter-
mined by the technique given in Ch. VI, Sec. 8, or from the formula
X II
U = ~ P (x, y) dx + ~ Q (x0 , y) dy
Yo
~see Ch. VII, Sec. 9).
Example I. Find the general integra! of the differential equation
(3x 1 + 6xy 1 ) dx + (6x 2y + 4y 1 ) dy = 0.
2
6
Solution. This is an exact differential equation, since a(Jx ;
y
xyz) =
a(6~'y+4u•>
= ax = l2xy and, hence, the equation is of the form dU =0.
Here,

whence
U= ~ (3.\ 2 + 6xy1 ) dx + <p (y) = x• + 3x 2y 2 + <p (y).

Differentiating U with respect toy, we findi!.._U =6x 2y+<p' (y)=6x 2y+4y1 (by
0y
hypothesis); from this we get <p' (!/)=411' and <p(f)=y4 -j-C. We finally get
U ('1', y)=x'+3x'y"+ y 4 + C1,. consequently, x'+3x y'+y•='i! is the sought-for
general integral of the equation.
2°. Integrating factor. If the left side of equation (I) is not a total (exact)
differential and the conditions of the Cauchy theorem are fulfilled, then there
exists a function lJ.=lJ.(X, y) (integrating factor) such that
)L (P dx+ Q dy) =dU. (2)
Whence It Is found that the function l1 satisfies the equation
a a
ay- <llP>=ax <J.LQ).
The integrating factor J.L is readily found In two cases:

1) ~(:-:~)=F(x), then J.L=J.L(x);

2) -~ (~:- :~) = F 1 (y), then ~L =It (y).


336 Dif1erent Ia/ Equations [Ch. 9

Example 2. Solve the equation (2xy+x 2y+~ )dx+tr+Y 2 )d~=0.


Solution. HereP=2xy+x 2y+~, Q=x 2 +y 2
1 (iJP iJQ) 2x+x 2 +y"-2t
and Q iJy- iJx = x•+y2 =1, hence, f-L=J-4 (x).
. iJ(f-LP) iJ(f!Q) oP oQ +QdJ-4
S tnce -ay=-ax or fJ. oy = fJ. OX dx.
it rollows that

Multiplying the equation by !J.=e", we obtain

ex ( 2xy+ x1y + ~) dx-j-e" (x1 + y 1 ) dy-==0


\ 3

which is an exact differential equation. Integrating it, we get the general


integral

Find the general integrals of the equations:


2802 (x+y)dx + (x + 2y) dy = 0.
2803. (x' + y• + 2x) dx + 2xy dy = 0.
2804. (x'-3xy 2 + 2) dx- (3x'y-y 2 ) dy =0.
x dy-y dx
2805. xdx-ydy= ~·+yz •

y y
+
2806. 2x ~x y•-43xz dy = 0.
2807. Find the particular integral of the equation

(x+e~) dx+e~ ( 1-~) dy=O,


which satisfies the initial condition y (0) = 2.
Solve the equations that admit of an integrating factor of the
form 11 = !l (x) or 11 = 11 (y):
2808. (x + y 1 ) dx- 2xy dy = 0.
2809. y (1 -~- xy) dx-x dy= 0.
2810. YX dx+(y'-lnx)dy=O.
28 I t. (x cosy- y sin y) dy + (x sin y + y cosy) dx = 0.
~ec. 7) First-Order DiOerential Equation~ not Solued for Deriuatiue 331

Sec. 7. First-Order Differential Equations not Solved


for the Derivative
1°. First-order differential equations of higher powers. If an equation
F (x, y, y')=O, (I)

"hich for example is of degree two in y', the.1 by solving (1) for tl we get
two equations:
(2)
Thus, generally sp('aking, through each pomt M 0 (x0 , !In) of some region
of a plane there pass two integral curves. The general integral of equation
(I) then, generally speaking, h~s the form
<l>(x, y, C)=<l> 1 (.r, y, C)<l> 2 (x, y, C)=O, (3)
where 1I> 1 and <1> 2 are the general integrals of equations (2).
Besides, there may be a Stn{Zular wtewal for equilllon \1). Geometrically,
a singular integral is the envelope of a family of curves (3) and may be ob·
tained by eliminating C from the system of equations
<D(x, y, C)=O, <I>~ (x, y, C)=O (4)
or by eliminating p=y' from the system of equations
F (x, !f, p) = 0, F~ (x, y, p) = 0. (5)
We note thilt the curves d('fined by the l'quations (4) or (5) are not
<.~lwavssolut10ns of eqnnt10n (I); therefore, in each case, a check is necessary.
Example I. Fmd the general and s111gular integrals of the equation
xu' 2 -f 2xt!'-y=0.
Solution. Solving for y' \\'e have two homogeneous equations:

y' = -
ddined in the region
I+ vI+ ~ . y' = - 1- vI t ~ I

x(x+y) >0,
the general integrals of which are

or
(v 1+~-1 )"=~.
(2x+y-C)-2 Yx 2
+xy=0,
Multiplying, we get the general integral of the given equation
(2x+y-C) 2 -4 (x 2 +xy)=0
or
(y-C) 2 =4Cx
(a family of parabolas).
Differentiating the general integral with respecf to C and eliminating C,
we find the singular integral
y+x=O.
(It may be verified that y+x=O is the solution of this equation.)
338 Differential Equations (Ch. 9

It is also possible to find the singular integral by differentiating


xp 2 +2xp-y=0 with respect to p and eliminating p.
2°. Solving a differential equation by introducing a parameter. If a first-
order differential equation is of the form
X=cp (y, y'),
th~n the variables y a11d x may be determined from the system of equations
1 _ acp +acp dp
11- au ady • x = cp <u. p),
where p=y' plays the part of a parameter.
Similarly, if y = \jJ (x, y'), then x and y are determined from the system
of equations
aw awdp
p= ax+ ap dx' Y=W (x, p).

Example 2. Find the general and singular integrals of the equation


x•
y=y'•-xy' + 2.

Solution. Making the substitution y' = p, we rewrite the equation in the


form

Diffetentiatiug with respect to x and considering p a function of x, we have


P= 2pdp -p-xdp +x
dx dx

or ~~ (2p-x)-(2p-x), or 1x=l.
Integrating we get p=x+C. Substituting
into the original equation, we have the general solution

Differentiating the general solution with respect to C and eliminating C, we


I 2
obtain the singular solution: y=x • (It may be verified that y=T is the
4
solution of the given equation.)
If we equate to zero the factor 2p -x, which was cancelled out, we get
p= ~
and, putting p into the given equation, we get y=~, which is the
same singular solution.

Find the general and singular integrals of the equations:


(Itt Problems 2812 and 2813 construct the field of integral
curves.)
2y
2812. y t2 --; y -.- 1 = 0 .
I I

2
2813. 4y' -9x=0.
Sec 8] The Lagrange and Clairaut Equations 339

2
2814. yy' -(xy+ l)y' -tx=O.
2815. yy' -2xy' + y = 0.
2

2816. Find the integral curves of the equation y' 1 + y• = 1


that pass through the point M ( 0. ; ) .
Introducing the parameter y' = p, solve the equations:
2
2817. x=siny' +lny'. 2820. 4y=x 2 +y' ,
12 Y'
2818 · Y = Y P • •+ '2
2821 ex- !j__Jf_
2819. y=y' 1 +2lny'. · - 2y' •

Sec. 8. The Lagrange and Clairaut Equaticns


1°, Lagrange's equation. An equatiOn of the form
Y=Xq.> (p) -f-'ljl (p), (I)
where p=y' is called Lagrange's equatiOn Equation (I) is reduced to a linear
equation in x by d1fferent1at10n and takmg into comideration that dy -~ p dx:
pdX=!p(p)dx+(xcp'(p)-l-'ljl'(p))dp. (2)
If p;t=cp(p), then from (I) and (2) we get the general solutiOn in parametric
form:
X= Cf (p) + g (p), y = (Cf (p) + g (p)) q• (p) + ~' (p),
where p is a parameter and f (p), g (p) are certa111 known functions. Besides,
there may be a s1ngular solutiOn that is found in the u~ual way.
2°, Clairaut's equation. If 111 equation (I) p =-" q' (p), then we get Clai-
raut's equatwn
y=xp-J-'Ijl(p).
Its general solution 1s of the form y ~ Cx + 'ljl (C) (a fa1TIJly of ~tratght lines).
There IS also a parttcular solutto•J (envelope) that results by eliminating the
parameter p from the system of equ~t10n~
X=-'ljl'(p),
{ y==px+~· (p).
Example. Solve the equation
I
y --2y'x+!7. {3)

Solution. Putting y' 0


"" p we have yo~ 2p\ + _!_p ; differentiating and replac-
ing dy by p dx, we get
p dx = 2p dx + 2x dp- ~
p
or
dx 2 1
-=--x+-
dp p p''

Solving this lineal equation, we will have

x= zI (lnp+C).
p
340 Di(Jerential Equations [Ch. 9

Hence, the general integral will be

f X=;. (In p-t-C),


l y=2px+! .

To find the singular integral, we iorm the system


I I
y=2px+-, 0=2x- 3
p p
in the usual way. Whence
I 2
X=2p"' y=p
and, con~equently,
y= ±2 V2x.
Putting y into (3) we are convinced that the function obtained Is not
a solution and, therefore, equation 13) does not have a singular integral.
Solve the Lagrange equations:
2
2822. y = ~ X ( y' +..it,) . 2824. lj = (1 + y') X+ y' •
, ,r- y ,z- 2825"'. y = - 21 y'(2x+ y') .
2823 . y=y +"1 1-y .
Find the general and singular integrals of the Clairaut equa-
tions and construct the field of integral curves:
2826. y = xy' + y·•.
2827. y=xy' +y'.
2828. y = xy' + l/~1-+-(y-')..,.-
1•

2829. y= xy' +~.y


2830. Firld the curve for which the area of a triangle formed
by a tangent at any point and by the coordinate axe<> is con-
stant.
2831. Find the curve it the distance of a given point to any
tangent to this curve is constant.
2832. Find the curve for which the segment of any of its
tangents lying between the coordinate axes has constant length l.

Sec. 9. Miscellaneous Exercises on First-Order Differential Equation>


2833. Determine the types of differential equations and indi-
cate methods for their solution:
a) (x + y) y' ~= x arctan ~ ; e) xy' + y = sin y;
b) (x- y) y' = y•; f) (y-xy')z = y'd;
c) y' = 2xy + x'; g) y = xeY';
d) y' = 2xy +y'; h) (y' -2xy) Vu=x';
Sec. 9] Miscellaneous Exercises on Fir.>t-Order Differential Equations 341

i) y'=(x+y) 1 ; I) (x 2 + 2xy 1) dx -\-


j) x cosy' + y sin y' = 1; + (y1 + 3x1 y 1 ) dy = 0;
k) (x'-xy)y' =y'; m) (x -3xy)dx+(x1 +3)dy=0;
3

n) (xy• +In x) dx = y1 dy.


Solve the equations:
2834. a) ( x- y cos ~) dx + x cos ~ dy-= 0;
X
b) x!n-ydy-ydx=O.
2835. xdx= (x; -y') dy.
2836. (2xy - y) dx + x dy "'- 0.
2

2837. xy' +Y'-=X!/ !nx.


2838. y=xy' +y' !ny'.
2839. y=xy' + V -ay'.
X (y + 1) dx + (x' -1) (y-1) dy = 0.
2
2840.
2841. (1 +y 2 )(e 2 ·>;dx-eYdy)-(1 +y)dy=O.
2
284 2 , y' -y 2x I= l. 2845. (1-x ) y' +xy =a .
.\ 2

2843. yeY = (y' -1 2xeY) y'. 2846. xy'- 1


-x= 0. x!
2844. y'+y cos X= sin x cos x. 2847. y' (x cosy +a sin 2y) =I.

2849. y· = ( 1 +~~I
2850. xy' dx -=--" (x 2 y t 2) dy.
r.
2848. (X 2 !J-X 2 -f-y--l)dx +(xy+2x-3y-6)dy=0.

' 3.t 2
2851 · ! J = .\a TLy- j- l '
2852. 2dx + ..VI..:_!J dy-- ..VIJ_ dx = 0.
X

2853. y' = Y -1- tan JLX . 2861. eY dx + (xeY- 2y) dtJ = 0.


2862. y=2xy' + V1 ~ y' •
X 2

2854. yy' -!- !J~ =COS X.


2855. x dy -\ y dx =-~ y' dx. 2863. y'=f(l +lny-lnx).
+
2856. y' (x sin y) = 1. 2864. (2ex + y') dy-
2857. ydr!.!!. = - p + p 2 -yexdx=O.
!I
2858. x' dx- (x~ ~ y') :fy ~ 0.

1 (

2865. y =2\x+y-l .
y+2 )I
2859. X1 !J' 1 + 3xyy' + 2866. xy (xy• + l)dy-dx=
+2y 1 =0. =0.
2860. xdx+yd~ -1- +
2867. a (xy' 2y) = xyy'.
Jfxz+y'
xdy-ydx -O 2868. xdy-ydx=y' dx.
+ yz - •
342 Differential Equations [Ch. 9

2869. (x'-1)* dy + (x' + 3xy li x• -1) dx = 0.


dy
2870. tan X dx- y =a.
2871. Va• + +
x• dy (x+ y- Va• + X2
) dx= 0.
+ +
2872. xyy' 1 - (x 2 y 2 ) y' xy = 0.
2873. y = xy' + y~ .
2874. (3x + 2xy-y
2 1
dx+ (x1 -2xy-3y 1 ) dy=- 0.
)

2875. 2yp :~ = 3p 2 -j. 4y •


2

Find solutions to the equations for the indicated initial con-


ditions:
1
2876. y' = Y+ ; y = 0 for x = I.
X
2877. e"-Yy'=1; y=l for x=l.
+
2878. cot xy' y=2; y=2 for x=O.
2879. eY (y' + 1) = 1; y = 0 for x = 0.
2880. y' + y =cos x; y=-} for X= 0.
I
2881. y'-2y=-x 1 ; y= 4 for x=O.
2882. y' +y=2x; !J=-1 for x=O.
2883. xy'=y; a) y=1 for x=1; b) y=O for x=O.
2884. 2xy'=y; a) y= 1 for x= 1; b) y=O for x=O.
2885. 2xyy'+x 2 -!J 2 =0; a) y=O for x=O; b)y==1 forx=O;
c) y = 0 for x = 1.
2886. Find the curve passing through the point (0, 1), for
which the subtangent is equal to the sum of the co01dinates of
the point· of tangency.
2887. Find a curve if we know that the sum of the segment::.
cut off on the coordinate axes by a tangent to it is constant and
equal to 2a.
2888. The sum of the lengths of the normal and subnormal
is equal to unity. Find the equation of the curve if it is known
that the curve passes through the coordinate origin.
2889*. Find a curve whose angle formed by a tangent and the
radius vector of the point of tangency is constant.
2890. Find a curve knowing that the area contained between
the coordinate axes, this curve and the ordinate of any point on
it is equal to the cube of the ordinate.
2891. Find a curve knowing that the area of a sector boun-
ded by the polar axis, by this curve and by the radius vector
of any point of it is proportional to the cube of this radius
vector.
2892. Find a curve, the segment of which, cut ofT by the
tangent on the x-axis, is equal to the length of the tangent.
Sec. 9] Miscellaneous Exerctses on First-Order Differential Equations 343

2893. Find the c·.uve, of which the segment of the tangent


contained between the coordinate axes is divided into half by
the para bola !/ = 2x.
2894. Find the curve whose normal at any point of it is
equal to the distance of this point from the origin.
2895*. The area bounded by a curve, the coordinate axes,
and the ordinate of some point of the curve is equal to the
length of the corresponding arc of the curve. Find the equation
of this curve if it is known that the latter passes through the
point (0, 1).
2896. Find the curve for which the area of a triangle formed
by the x-axis, a tangent, and the radius vector of the point of
tangency is constant and equal to a 2 •
2897. Find the curve if we know that the mid-point of the
segment cut off on the x-axis by a tangent and a normal to the
curve is a constant point (a, 0).
When forming first-order differential equations, particularly in ph\sical
problems, it is frequently advisable to apply the so-called m£1hod of dtf}eren-
tials, which con~ists in the fact that approximate relationships between
infimtesimal i~ocremenh of the des1red quantities (these relationships are
accurate to infinitesimals of higher order) are replaced by the corresponding
relationships between the1r dll'ft·rPnhals. Th1s doe~ not affect the result.
Problem. A tank conta1ns 100 litre~ of an aqueous solution containing
10 kg of salt. Water 1s entcnng the tank at the rate of 3 l1tres per minute,
and the mixture is llowinr:: out at 2 l!tres per minute. The concentration is
maintained un1form by stirring. How much salt will the tank contain at the
end of one hour?
Solution. The concentration c of a substance is the quantity of it in
unit volume. If the concentration is uniform, then the quantity of sub-
stance in volume V is cV.
Let the quant1ty of salt in the tank at the end of I minutes be x kg.
The quantity of solullon in the tank at that instant will be 100+1 litres,
and, consequently, the concentration c= x+l kg per litre.
100
During time dt, 2dl litres of the solution flows out of the tank (the
solution contains 2c dt kg of salt). Therefore, a change of dx in the quantity
of salt in the tank is given by the relationship
2x
-dx= 2c dt= 100 + t dt.

This is the sought-for differential equation. Separating variables and integrat-


ing, we obtain
In x=- 2ln (100+ t)+ InC
or c
X=(IOO+I) 1 '
The constant C is found from the fact that '' lwn t = 0, 10, that is,
1---=
C= 100,000. At the expiration of one hour, the tank will contain
x=~~~·~;m ::::::3.9 kilograms of salt.
344 DiOerential Equations [Ch. 9

2898*. Prove that for a heavy liquid rotating about a vertical


axis the free surface has the form of a paraboloid of revolution.
2899*. Find the relationship between the air pressure and the
altitude if it is known that the pressure is 1 kgf on 1 cm 2 at
sea level and 0.92 kgf on 1 cm 1 at an altitude of 500 metres.
2900*. According to Hooke's law an elastic band of length
l increases in length klF(k=const) due to a tenstle force F.
By how much will the band increase in length due to its weight
W if the band is suspended at one end? (The initial length of
the band is /.)
2901. Solve the same problem for a weight P suspended from
the end of the band.
When solving Problems 2902 and 2903, make use of Newton's
law, by which the rate of cooling of a body is proportional to the
difference of temperatures of the body and the ambient medium.
2902. Find the relationship between the temperature T and
the time t if a body, heated to T 0 degrees, is brought into a room
at constant temperature (a degrees}.
2903. During what time will a body heated to 100° cool off
to 30° if the temperature of the room is 20° and during the first
20 minutes the body cooled to 60°?
2904. The retarding action of friction on a disk rotating in
a liquid is proportional to the angular velocity of rotation. Find
the relationship between the angular velocity and time if it io;
known that the disk began rotating at 100 rpm and after one
minute was rotating at 60 rpm.
2905*. The rate of disintegration of radium is proportional
to the quantity of radium present. Radium disintegrates by one
half in 1600 years. Find the percentage of radium that has disinte-
grated after 100 years.
2906*. The rate of outflow of water from an aperture at
a vertical distance h from the free surface is defined by the
formula
u=cV2gh,
where c ~ 0.6 and g is the acceleration of gravity.
During what period of time will the water filling a hemi-
spherical boiler of diameter 2 metres flow out of it through a cir-
cular opening of radtus 0.1 m in the bottom.
2907*. The quantity of light absorbed in passing through
a thin layer of water is proportional to the quantity of incident
light and to the thickness of the layer. If one half of the original
quantity of light is absorbed in passing through a three-metre-
thick layer of water, what part of this quantity will reach a depth
of 30 metres?
Sec. 10] 1/i~her-Order DtOerenttal Equations 345

2908*. The air resistance to a body falling with a parachute


is proportional to the square of the rate of fall. Find the limit-
ing velocity of descent.
2909*. The bottom of a tank with a capacity of 300 litres
is covered with a mixture of salt and some insoluble substance.
Assuming that the rate at whtch the salt dissolves is proportion·
al to the ditYerence between the concentration at the given time
8nd the concentration of a saturated solution (1 kg of salt per 3
litres of water) and that the given quantity of pure water dis-
solves 1}3 kg of salt in 1 minute, find the quantity of salt in solu-
tion at the exptration of one hour.
2910*. lhe electromotive force e in a circuit with current i,
resistance R and self-induction L is made up of the voltage drop
Ri and the electromottve force of self-induction L :; . Determine
the current i at timet if e=Esinrot (£and ro are constants)
and i = 0 when t = 0.

Sec. 10. Higher-Order Differential Equations


1". The case of direct integration. If
y1111 = f (.\),

y=~dx~ ... )f(.\)dx-)-C 1.t 11 - 1 +C 2.t11 -


-------
2
-)- ••• -)-C11 •
11 , !fl1('$

2°. Ca~es of reduction of order. I) If a dtilerential equation does not


contatn y cxpl!cttly, for instanco.>,
F {x, t/, y•) = 0,
then, assuming y' = p, we get an equation ol an order one umt lower:
F (x, p, p')=O.
Example I. Find the particular solution of the equation
xy"-)-y'+x=O,
that satisfies the conditions
y=O, y' =0 when x=O.
Solution. Putting y' =p, we have !I= p', whence
xp'-)-p-)-x=O.

Solving the latter equation as a linear equation in the function p.


we get
346 DtUerenltal Equations [Ch 9

From the fact that y'=p=O when x=O, we have 0=C 1 -0, i.e., C1 =0.
Hence,
X
p=--
2
or

whence, integrating once again, w~obtatn


x'
y=-4t-C2

Putting y~O when x=O, we lind C 2 =0. Hence, the desired particular
solu lion is

Y = _ _..!_xz
4 .
2) If a differential equation does not contain x explicitly, for instance,
F(y, y', y")=O
then, puttingy'=p,y"=pddp' we get an equation of au order one unil
y
lower:

F(y. p, p:~)=o.
Example 2. Find the particular solution of the equation
yy• -y'Z=y'
provided that Y= 1, y'=O when x=O.
Solution. Put y'=p, then if=p:~ and our equation becomes

IJP ~~ -pz=~''·
We have obtained an equation of the Bernoulli type in p (y is considered
the argument). Solving it, we find
P=±y YC,+y 2 •
From the fact that y'=p=O when y=l, we have C1 =-1. Hence,

P= ±Y Yy 2
-1
or
:~ = ± y vyZ-1.
Integrating, we have
I
arc cos - ± x = C 2•
lJ
I
Putting y= I and x=O, we obtain C2 =0, whence -=cosx or y= st>c x.
y
Sec. 10] Higher-Order DiUerential Equations

Solve the following equations:


2911. y" = _!_.
X
2920. yy" = y•y' + y''.
2912. y" = - 2!' . 2921. yy"- y' ( 1 + y') = 0.
1
2913. y" = 1- y' • 2922. y"=- x•.
y
2914. xy" ·l y' = 0. 2923. (x+l)y"-(x-t-2)y'+x+
+ 2 = 0.
1
2915. yy" = y' • 2924. xy" =y' In y'.
X
yy" + y' = 0.
1
2916. ' + 1 ,,1 ,
2917. (1 !-x 2 )y"+y' 1 +1=0. 2925 · Y 4Y =xy.
2918. y'(l+y' 2 )=ay". 2926. xy'"+y"=1+x.
x•y" + xy' = 1. 2927. y"' + y" = 1.
2 2
2919.
Find the particular solutions for the indicated initial con-
dit ions:
2928. (1+x 2 )y"-2xy'=0; y=O, y'=3 for x=O.
2929. 1 + y'• = 2yy"; y = I, y' = 1 for x = 1.
2930. yy" + y'" = y'~; y = I, y' = 1 for x = 0.
2931. xy"=y'; y=O, y'=O for x=O.
Find the general integrals of the following equations:
2932. yy' = vY + y'' y"- y' y".
2

2933. yy" = y'' + y' Vy• + y'


2

2934. y'"- yy" = y•y'.


+ 2
2935. yy'' y' - y'"ln y = 0.
Find solutions that satisfy the indicated conditions:
I
2936. y"y 3 =1; y=1, y'=1for x= 2 .
2
2937. yy"+y' =I; y=l, y'=l for x=O.
2938. xy"= V1 +y' 2
; y=O for x= 1; y= 1 for x=e1 •
I I
2939. y"(1 +lnx)+-x·Y'=2+lnx;y= 2 , y'=l tor x=l.

2940. y"=~(l+ln~'); y=-}, y'=l for X=l.


2
2941. y"-y' +y'(y-l)=0; y=2, y'=2 for x=O.
2942. 3y'y"=y+y'''+-l; y=-2, y'=O for x=O.
2943. y'+y''-2yy"=0; y=l, y'=1 for x=O.
2944. yy'+y''+yy"=O; y=l for x=Oandy=Oforx=-1.
348 Differential Equations [Ch. 9

2y' + (y' -6x)·y" = 0; y =0, y' = 2 for X= 2.


2
2945.
2946. y'y' + yy" -y'' = 0; y= I, y' = 2 for X= 0.
1
2947. 2yy"-3y' =4y 2 ; y= I, y'=O for x=O.
1
2948. 2yy"-t-y'-y' =0; y=I, y'=I for x=O.
2949. y" = y'•-y; y= -}, y' =; for X= 1.
2 2
2950, y" -f- ; 2 eY y' -2yy' = 0; !/=I, y' = e for X o-=- ;e,
2
2951. I+yy"-1-!1' =0; y=O, y'=I for x=l.
2
2952. (I+-yy')y"=(l+-y' )y'; y=I, y'=I for x=O.
2
2953. (x+ l)y" +-xy' =y'; y= -2, y' =4 for X= 1.
Solve the equations:
2954. y' = xy"' + y" •
2

2955. y' =xy" + y"-y" ,


2

2956. y'""=4y".
2957. yy'y" = y' + y" • Choose the integral curve passing through
3 2

the point (0, 0) and tangent, at it, to the straight line y+x=O.
2958. Find the curves of constant radius of curvature.
2959. Find a curve whose radius of curvature is proportional
to the cube of the norma I.
2960. Find a curve whose radius of curvature is equal to the
normal.
2961. Find a curve whose radius of curvature is double the
normal.
2962. Find the curves whose projection of the radius of cur-
vature on the y-axis is a constant.
2963. Find the equation of the cable of a suspension bridge
on the assumption that the load is distributed uniformly along
the projection of the cable on a horizontal straight line. The
weight of the cable is neglected.
2964*. Find the position of equilibrium of a flexible nonten-
sile thread, the ends of which are attached at two points and
which has a constant load q (including the weight of the thread)
per unit length.
2965*. A heavy body with no initial velocity is sliding along
an inclined plane. Find the law of motion if the angle of incli-
nation is a, and the coefficient of friction is J..l..
(Hint. The frictional force is JLN, where N is the force of reaction of the
plane.)
2966*. We may consider that the air resistance in free fall
is proportional to the square of the velocity. Find the law of
motion if the initial velocity is zero •.
Sec. 111 Linear Di{Jerential Equations 349

2967*. A motor-boat weighing 300 kgf is in rectilinear motion


with initial velocity 66 m;sec. The resistance of the water is pro-
portional to the velocity and is 10 kgf at 1 metre;sec. How long
will it be before the velocity becomes 8 m/sec?

Sec. 11. Linear Differential Equations


1°. Homogeneous equations. The functions y 1 =q>,(x), y 1 =q> 2 (x), oo•
00.,y = 'Pn (x) are called lweor/y dependent if there are constants C1 , C1 , ••• , Cn
not at1 equal to zero, such that
C1y 1 + CtYa + ... + C,y, """0;
otherwise, these functions are called linearly Independent.
The general solution of a homogeneous lmear differential equation
y<nt-j-P 1 (x)y<"- 11 -j- .•. -J-Pn(x)y=0 (I)
w; th continuous coefficients P; (x) (i =I, 2, 00 • , 11) IS of th~ form
y= C 1!J 1 + C~y 2 -j- oo. + C,yn,
where y 1 , y 1 ,
00 !In are linearly independent solutions of equation (I)
.,

(fundamental system of solutiOns).


2°, Inhomogeneous equations. The general solution of an inhomogeneous
ltnear di!Terential equation
y<nl +PI (.v) y<n-1) + •', + P, (x) y =f (x) (2)
with continuous coefficients P; (x) and the right side f (x) has the form
Y=Yo+ Y,
where Yo is the general solution of the corresponding homogeneous equation (I)
and Y is a particular solution of tho? given inhomogeneous equation (2).
II the fundaml'ntal system of solulions l/ 1 , y 2 , ••• , Yn oi the homogeneous
equation (I) is known, then the general solution of the corresponding inho·
mogeneous equation (2) may be found from the formula
y= C, (x) Y1 +C 1 (x) Ya+ • · • +Cn (X)Yn•
where the functions C;(x) (i=l, 2, ... , 11) are determined from the follow-
ing system of equations:
c; (x) y -t- C~ (x) Y1
1 + ... +C;, (x) y 1 =0,
c; (x) u; +c; (x) u: + ... + c~ <\ >y~ = o.

c; (x) Yln-•l +c; (x) y~n-al + ... +C~ (x) y~n-•> =0,
c: (x) uln-l) + c~ (x) y~n-t) + ... + c~ (x) y~•-•l =f (x)
(the method of uariation of parameters).
Example. Solve the equation
l (3)

xy• +Y'=x•. (4)


350 DiUerential Equations [Ch. 9

Solution. Solving the homogeneous equation


xy"+y' =0,
we get
y=C1 1nx+C1 • (5)
Hence, It may be taken that
y 1 =1nx andy 2 =1
and the solution of equation (4) may be sought in the form
y=C1 (x) lnx+Cz (x).
Forming the system (3) and taking into account that the reduced form of
the equation (4) is y" + y' =x, we obtain
X

c;(x)lnx+C~(x)I=O,
, I ,
{ C,lx) -+C 2 (x) 0= X.
X
Whence
x' x• x•
C1 (x)=-a+A and C1 (x)= -
3 1n x+ -g+B
and, consequently,
x•
Y=g-+A lnx+B,

where A and B are arbitrary constants.

2968. Test the following systems of functions for linear rela-


tionships: ,
a) x, x+ 1; e) x, x•, x•;
b) x•, -2x"; f) e", ezx, e'";
c) 0, 1, x; g) sin .t, cos x, 1;
d) X, X+ 1, X+ 2; h) sin 2 X, COS 2 X, 1.
2969. Form a linear homogeneous differential equation, know-
ing its fundamental system of equations:
a) y, =sin x, Yz = cosx;
b) y, = e", Yz = xe";
c) y,=x, Y.=x".
d) y, = e", Y2 = e" sin x, Ya = e" cosx.
2970. Knowing the fundamental system of solutions of a linear
homogeneous differential equation
y,=x, Y2 =X1 , y 1 =X1 ,
find its particular solution y that satisfies the initial conditions
YIX=1 =0, y' 1.~=1 = -1, y" lx•n = 2.
Sec. 12} Linear DiOerential Equations with Constant Coefficients 351

2971*. Solve the equation


y" +!
y
y' + y = 0,
knowing its particular solution y, = sinx.
X
2972. Solve the equation
1
X (lnx-I)y"-xy' +y=O,
knowing its particular solution y, = x.
By the method of variation of parameters, solve the following
inhomogeneous linear equations.
2973. x'y" -xy' = 3x•.
2974*. x'y"+xy'-y=x'.
+
2975. y"' y' =sec x.

Sec. 12. Linear Differential Equations of Second Order


with Constant Coefficients
1°. Homogeneous equations. A second-order lmear equation with const~nt
coefficients p and q w1thout the right side IS of the form
y"+py'+qy=O (I)

If k 1 and ~2 are roots of the characteristic rquatwn


<r (k) =k'+pk+q =0, (2)
then the general solution of equation (I) is \Hillen in one of the following
three ways:
I) y~~ C,ek,-'"+Ccek.x if k 1 and k, are real and k 1 :F k<;
2) y -= e'<,x (C 1 + C 2x) if k 1 == k 2 ;
3) y =e'" (C 1 ens~ x+C 2 stn ~-\} if k 1 =a-t-~t and k 2 -=-a-~l (~ :F 0).
2°. Inhomogeneous e4uations. The general solution of a linear lnhomo~e­
neous differential equation
y" + py' + qy = f (x) (3)
may be wntten in the form of a sum:
Y=Yo+ Y,
where Yo is the general solution of the corresponding equation (I) without
rif.(ht side and determined from formulas (I) to (3), and Y is a particular
solution of the given equation (3).
The function Y may be found by the method of undetermined roe{ficunts
in the following simple cases:
I. f (x) = eax P,. (x), where P11 (x) is a polynomial of degree n.
If a is not a root of the characteristic equation (2), that is, q> (a) r:f: 0,
then we put Y=e•x Q,. (x) \\here Q,. (x) is a polynomial of degree n with
undetermined coeflicients.
If a 1s a root of the characteristic equation 12), that is, q> (a) =0, then
Y=x'eaxQn(X), \\here r is the multiplicity of the root a(r=l or r=2).
2. f (x)=e0 x [P 11 (x) cos bx+ Q111 (x) sin bx).
352 Difjerential Equations (Ch. 9

If q> (a ± bi) :p 0, then we put


Y =e0 " [SN (x) cos bx+ TN (x) sin bx),
where SN (x) and T 1:1 (x) are polynomials of degree N-max {n, m}.
But if q> (a± b1) =0, then
Y=xreax [SN(x) cos bx+TN (x) sin bx),
whrre r is the multiplicity of the roots a ± b1 (for second-order equations,
r= 1).
In the general case, the method of variation of parameters (see Sec. II)
is usl'd to solve equatiOn (3).
Example l. Find the ~eneral solution of the equation 2y"-y'-y=4xe2".
Solution. The characteristic equation 2k 2 - k - l =0 has roots k 1 =I and
k2 =-; . The general solution of the corresponding homogeneous equation
X

(first type) is lfo= C1e" +C,e 2 • 1 he right side of the given equation is f (x) =
=4xe2"=t 0 "Pn(x). Hence, Y=e 2"(Ax+Bl, since n=l and t=O. Difieren-
tiating Y tw1ce and putting the derivatives into the given equation, we
obtain:
2e 1" (4Ax+ 48 + 4A)-e 2" (2Ax+ 28 + A)-e 2" (Ax+ B)= 4xe~".
Can,crlling out e2" and rquating the coefficients of identical powers of x and
the absolute terms on the left and right of the equality, we have 5A =4 and
4 28
1A+5B=0, whence A=s and B=- .
25
Thus, Ye 2" (: x-~~), and the general solution of the given equation 1s
I

y= C1ex + C2e --. '+ e·•x ( 4x- 2528) .


5
Example 2. Find the general solution of the equation y"-2y'+y=xe".
Solution. The <"haractensllc equatiOn k 2 -2k+ I =0 has a double root
k=l The ri:~;ht s1de of lht> equ<~tiOn 1~ of the form f(x)=xex; hert>, a=l
and n=l. The parlt<"ular solution is Y=x 2e"(Ax+B), since a cOincides \1J'Ith
the doublr root k=l and, consequently, r=2.
Diflerent1ating Y h\ ice. substituting into the equation, and equating the
coefficients. we oblatn A= i,. 8 = 0, Hence, the general solution of the given
equation will be \Httten in the form
y=(C 1 -j-C 2x)e"+! x3e".

Example 3. Find the general solution of the equation y• +If= X sin x.


Solution. The characteristic rquation k2 +I = 0 has roots k 1 = i and
k 2 = - 1 . The general solul10n ol the corresponding homogeneous equ<~tion
will lsee 3, where a.=O and p = IJ be·
y 0 =C1 cos x+ C1 sin x.

The right side is of the form


f {xJ =eax (Pn (x) cos bt + Q m (x) sln bxJ,
Sec. 12] Linear Differential Equations with Constant Coefficients 353

where a=O, b=l, Pn(x)=O, Qm(x)=x. To this side there corresponds the
particular solution Y,
Y=x [(Ax+B) cosx+(Cx+D) slnx)
(here, N=l, a=O, b=l, r=l).
DifTerentiating twice and substituting into the equation, we equate the
coE'ffic1ents of both sides in cos x, x cos x, sin x, and x sm x. We then get four
equations2A+2D=0,4C=0, -2B+2C=O, -4A=l. from which we deter·
. I 1 x• x
mme A=- , 8=0, C=O, D= . Therefore, Y=--;rcosx+-;rsinx.
4 4
The general solution is
x2 X
y=C 1 cos x+C 2 slnx- cos x+ 4 sin x.
4
3°. The principle of superposition of solutions. If the right side of equa-
tion (3) is the sum of several funct:ons
f (x) = f 1 (x) + f 2 (x) + ·· · + f n (x)
and Y; (i =I, 2, 3, ... , n) are the corresponding solutions of the equations
y"+py'+qy=,fj(x) (i=l. 2, ... , n),
then the sum
y=Y 1 +Y2 + ... +Yn
is the solution of equation (3).

Find the general solutions of the equations:


2976. y"- 5y' + 6y = 0. 2982. y" + 2y' + y = 0.
2977. y"-9y=0. 2983. y"-4y' +2y=0.
2978. y"- y' = 0. 2984. y• + ky = 0.
2979. y" + y= 0. 2985. y= y" + y'.
2980. y"- 2y' + 2y = 0. y' -y
2981. +
y" 4y' + 13y = 0. 2986. 7 = 3.
Find the particular solutions that satisfy the indicated condi·
tions:
2987. y"-5y'+4y=0; y=5, y'=B for x=O
2988. y" +3y' +2y=0; y= 1, y' = -1 for x=O.
2989. y" + 4y = 0; y = 0, y' = 2 for x = 0.
2990. y" + 2y' = 0; y = 1, y' = 0 for x = 0
2991. y" = a11.; y =a, y' = 0 for x = 0.
2992. y"+3y'=0; y=O for X=O and y=O for x=3.
2993. y" + n 2 y=0; y=O for x=O and y=O for X= I.
2994. Indicate the type of particula1 solutwns for the given
inhomogeneous equations:
a) y"- 4y = x 2 e""";
b) y"+9y=cos2x;
12-J 900
354 DiUerential Equations (Ch. 9

c) y"-4y' -j-4y= sin 2x+eu;


d) y" + 2y' + 2y= e" sin x;
e) y'' -5y' + 6y = (x1 +- 1) e" + xeu;
f) y"- 2y' + 5y = xe" cos 2x-x1 e" sin 2x.
Find the general solutions of the equations:
2995. y"-4y' +4y=x1 •
2996. y"-y' +u=x'+6.
2997. y" + 2y' + y = ezx.
2998. y" -By'+ 7y= 14.
2999. y" -y=e".
3000. y" + y =COS X.
3001. y" + y' -2y = 8 sin 2x.
3002. y" + y' -6y = xe 2 " .
3003. y"-2y' +Y= sinx+sinhx.
3004. y" + y' = sin2 x.
3005. y"- 2y' -t- 5y = e" cos 2x.
3006. Find the solution of the equation y"+4y= sinx that
satisfies the conditions y = 1, y' = 1 for x = 0.
Solve the equations:
3007. ~:~ + ffi 1 x = A sin pt. Consider the cases: l) p =1= ffi;
2) p = (l),
3008. y"-7y' + l2y=-eu.
3009. y''-2y' =X1 -l.
3010. y" -2y' + y= 2e".
3011. y" -2y' = ezx + 5.
3012. y" -2y' -By= e"'-8 cos 2x.
3013. y"~tu'=5x+2e".
3014. y"- y' = 2x-1-3e".
3015. y"+2y'+y=e"+e-".
3016. y"-2y' +lOy= sin 3x +e".
3017. y"-4y' + 4y=2ezx+;.
3018. y"-3y' =x+cosx.
3019. Find the solution to the equation y"-2y'=eu+x1 - l
that satisfies the conditions y = i·
y' = 1 for x = 0.
Solve the equations:
3020. y"- y = 2x sin x.
3021. y" -4y = e2 " sin 2x.
3022. y" + 4y= 2 sin 2x-3 cos 2x+ 1.
3023. y"- 2y' + 2y = 4e" sin x.
3024. y" = xe" + y.
3025. y" + 9y = 2X sin X+ xeax,
Sec. 12) Linear Differential Equations with Constant Coefficient., 355

3026. y"-2y'-3y=x(1 +e"').


3027. y"-2y' =3x+2xe".
3028. y"- 4y' + 4y = xe"'.
3029. y" + 2y' -3y= 2xe-"' + (x-t 1) e".
3030*. y" + y = 2x cos x cos 2x.
3031. y"-2y=2xe"(cosx--sinx).
Applying the method of variation of parameters, solve the
following equations:
3032. y" + y =tan x. 3036. Y"+ Y=cosx·
I

3033. y" + y =cot x. 3037. y" + y=-!-.


smx
e"
3034. y"-2y' +y=-.
X
3038. a) y"-y=tanhx.
3035. y" + 2y' + y = 7
e-" . b) y" -2y = 4x 1ex•.
3039. Two identical loads are suspended from the end of a
spring. Find the equation of motion that will be p~rformed by
one of these loads if the other falls.
Solution. Let the increase in the length of the spring under the action
of one load in a state of rest be a and the mass of the load, m. Denote by x
the coordinate of the load reckoned vertically from the position of equilib-
rium in the case of a single load. Then
d2x
m dt•=mg-k (x+a),

mg d 2x g
where, obviously, k=a and, consequently, dt·=-a x. The general ~olu-

tion is X=C,cos
d~
y! t+C.sln v"! t. The initial condttiOnS yteld X=U

and dt =0 when t =0; whence C 1 =a and C1 =0; and so

x=acos y! t.
3040*. The force stretching a spring is proportional, to the
increase in its length and is equal to 1 kgf when the length
increases by 1 em. 1\ load weighing 2 i<gf is suspended from the
spring. Find the period of oscillatory motion of the load if it
is pulled downwards slightly and then released. ·
3041*. A load weighing P=4 kgf is suspended from a spring
a_nd increases the length of the spring by 1 em. Hnd the law
of motion of the load if the upper end of the spring performs
a vertical harmonic oscillation y=2sin30tcm and if at the
initial instant the load was at rest (resistance of the medium is
neglected). L

12"
356 DiUerentlal Equations [Ch. 9

3042. A material point of mass m is attracted by each of two


centres with a force proportional to the distance (the constant
of proportionality is k). Find the law of motion of the point
knowing that the distance between the centres is 2b, at the ini-
tial instant the point was located on the line connecting the
centres (at a distance c from its midpoint) and had a velocity
of zero.
3043. A chain of length 6 metres is sliding from a support
without friction. If the motion begins when l m of the chain
is hanging from the support, how long will it take for the entire
chain to slide down?
3044*. A long narrow tube is revolving with constant angular
velocity ro about a vertical axis perpendicular to it. A ball in-
side the tube is slidin5 along it w1thout friction. Find the law
of motion of the ball relative to the tube, considering that
a) at the initial instant the ball was at a distance a from
the axis of rotation; the initial velocity of the ball was zero;
b) at the initial instant the ball was located on the axis of
rotation and had an initial veloc1ty v 0 •

Sec. 13. Linear Differential Equations of Order Higher than Two with
Constant Coefficients
1°, Homogeneous equations. The fundamental system of solutions y 1 ,
y 1, Yn of a homogeneous linear equation with constant coefficients
••• ,

y<">-ta1y<"- 1'+ •.. +a,._ 1y' +a,.y=O (I)

Is constructed on the basis of the character of the roots of the characteristic


equation
(2)
Namely, I) if k is a real root of the equation (2) of multiplicity m, then to
this root there correspond m linearly independent solutions of equation (1):
Yl=ekx, u.=xelex, ... , Ym=xm-tekX;

2) if a±~~ Is a pair of complex roots of equation (2) of multiplicity m,


then to the latter there correspond 2m linearly independent solutions of
equation (1):
Yl =e~" cos ~x. u.=e~X sin ~x. y,=xe~" cos ~x. y, =xe~X sin ~x • ...
. . . , Yzm- 1 = xm-le~" cos ~X, Yzm = xm- 1 e~x sin ~x.

2°. Inhomogeneous equations. A particular solution of the inhomogeneous


equation
y<n> + a1y<n-ll + ... + a11 _ 1 y' + a,.y = f (x) (3)

Ia sought on the basis of rules 2° and 3° of Sec. 12.


Sec. 14] Euler's Equations 357

Find the general solutions of the equations:


3045. y'" -13y" + 12y' = 0. 3058. yiV + 2y" y = 0. +
3046. y'"- y' = 0. . n y(n-1) -t-
3059. y'"l ..,T
3047. y"' + y = 0.
3048. ylV -2y" = 0. -l
_n(n-1) (n-z)+
1·2 y ..•
+
3049. y"'-3y" + 3y'-y=0.
3050, yiV + 4y = 0. +-Tu'+y=O.
3051. y 1vt·8y"+l6y=0.
3052. ylV + y' = 0. 3060. yiv -2y"' + y'' = e".
305:3. ytV -2y" + y = 0. 3061. ylV -2y"' +y" =X1 •
oBt>54. y 1v -a 4 y = 0. 3062. y'" -y = x'-1.
3063. yiV + y"' = cos 4x.
3055. yiV -6t/' + 9y = 0.
3064. y''' +- y" = x• + I + 3xe.c,
3056. y 1v + a•y" = 0.
3057. 3065. y"' + y" + y' + y = xe".
ylV + 2y''' -j- y" = 0.
3066. y'"+y'=tanxsecx.
3067. Find the particular solution of the equation
y"' +2y" +2y' +y=x
that satisfies the initial conditions y(O)=y'(O)=y"(O)=O.

Sec. 14. Euler's Equations


A linear equatiOn of the form
(ax+ b)"y'"'+ A, (ax+ b)''-'y'"-n+ ... + A,,_ 1 (ax+ b) Y+ A,.y=f(x), (1)
where a, b. A 1 , ••• , A,_,, A, are constants, IS called E•Jler's equation.
Let us mtroduce a new Independent vartable I, putting
ax-j-b-=e1.
Then
Y , =ae-1 t!:!J.
dt'
y" =a2e-21 (d'l/- dy)
dt' dt •
I" 3 31 ( d' f/~ d!l/ 2 dt/) d f th
Y =ae- dt·-· dt'+ dt an so or
and Euler's equation is transformed 1nto a linear equatwn with constant
coefliCJents.
fxample 1. ~olve the equat10n x•y" xy' y = 1. + +
Solution. Putting x=e 1, we get
2 2
dy -I dq d 1!- -21 ( d y dq)
dx=e dt' dx 2 -e \dt 2 -dt 1 '
Consequently, the given equatio 1 takes on the form
d•y
dl 2 +y=l,
whence
y= cl cos t +c. sin t + 1
or
g=C 1 cos (lnx)+C1 sln (In x)+ 1.
358 DiOerential Equations [Ch. 9

For the homogeneous Euler equation


xny<n>+ Alxn-lyin-ll+ ... +An-lxy' +Any=O (2)

the solution may be sought in the form


y=xk. (3)

Putting into (2) y, y', ... , y<n> found from (3), we get a characteristic equa·
tion from which we can find the exponent k.
If k is a real root of the characteristic equation of multiplicity m, then to it
correspond m linearly independent solutions
Y1 =xk, Y1 =xk·lnx, y1 =xk(lnx) 1 , . . . , Ym=xk(lnx)m- 1 •
If a± ~i is a pair of complex roots of multiplicity m, then to it there
correspond 2m linearly independent solutions
y 1 =x~cos (~lnx), y 1 =x" sin(~ In x), y1 =x" lnxcos (~In x),
y,=x" In X·sln (~In x), ... , Yam-l =x" (In x)m- 1 cos(~ In x),
Yzm=X" (In x)m-• sin(~ lnx).
Example 2. Solve the equation
x 2y"- 3xy' + 4y = 0.
Solution. We put
y=xk, y' = kxk- 1 , y"=k (k-1) xk-a.
Substituting into the given equation, after cancelling out xk, we get the
characteristic equation

Solving it we find

Hence, the general solution will be


y = C1 x2 + C1x2 In x.

Solve the equations:


2 d2Y dy
3068. x dx•+ 3x dxy= 0.
+
3069. x'y"-xy'-3y=0.
X y" -1- xy' + 4y = 0.
2
3070.
3071. x•y"' -3x'y" + 6xy' -6y = 0.
3072. (3x + 2) y" + 7y' = 0.
3073. y" = :~.
y' y
3074. y" +-+-.=0.
X X
3075. x•y"-4xy' +6y=x.
3076. (1 + x) y•- 3 ( 1 + x) y' + 4y = (1
2
+x)'.
Sec. 151 Systems of Differential Equations 359

3077. Find the particular solution of the equation


x•y" -xy' + y= 2x
that satisfies the initial conditions y=O, y' = 1 when X= 1.

Sec. 15. Systems of Differential Equations


Method of elimination. To find the solution, for instance, of a normal
system of two first-order differential equations, that is, of a system of the
form
dy dz
dx=f(x, y, z), dx =g (x, y, z), (I)

solved for the derivatives of the desired functions, we differentiate one of


them with respect to x. We have, for example,
d 2 y of of of
dx 2 = ox +-ay +
f ih g. (2)

Determining z from the first equation of the system (I) and substituting the
value found,
(3)

into equation (2), we get a second-order equation with one unknown func-
tion IJ. Solving it, we find
(4)
where C1 and C1 are arbitrary constants. Substituting function (4) into for-
mula (3), we determine the function z without new integrations. The set of
formulas (3) and (4), where y is replaced by lj>, yields the general solution
of the system (I).
Example. Solve the system

~~ +2y +4z= 1+4x,


dz 3 1
{ dx+y-z=2x ·
Solution. We differentiate the first equation with respect to x:
d•y +2~+4 dz=4.
dx 1 dx dx

From the first equation we determine z=} ( 1+4x-~~-2y) and then

from the second we will have dz =~ x'+x+ _!_-~y-_!_ dy_ Putting z


dx 2 4 2 4 d~
and:~ into the equation obtained after differentiation, we arrive at a secord•
order equation in one unknown y:
d•y +~-6y=-6x 1 -4x+3.
dx 1 dx.
360 DiUerential Equations [Ch. 9

Solving it we find:

and then
dy
1 ( 1+4x---2y
z=- c - 3x--x
) =-C1e1X+-!e 1 1
4 dx 4 2'

We can do likewise in the case of a system with a larger number of


equations.

Solve the systems:


'!Jf_=Z :~ + 3y + 4z = 2x,
3078. dx '
{ dz
dx=-y.
3085. { dx-y-Z=X,
dz

:~=y+5z, y=O, z=O when x=O.


3079. { dz ~;-4x-y+36t=0,
dx+Y+3z=0.
df/
- =-3y-z
dx '
3086.
j d
_]!.
dt + 2x -- y + 2e1 = 0,
3080. x = 0, y = 1 when t = 0.
{ dz
dx = y-- z. dy !12

(~]=y.
dlJ
3087. { ~=~·
dx-2Y·
3081. dt=Z,
3088* • a ) x 3 +dx3xy2 = 2y
dy dz .
t dz
· dt =X.
b)~= dy =dz.
3 = 2y 2 z'

~~ =Y t
x-y xty z'
Z,
c) ~ = dy = ..!!!__
dy y-z z-x x-y'
3082. dt=x+z.
ft dz
dt=x+y.
isolate the integral curve pas-
sing through the point (1, 1, -2).
dy+ -1
:~=y+z. d~ z- '
3083.
l dz
dx=X+y+z.
3089.
{ dz
dx+X2Y=lnx.
d2
2

~+2y+z= sinx, d~ +2y+4z =ex,


3084. { dz 3090.
{ dz
2
dx -4y-2z = cosx. dx2-y-3z = -x.
Sec. /6) Integration of Differential Equations by Power Serles 361

3091**. A shell leaves a gun with initial velocity tJ 0 at an


angle a to the horizon. Find the equation of mE>tion if we take
the air resistance as proportional to the velocity.
3092*. A material point is attracted by a centre 0 with a
force proportional to the distancE'. The motion begins from point A
at a distance a from the centre with initial velocity tJ 0 perpen-
dicular to 0 A. Find the trajectory.

Sec. 16. Integration of Differential Equations by Means of Power Series


If it is not possible to integrate a differential equation with the help of
elementary functions, then in some cases 1ts solution may be sought in the
form of a power series:

(1)
n=o
The undetermined coefficients en (n=l, 2, ... ) are found by putting the
series (I) into the equation and equating the coefficients of identical powers
of the binomial x-x 0 on the left-hand and right-hand sides of the resulting
equation.
We can also seek the solution of the equation
y'=f(x, y); y(xo)=Yo (2)
In the form of the Taylor's series
"'
y(x) = L. y<n~~Xo) (X-Xo)n, (3)
n=o
where y (x 0) = y0 , y' (x 0 ) = f (x 0 , y 0) and the subsequent dPrivatives y<m (x 0 )
(n =-- 2. 3, ... ) are successively found by difTcr~ntiating e'luation (2) and by
putting x0 in place of x
Example 1. Find the solution of the equation
y"-xy=O,
If Y=Yo· y' =Y: for x=O.
Solution. We put

whence, differentiating, we get


y" = 2·1c1 + 3·2c1x + •.. + n (n-1) CnXn-z + (n + 1) ncn+Jxn-J +
+ (n+2) (n+ 1) Cn+zxn+ .. ,

Substituting y and y" into the given equation, we arrive at the identity
[2·lc1 + 3·2C1X + ... + n (n-1) Cnxn-z+ (n + l) ncn+Jxn-J +
+<n +2) (n + 1) cnHxn+ .. . ]-x Jc0+c 1x+ ..• +cnx"+ •• • ] :50,

Collecting together, on the left of this equation, the terms with identical
powers of x and equating to zero the coef1ic1ents of these powers, we wlll
362 Differential Equations [Ch. 9

have
c1 =0; 3·2c1 -c0 =0, c1 = Co. ; 4·3c4 -c1 =0, c,= cl. ., 5 ·4c1 -c1 =0.
3 2 43
c, = ;.• and so forth.
4
Generally,
Co cl
Cak 2·3·5·6· ... · (3k-1) 3k' Cak+J =
3·4·6· 7 • ... ·3k (3k + 1)'
Cak+a=O (k= 1, 2, 3, ... ).
Consequently,
x• x• x•k )
Y =Co ( 1 + 2·3 +2·3·5·6 + ·' · +2·3·5·6· ... ·(3k-l) 3k + ·" +
x• x7 x•kq. J )

+c1 ( x+3.4 +3·4·6·7+ .. · + 3·4·6·7· ... ·3k (3k+ 1) + .. · ' (4)

where c0 =y0 and c 1 =y~.


Applying d' Alembert's test, it is readily seen that series (4) converges
for - oo < x < + oo.
Example 2. Find the solution of the equation
y' =x+ y; Yo=Y (0)= 1.
Solution. We put , ,,,
_ ' ·yo 2 Yo 1
Y=-Yo+Yox+-wx+3lx -j- ...
We have y 0 = 1, y~ = 0 + 1 = l. Differentiating equation y' = x + y, we succes-
sively find y"=l+y', y:=l-l-1=2, y"'=y", y;"=2, etc. Consequently,

-1 -j-x +~
Y- XI -J- !31 XI -j- ...
21
For the examP,le at hand, this solution may be written in final form as
y=l+x+2(ex-1-x) or y=2eX-1-x.
The procedure is similar for differential equations of higher orders. Test-
ing the resulting series for convergence is, generally speaking, complicated
and is not obligatory when solving the problems of this section.

With the help of power series, find the solutions of the equa-
tions for the indicated initial conditions.
In Examples 3097, 3098, 3099, 3101, test the solutions
obtained for convergence.
3093. y' =y+x2 ; y= -2 for x=O.
3094. y'=2y+x-1; Y=Y 0 for X=l.
3095. y' = y• + x•; y = ; for x = 0.
3096. y'=x1 -y 2 ; y=O for x=O.
3097. (1-x)y' = 1 +x-y; y=O for x=O.
Sec. /7) Problems on Fourier's Method 363

3098*. xy" + y = 0; y = 0, y' = 1 for x = 0.


3099. y" +xy = 0; y= 1, y' =0 for x=O.
3100*. y"+~y'-t-y=O;
X
y=l, y'=O for x=O.
3101*. y"+.I_y'+y=O;
X
y=l, y'=O for x=O.
d 2x dx
3102. dt• +xcost=O; x=a; dt =0 for t=O.

Sec. 17. Problems on Fourier's Method


To find the solutions of a linear homogeneous partial differential equation
by Fourier's method, first seek the particular solutions of this special-type
equation, each of which represents the product of functions that are dependent
on one argument only. In the simplest case, there is an infinite set of such
solutions un (n= I, 2, ... ), which are linearly independent among themselves
in any finite number and which satisfy the given boundary conditions. The
desired solution u is represented in the form of a series arranged according
to these particular solutions:

(I)
n=•
The coefficients Cn which remain undetermined are found from the initial
conditions.
Problem. A transversal displacement u =u (x, t) of the points of a string
with abscissa x sat isfit>s, at time t, the eq nation

(2)

wht>re a• = ~ (T0 is the tensile force and Q is the linear density of the
Q
string). Find the form of the string at time t if its ends x=O and x=l are

Fig. 107

fixed and at the initial instant, t =0, the string had the form of a parabola
u = ~: x (l-x) (Fig. 107) and its points had zero velocity.
Solution. It is required to .~nd the solution u=u (x, t) of equation (2)
that satisfies the boundary condthons
u(O, t)=O, u(l, t)=O (3)
364 Differential Equations (Ch. 9

and the initial conditions


4h
u(x,O)=px(l-x), u;(x,O)=O. (4)

We seek the nonzero solutions of eouation (2) of the special form


u=X (x) T(t).
Putting this expression into equation (2) and separating the variables, we get
T" (t) X" (x)
(5)
a2 T ll)= X (x) ·
Since the variables x and t are independent. equation (5) is possible only
when the general quantity of relation (5) is constant Denoting this constant
by - i.. 2 , we find two ordinary ddTerential equations:
T"(t)+(ai..) 2 ·T(t)=0 and X"(x)+i..'X(x)=O.
Solving these equations, we get
T (t) =A cos ai..t + B sin ai..t,
X (x) = C cos i..x + D sin A.x,
where A, 8, C, D are arbitrary constants. Let us determine the constants.
From condition (3) we have X (0) = 0 and X(/)= 0; hence, C = 0 and
sin U=O (since D cannot be equal to zero at the same time as C is zero).
For this reason, A.k= ~n, where k is an mteger. It will readily be seen that
we do not lose generality by taking for k only positive values (k = 1, 2, 3, ... ).
To every value A.k there corresponds a particular solut10n
kan kan ) knx
uk= ( Ak cos--t+Bksin-- t sin--
1 1 1
lhat satisfies the boundary conditions (3).
We construct the series
"' (
~ kant . kant) . knx
U=~ Akcos--+Bksm- - sm- -,
1 1 1
k=l

whose sum obviously satisfies equation (2) and the boundary conditions (3).
We choose the constants A" ancl Bk so that the sum of the series should
satisfy the initial conditions (4). Since
"'
~ kan ( knx
iJu
at=~ 1- - . kant
- Ak sm - 1- + Bk cos k2nt)
- - sin - - ,
1 1
k=J
it follows that, by putting t = 0, we obtain

~"' . krr.x 4h
u (x, 0) = ~ Ak sm - -== jl x (l-x)
1
k=l
and
iJu (x, 0) _ ~"' kan B . krr.x
at - ~ - 1- k sm - - oaO,
1
k=l
Sec. 17] Problems on Fourier's Method 365

Hence, to determine the coefficients A11 and 8 11 it is necessary to expand in


a Fourier series, in sines only, the function u (x, 0) = ~ x (l-x) and the
.
f Unc t IOn
au (x, O)
iJt e= 0 .
From familiar formulas (Ch. VIII, Sec. 4,3°) we have
l

A.t=T J
2 r 4h x(l-x)sln-
kn.x 32h
dx=n¥,
11 1
0

if k is odd, and A11 =0 if k is even;

-kan 2
- B =y
1 11
s l
knx
Osln--dx=O, 8 11 =0.
1
0
The sought-for solution will be
(2n +I) ant
rn cos-----
- 32h ~ J • (2n +I) nx
u- na L +
l2n I )1 sm l '
n=o

3103*. At the initial instant t=O, a string, attached at


its ends, x = 0 and x = /, had the form of lhe sine curve
u =A sin 7,
and the points of it had zero velocity. Find the
form of the string at time t.
31 04*. At the initial time t = 0, the points of a straight
string 0 < x < l receive a velocity ~ = I. Find the form of the
strin~ at time t if 1he ends of the string x=O and x=l are
fixed (see Problem 3103).
3105*. A string of length l = 100 em and attached at its ends,
x=O and x=l, is pulled out to a distance h=2 em at point
x ==50 em at the initial time, and is then released without any
impulse. Delermine the shape of the string at any time t.
3106*. In longitudinal vibrations of a thin homogeneous
and rectilinear rod, whose axis coincides with the x-axis, the
displacement u = u (x, t) of a cross-section of the rod with
abscissa x satisfies, at time t, the equation
iJ2u z iJ2u
dtz =a axz •
where a• =!!.. (E is Young's modulus and Q is the density of the
Q
rod). Determine the longiturlinal vibrations of an elastic hori-
zontal rod of length l = 100 em fixed at the end x = 0 and pulled
back at the end x = 100 by fll = 1 em, and then released without
1m pulse.
366 · DiUerential Equations [Ch. 9

3107*. For a rectilinear homogeneous rod whose axis coincides


with the x-axis, the temperature u = u (x, t) in a cross-section with
abscissa x at time t, in the absence of sources of heat, satisfies
ihe equation of heat conduction
au 1 au
2

at =a ax 1 '

where a is a constant. Determine the temperature distribution


for any time t in a rod of length 100 em if we know the initial
temperature distribution
u (x, 0) = 0.01 x (100-x).
Chapter X

APPROXIMATE CALCULATIONS

Sec. 1. Operations on Approximate Numbers


1°. Absolute error. The ab~olute error of an approximate number a which
replaces the exact number A is the absolute value of the difference between
them. The number !::., which satisfies the inequality
IA-alt;;;;;;t:., (I)
is called the limtting absolute error. The exact number A is located within
the limits a-t:..,;;;;; A<: a+!::. or, more briefly, A =a± /::,.
2°. Relative error. By the relative error of an approximate number a
replacing an exact number A (A > 0) we understand the ratio of the absolute
error of the number a to the exact number A. The number {},which satisfies
the inequality
I A-al _.. 6 (2)
A -'
Is called the limiting relative error of the approximate number a. Since in
actual practice A ~a. we often take the number 6=_! for the limiting
a
rei a ti ve error.
3°. Number of correct decimals. We say that a positive approximate
number a written in the form of a decimal expansion has n correct decimal
places in a narrow sense if the absolute error of this number does not exceed
one half unit of the nth decimal place. In this case, when n >I we can
take, for the limiting relative error, the number

where k is the first significant digit of the number a. And conversely, if it

2 1
)n-t
is known that 6.,.;;; (k1+ ) ( lO
1 , then the number a has n correct decimal
places in the narrow meaning of the word. In particular, the number a
definitely has n correct decimals in the narrow meaning if 6.,.;;; ~ (I~
If the absolute error of an approximate number a does not exceed a
r.
unit of the last decimal place (such, for example, are numbers resulting
from measurements made to a definite accuracy), then it is said that all
decimal places of this approximate number are correct in a broad sense. If
there is a larger number of significant digits in the approximate number,
the latter (if it is the final result of calculations) is ordinarily rounded off
so that all the remaining digits are correct in the narrow or broad sense.
368 Approximate Calculations [Ch. 10

Hencelorth, we shall assume that all digits in the initial data are
correct (if not otherwise staled) in the narrow sense. The results of inter·
mediate calculations may contain one or two reserve digits.
We note that the examples of this secliun are, as a rule, the results of
final calculataons, and for this reason the answers to them are given as
approximate numbers with only correct decimals.
4°. Addition and subtraction of approximate numbers. The limiting ab-
solute error ol an algebraic sum of sevE'ral numbers is equal to the sum of
the limiting absolute errors of these numbers. Therefore, in order to have,
in the sum of a ~mall number of approximate numbers (all decimal places
of which are correct), only correct dig1ts (at least in the broad sense), all
summands should be put into the form of that summand which has the
smallest number of decimal places, and in each summancl a reserve digit
should be retained. Then add lhe resulting numbers as exact numbers, and
round off the sum by one decimal place
If we have to add approximate numbers that have not been rounded off,
lhey should be rouncled off and one or two reserve digits ~hould be retained.
Then be guided by thE' forego~ng rule of addition while rt>taining the appro·
priate extra digits in the sum up to the end of the calculations.
Example 1. 215.21 + 14.182 + 21.4 = 215.2tl) + 14.1 (8) + 21 4 = 250.8.
The relative error of a sum of po~itive terms l1es between the least and
greatest relative errors of these terms.
The relative error of a d1fl'erence is not amenable to simple counting.
Particularly unfavourable in this sense is the dilference of two close numbers.
Example 2. In subtracting the a;,proximate numbers 6 135 and 6.131 to
four correct decimal places, we get the difference 0 004. The limiting relative
. ~ 0.001 + ; 0.1'01 I
error IS 6= 0.00 =-;r=0.25. Hence, not one of the decimals
4
of the difference is correct. Therefore, it is always advisable to avoid
subtracting close approximate numbers and to tnmsform the given expression,
if need be, so that this undesir~l;;le operation is omitted.
5°. Multiplication and division of approximate numbers. The limiting
relative error of a product and a quotient of approximate numhers is equal
to the sum of the limiting relative errors of these numbers Proceeding from
lhis and applying the rule for the number of correct decin.als (3°), we retain
In the answer only a definite number of decimals
Example 3. The product of the approximate numbers 25.3·4.12= 104.236.
Assuming that all d£cimals of the factors are correct, we find that the
limiting relative error of the product is
I I
6=2-20.01 + 4-20.01 ~0.003.
Whence the number of correct decimals of the product Is three and the
result, if it is final, should be written as follows: 25.3·4 12= 104, or more
correctly, 25 3-4.12= 104 2 ± 11.3.
6°. Powers and roots of approximate numbers. The limihng relative error
of the mth power of an approximate number a is equal to the m-fold limiting
relative error of this number
The limiting relative error of the mth root of an approximate number a
I
Is the - th part of the limiting relative error of the number a.
m
7°. Calculating the error of the result of various operations on approxi-
mate numbers. If Aa1 , ... , lla,. are the limiting absolute errors of the appro-
Sec. I) Operations on Approximate Numbers 369

ximate numbers a 1, •• , , an, then the limiting absolute error ll.S of the result
S = f (ar. ... , an)
may be evaluated approximately from the formula

t:.S =I :1.1 t:.a 1 + ' '· +I :L I !:.an.

The limiting relative error S is then equal to

68
= ~; 1=I:!. I· fn +···+I :!J ~~~n =
= olnfl
aa. t:.a. + ... + latnf/
dan !:.an.

Example 4. Evaluate S =In (10.3+ V 4.4 ); the approximate numbers


10.3 and 4.4 are correct to one decimal place.
Solution. Let us first compute the limiting absolute error ll.S in the
general form: S=ln (a+ Yb). ll.S= a+ IV b(t:.a +~ Vbb) . We have

Jl.a=t:.b~ 210; vn = 2.0976... ; we leave 2.1, since the relative error of

the approximate number Y4.4 is equal to:::::~ ·~=;0 ; the absolute error

is then equal to ::::::: 2 ~ = k; we can be sure of the first decimal place. Hence,
I ( I I I ) I ( I ) 13
ll.S= 10.3+2.1 20 +lr. 20·2.1 = 12.4·20 I+ 4.2 =2604::::::: 0 · 005 ·
Thus, two decimal places will be correct.
Now let us do the calculations with one reserve decimal:
log (10.3 + V44) ~log 12 4 = 1.093, In (10 3 + YU):::::: 1.093·2.303 = 2.517.
And we get the answer: 2 52
8°. Establishing admissible errors of approximate numbers for a given
error in the result of operations on them. Arplying the formulas of 7° for
the quantities ll.S or 6S given us and considering all particular differentials
1a~k 1 flak or the quantities 1a:~ 1~ti equal. we calculate the admissible
ab~olute errors t:.a 1 , ••• , ll.an, ... of the approximate numbers a 1 , ••• , an, .••
that enter into the operations (the principle of equal effects).
It should be pointed out that sometimes when calculating the admissible
error~ of the arguments of a funcbon it is not advantageous to use the
principle of equal effects, since the latter may make demands that are
practically unfulfilable In these cases it is advisable to make a reasonable
redistribution of errors (if !hi!. is possible) so that the overall total error does
not exceed a specified quantity. Thus, strictly speaking, the problem thus
posed is indeterm !nate.
Example 5. The volume of a "cylindrical segment", that is, ·a mlid cut
off a circular cylinder by a plane passing through the diameter of the base
(equal to 2R) at an angle a to the base, is computed from the formula
2
V ='3 R1 tan a. To what degree of accuracy should we measure the radius
370 Approximate Calculations [Ch. 10

R::::::: 60 em and the angle of Inclination a so that the volume of the cylindrical
segment is found to an accuracy up to 1%?
Solution. If LlV, LlR and .!la are the limiting absolute errors of the
quantities V, R and a, then the limiting relative error of the volume V that
we are calculating is
6 =MR 2Aa ,.;;;;-1
R +
sin2a 100 ·

MR I 2Aa 1
We assume R,.;;;; 200 and sin a,.;;;;
2 200
. Whence

R 60 em
AR ,.;;;;600::::::: 600 = 1 mm;
sin 2a 1 d' ,
6 a ,.;;;; 400 ,.;;;; 400 ra tan::::::: 9 •

Thus, we ensure the desired accuracy in the answer to I% if we measure


the radius to 1 mm and the angle of inclination a to 9',

3108. Measurements yielded the following approximate numbers


that are correct in the broad meaning to the number of decimal
places indicated:
a) 12°07'14"; b) 38.5 em; c) 62.215 kg.
Compute their absolute and relative errors.
3109. Compute the absolute and relative errors of the follow-
ing approximate numbers which are correct in the narrow sense
to the decimal places indicated:
a) 241.7; b) 0.035; c) 3.14.
3110. Determine the number of correct (in the narrow sense)
decimals and write the approximate numbers:
a) 48.361 for an accuracy of 1%;
b) 14.9360 for_an accuracy of 1%;
c) 592.8 for an accuracy of 2%.
3111. Add the approximate numbers, which are correct to the
indicated decimals:
a) 25.386 +0.49 + 3. I0 + 0.5;
b) 1.2·101 +41.72+0.09;
c) 38. I+ 2.0 + 3.124.
3112. Subtract the approximate numbers, which are correct
to the indicated decimals:
a) 148.1-63.871; b) 29.72-II.25; c) 34.22-34.21.
3113*. Find the difference of the areas of two squares whose
measured sides are 15.28 em and 15.22 em (accurate to 0.05 mm).
3114. Find the product of the approximate numbers, which
are correct to the indicated decimals:
a) 3.49·8.6; b) 25.1·1.743; c) 0.02·16.5. Indicate the possible
limits of the results.
Sec. I] Operations on Approximate Numbers 371

3115. The sides of a rectangle are 4.02 and 4.96 m (accurate


to 1 em). Compute the area of the rectangle.
3116. Find the quotient of the approximate numbers, which
are correct to the indicated decimals:
a) 5.684 : 5.032; b) 0.144 : 1.2; c) 216:4.
3117. The legs of a right triangle are 12.10 em and 25.21 em
(accurate to 0.01 em). Compute the tangent of the angle opposite
the first leg.
3118. Compute the indicated powers of the approximate
numbers (the bases are correct to the indicated decimals):
a) 0.4158 1 ; b) 65.2'; c) 1.51 •
3119. The side of a square is 45.3 em (accurate to I mm).
Find the area.
3120. Compute the values of the roots (the radicands are
correct to the indicated decimals):
a) V2.715; b) V65.2; c) ~/81.1.
3121. The radii of the bases and the generatrix of a truncated
cone are R=23.64 cm±0.01 em; r=17.31 cm±0.01 em; l=
= 10.21 em+ 0.01 em; n = 3.14. Use these data to compute the
total surface of the truncated cone. Evaluate the absolute and
relative errors of the result.
3122. The hypotenuse of a right triangle is 15.4 em+ 0.1 em;
one of the legs is 6.8 em± 0.1 em. To what degree of accuracy
can we determine the second leg and the adjacent acute angle?
Find their values.
3123. Calculate the specific weight of aluminium if an alumin-
ium cylinder of diameter 2 em and altitude 11 em weigh~
93.4 gm. The relative error in measuring the lengths is 0.01,
while the relative error in weighing is 0.001.
3124. Compute the current if the electromotive force is equal
to 221 volts ± 1 volt and the resistance is 809 ohms± 1 ohm.
3125. The period of oscillation of a pendulum of length l is
equal to
T=2n
yT- ,
g
where g is the acceleration of gravity. To what degree of accuracy
do we have to measure the length of the pendulum, whose period
is close to 2 sec, in order to obtain its oscillation period with a
relative error of 0. 5%? How accurate must the numbers n and g
be taken?
3126. It is required to measure, to within I%, the lateral
surface of a truncated cone whose bast- radii are 2 m and 1 m,
and the generatrix is 5 m (approximately). To what degree of
372 Approximate Calculations [Ch. JQ

accuracy do we have to measure the radii and the generatrix and


to how many decimal places do we have to take the number n?
3127. To determine Young's modulus for the bending of a
rod of rectangular cross-section we use the formula
I I'P
E=4 'd 1 bs'
where 1 is the rod length, b and d are the basis and altitude of
the cross-sect ion of the rod, s is the sag, and P the load. To
what degree of accuracy do we have to measure the length l and
the sag s so that the error E should not exceed 5.5%, provided
that the load P is known to 0.1%, and the quantities d and b
are known to an accuracy of 1%, l ::::;:, 50 em, s ~ 2.5 em?

Sec. 2. Interpolation qf Functions


1°. Newton's interpolation formu Ia. Let x0 , x 1 , ••• , xn be the tabular val-
ues of an argument, the difference of which h=flx; (fl.x;=x;+ 1 -x;; i=O,!,
, .. , n-1) is constant (table tnlerval) and y0 , y,, ., y, art> the correspond-
in~ values of the functiOn y Then the valuP of the funct1on y lor an inter-
mt>dlate value of the argumt-nt x is approximately given by Newton's inter-
polation formula
_ + .fl. +q(q-1) fl.Z + +q(q-1) .. (q-n+l) fl.n
Y-Yo q Yo 2! Yo · · • nl Yo. (1)

where q=T
x-x
and fly 0 =y,-y0 , fl 2 y 0 =fly,-fly0 , ... are success1ve finite
dil,erences of tht> furction y. \\hen x=x; (t=O, I, ... , n), the rolynomial
(1) takes on, accorclingly, the tabular values Y; (t=O, 1, . . , n). As partie·
ular cases of 1\:ewtc.n's formula we obta1n: for n =I, linear wterpolalwn;
for n =- 2. qupdratrr rnterpolatwn. To simplify the use of Newton's formula,
it 1s advisable first to st>t up a table of finite clJfferences.
If y=f (x) is a polynomial of degree n, then
fl.ny; = const and fl.n+'y;=-0
and, hence, formula (I) i~ exact
In t11e general case. if f (x) ha~ a continuous derivative f Cn+n (x\ on the
interva I ta. b), which includes the points x0 , x1 , ••• , xn and x, then the error
of formula (l) is

R ()- "-"...,q(q-l) ... (q-i+l)fl.' _


n X - Y- ~ rl Yo-
l=o
=hn+t Q (q-J).' .(Q-M) f'B+Il m (2)
(n+ 1)1 '

wht>re; Is some intermediate value between x; (t =0, I, ... , n) and x. For


pracllcal use, the following approximate formula is more convenient:
Sec. 2) Interpolation of Functions 373

If the number n may be any number, then it is best to choose it so that


the difference ~n+'y 0 :::::: 0 w1thin the hmits of the given accuracy; in other
words. the diflerences !J."y0 should be constant to within the g1ven places of
decimals
Example t. Find sm 26nl5' using the tabular data sin26°=0.43837,
sin 27"-= 0.45399, sin 28° _,_ 0.46947.
Solution. We set up the table

I
I x,
I '" I "'' · I fl21/,

0
I
26°
27~
o 42837
0 45399
I 15621
1548
-14
2 28° 0 46947

26°15'- 26°
Here, h=60', q=
60
,
4 .
Applying formula (I) and usmg the first horizontal line of the table, we
have
_!_
4
(-!--1 )
sin 26°15' =0.43837 + ,}- 0.01562+ ~1 ' • (-0.00014) =0.44229.

Let us evaluate the error R 2 Usmg formula (2) and taking into account
that if y = sm x, then 1y'no 1 ~ l, we will have:

IR J..;;;
1
i(i- )(*- )(:n)'
1
31
2
180
7 1 l -·
=12o'5.1.33':::::4' 10 '
Thus, all the dec1mals of sin 2fl 0 15' are correct.
Using Newlon's formula, it is alsc posstble, from a given intermediate
value of the function y, to flntl the correspo.1d1ng value of the argument x
(inuerse interpolation). To do lhts, first determtne the corresponding value q
by the method of success1ve approxm1at10n, pulttng

q'"' =Y-Yo
!J.y.
and
q 10 (q 1' 1- l ) Ny q1' 1(qu 1-l) .. . (q 111 -n+ I) ~ny
q''+''=q'"'-----. - - · - -
21 lly0 • • • nI !'l.rJ00
(1 =0, I, 2 .... ).

Here, for q we take the common value (to the given accuracy!) of two sue.
cessive approximations q1rn' =·i'rn+•l. Whence x=x.+q·h.
Example 2. Us1ng the table

• I 11=-•inh •
I "'" I !J.>y

2 2 4.457 1.009 0.220


2 4
2 6 I 5.466
6.695 I
I 229
I
approximate the root of the equation swh x = 5.
'874 Approximate Calculations [Ch 10

Solution. Taking Yo= 4.457, we have


(0)_5-4.457 0.543_0 538·
q - 1.009 == 1.009- . •
(I)_ (O)+ qiO) (J-q<Ol) , ~ y 0 =0
2

538 + 0. 538·0.462, 0.220 =


q -q 2 ily 0 • 2 1.009
+
= 0. 538 0. 027 = 0. 565;
q<•>=0.538+ · ; .4 • ~:~~~=0.538+0.027=0.565.
0 565 0 35

We can thus take


x=2.2+0.565·0.2=2.2+0.ll3=2.313.
2°. Lagrange's interpolation formula. In the general case, a polynomial of
degree n, which for x=xi takes on given values Yi (i=O, I, ... , n), is given
by the Lagrange interpolation formula
(x-x1 ) (x-x2 ) . • . (x-xn) -1- (x-x0 ) (x-x 0 ) ••• (x-xn) +
JJ =(x0 -x1) (x0- X 2) . . . (x0-xn)Yo' (x1 -x0) (x1 -x 2) •• . (x 1 -xn)y1 · ' '
+ (x-x 0)(x-x1 ) • •• (x-xk_ 1 )(x-xk+ 1) ••• (x-xn) +
•. . (xk-xo) (xk-x,) ... (xk-xk_,)(xk-xk+,)· .. (xk-xn) Yk ...
+ (X-X0) (X-X1) .. . (X-Xn- 1)
. . . (Xn-Xo) (Xn-XJ) ... (Xn-Xn-1) Yn•

3128. Given a table of the values of x and y:


X
I 2 3 4 5 6

y 15 12 5
I 3 10 9

Set up a table. of the finite differences of the function y.


3129. Set up a table of differences of the function y=xs-
- 5x 2 + x--1 for the values x = 1, 3, 5, 7, 9, 11. Make sure that
all the finite differences of order 3 are equal.
3130*. Utilizing the constancy of fourth-order differences, set
up a table of differences of the function y=x•-10x'+2x1 +3x
for integral values of x lying in the range 1 .s;;;;; x .s;;;;; 10.
3131. Given the table
log 1 = 0.000,
log2=0.301,
log3=0.477,
log 4 = 0.602,
log 5 = 0.699.
Use linear interpolation to compute the numbers: log 1. 7, log 2.5,
log3.1, and log4.6.
Sec. 2) Interpolation of Functions 375

3132. Given the table


sin 10° = 0.1736, sin 13° = 0.2250,
sin 11°=0.1908, sin 14°=0.2419,
sin 12° = 0.2079, sin 15° = 0.2588.
Fill in the table by computing (with Newton's formula, for n = 2)
the values of the sine every half degree.
3133. Form Newton's interpolation polynomial for a function
represented by the table

X 0

y 4 15 1 40 1 85

3134*. Form Newton's interpolation polynomial for a function·


represented by the table

X
I 2
I 4 6 8 10

y
I 3
I 11 27 50 83

Find y for x=5.5. For what x will y=20?


3135. A function is given by the table

X -2 I I I 2 4

y 25 -8 /-151 -23

Form Lagrange's interpolation polynomial and find the value of


y for x=O.
3136. Experiment has yielded the contraction of a spring (x nun)
as a function of the load (P kg) carried by the spring:

X I 5 10 15 20 25 30 35 40

p 1 49 105 172 2.53 352 473 619 793

Find the load that yields a contraction of the spring by 14 mm~


376 Approximate Calculations [Ch. 10

3137. Given a table of the quantities x and y

X 0 3 4 l 5

y -3 25 129 1 381

('...ompute the values of y for x=0.5 and for x=2: a) by means


of linear interpolation; b) by Lagrange's formula.

Sec. 3. Computing the Real Roots of Equations


1°. Establishing initial approximations of roots. The approximation of the
roots of a given equation
f (x) =0 ( !)

consists of two stages: 1) separating the roots, that is, establishing the inter-
vals (as small as possible) within which lies one and only one root of equa-
tion (!); 2) computing the roots to a given degree of accuracy
If a function f (~) is defined and continuous on an interval (a, b) and
f (a)· f (b) < 0, then on [a, b) there is at least one root ~ of rq uaticn ( 1).
This root will definitely be the only one if f' (x) > 0 or f' (x) < 0 when
a<x< b.
In approximating the root ~ it is advisable to use millimetre paper and
construct a graph of the function y = f (x). The abscissas of the roints of
intersection of the graph with the x-axis are the roo!s of the equation f (x) =0.
It is sometimes convenient to replace the given equation with an equivalent
equation <p (x) o-: 'IP (.1.). Then the roots of the equatiOn are found as the abs-
cis~as of roints of intersection of the graohs 1/=ffl (x) and y=¢ (x).
2°. The rule of proportionate parts (chord method). If on an interval [a, b)
there i~ a unique root ~ of the f'qL•ation f (J.) = 0, where the function f (x)
is continuous on [a, b), then by repla< ing the curve y = f (x) by a chord
passing through the roints [a, f (a)] and [b, f (b)]. we obtain the first
approximation of the root
f (a)
C1 =a-f(b)-f(a)(b-a). (2)

To obtain a second approximation c2 , we apply formula (2) to that one of


the intervals [a, c,] or [c,, b) at the ends of which the function f (x) has
values of oppcslte sign. The succeeding approximations are constructed in the
same manner. The sequence of numbers en (n = 1, 2, ..• ) converges to the
root 6. that is,
lim cn=s.
n-+<~>

Generally sreaking, we should continue to calculate the approximations c1 ,


c2 ,• • • , until the decimals retained in the ar.swer cease to change (in accord
with the specified degree of accuracy!); for intermediate calculations, take
one or two reserve decimals Thi~ is a general remark.
If the function f (.t) has a nonzero conttnuous derivative f' (x) on the
interval [a, b), then to evaluate the absolute error o[ the approximate root
Sec. 3] Computing the Real Roots of Equations 377

en, we can make use of the formula


lt-c /o;;;;;lf(c,.)l
'o n J.l. '
where !l = min If' (x) I·
a<x<b
3°, Newton's method (method of tangents). If f' (x) ::/= 0 and f" (x) ::/: 0 fol'"
a.,;;;;; X.,;;;;; b, where f (a) f (b) < 0, f (a) f' (a) > 0, then the SUCCessive approxima·
tions x,.(n=O, I, 2, ... )to the root~ of an equation {(x)=O are computed
from the formulas
(3)

Under the given assumptions, the sequence Xn (n =I, 2, ... ) is mono-


ionic and
n-+"'
To eva I uate the errors we can use the formula

where f.l. = min If' (x) I·


a<x<b
For practtcal purposes it is more convenient to use the simpler formulas
x0 =a, x,.=x,_,-af(x,._,) \n=l, 2, ... ), (3')
where a= f' ~a), which yield the same accuracy as formulas (3).
If f (b) f" (b)> 0, then in formula·; (3) and (3') we should put x0 =b.
4°. Iterative method. Let the given equation be reduced to the form
x=,w. w
where 1q/ (x) 1.,;;;;; r < 1 (r is constant) for a.,;;;;; x.,;;;;; b. Proceeding from the ini-
tial value x0 , whtch belongs •to the interval (a, bJ, we butld a sequence of
numbers x1 , x2 , •.• according to the followtng law:
x,=,(x 0 ), x2 =,(X 1), ••• , x,.=,(x 11 _ 1), (5)
If ao;;;;;x,.o;;;;;b (n=l, 2, ... ),then the limit
~=lim x 11
n-->"'
is the only root of equation (4) on the interval (a, b]; that is, x,. are succes·
sive approximations to the root ~.
The evaluation of the absolute error of the nth approximation to x,. is
given by the formula

Therefore, if x 11 and x 11 + 1 coinctde to within e, then the limiting absolute


8
error for x,. will be - •
1- r
In order to transform equation f (x) = 0 to (4), we replace the latter with
an equivalent equation
x=x-'}..f (x),
where the number 'A ¢; 0 is chosen so that the function d~X [x-'J..f (x)] = 1-'A.f' (x)
.378 Approximate Calculations [Ch. 10

should be small in absolute value in the neighbourhood of the point x0 (for


example, we can put 1-A.f' (x0)=0).
Example 1. Reduce the equation 2x-ln x-4 =0 to the form (4) for the
initial approximation to the root x0 = 2.5.
Solution. Here, f(x)=2x-lnx-4; f'(x)=2-J... We write the equiva·
X
lent equation x=x-A. (2x-Jn x-4) and take 0.5 as one of the suitable
values of A.; this number Is close to the root of the equation
,jt-i..(2-J..)j_ =0, that is, close to _!_ :::::::0.6.
, X X-2.5 1 ·6
The initial equation is reduced to the form
x=x-0.5 (2x-ln x-4)
or
I
x=2+
2 lnx.
Example 2. Compute, to two decimal places, the root ~ of the preceeding
equation that lies between 2 and 3.
Computing the root by th~ iterative method. We make use of the result
of Example I, putting x0 =2.5. We carry out the calculations using formulas
(5) with one reserve decimal.
I
x1 =2+ ln2.5:::::::2.458,
2
I
x,=2+
2 1n2.458:::::::2.450,
I
x1 = 2 + In 2.450:::::::2.448,
2
I
X4 =2+2" In 2.448:::::::2.448.

Ancl so ~::::::: 2 45 (we can stop here since the third decimal place has
llecome fixed)
Let us nqw ,evaluate the error. Here,

1Jl(X)=2+{lnx and IP'(x)=.Jx.

Considering that all approximations to Xn lie In the interval [2.4, 2.5], we


get
r =max liP' (x) I= 2 .~. 4 =0.21.
Hence, the limiting absolute error in the approximation to x1 Is, by virtue
of the remark made above,

ll= 1 ~~~~~ 0.0012:::::::0.001.

Thus, the exact root ~ of the equation lies within the limits
2 447 < ~ < 2.449;
we can take ~::::::: 2.45, and all the decimals of this approximate number will
be correct in the narrow sense.
Sec. 8) Computing the Real Roots of Equations 379'

Calculating the root by Newton's method. Here,

f (x) =2x-ln x-4, f' (x) = 2 _ _!_,


X
r <x)=..;..
X

On the interval 2.,;;;;; X...;;;; 3 we have: f' (x) > 0 and r


(x) > 0; f (2) f (3) < 0;
f (3) r (3) >
0. Hence, the conditions of 3° for Xo = 3 are fulfilled.
We take
a= ( 2- I ) -• =0.6.
3
We carry out the calculations using formulas (3') with two reserve decimals:
X 1 =3-0.6 (2·3-Jn 3-4) =2 4592;
x 1 =2.4592-0.6 (2·2 4592-ln 2 4592 -4) = 2 4481;
x, =2.4481-0.6 (2·2.4481-ln 2.4481-4) =2.4477;
x4 = 2.4477-0.6 (2·2 4477 -In 2.4477 -4) =2 4475.
At this stage we stop the calculations, since _the third decimal place
does not change any more. The answer is: the root ,;=2.45. We omit the
evaluation of the error.
5°. The case of a system of two equations. Let it be required to calcu-
late the real roots of a system of two equations in two unknowns (to a gtven
degree of accuracy):
f (x, y)=O,
{ rp (x, y) =0,
and let there be an initial approximation to one of the solutions (;, TJ) of
this system x = x 0 , y =Yo·
This initial approximation may be obtained, for example, graphically,
by plotting (in the same Cartesian coordinate system) the curves f (x, y) =0
and rp (x, y) ,=0 and by determining the coordinates of the points of inter-
section of these curves.
a) Newton's method. Let us suppose that the functional determinant
a
I= (f. rp)
a
(x, y)
does not vanish near the Initial approximation x =X0, y =Yo· Then by New-
ton's method the first approximate solution to the system (6) has the form
x1 =x 0 +a0 , y 1 =Yo+~o· where a 0 , ~ 0 are the solution of the system of two.
linear equations
I (Xo• Yo)+aof~(Xo, Yo)+~of~(Xo, Yo)=O,
{
rp (Xo, Yo)+ Cloq>: (Xo, Yo)+ ~orp~ (Xo, Yo)= 0.
The second approximation is obtained in the very same way:
X 1 =x.+a., Yz=Y.+~t•
where a 1, ~. are the solution of the system of linear equations

f(x 1 , y 1 )+a 1 {~(x 1 , Y 1 )+~.f~.(x 1 , y 1)=0,


{ rp (x , y ) + a rpx (x., y ) + ~ rpy (xp y ) =0.
1 1 1 1 1 1

Similarly we obtain the third and succeeding approximations.


380 Approximate Calculat cons [Ch J(J

b) Iterative method. We can also apply the Iterative method to solving


the system ol equations (6), by transforming this ~ystem to an equivalent one
x=F (x, y), ( )
{ y=ll> (x, y) 7
and assuming that
IF~(x,y>j+I<D~(x, y)IEO;;r<l; IF~(x, Y>I+I<D;,(x, Y>lo;;;;;r< 1 (8)
in some two-dimensional neighbourhood U of the initial approximation (x0 , y 0 ),
which neighbourhood also contains the exact solut1on (;, fJ) of the system.
The sequence of approximations (xn, y 11 ) (n = I, 2, ... ). wh1ch converges
to the solutton ol the system (7) or, what 1s the same thing, to the solution
~I (6), is constructed according to the following law:

X1 = F (Xo, Yo), Ys = !l> (Xo, 1/o),


x 2 =f(x 1 , y1), y2 =<l>(x 1 , y1 ),
Xa= F (Xz, Yz), Ya = 11> (Xz, Yz),

If all (Xn, y11 ) belong to U. then Jim Xn = ;. lim y11 =fJ.


n-+oo n ..... .:x~

The following technique 1s advtsed for transformtng the system of equa·


lions (6) to (7) with cond1tion (8) observed. We consider the system of
equations
a/ (x. y) + ~c:p (x, y) =0,
{ y/ (x, y) + bc:p (x, y) = 0,

which is equivalent to (6) provided that I~: ~ J =1= 0. Rewrite it in the form
x=x+af (x, y) +
~c:p (x, y) ~ F (x, y),
Y=Y+Yf(x, y)+l~lp(X, y)-c-=;QJ(x, y).
Choose the parameters a, ~. y, tl such that the partial deriv11tives of the
fum·ttons F (~. y) -:nd !l> (x. y) will be equal or close to zero tn the tnitial
arprOXIIlllliiO.Il; Ill Olher Words, We find U, ~. y, b as apprOXImate SOIUltOilS
o the system of equattons
1 +at: (X0 , Yo) + ~c:p: (Xo, Yo) = 0,
af~ (Xo. Y0 ) + ~c:p;, (Xo, Y0 ) = 0,
{ Yf~ (xo. Yo)+ tlc:p: (X0, 1/0) = 0,
1 + vt;, (xo. Yol + tlc:pl/ (Xo, 1/ol =0.
Condition (8) will he ohserved in such a choice of parameters a, ~. y, b
on the 11ssumption that the p11r!tal derivatives of the functwns f (x, y) and
cp (.\, y) do not vary very rapidly in the neighbourhood of the initial approx·
imation (x 0 , y 0 ).
Example 3. Reduce to the form (7) the sy~tem of equations
X
2
+y 2 - l =0,
3
{ x -lt=0
,given the initial approximation to the root x0 =0.8, y0 =0.55.
Sec. 3] Computing the Real Roots of Equations 381

Solution. Here, f(x, y)=x2 +y 2 -l, IJ'(X, y)=x'-y; f~(x0 , y8 )=1.6,


f, (xo, Yo)= 1.1; 'P~ (xo, Yo)= 1.92, 'P~ (Xo, Yo)=- l.
Write down the system (that is equivalent to the initial one)
a(x +y -l)+~(x -y)=O,
2 2

{ y(x 2 +y 2 -l)+t'l(x3 -y)=0


3
(Ia,y, 6~~=~:o)
in the form
x=x+a (x 2 + y 2 - l ) + ~ (x'-y),
Y=Y+Y (x 2 +Y2 -1)+6 (x3 -y).
For suitable numerical values of a, ~. y and t'l choose the solution of the
system of equations
1+1.6a+l.92~=0,
l.Ia-~=0,
{ l.6y+ 1.92~ =0,
I+I.Iy-il=O;
I. e., we put a ~-0.3, ~ ~-0.3, y ~-0.5, t'l ~ 0.4.
Then the system of equations
x=x-0.3 (x 2 + y2 -l)-0.3 (x1 -y),
{ y=y-0.5 (x 2 + y 2 -l) +0 4 (x 3 -y),
which is equivalent to the initial system, has the form (7); and in a suffi.
ciently small neighbourhood of the pomt (x 0 , y 0 ) condition (8) will be fulfilled.

Isolate the real roots of the equations by trial and error, and
by means of the rule of proportional parts compute them to two
decimal places.
3138. x•- x ·I 1 = 0.
3139. x• + 0 5x-1.55 = 0.
3140. x'-4x --1 =0.
Proceeding from the graphically found initial approximations,
use Newton's method to compute the real roots of the equations
to two decimal places:
3141. X 1 -2x-5=0. 3143. 2x=4x.
I
3142. 2x-lnx-4=0. 3144. logx=-.
X

Utilizing the graphically found initial approximations, use the


iterative method to compute the real roots of the equations to
two decimal places:
3145. x• -5x -1 0.1 = 0. 3147. x' -x-2 = 0.
3146. 4X= COSX.
Find graphically the initial approximations and compute the
real roots of the equations and systems to two decimals:
3148. x'-3x 1 = 0.+ 3151. x-ln x--14 = 0.
3149. x•-2x 2 +3x-·5=0. 3152. x•+3x-0.5=0.
3150. x•+x1 -2x-2=0. 3153. 4x-7sinx=0.
382 Approximate Calculations [Ch. 10

3154. XX +2x-6 = 0. 3157. { xa+y-4=0,


3155. ex+e-ax-4=0. y-logx-1 =0.
3156. { x•+y•-t=O,
x'-y=O.
3158. Compute to three decimals the smallest positive root of
the equation tan x = x.
3159. Compute the roots of the equation x-tanh X= 1 to four
decimal places.

Sec. 4. Numerical Integration of Functions


1°. Trapezoidal formula. For the approximate evaluation of the integral
b

~ f (x) dx
a
[/ (x) is a function continuous on [a, b]) we divide the interval of integration
[a, b] into n equal parts and choose the interval of calculations h = b-a .
I!
Let xi=x0 +ih (x 0 =a, Xn=b, i=O, l, 2, ... , n) be the abscissas of the par-
tition points, and let Yi = f (xi) be the corresponding values of the integrand
y = f (x). Then the trapezoidal formula yields
b

Sf(x) dx~h (Yo~Yn +Y1+Ya+ ... +Yn-1)


a
with an absolute error of
hi
Rn ~ f2 (b-a)·M 2 ,
where M 2 =~axJr(x)j when a~x~b.
To attain the specified accuracy e when evaluating the integral, the in-
terval h is found from the inequality
h2-- 12e 2
._(b-a) M 1 t l

That is, h must be of the order of Vi". The value of h obtained is rounded
off to the smaller value so that
b-a
-h-=n

should be an integer; this is what gives us the number of partitions n.


Having established h and n from (l), we compute the integral by taking the
values of the integrand with one or two reserve decimal places.
2°. Simpson's formula (parabolic formula). If n is an even number, then
in the notation of 1° Simpson's formula
b

5f (x) dx ~~ HYo+ Yn)+4 (y, + Ya + •.. +Yn-1) +


a +2(Ya+y,+ .. •+!ln-aH (3)
Sec. 4) Numerical Integration of FunctiOfls 383

holds with an absolute error of


h'
Rn.;;;; lBO (b-a) M,, (4)

I/ I
where M 4 =max 1v (x) when a.;;;;x.;;;;b.
To ensure the specified accuracy e when evaluating the Integral, the
interval of calculations h is determined from the inequality
h'
i8o (b -a) M,,..;;;; B. (5)

That is, the interval h is of the order ve. The number h is rounded off
to the smaller value so that n = b-a is an even integer.
n
Remark. Since, generally speaking, it is difficult to determine the inter-
val h and the number n associated with it from the inequalities (2) and (5),
in practical work h is determined in the form of a rough estimate. Then,
after the result is ohtained, the number n is doubled; that is, h is halved.
If the new result coincides with the earlier one to the number of decimal
places that we retain, then the calculations are stopped, otherwise the pro-
cedure is repeated, etc.
For an approximate calculation of the absolute error R of Simpson's
quadrature formula (3), use can also be made of the Runge princtple, accord-
ing to which

where 11 and l: are the result~ of calculations from formula (3) with interval
h and H =2h, respectively.

3160. Under the action of a variable force F directed along


the x-axis, a material point is made to move along the x-axis
from x = 0 to x = 4. Approximate the work A of a force F if a
table is given of the values of its modulus F:

_x-+l_o.o~lo_.s~l_t.o~l1_.s~l_2.0~12_.s~l_3.o-+1_3_.5~4.0
I I
F 11.50 0. 751 0.50 0.7511.5012.751 4.50 I 6. 75 110.00
L------------------------------------------
Carry out the calculations by the trapezoidal formula and by
the Simpson formula.
I

3161. Approximate ~ (3x1 -4x) dx by the trapezoidal formula


0
putting n= 10. Evaluate this integral exactly and find the abso-
lute and relative errors of the result. Establish the upper limit Ll
of absolute error in calculating for n= 10, utilizing the error
formula given in the text.
384 Approximate Calculations (Ch. 10

3162. Using the Simpson formula, calculate 5;~xi to four


0
decimal places, taking n= 10. Establish the upper limit ll of abso-
lute error, using the error formula given in the text.
Calculate the followmg definite integrals to two decimals:
I 2

3163. 5 +x . 1
dx
3168. 5sl:x dx.
0 0
1 :rt
dx
3164.
5 +x
0
1 2
'
3169.
5sin x dx
0
X •

3165.
I

5 :x'' 1
3170. 5co:x dx.
0 I
n
I
2
3166. ~ xlogxdx.
I
3171. 5cosx d 1+x X.
I 0
I
3167. 5lo~x dY. 3172. ~ e-x• dx.
I 0

3173. Evaluate to two decimal places the improper integral

x=+·
tl>

5 !xx 1 2 by applying the substitution Verif_r the calculations

by applying Simps~~·s formula to the integral 5 ~x•,


1
where b

is chosen ;;o that 5 1


~\. < ~ · 10-=.
b
3174. A plane figure bounded by a half-wave of the sine curve
y = sin x and the x-axis is in rotation about the x-axis. Using the
Simpson formula, calculate the volume ot the solid of rotation
to two decimal places.
3175*. Using Simpson's formula, calculate to two decimal
x• y•
places the length ot an arc of the ellipse 1 (O.fi 222 ).= 1 situated +
in the first quadrant.

Sec. 5. Numerical Integration of Ordinary Differential Equations


1°. A method of successive approximation (Picard's method). Let there
be given a first-order differenhal equation
y'=f(x, y) 1 1)

subject to the initial condition Y=Yo when X=x 0.


Sec. 5) Numerical Integration of Ordinary Differential Equations 385

The solution y (x) of (1), wh1ch satisfies the given initial condition, can,
generally speaking, be represented in the form
y (x) = lim Yi (x) (2)
I_,. "'

where the successHJe approximations Yi (x) are determined from the formulas
Yo (x) =y •.
X

yj(x) =Yo+~ f (x, Yt-dX)) dx


Xo
(l=O, 1, 2, ... ).

If the right side f (x, y) is defined and continuous in the neighboe~rhood


R{lx-x.lo;;;;a, IY-Yolo;;;;b~

and satisfies, in this neighbourhood, the Lipschitz conditiOn

If (x, Y1)-f (x, Y2 ) I,.;;;; L I y,-y.l

(l. is constant), then the process of successive approximation (2) definitely


ronverges in the interval
lx-x 0 jo;;;;h,
where h=mJn(a. ~)and M=m;xlf(x, y)l. And the error here is

I x-x. In+•
Rn=ly(x)-Yn(X)I~MLn (n+l)l •
If
lx-x 0 lo;;;;h.

The method of successive approximation (Picard's method) is also appli-


cable, with slight modifications, to normal systems of differential equations.
Differential equations of higher orders may be written in the form of systems
of differential equations.
2°. The Runge-Kutta method. Let it be required, on a given 1nterval
x0 o;;;;xo;;;;X, to find the solution y(x) of (I) to a specified degree of accuracy e.
To do this, we choose the interval of calculations h=X-xo by dividing
n
the interval (x 0 , X] into n equal parts so that h 4 <e. The partition points
x 1 are determined from the formula
Xt=x 0 +ih (1 =0, 1, 2, ... , n).

By the Runge-Kutta method, the corresponding values Yt=Y (x1) of the desired
function are successively computed from the formulas
Yi+t =Y;+~Yi·
~y.-.!.
1-6 ( k(i)
I
+ 2k(t)
I
+ 2k(t)
I
+ k(l))
4 •

13-tooo
38~6__________________A~p~p~ro=x=im~a=te~C~a=lc~u=la=t=io=n=s~------------~[C~'h~·~JO

where

(3)

To check the correct choice of the interval h it is advisable to verify


the quantity

The fraction e should amount to a few hundredths, otherwise h has to be


reduced.
The Runge-Kutta method is accurate to the order of h'. A rough estimate
of the error of the Runge-Kutta method on the given Interval (x 0 , X] may
be obtained by proceeding from the Runge pnnciple:

R =I Y.,.--Ym
15
I
'
where n=2m, y 214 and Ym
are the results of calculations using the scheme (3)
with interval h and interval 2h.
The Runge-Kutta method is also applicable for solving systems of dtffe-
rential equations
y'=f(x, y, z), z'=qJ(x, y, z) (4)
with given initial conditions y=y 0 , z=z 0 when X=X 0 .
3°. Milne's method. To solve (1) by the Mtlne method, subject to the
initial conditions Y=Yo when x=x 0 , we in some way find the successtve
values
Y1 =y (xl), Yz=Y (x.), Ya =Y (x,)
of the desired function y (x) [for instance, one can expand the solution y (x)
In a series (Ch. IX, Sec. 17) or find these values by the method of successive
approximation, or by using the Runge-Kutta method, and so forth]. The ap-
proximations iiiand lf7 for the following values of y 1 (i =4, 5, ... , n) are
successively found from the formulas

Yi=Yt-4+ ~h (2ft_,-ft-•+2fi_l), }
= h- ~
Yt=Yi-z+ 3 (fi +4fi-l +It-a>•
Where ft=f(xi, Yi) and h=f(x1, Yj). To check we calculate the quantity
1
1- =I •
8t= 29 Yi-Yi (6)
Sec . .5] Numerical Integration of Ordinary Differential Equations 387

If E; does not exceed the unit of the last decimal w-m


retained in the
answer for y (x), then for Yi we take Yi and calculate the next value Yi+t>
repeating the precess. But 1f E; > w-m,
then one has to start from the be-
gmning and reduce the interval of calculations. The magnitude of the initial
interval is determined approximately from the inequality h 4 < w-m.
For the case of a solution of the system (4), the Milne formulas are
written separately for the functions y (x) and z (x). The order of calculations
remains the same.
Example I. Given a differential equation y' =y-x with the initial con-
dition y (0)= 1.5. Calculate to two decimal places the value of the solution
of this equation when the an;ument is x= 1.5. Carry out the calculations
by a combined Runge-Kutta and Milne method.
Solution. We choose the initial mterval It from the cond1tion h4 < 0.01.
To avoid involved writing, let us take h=0.25. Then the entire interval of
integration from x=O to X= 1.5 is divided into six equal parts of length
0.25 by means of points x; (i=O, I, 2, 3, 4, 5, 6); we denote by Yi and y;
the corresponding values of the solution y and the derivattve y'.
We calculate the first three values of y (not counting the initial one) by
the Runge-Kutta method [from formulas (3)]; the remaining three values
- y 4 , y5 , y 6 - we calculate by the Milne method [from formulas (5)]
The value of Yo will obviously be the answer to the problem.
We carry out the calculations with two reserve decimals according to a
definite scheme consisting of two sequential Tables I and 2. At the end of
Table 2 we obtain the answer.
Calculating the value y~" Here, f(x, y)=-X+!J, X0 =0, Y0 =1.5

h = 0.25. l'ly 0 = ~ (k\ 0 ) + 2k~o) + 2k~o) + k~0 )) =


= ! (0.3750 + 2·0.3906 +2·0.3926 + 0.4106) =0.3920;
k\ 0 ) = f (x 0 , y 0) h = (- 0 + 1.5000)0.25 ~ 0.3730;
I k(o) )
k~0 )=f ( Xo-t-]-, !/d·+ h=(-0.125-t-1.5000-t-0.1875)0.25=0.3906;
k(O)
k~0 )= f ( x 0 -f- ~, Yo++) h=(-0 125+ 1.5000+0.1953) 0.25=0.3926;

k~o) = f (x 0 -t- ll, Yo+ k~ 0 )) h = (- 0.25 + 1.5000 + 0.3926) 0.25 =0.4106;

y 1 =Yo+ tly 0 = 1.5000+0.3920= 1.8920 (the first three decimals In thts


approximate number are guaranteed).
Let us check:

8= Iklk~o)0 >-k~o)
-k~o) I= I 0.3906-0.39261
I 0.3750-0.39061-
20
0 13
156= · ·

By this criterion, the interval ll that we chose was rather rough.


Similarly we calculate the values y 2 and y1 • The results are tabulated
in Table 1.

13*
388 Approxtmate Calculations [Ch. 10

Table /. Calculating y 1 , y,, Ya by the Runge-Kutta Method.


f(x, y)=-x+y; h=0.25

'
t(xi+4,
Value of i X[ Yi Yt"'= k(l) k(l)
=f (Xj, Yi)
I
k\1)) '
y,-1-2

0 0 I.5000 1.5000 0.3750 I.5625 0.3906


I 0.25 I.8920 I.6420 0.4I05 1.7223 0.4306
2 0.50 2.3243 I .8243 0.456I I.9273 0.4818
3 0 75 2.8084 2.0584 0.5I46 2.I907 0.5477

Value of i
t( -"i+i. k(/) f (xi+h. k(i) !lyi
k~l)) I Yt -1- k~1 l) 4 Yi+t
Yi-1-y

0 I .5703 0.3926 I.6426 0.4I06 0.3920 I . 89-20


I I. 7323 0.433I I.825I 0.4562 0.4323 2.3243
2 I.9402 0.4850 2.0593 0.5I48 0.484I 2.8084
3 2.2073 0.55I8 2.3602 0.5900 0.5506 3.3590
·-
Calculating the value of y 4 • We have: f(x, y)=-x-j-y, h=0.25, x4 =1;
· Yo= I.5000, y 1 = 1 .8920, y 1 = 2.3243, Ya = 2.8084;
y~ = 1.5000, u;
= 1. 6420, u;
= 1.8243, = 2.0584.u;
Applying formulas (5), we find
-
y,=yo + 4h
3 (2y,-y&' + 2y,)' =
= 1.5000 + 4 ' 0325 (2·1.6420-1.8243 +2·2.0584) =3.3588;
!l, = f (x,, i.> =- I + 3.3588 = 2.3588;
=
u,=u. + 3h <Y_, +4u,' + u.>' = 2.3243+ 0.25
4
3 (2.3588+4·2.0584+ 1.8243)=3.3590;

e.,-
_IU:-Y:I
29
13.3588-3.3590!_0.0002
29 -29
7 lO-•< 1 0001·
:::::: ' 2' · '

hence, there is no need to reconsider the mterval of calculations.


Sec. 5) Numerical Integration of OrdiMry Differential Equations 389

We obtain y 4 = ~ = 3.3590 (in this approximate number the first three


decimals are guaranteed).
Similarly we calculate the values of y1 and y0 • The results are given in
Table 2.
Thus, we finally have

y(l.5)=4.74.

4°. Adams' method. To solve (1) by the Adams method on the basis of
the initial data y (x 0 ) =Yo we in some way find the following three values
of the desired function y (x):

(these three values may bli' obtained, for instance, by expanding y (x) in a
power sertes (Ch IX, Sec. 16), or they may be found by the method of sue·
cessive approximation (\ 0 ), or by applying the Runge-Kutta method (2°)
and so forth).
W1th the help of the numbers x 0 , x 1 , X 2 , X 1 and y0 • y 1 , y2 , Ya we calcu-
late q0 , q 1 , q 2 , q1 , where

Q0 = hy~ = hf (X0 , Y0), q1 = hy; = hf (x 1, Y1),


q2=hy~=hf(x2, y.). q,=hu:=hf(x•. y,).

We tlwn form a diagonal table of the finite differences of q:

lly= 1/=f (X, y)


q=y'h ll' q=
llq=q,.+,-q,. =llq,.+,-llq,.
ll 'q ~
X II ;;::;lin+ ,-tJn =fl 1 Qr, + 1 -
-ll'q,
--
I I
Xo !lo t>yo
I f (xo. Yo)
I Qo
I 6-qo
I 6_2qo
I t.•q,,

x. YtI I 6-y.
I f (x., Y.) I !It I ~q.
I t.•q. I ,\'q,

x.l Y21 tJ.y.


I f (x •. Y2) I q2 I 6-q.
I 6-'q.
I tJ,•q.

Xa It/a I tJ.y.
r
f (x •• Yal I q. I 6-q,
I tJ.'q.
I
x.j u.j !!.y.
I I (x.. Y.) I q. I !!.q.
I I
x.i y,J !!.y.
I f (x,, Ys) I q, I I I
x, IYs I I I I I- I
390 Approximate Calculations [Ch. /0)

The Adam5 method consists in continuing the diagonal table of differen-


ces with the aid of the Adams formula

l A
A
oYn=qn+2 oqn-l + 152 '-' Qn-z+ B3 '-' qn-1·
A2 AI
(7)

Thus, utilizing the numbers q., 11q 2 , fl2q 1 , 11'q0 situated diagonally in
lhe difference table, we calculate, by means of formula (7) and puttingn=·3
in 1t, fly 1 =qa+{ flq 2 +~ fl 2q1 + ~ fl'q 0. After finding fly 1, we calculate
y,=Ya+ fly 1 • And when we know x 4 and y,, we calculate q4 =hf (x 4 , y 4 ),
introduce y,, fly 1 and q4 into the difference table and then fill into it the
finite differences flq 1, fl 2q2 , fl'q~o which are situated (together with q4 ) along
a new diagonal parallel to the f1rst one.
Then, utilizing the numbers of the new diagonal, we use formula (8)
(putting n=4 in it) to calculate fly 4 , y5 and q5 and obtain the next diagonal:
q1 , flq 4 , 11'q1 , t11q2 • Using this diagonal we calculate the value of y8 of the
desired solution y (x), and so forth.
The Adams formula (7) for calculating 11y proceeds from the assumption
that the third finite differences /l 3 q are constant. Accordingly, the quanti tv h
of the initial interval of calculations is determined from the inequality
h' < w-m [if we wish to obtain the value of y (x) to an accuracy of
to-m].
In this sense the Adams formula (7) is equivalent to the formulas of
Milne (5) and Runge-Kutta (3).
Evaluation of the error for the Adams method is complicated and for
practical purposes is useless, since in the general case it yields results with
considerable excess. In actual practice, we follow the course of the th1rd
finite d1fferences, choosing the interval h so small that the adjacent diffe-
rences /1 3 qi and fl'qi+l differ by not more than one or two units of the given
decimal place (not counting reserve desimals).
To increase the accuracy of the result, Adams' formula may l.Je extended
by terms containing fourth and higher differences of q, in which case there
is an increase in the number of first values of the function y that are needed
when we first fill in the table. We shall not here give the Adams formula
for higher aceuracy.
Example 2. Using the combined Runge-Kutta and Adams method, calcu-
late to two decimal places (when x= 1.5) the value of the solution of the
differential equation y'=y-x with the 1nitial condition y(0)=1.5 (see
Example 1).
Solution. We use the values y1 , y 2 , y1 that we obtamed in the solution
of Example I. Their calculation is given in Table l.
We calculate the suhsequent values y,, y 5 , y 8 by the Adams method (see
Tables 3 and 4).
The answer to the problem is y 8 =4.74.
For solving system (4), the Adams formula (7) and the calculation scheme
shown in Table 3 are applied separately for both functions y (x) and z (x).

Find three successive approximations to the solutions of the


differential equations and systems indicated below.
+
3176. y' = X 1 y 1 ; y (0) = 0.
3177. y'=x+y+z, z'=y-z; y(O)=l, z(0)=-2.
3178. y" = - y; y (0) = 0, y' (0) = 1.
Table 2. Calculating Y 4 , y 5 , Y 6 by the Milne Method.
f (x, y) = - x + h; h = 0.25. (Italicised f1gures are mput data)
'
' I'
II
Reconsider interval
\' alue
ol I x, y,
'
Y,=f (x,. y,)
-
y,
-'
Y,=f
-
(X,, y,)
=
y, e, y,
'
v,=f (X,, y,)
of calcula twns,
followmg IndicatiOnS
of formula (6).

o I o 11. 5000
1 1.
5000
lllllllllllllllll/llllllllllll/ll!lllllllll/illlll/11/llllllll/l/1111/llllllllllllllllll/11/111/l/1/l/lllll///lll/ll/ll/llllllllllllllllll/1
2 8920 6420
I Jo. SI1. 1 1. /lllllllllllllllllllllllllllllll/111111/11111/ll/1/llllilll/i/llll/111111//llllllllllll/!//ll/llllllll/1/llllllllllllllll[lllllllllllllllllll
2 2 3243 8243
Jo.so / - 1 1. /I/JIJJJJIIIJIIJJ/////JJJ/JI/JIIIIIJ\J/JJJII/I//IIII\111/IJJIIJIIJIJI\/I\11\\I\I\IIIIIIIII/IIIJIIIIIIIIIIJIIIJ/III///I//////I/JIIIJII/JjJJI

75 2 8084 2 0584
_ a 1°- / ' 1 ' 11/JIIII/1//1//JIII/1////[////J//J/[ //!I///J/II//J///////I//III/I///////I//I////IIJI///////I////I//////11111/III/////II//IIIIII//I/II/!IIJ
4
ll.OO 111111111111111//11/1/llllllllll/111 3.3588 I 2.3588 I 3.3590 I ~7.Jo-• 13.3590 I 2.3590 /Do not reconsider

5
/1. 25 /ll/llllllllllllllllll//llllllllllll 3 · 9947 1 2. 7447 I 3.9950 I ~ to- ,3.9950 I 5
2. 7450 I Do not reconsi_der :

I 4"7406 I~ w- ~~~~
6 11. 50 IIIIIIIIIIIIIIIIIIJIIIIIJ/1/IJ//111 I 4 7402 I 3 2402 5 4
" " 1. • I Do not reconsider I

Answer: y(l.5)=4.74
Table 3. Basic Table for Calculating y,,
.)11 , .)11 by the Adams Method.
f (x, y) = - x y; h = 0.25 +
(Italicised figures are input data)

·~

0 6.y, '
y{= Qj= y;h 6.q, 6.'q, 6.'q,
"' t,
Yl
2 =f (Xt, yj)

>"

oI o I 1.5000 111111111111111111
1.sooo 1 0.3750
I o 0355 1 o.OIOI 1 o 0028

110.251 1.8920
111111111111111111
1.6420 1 0.4105
I 0.0456 1 0.0129 1 o 0037

210 501 2.3243 111111111111111111


1.8243 1 0.4561
I 0.0585 1 0.0166 1 o.oo47

310.751 2.8084 1 0.5504 1 2.0584 1 0.5146 1 o.o751 1 0.0213 1

411 ool 3.3588 1 0.6356 1 2.3588 1 o.5897 1 0.0964


I I
I

511 251 3.9944 I 0.7450


I 2.7444
I 0.6861
I I I
611.50114 .7394f
--
1
I I I I I
An5wer: 4.7<1

Table 4 Auxiliary Table for Calculating by the Adams Method


Ill
11 y,=q;+2 5u 3
qi-1+ 12u qi-2+8 I'.'Q;-a

~ 6.'q,_,
Value of t q, I I
2 6.q,_, 1Z
... 6.'Ql-. 6.y,

3
I 0.5146
I 0.0293
I 0.0054
I 0.0011
I 0.5504
4
I 0 5897
I 0.0376
I 0.0069
I 0.0014
I 0.6356
5
I 0.6861
I 0.0482
I 0.0089
I 0.0018
I 0. 7450
!Sec. 6) Approximating Fourier Coefficients 393

Putting the interval h = 0.2, use the Runge-Kutta method to


calculate approximately the solutions of the given differential
equations and systems for the indicated intervals:
3179. y'=y-x; y(0)=1.5 (O:s;;;;;x:s;;;;;1).
3180. y' = }!_- y•;
X
y ( 1) = 1 ( 1 :s;;;;; X ~ 2).
3181. y'=z+l. z'=y-x, y(O)=l, z(0)=1 (O~x:s;;;;;1).
Applying a combined Runge-Kutta and Milne method or
Runge-Kutta and Adams method, calculate to two decimal places
the solutions to the differential equations and systems indicated
below for the indicated values of the argument;
3182. y'=x+y; y=l when x=O. Compute y when x=0.5.
3183. y' =X 2 -\-y; y= 1 when x=O. Compute y when X= l.
3184. y' =2y-3; y= 1 when x=O. Compute y when x=0.5.
3185. { y'=-x+2y+ z,
z' =x+2y+3z; y=2, Z=-2 when x=O.
Compute y and z when x = 0.5.
3186. { y' = - 3y-z,
z' =y-z; y=2, z=-1 when x=O.
Compute y and z when x = 0.5.
3187. y"=2-y; y=2, y'= -1 when x=q.
Compute y when x = 1.
3188. y 3 y"+1=0; y=l, y'=O when x=l.
Compute y when x = 1.5.
2
3189. ddta+
x x '
2 cos2t=0; x=O, x =·I when t=O.
Find x (n) and x' (n).

Sec. 6. Approximating Fourier Coefficients


Twelve-ordinate scheme. Let Yn = f (xn) (n = 0, 1, ... , 12) be the values
of the function y=f(x) at equidistant points Xn=:n; of the interval lU,2n),
and Yo ~ Yu We set up the tables:

Yo Yt Ya Ya Y4 Ys Ya
I Yn Yto Ye Ya Y7

Sums (~) U0 U1 U 2 u1 u4 u, u8
Differences (~) \ V 1 V 2 Va V 4 V 5

Sums Sums
Diflerences Differences
394 Approximate Calculations [Ch 10

The Fourier coefficients an, bn (n = 0, I, 2, 3) of the function y = f (x)


may be determined approximately from the formulas:
6a0 = s0 +s1 + s2 + s1 , 6b1 = 0.50' 1 +0.866cr! + cr8 ,
+
6a 1 = t0 0.866t 1 0.5t 2 , + +
6b 2 = 0.866 (T 1 T 2),
6a 2 = s0 -s1 + 0.5 (s1 -s1 ), 6b1 = 0'1 -0'1 ,
6a 1 = t 0 - t 2 , • (1)

Y3 1 1
where 0.866 = - -::::::: 1- IO- •
2 30
We have
I

/(x)::::::: ~ + L (an cos nx+ b,. sin nx).


n=J

Other schemes are also used. Calculations are simplified by the use of
patterns.
Example. Find the Fourier polynomial for the function y= f (x) (O~x,;;;;2n)
represented by the table

Yo Y1 Ya
38 38 12 4

Solution. We set up the tables:

y 138 38 12 4 14 4 - 18
32 8 - 24 - 27 - 23
u 138 70 20 -20 -13 -19 -18
v 6 4 28 41 27
38 70 20 _ 20 I 6 4 28
I
u - 18 - 19 - 13 v 27 41

b
t
I 20
56
51
89
7 _ 20
33
0'
't
I-21-37
33 28
45

From formulas (1) we have


a 0 =9.7; 0 1 =24.9; 0 2 =10.3; a 1 =3.8;
b1 =13.9; b2 =-8.4; b1 =0.8.
Consequently,
f (x)::::::: 4.8 + (24.9 cos x + 13.9 sin x) + (10.3 cos2x-8.4 sin 2x) +
+ (3.8 cos 3x +0.8 sin 3x).
Using the 12-ordinate scheme, find the Fourier polynomials
for the following functions defined in the interval (0,2n) by the
Sec. 6] Approximating Fourier Coefficients 395
tables of their values that correspond to the equidistant values
of the argument.
3190. y0 =-7200 y3 =4300 y8 =7400 lj 9 =7600
y,=300 yol=O Y1 =-2250y 10 =4500
lj 2 = 700 Y5 = -5200 y 8 = 3850 Y11 = 250
3191. Y0 =0 Ya=9.72 y6 =7.42 Yu =5.60
Y1 = 6.68 Y4 = 8.97 Y1 = 6.81 Y10 = 4.88
Y2 = 9.68 Ys = 8.18 Y8 = 6.22 Y11 = 3.67
3192. lj 0 =2.7l4 Y3 =1.273 Y6 =0.370 Ya =-0.357
Y1 = 3.042 Y4 = 0.788 y 1 = 0.540 y 10 = - 0.437
Y2 =2.134 Y5 =0.495 y8 =0.19l Y11 =0.767
3193. Using the 12-ordinate scheme, evaluate the first several
Fourier coefficients for the following functions:
I
a) f (x) = 2112
(x 3 - 3nx 2 + 2n 2 x) (0 ~ x ~ 2n),

b) f (x) = ~
n
(x- n) 2
(0 ~ x ~ 2n),
ANSWERS

Chapter I
1. Solution. Since a=(a-b)+b, then 1aj <;ja-b I+ 1b j. Whence 1a-b I~
~fal-l bl and la-b 1=1 b-a I~ jb I-I a 1. Hence, la-b I~ I a 1-1 b I·
Besides, la-bi=la+(-b)lo;;;;;lal+l-bl=lal+lbl. 3. a) -2<x<4;
b) X<- 3, X> 1; C) -1 <X< 0; d) X> 0. 4. -24; -6; 0; 0; 0; 6. 5. I;
I .r--
t
4 ; r l+x 1 ; lxl- 1 .r r - - ,r;--;--::o
1+x2 ; 1/r 1+x2 .6.n;
n :0. 7. f(x)=- 5 x+ 1 .
2 3 3
7 I 13 I
8. f(x)= x - x+l. 9. 0.4. 10. (x+fxj). 11. a) -1<;x<+oo;
6 6 2
b)- oo < x < + oo. 12. (-oo,-2), (-2, 2), (2, + oo). 13. a) -oo < x..;;;;- V2,
V2o;;;;;;x<+oo; b) x=O, lxl~ ¥2. 14. -1o;;;;;xos;;;;2. Solution. It should
be 2+x-x 1 ~0. or x2 -x-2E;;;;O; that is, (x+1)(x-2)E;;;O. Whence either
x+I::;?-:0, x-2,.;:;;0, i.e., -Ioe;;;;xE;;;2· or x+1E;;;O, x-2~0,i.e.,xoe;;;;-1,
x~2. but this is impossible. Thus, - 1E;;;;xE;;;;2. 15. -2<xoe;;;;O.
16. -oo < x.;;;;;-1, Ooe;;;;xoE;;;; I. 17. -2 < x < 2. 18. -1 < x <I, 2<x< + oo.
I :t
19. - o;;;;;;xo;;;;;;l. 20. 1E;;;xoe;;;;lOO. 21. knE;;;;xE;;;;kn+ (k=0, ±1, ±2 .... ).
3 2
22. !Jl (x)=2x4 -5x 2 -10, 'ljl (x)=- 3x'+6x. 23. a) Even, b) odd, c) even, d) odd,
e) odd.24. Hin!. Utilize the identity f (x) ={If (x) + f (-x)] + {11 (x)- f ( -x)).
2
26. a) Periodic, T =i :t, b) periodic, T = ~, c) periodic, T =:t, d) periodic

T=:t, e) nonperiodic. 27. y=~x, If O...;;xoE;;;;c; y=b if c<xE;;;;a; S=&x 2

if OE;;;;xo;;;;;;c; S=bx-~ if c<xE;;;;a. 28. m=q 1x when O.;;;;;xoE;;;;/ 1; m=


=q 1l 1 + q2 (x-11 ) when / 1 <X EO;; 11 +1 2 ; m=q 1l 1 +q 2 l 2 +q 1 (x-1 1 - / 2 ) when
l 1 +l 2 <xEO;;l1 +l2 +l1 =l. 29. !Jl['ljl(x)]=2 2x;'ljl[!Jl(X)]=2x 2 30.x.3t.(x-j-2) 2 •
:t
37. -
2 ; 0; :t . 38. a) y=O when x=-1, y>O when x>-1, y<O
4
when x<-1; b) y=O when X=-1 and x=2,y>Owhen -1<x<2,
y<O when -oo <x<-1 and 2<x<+oo; c) y>O when -oo<x<+oo;
d) y=O when x=O, x=- V3 and X= ¥3, y>O when - V3 < x < 0 and
VS<x< + oo, y<O when - oo <x<- V3 and O<x< V3; e) y = 0 when x = 1,
I
y>O when-oo<x<-1 and 1<x< + oo, y<O when O<x<I 39. a) x= (y-3)
2
(-oo<y<+oo); b) x=YY::F1 and x=-Vu+I ~-IE;;;y<+oo);
Answers 391

c) x= Vl-y' (-oo<y<+oo); d) x=2·10Y (-oo<y<+oo); e) x=


:n: <y< :n: ) .
=3"1 tany ( -2 x=y when -oo<y~O; x= .r-
40. r y when
2
Q<Y<+oo. 41. a) y=U , U=2x-5; b) y=2a, U=COSX; c) y=\ogu,
10

u=tanv, v=-f; d) y=arc sinu, u=3v, v=-x•. 42. a) y=sin 1 x; b) y=


=arctan Ylogx; c) y=2(x 2 -l) if IXI<;1, and y=O if jxl> I.
43. a) y=-cosx•, Vn<;lxl~ Y21i; b) y=log(10-10x), -oo <x< 1;
c) y= i when -oo<x<O and y=x when O...;;x< + oo. 46. Hint. See Appen-
dix VI, Fig. I. St. Hint. Completing the square in the quadratic trinomial
we will have y= y0 +a (x-x 0) 1 where x0 = -bf2a and Yo= (4ac-b 2 )f4a.
Whence the desired graph is a parabola y=ax 1 displaced along the x-axis by
x 0 and along the y-axis by y0 • 53. Hint. See .Appendix VI,
Fig. 2. 58. Hint. See Appendix VI, Fig. 3. 61. Hint.
The graph is a hyperbola y= m , shifted along the x-axis by x0 and along
X

the y-axis by Yo· 62. Hint. Taking the integral part, we have y={- ~I
( x+-}) (Cf. 61*). 65. Hint. See Appendix VI, Fig. 4. 67.Hint. See Appendix VI,
Fig. 5. 71. Hint. See Appendix VI, Fig. 6. 72. Hint. See Appendix VI,
Fig. 7. 73. Hint. See Appendix VI, Fig. 8. 75. Hint. See Appendix VI,
Fig. 19 78. Hint. See Appendix VI, Fig. 23. 80. Hint. See Appendix VI,
Fig. 9. 81. Hint. See Appendtx VI, Fig. 9. 82. Hint. See Appendix VI,
Fig. 10 83. Hint. See Appendix VI, Fig. 10. 84. Hint. See Appendix VI,
Fig II. 85. Hint. See Appendix VI, Fig. 11. 87. Hint. The period of the function
is T = 2:nfn. 89. Hint. The desired graph is the sine curve y = 5 sin 2x with am-
plitude 5 and period :rt displaced rightwards along the x-axis by the quantity
1 {- . 90. Hint. Putting a= A cos q> and b=-A sin q>, we will have y=A sin (x-q>)

where A=Va 1 +b 2 and q>=arctan( -:)'Inourcase, A=10, q>=0.927. 92.

Hint. cos 2 x = ~ (1 +cos 2x). 93. Hint. The desired graph is the sum of the graphs
y,=x and y 2 =sinx. 94. Hint. The desired graph is the product of the graphs
y 1 =X and y 1 =sinx. 99. Hint. The function is even For x>O we determine
the points at which 1) y=O; 2) y= 1; and 3) y=-1. When x---. oo,+
y-+ 1. tOt. Hint. See Appendix VI, Fig. 14. 102. Hint. See Appendtx VI,
Fig. 15. 103. Hint. See Appendix VI, Fig. 17. 104. Hint. See Appendix VI,
Fig. 17. 105. Hint. See Appendix VI, Fig. 18. 107, Hint. See Appendix VI,
Fig. 18. 118. Hint. See Appendix VI, Fig. 12. 119. Hint. See Appendix VI,
Fig. 12. 120. Hint. See Appendix VI, Fig. 13. 121. Hint. See Appendix
VI, Fig. 13. 132. Hint. See Appendix VI, Fig. 30. 133.Hint. See Appendix VI,
Fig. 32. 134. tlint. See Appendix VI, Fig. 31. 138. Hint. See Appendix VI,
Fig. 33. 139. Hint. See Appendix VI, Fig. 28. 140. Hint. See Appendix VI,
Fig. 25. 141. Hint.
398 Answers

Form a table of values:

0 2 3
I -I -2 I -3
X 0 8 27 .. I -I -8 1-27

y 0 4 9 .. I 4
I 9

Constructing the points (x, y) obtained, we get the desired curve (see Appen-
dix VI, Fig. 7). (Here, the parameter t cannot be laid off geometrically!)
142. See Appendix VI, Fig. 19. 143. See Appendix VI, Fig. 27. 144. See
Appendix VI, Fig. 29. 145. See Appendix VI, Fig. 22 150. See Appendix VI,
Fig. 28. 151. Hint. Solving the equation for y, we get y= ± Y25-x 2 • It is
now easy to construct the desired curve from the points. 153. See Appen-
dix VI, Fig. 21. 156. See Appendix VI, Fig. 27. It is sufficient to construct
the points (x, y) corresponding to the abscissas x=O, ± f, ±a. 157. Hint.
Solving the equation for x, we have x=IO logy-y<*l. Whence we get the
points (x, y) of the sought-for curve, assigning to the ordinate !' arbt trary
values (y > 0) and calculating the abscissa x from the formula l*l Bear in
mind that logy--+-- oo as y-+ 0. 159. Hint. Passing to polar coordinates
r=Vx 2 +y1 and tancp=J!., we will have r=e·~ (see Appendix VI, Fig 32)
X
160. Hint. Passing to polar coordinates x=rcos<p, and y=rsin cp, we wtll
3
haver= sincpcoscp (see Appendix VI, Fig. 32) 161. f=32+1. 8C
cos 3 cp + sin 3 cp
ab . ab
162. y=0.6x(IO-x); Ymax=l5 when x=5. 163. y=2smx; Ymax=2
n I
whenx=T 164. a) x1 = • X 2 =2; b) x=0.68; c) x1 =1.37, x2 =10;
2
d) x=0.40; e) x=l.50; f) x=0.86. 165. a) X 1 =2, y 1 =5; X 2 =5, y 2 =2;
b) X1 =-3, y 4 =-2; X 2 =-2, Y2 =-3; x3 =2,y 3 =3;x,=3, y,=2; c) x1 =2,
Y1 =2; X 2 :::::3.1, y 2 :::::-2.5; d) X 1 :::::-3.6, y,::::::-3.1; x 2 ::::::-2.7, y 2 ::::::29;
n Y2
xa=::::2.9, y1 ::::::1.8; x4 ::::::3.4, y,::::::-1.6; e) X 1 =4• y1 = -- ; X 2 = 4.
5n
2
Y2 I - I
Y2 = - -- . 166. n>
2
Ye. a) n~4; b) n> 10; c) n;;=,32. 167. n >--e-
e
- I =N. a) N= 9; b) N=99; c) N=999. 168. 11=-g (e <I). a) 0 02;
b) 0 002; c) 0.0002. 169. a) log x < -N when 0 < x < 1\ (N); b) 2x > N when
7
x > X (N); c) If (x) I > N when I xI > X (N). 170. a) 0; b) I; c) 2; d) 3o .
I 3
171. . 172. l. 173. -
2 . 174. l. 175. 3. 176. l. 177. 3 . 178. I . Hint.
2 4 3
I
Use the formula !2+2 2 + ... +n 2 =
6 n(n+l)(2n+l). 179. 0. 180.0.181.1.
182. 0. 183. oo. 184. 0. 185. 72. 186. 2. 187. 2. 188. oo. 189. 0. 190. I. 191. 0.
I a-1 I
192. oo. 193. -2. 194. oo. 195. 196. a• . 197. 3x2 . 198. -I. 199.
2 3 2
Answers 399

4 I I 3
200. 3. 201. 3 . 202. 9. 203. -56. 204. 12. 205. 2 . 206. 3 . 207. I.
2o8.
2
I
v-x·
209.
3
I
2
• 210. -
1
Vx
3 .211. o. 212.
a
2 . 213. -
5
2 .214.
I
2.
215. 0. 216. a) ~ sin 2; b) 0. 217. 3. 218. { . 219. ~ . 220. n. 221.
2 ,
222. cos a. 223. -sin a. 224. :rt. 225. cos x. 226. - j 2. 227. a) 0; b) I.
2 I I I 1
228. "i'". 229.
2. 230. 0. 231. - .,,..
3
. 232.
2 (n 2 -m 2 ). 233.
2 234. 1.
2 2 I I I
235.
3 . 236. "'it . 237. - 4 . 238. n. 239. 4 . 240. I. 241. I. 242. 4 .
3
243. 0 244. -n. 245. 0. 246. e- 1 • 247. e2 • 248. e- 1 • 249. e-'.
"'
250. ex. 251. e. 252. a) I. Solution. lim (cos x) x = lim [1-(1-cos x)] x =
X-+0 X-+0

2•in•-f
I 1 ]-
--- k
1 2•in• .!!_
=lim (1-2sin 2
x -70
!.._)x
2
=lim l(l-2sin !.._)
x~o 2
2
2

lim ___ 2•in'-=.)


( 2
X--+0 X
=e

2sin ; )
Since lim ( - - - =-2lim [(sin
X-?0
--
X
2

X~O
2
X
i) 2

xz] x
- . =-2·1·1im-;=0, it follows
4X X-?0 'I

I
that lim (cosx) x =e0 = 1. b) .r-e. Solution. As in the preceding
.t-->0 v

case (see a), . (cos x)Xi


I tm
1

=e X-+0
2
hm ( - •in•
xt
f)
. s·mce I'tm
(- 2 sin 2 i) =
2
X-+0 X-+0 X

= -2 .t-+o
lim
x
l -- _.:_
r-( sin 2x)z ]
4x 2
=
2
2 1
- - ,
2
tt follows that lim (cos x)i> =e
X-+0
1 I
2
=

= Ve. 253. In 2. 254. 10 loge. 255. 1. 256. 1. 257. -~. 2.'58. 1. Hint.
1
Put ex-l=a, where a-+0. 259. Ina. Hint. Utilize the identity a=e "a.
260. In a Hint. Put ..!... =a, where a-+ 0 (see Example 2.59) 261. a-b.
n
262. I. 263. a) I; b) } . 264. a) -1; b) I. 265. a) -1; b) I. 266. a) 1; b) 0.
267. a) 0; b) 1. 268. a) -1; b) 1. 269. a) -1; b) 1. 270. a) -oo; b) + oo.
400 Answer~

271. Sol uti on. If x '¢: kn (k = 0, ± 1, ± 2, ... ), then cos1 x <1 and y = 0;
1 1
but if x=kn, then cos x=1 and g=l. 272. g=x when Oc;;x< 1; u=
2
n
when x=1; g=O when x> I 273. Y=IXI. 274. g=- when x<O; g=O
2
n
when x=O;g=T when x>O. 275. g=1 when Oc;;;xo;;;;;1; y=x when
61 c
1<x<+oo. 276. . 277. x1 ..... -7i; x1 ..... oo. 278. n. 279. 2nR.
450

284. lim AC - !._ 285 ~ 286 k-1


r:""tt":fi .
1
280 e 281 I - . 282.
e-1 · ' 3 n-u> n- 3 · ' 2 · ' -'
2
e -1
' x'+ 1
b=O; the straight line y=x is the asymptote of the curve y=x•+
1.
287. Q~n'=Q 0 ( 1 +~r. where k is the proportionality factor (law of

compound interest); Q1 = Qgek1• 2!!8. I xI> _!_, a) I xI> 10; b) I x 1 > 100;
e
8
c) I xI > 1000. 289. I x-l I < when 0 < e < 1; a) 1x-1 I < 0.05;
2
l
b) \X-11<0.005; c) lx-11<0.0005 290. lx-2\< N=b; a) b=0.1;

b) {) = 0.01; c) {) = 0.001. 291. a) Second, b) third. ~, { . 292. a) I; b) 2;


1 2
c) 3. ~93 a) 1; b)
4 ; c)
3; d) 2; e) 3. 295. No 296. 15. 297. -l. 298. -1.
29!1. 3. 300. a) 1.03 (I 0296); b) 0.985 (0.9849); c) 3.167 (3.1623) Hint.
ViO=V9+1=3 V' +{: d) 10.954(10.954). 301. I) 0.98(09804);
2) 1.03 ( 1.0309); 3) 0.0095 (0.00952); 4) 3.875 (3.8730); 5) 1.12 ( 1.125);
6) o 72 (0.7480); 7) 0.043 (0.04139). aoa, a) 2; b) 4; c) ~ ; d) -}. 307. Hint.
If x>O, then when l~xl<x we have \Vx+~x-Vxl=
= 1 ~x \/( Vx+ M+ Vx)..:;;; 1 t\x If Vx. 309. Hint. Take advantage of the
inrquality I cos (x + t\x) -cos x !..:;;; 1 t\x 1 310. a) x -:j:: ~ + kn, where k is an
integer; b) x-:/= kn, where k is an integer 311. Hint. Take advantage of the
inequality llx+t\xl-1>:1\..:;;;lt\xl 313. A=4. 3t4. f(O)=l. 315. No
316. a) f(O)=n; b) f(O)=;; c) f(0)=2; d) f(0)=2; e) {(0)=0; f) f(O)=l.
317. x= 2 is a discontinuity of the second kind. 318. x=-1 is a removable dis-
continuity. 319. x=-2 is a discontinuity of the second kind; x=2 is a removable
discontinuity 320. x=O is a disconttnuity of the first kind. 321. a) x=O is
a discontinuity of the second kind; b) x=O is a removable discontinuity. 322. x=O
h a removable discontinuity, x= kn (k= ±I, ± 2, ... ) are infinite discontinuities
1t
323. x= 2nk ±2 (k=O, ± 1, ± 2, ... ) are infinite discontinuities.
324. x=kn (k=.O, ± 1, ± 2, ... ) are infinite discontinuities. 325. x=O is a
discontinuity of the first kind. 326. x=- I is a removable discontinuity:
x= 1 is a point of dis<.ontiiluity of the first kind. 327. x=-1 is a discon-
Answers 401

tinuity of the second kind. 328. x=O is a removable discontinuity. 329. x= 1


is a discontinuity of the first kind. 330. x = 3 is a discontinuity of the first
kind. 332. x= 1 is a discontinuity of the first kind. 333. The function is
continuous. 334. a) x= 0 is a discontinuity of the first kind; b) the function
is continuous; c) x= kn (k is integral) are discontinuities of the first kind.
335. a) x=k (k is integral) are discontinuities of the first kind; b) x=k
(k ~ 0 is integral) are points of discontinuity of the first kind. 337. No, since
the function y=E(x) is discontinuous at x=l. 338. 1.53. 339. Hint. Show
that when x0 is sufficiently large, we have P (-x 0) P (x0) < 0.

Chapter II
341. a) 3; b) 0.21; c) 2h+h'. 342. a) 0.1; b) -3; c) Va+h-Va.
344. a) 624; 1560; b) 0.01; 100; c) -1; 0.000011. 345. a) aAx; b) 3x2 Ax
2
+
A )' (A )'· 1 A +(A 2• ) 2xAx+(Ax) • 2x+Ax .
+ 3x ( LlX + ux ' 3x + 3x ux ux) • c - x• (x + Ax)• , x• (x + Ax)• ,
Vx+l\x-Vx; 1 ; 2x(2~x-1); 2x( 2 ~~- 1 );
d) Y x+ Ax+ V x e) Ax
x+l\x 1 (
f) In - x - ; Ax In 1+ Ax) x .
346. a) -1; b) 0.1; c) -h; 0. 347. 21.
348. 15 em/sec. 349. 7 .5. 350. f (x + Ax)- f (x) . 351. f' (x) = lim f (x +Ax)- f (x) .
Ax ~x-+o Ax
352. a) 1\cp; b) dcp = Jim Acp, where cp is the angle of turn at time t.
At dt M-+o At
353. a) AT; b) dT = lim AT, where T is the temperature at time t.
l\t dt 61->0 At

354. dQ = lim AQ, where Q is the quantity of substance at time t.


dt M-..o At
356. a) - 1 :::::::-0.16; b) - ~ :::::::-0 238;
355. a) Am;
Ax
50
b) lim Am
~x-+o Ax
.
6 21
c) - ::::::: -0.249; Yx=• = - 0.25. 357. sec 2 x. Solution.
201
y' = lim tan (x + Ax)- tan x = lim sin Ax = lim sin Ax X
~x ... o Ax ~x-..o Ax cos x cos (x +Ax) ~x-+o Ax
x lim 1 = _l_ = sec2 x. 358. a) 3x2 ; b) - '!... ; c) _ 1_ ;
~X -+0 cos X cos (x + Ax) cos• X x• 2v-r
d) --=--!.__. 359. .!._ Solution. f' (8)= lim f (8 +Ax) -f (8) =
sin 2 x 12 ~x-..o Ax
= lim VB=iTx- V8 _ Jim 8+ Ax-8 =
~x-..o Ax Ax-...o Ax lV<8 + Ax) 2 + V
(8+ A.~) B+ V 81]
1
=lim =.!... 360. f'(0)=-8, {'(1)=0,
AHo V<B+ Ax)*+2 VB+Ax +4 12
f' (2) =0. 361. x1 =0, x2 =3. Hint. For the given function the equation
f'(x)=f(x) has the form 3x2 =x 8• 362. 30m/sec. 363. 1, 2. 364. -1.
365. f' (x0) =-1
- . 366. -1, 2, tan cp =3. Hint. Use the results of Example 3
x•
o
~x-+o
V<Ax) 2
and Problem 365. 367. Solution. a) /'(0)= lim---= hm
l\x ~x-...o
v-=±
1
l\x
oo:
402 Answers

b) f'(l)= lim
&x-+o
VT+Tx-
!'lx
1
= lim
1
&x-.o V (l'lx)'
=oo; c) f'- ( 2k +
2
1 n)=
I
2
eos( k + 1 n+l'lx)\ .
= lim 2 = lim 1 sm !'lx 1 =- 1; f' + (2k + 1 n) =
&x~-o !'lx &x~-o llx 2
= lim I sin !'lx I = 1. 36k. 5x'-
2. 369. - .!__ + 2x- 12x2 +
370. 2ax +b. 2x8•
Ax-++ o l'lx 3
5
371. _ISx•. 372. matm-t+b(m+n)tm+n-t. 373. .r6ax n
. 374. - 1 .
a r B2+b2 X
1 a .!_ 2 8
8
375. 2x 8
-5x 2 -3x-•. 376. 3x 8

Hmt. y=x 2x
a
=X
a
. 377.
3x2
V -
4b
x

--vX2 .
3x
2a
378.
be-ad
(c+dx) 1
379.
-2x 2 -6x+25
(x 2 -5x+W
380.
1-4x
x 2 (2x-l) 2

381. Y z (1 ~ Y z)•. 382. 5cosx-3sinx. 383. sin: 2x 384. (sinx=:osx)•


385. 12 sint. 386. y'=O. 387. cotx-+. 388. arcsinx+.rx .
sm x r l-x•
2 393 . fix"-
389 . x arc t an x. 390 . xeeX ( x+ 7) . 39 I. xe X . 392 . eX ----x'.
X- _e_"_ XS .
1 2
394.ex(cosx-sinx).395.x2ex.396.ex(arcsinx+.r ) .397.x( lnx-l).
r I -x 2
ln 2 x
2 In x 2 2ln x 1 .
398. 3x2 lnx. 399. - + - 2 - 2 . 400. - - - . 401. smh x+x coshx.
X X X Xn 1 10 X
2xcoshx-x 2 sinhx - t h2 -3(xlnx+sinhxcoshx)
402 · cosh 2 x 403 · an x. 404 · x ln 2 X•Sinh 1 x
2 2 1 1
405. - x 406. .r arcsinhx+,;- arcsinx.
•1-x4 r l-x• r l+x 2
408 . 1+2xarctanhx. 410 . ~(ax+b)
2
407 . x-Yii=Tarccoshx
x 2 x - 1
2 y ( 1- x )
2 2 c c
41 t. 12ab + 18b2y. 412. 16x (3 + 2x 2 ) 1 • 413. x•-_.!____. 414. .r -x

v va•
(2x-1) 8 r 1-x2
bx 2
I 1-tan• x+tan' X
415. V (a+ bx')2. 416. - xz -1. 418. cos• x

419. -1,;- . 420 . 2 - 15 cos 2 xsmx.


• 421. -16 cos --.
21 H"mt .x=sm- . z t+
2 sin 2 x r cot x sin 8 21
sin x sin 8 x . 3 cos x + 2 sin x .
+ cos- 2 t. 422. 423. 424 ;::--;,;-r:;:;:;::::===:::::;:;;===
(1-3cosx) 8
cos 4 x 2 r 15sinx-10cosx
2 cos x + 3 sin x 1
425.
3 Slll Xv C0~ X •
4
426.
2 Y 1-x V 1 + arc sin x •
2

1 3 (arc sin x) 2 -1
427. 428.
2 (1 +x 2)Yarc tanx Y1-x2
Answers 403

ex +xex + 1 2ex-2x In 2 51n4 x ·


429. 430. + -- . 432. (2x-5) X
2 Vxex
+x 3 V (2eX-2X + \) 1 X
a
X cos(x5 -5x+ 1 ) - - - - 433. -a sin (ax+~). 434. sin (2t + Ql ).
x• cos• !!...
X
_ cosx -1
435. 2 436. 437. x cos 2x 2 sin 3x8 • 438. Solution,
sin' x ·
sin 2 ..:_
a
1 (2x)' = 2 . 439.
y 1-(2x) 2 v 1-4x 2 v- 2 . 440. 2 y--1
x x4 -1 x-x 2
-1
. 441. 1+x"
442. - 2 .443. -10xe-x'. 444. -2~s-x•tn5. 445.
1
2xl0 2x(l+xlnl0).
1 +x
446. sin 21 + 21t cos 21 In 2. 447. V . 448. 2 -ex
449. cot x log e.
1-e2 x 2x + 7
450 . -2x. 1. 21nx _ _ l_. 452 . (ex+5cosx) Vf=X!-4
45
l-x• x xlnx (ex+5sinx-4arcsinx)Y1-x 2
453.
1
+ l 454. y1 + yI •
(1 + ln 2 x) x (1 +x 2 ) arc tanx 2x In x+ I 2 ( x+x)
455. Solution. y' =(sin' 5x)' cos 2 ~ +sin' 5x ( cos 2 ; )' = 3 sin 2 5x cos 5x 5 cos 2 ~ +
+ sin 3 5x 2 cos ~ ( -sin i) { = 15 sin 2 5x cos 5x cos•
1
i- ~sin' 5x cos i sin T,
456. 4x+ 3 457. x"+ 4x- 6 . 458. x • 459. x-! .
(x-2) 3 (x-3) 5 (l-x 2 ) 5 x 2 V2~ 2 -2x-t-l

460. - . _ I-
Y
(a 2 -j-~ 2 ) 8 •
461. f x•
(I +x2 ) 5
462. (J +(x x)' . 463. X5 v (! +x') 2 ,

I
464. 465. 4x1 (a-2x1) (a- 5x3 ),
t/(x-1) 3 (x+ 2) 5
2ubmnx 11 - 1 (a-j-bA")m-J x'-! a-3x
466. 467. 468.
(a-bx")m+ 1 (x+ 2) 6 2 Jfa-x ·
469.
3x 2 + 2 (a+ b -j-c) x +ab +be+ ac 470.
1+2 vy
2 Jf(x+a) (x+b) (x+c) 6Vy tf(y+ v y)" •
471. 2(7t+4>V3t+2. 472. y-a .473. ,r I , 474 . SIO
· ' XCOS • X.
f(2ay-y 2) 3 r ex+ I
475 476 10 tan 5x sec 2 5x. 477. x cos x1 • 478. 3t 2 sin 2t 3 •
· sin 4 x cos 4 x · •
479. 3cosxcos2x. 480. tan4 x. 481. c_·o_s2 _x. 482. (u-~)sin 2x .483.0.
sin4 x 2 V a sin 2 x + ~ cos• x
1 arc sin x (2 arc cos x-arc sin x) 2 1
484. - .r . 485. .r . 486.
2 r !-x• x r 2x 2 -1 l+x 2
487. X arc cos X -
(l-x1 ) I•
rr=xz 488. I
Va-bx 2
. 489. .. I~ . 490.
V a+x
2 Va•-x•.
5
491. -x . 492. arc sin Vx. 493. .r •
Y2x-x• r 1-25x1 arcsin5x
404 Answers

sin a.
494.
x
y 1-ln•x 4915.
l-2xcosa+r
496.
5+4 sin x
:n• ~ Y eax. 500. sin 2xe'1n•x •
501.
v
497. 4x , /b x .
2m p (2mamx +
1
-X
498.
1 COS X
b)P- 1amx In a. 502.
499. a
2
ea.t (a. cos pt- p sin pt). 503. e~" sin px.
504. e-x cos 3x. 505. xn- 1a-x• (n-2x 2 ln a). 506. - { y tan x (I+ Ycosx In a).
I
3cot In 3 x 2ax+b 1
507. ( . 1 )• . 508. 2 . 509. .r 2 . 810.
x sm _ ax + bx + c ,. a + x•
X
1 -2 1 x-1 2x+11
511. Y 2ax +x . 512. 2
- - . 513. - - t a n - . 514.
x ln 1 x x• x x•-x-2
Hint.
2
3x -16x+19
y=51n(x-2)-31n(x+l). 515. (x-l1(x-2)(x-3) 516. sin'xcosx .
y--- -6x 15a In" (ax +b)
517. x•-a•. 518. (3-2x') In (3-2x') 519. ax+b
2
520. y . 521. mx + n 522. Y2 sin In x. 523.
x 2 + a2 x 2 -a• sln 1 x
1 3
524. V'f+X'i. 525. x+ . 526. Y [2arcslnaxln2+2(l-arccos3x)}.
x x•- 1 1-9x 2

527 . ( 3:~:~:-1 n 3 +sin 22abx)acosaxcosbx-';bbsinaxsinbx. 828 . d. .


COS X 1 + Sl11 X
COS X

529. l 530 . V 1 +In x -l- V 1


x(l+ln 2 x) l-x 2 arcsinx x 'x l-ln 2 x
~
2
1 3
531. . 532. x• 533. . 534. x - x
2
x(l+ln x) 4 2
x +x -2 cosx slnx 4
x -l
+ 1 • 536. arcsin~ . 537. 6sinh 2 2x·cosh2x. 538. ea"(ucosh~x+
1
535.
1 X (1 -11:2) •
+~sinh px)~ 539. 6 tanh 2 2x (1-tanh 2 2x) 540. 2 coth 2x. 541. --r~x .
}' a• + x4
542. v I
x ln x-1 1
543. - - . 544. -
cos2x slnx
1
. 545.
l-x 2
-1
546. x arc tanh x
2

547. x arc sinh x. 548. a) y' = 1 when x > 0; y' = - 1 when x < 0; y' (0\ dues
not exist·, b) '=12xl. 549. '=...!.... 550. f'(x)={ - l when x.;;;;;O,
Y Y x -e-x when x > 0.
1 ¥3 ' ' ' 2
552.
2 + 3 . 553. 6lt.554. a) /_(0)=-1, /+(0)=1; b)/_(0)=-a·
, -2 , , , , '
t + (0) = a;
c) I_ (O) = l, f + (O) = o; d) I_ (0) = f + (O) = o, e) f _ (0) and
· x-3
f + (0) do not exist. 555. 1-x. 556. 2+ - -. 557. -1. 558. 0 561. Solu-
4
tion. We have y' =e-x (l-x). Since e-x =JL , it follows that y' = 1L (1-x)
X X
or xy'=y(l-x) 566. (1+2x)(l+3x)+2(1+x)(l+3x)+3(x+l)(l+2x).
567• _ (x + 2) (5.\ 1 + l9x + 20) , . x2 -4x + 2
568
(x+ 1)" (x+3)• 2 Yx(x-1)(x-2)'
Answers 40&

669
·
3X1 5+
3(x1 +I)
v---x.-
x1 +I"
570
"
(x- 2)0 (XI_:_ 7X+ I)
(x-l)(x-2)(x -3)V(x-I) 1 (x-3) 4 •

•~x•+x-.~
4
571. •r. 572. xx(l+lnx). 573. xx'+J(l+21nx) .
3(x-l) • (x+2) •(x+3) •
¥"~-.!...
1
x~nx.575.
1
574.y/x x (I+;Inx). 516.xxxxx(f+ lnx+In•x)·
677. Xsinxci~X+COSXJnx),
+r
578. (COSX)&IRX (COSXJnCOSX- SinxtanX).

579. (I+ [In (I+ +)+I ~x]. 580. (arc tan x)x x
x [lnarctanx+( l +x•tarctanx] . 581. a) x~= 3 (l~x 2 );
' 2
b) xy

= 2 -cos x ; c) xy = ~ . 582. 2 t . 583. tt -I
IO 3 I -2t
+I . 584. I-ta •
I+5e•
585. t (2 -t') . 586. ~ . 587. t +I 588. tan t. 589.
b
--.
I-2t 1 3ift t(t 1 +I) a
b t t 3 , { - I when t < 0, 93 - 2eat.
a an . 591. -tan t. 592. Yx =
590. -- I w hen t
> 0. 5 ·
594. tan t. 596. I. 597. oo. 599. No. 600. Yes, since the equality is an Jden-
tity.
2 b2 x x2
601. -5. 602. --~-. 603. -~· 604. - x(3x+2y) 2+2
y!i
v!i . y . 605. -
ay y X
-.
X

606. -
x
607. 3-- I 2y22 2 =I ; - ( 4 a • 608. 10
, (x - y ) + xy + xy + y - cosy
;o .
ycos 2 y y I-x 2 -Y 2 ,
609. -I. 610. I - X COS 2 611. - I + 2 +--z. 612. (x+y). 613. y =
2
y X X y .
!I
__ I_= I
-eY-I x+y-I ·
614. JL.+ ex. 615. _Y_ 616.
x x-y x-y
617. cy+x v~. 618. xlny -y 1L.. 620. a) 0; b)_!_; c) 0. 622. 45°;
cx-y Vx 2 +Y 2 ylnx-x x 2
arctan2:::::::63°26'. 623. 45°. 624. arctan~:::::::36°21'. 625. (0,20); (1, 15);
e
(-2, -I2). 626. (1, -3). 627. y=x•-x+ 1. 628. k=u. fi29. -1 (I8 I)
, - 16 .
631. y-5=0; x+2=0. 632. x-I =0; y=O. 633. a) Y=2x; y= - I- x;
2
b) x-2y-I =0; 2x+y-2=0: c) 6x+2y-:t=0; 2x-6y+3rr=0;
d) y=x-I; y=I-x; e) 2x+y-3=0; x-2y+I=O for the point (I, I);
2x-y+3=0; x+2y-1=0 for the point (-1, I). 634. 7x-10y+6=0,
:t V2
2
IOx+1y-34=0 . 635. y=O; (n+4)x+(n -4)y-- -=0. 636. 5x+6y-
4
-13=0, 6x-5y+21=0. 637. x+y-2=0. 638. At the point (1, 0):
y=2x-2; y=- 1-x
- ; at the point (2, 0): y= -x+2; y=x-2; a t the po111
. t
2
3-x
(3, O): u=2x-6; u=-- . 639. l4x-I3y+I2=0 ; I3x+I4y-41=0~
2
406 Answers

640. Hint. The equation of the tangent is ~ + - Y =I. Hence, the tangent
2 Xo 2Yo
crosses the x-axis at the point A (2x 0 , 0) and the y-axis at B (0, 2y 0 ). Finding
the midpoint of AB, we get the point (x 0 , y 0 ). 643. 40° 36'. 644. The para-
bolas are tangent at the point {0, 0) and intersect at an angle
1
arctan7~808' at the point {1, 1). 647. St=S 11 =2; t=n=2r 2.
.r-
1 t tt an2;
T= 2asin t tan t . 2
S = 2asm t
648. In .
2
652.
2 2 ; N=2asin
2 ; 1 2
1 n
S 11 =asint. 653. arctank. 654. +2tp. 655. S 1 =4n 2a; S11 =a;
2
t=2na Vl+4:rt 2 ; n=a Vl+4n2 ; tan~t=2:rt. 656. S1=a; S 11 =~;
cpu•
t=Va2 +Q!; n=~Va2 +Q!; tantt=-cp 0 • 657. 3cmfsec; 0; -9cm/sec

658. 15 em/sec. 659. -


g
f
mjsec. 660. The equatton of the trajectory isy=x tan u-
v :sin 2u
- - - x2 • The range is The velocity,
2v:co~u g
~1 v 0 stna-gt
v v~-2v
• 0 gt sin a+g t ;
2 2
the slope of the velocity vector is
V 0 cos u
Hint. To determine the trajectory, eliminate the parameter t from the gtven
system. The range is the abscissa of the point A {Fig. 17)0 The projections
~ ~
v(:r C!itY;
of velocity on the axes are

trajectory
+

661. Diminishes
and

with the
The magmtude of the velocity is
the velocity vector is directed along the tangent to the

velocity 0.4 62. ( {, ~) 0


663. The dtagonal tncreases at a rate of - 308 cmfsec, the area, at a rate
of 40 cm 2 fsec 664. The surface area increases at a rate of 0 2n m 2 jsec,
the volume, at a rate of Oo05 n m 1 jsec. 665. ~ cm{sec 666. The mass of the rod
Is 360 g, the density at M is 5x gjcm, the density at A is 0, the density
at B is 60gjcmo 667. 56x6 +210x4 o 668. ex'(4x2 -f-2)o 669. 2cos2x
2(1-x2 ) -x 2x
670. 671. .r 2 2 3 672. 2 arc tan x + - - 0
3 (I + x") 2 r {a + x ) l + x•
673. - - + x arc si~/x . 674. _!_cosh!.. . 679. y"' =60
2 2
2
680. f"' {3) = 4320
l-x (l-x2) • a a
1
681. y"=(x!\) 5 0 682. yv = -64sin2x 684. 0; I; 2; 20 685. The velocity
is v=5; 4 997; 4o7o The acceleratiOn, a=O; -0.006; -0060 686. The law
of motion of the point M 1 is x =a cos wt; the velocity at time t is
-awsinwt; the acceleration at time t is -aw 2 coswto Initial velocity, 0;
initial acceleration: -aw•; velocity when x=O is -aw; acceleration when
x=O is 0. The maximum absolute value of velocity is aw; the maximum
absolute value of acceleration is aw 2 o 687. y<ll)=nla"o 688. a) nl {l-x)-<11 + 1l,
b) (-l) 11 +
11
'l..o ~~~ 3 ) 689. a) sin ( x+ n ~ ; b) 2" cos ( 2x +n ~) ;
2"x •
Answers 407

1
c) (- 3)" -ex. d) ( - 1)n- 1 (n-1)!. ) ( -1)"+ .nl. f) 2n!
e ' (I +x)"' e (I +x)"-1 1 ' (1-x)"+•'
g) 1
2"- sin [ 2x + (n-1) ~ J; h) ( -l)(::~nb)" l)!a". 690. a) x·e"" +ne"".'
b) 2"-•e- 2"" [2(-1) 11 x1 +2n(-l)"- 1x+"(n -l)(-1)"- 2 ]; c) (l-x 2 ) x
2
2
X cos (x+ ";) -2nx cos ( x+ (n-/) :rt) -n (n-1) cos ( x+ (n -; >:rt);
1
d) (-l)"- ·1·12 ... (2n-3)[ -(2 -I)]'
zn+l x n , e
) (-1)"6(n-4)l
n
ror 4
n :;;=. •
1
- X
2
2"x
691 y<h> (0) ~ (n- I )I
1
692. a) 9t 1 ; b) 21 2+ 2; c) - V l-t 1
• 693. a) -:- , t ;
asm
1 . -1 -1 t
b) 3a cos' t sin t , c) t ; d) at sin' t . 694. a) 0; b) 2e•a . 695. a) (I +tz) X
4
4a sin
2
X(l+3t2); b)
1+1
1(1-t)'' 696.
-2e- 1
(cosl+sint)'' 697.
(ddxzy) t=o=l.
2

699 2 cot' t 700 4e21 (2sin t -cos t) 70 -6 at 1 31 t2) 702 "1m


'sinl · · (sint+cost)s I. e ( + + · · m ·
d1x -f" (x) d'x 3 [f" (x)J2-f' (x) f"'(x) p2 b4
703. dy 2 = [f' (x)]' ; d y' = [/' (x)] 5 705. -!ji· 706. - a•y• .
2y 2 2 + d2y y d2x 1 Ill I
707. -----ya-· 708. dx 2=(1-y)'; dy•=yz. 709. 256 . 710. 16 .
1 3a2 r
711. a) 3: b) -yi-. 712. ~y=0.009001; dy=0.009. 713. d (1-x1)= 1 when
1
~x=-
x=l and
3. 714. dS=2x~x. ~S=2x~x+(~x) 2 • 717. For x=O.

718. No. 719. dy=-


:rt :::::::.-0.0436.
1
720. dy=2700=:::::.0.00037.
72
:rt dx -mdx dx
724 . .r---=.
721. dy=4S =:::::.0.0698. 722.
·) • xm+• . 723. (I
-x 2 I' az-x•
a dx 2 -2dx I +cos tp
725. x• + az . 726, - 2xe-" dx. 727. In x dx. 728. - - . . 729. - . d<p.
1-x sm 2 <p
X

730. - e'dlzl 732.- 1°x+ 8Ydx. 733. -ye Ydx -Ldx. 734~+Ydx.
1 +e · 7x+ 5y _.!_ x-y x-y
y2 -xe Y
12 :rt
735. ITdx. 737. a) 0.485; b) 0.965; c) 1.2; d) -0.045; e) +0.025=:::::.0 81.
4
738. 565 cm 1 •
739. yg-=:::::.2.25; Y17=:::::.4.13; ¥70:::::::.8.38; Jf640=:::::.25 3.
74o. Vto:::::::.2.16: Vro:::::::.4.13: V2oo:::::::.s.8s. 741. a> s: b) 1.1: c> o.93~
• 749. -x(dx~; .
2
d) 0.9. 742. 1.0019. 743. 0.57. 744. 2.03. 748. -(dx)
(l-x 2 ) 11• (l-x1 ) •
. 2 cos
750. ( -smxlnx+-x--7 (dx) .
sin 2 751.
21 x 3 2x)
" ~- (dx) • 752. - e-xx
X
6 6
x(x- ::+ )( x .
2 d )' 75 3. 384 (dx)• 7
l2 -x)s . 54. 3·2" sin ( 2x+5 (dx)". + n;)
408 Answers

755. ex cos a sin (x sin a+ na) · (dx)n. 757. No, since f' (2) does not exist.
758. No. The point x=-; is a discontinuity of the function. 762. 6=0.
14 n 1
765. a) ~=g; b) £=
4 .
2
768. lnx=(x-l)-
763. (2, 4).
2 (x-1) +
2 (x -1) 1 • x• x• . t
+
31 ~, , where 6=1+0(x-1), 0<0<1. 769. smx=x-3f+4fsm , 1 ,
. x1 x5 x1
1Vhere 61 =0 1 x, 0<0 1 <1; SlnX=X-3f+fif -6fslns 2 , Where s =0 X,
1 2

x• x•
xn " xn- 1
()<0.<1. 770. ex=1-f-x+2f+3f+···+(n- 1)!-j-iile·, where s=Ox,
I x• 5 x1
0 < !l <1. 772. Error: a) -
16
• ; b) _
1

•t ; 1n both cases =Ox; s
+~> o81 • (l+s> •
0 < 8 < I. 773. The error is less than ~ = ~. 775. Solution. We have
I 1

.,/a+x=(l+-=.)
1
(t-~)
1
V a-x a a
. Expandmg both factors in powers of x,
1 I

we get: x)
( I+a
-
1
lx
:::::: 1 + - - -l
2 a
-x- '· ( 1 - -
8 a•'
x)
a
1
1
::::::1+--
2 a
X 3
+--
8 a• ·
X
1

Multipl) 1n:;. we will have:


Y a+x x x2 a
--:::::: 1 + - + 2 . Then, expanding e in
a-x a 2a
X
- x x1 I
rower~ of ~, we get the same polynomial e a :::::: 1 + + 2a• . 777. - 3 a
a
779. 1 780. 3. 781. 2
1 n•
778 00 782. 5. 783. oo. 784. 0. 785. 2
2
786. \. 788. - . 789. I. 790: 0. 791. a. 792. oo for n > 1; a lor n = 1;
n
I
0 for n < I. 793. 0. 795. . 796. f21 797. -1. 799. 1. 800. e•. 801. 1.
5
I I 1
802 I 803. 804. - . 805. - . 806. -. 807. I. 808. 1. 810. Hint.
e e e
find lim S , whereS=!f(a-sina) is the exact expression for the area
u -· 0 -
2 bh 2
3
Dl the segment (R is the radius of the corresponding circle).

Chapter Ill
81 t. (-oo, -2), increases; (-2, oo ), decreases. 812. (-oo, 2), decrease~;
(2, oo), increases. 813. (-oo, oo), increases. 814. (-oo, 0) and (2, oo),
increases; (0, 2), decrt!ases 815. (-oo, 2) and (2, oo), decreases. 816. (-oo, 1),
increast!s; (1. oo), decreases. 817. (-oo, -2), (-2, 8) and (8, oo), decreast!s.
818. (0, 1), decreases; (1, oo), increases. 819. (-oo, -1) and {1, oo), in·
creases; (-1, 1), decreases 820. (-oo, oo), increases 821. ( 0,-}- ). de-

.crea~es; ( ~, oo). increases. 822. (-2, 0), increases. 823. ( - oo,2), decreases;
Answers 409

(2, oo), increases. 824. (- oo, a) and (a, oo ), decreases: 825. (- oo, 0) and
9 1
(0, I), decreases; (1, oo), increases 827. Ymax=T when x= . 828. No 2
extremum. 830. Ymtn=Owhen X=O; Ymtn=Owhen X= 12; Ymax= 1296when x=6.
831. Ymln:::::-0.76 when x:::::0.23; Ymax=O when x=1; Ymin:::::·-0.05 when
x::::: 1.43. No extremum when X=2. 832. No extremum. 833. Ymax=-2
9
when X=O; Ymln=2 when x=2 834. Ymax=l6 when x=3.2. 835. Ymax=
.r- 2 .r- 2 .rn
=-3 r 3 when X=-YJ; Ymin=3r 3 when X=ltJ 836. Ymax= r 2
when x=O 837. Ymax=- V3 when x=-2V3; Ymin= V3 when x=2V3.
3,r-
838. !lmm=O when X=±l; Ymax=l when x=O 839. Ymin=-2 r3 when

X-=(k-i)n; Ymax=~V3 when x=(k+{n) (k=O, ±1, ±2, .. ).

840. Ymax--=5 when x=l2 kn; Ymax=5cos~ when x=l2( k±{)n; Ymin=

=-5cos;whenx=12(k±i) n; Ymtn=l when x=6(2k+1)n (k=O,


I I
±1, ±2.... ). 841. Ymin=O when x=O. 842. Ymtn=--
e
when x=-
e .
4 I
843. Ymax = 1 when X= 1 ; Ymin = 0 when X=l 844. Ym;n = 1 wh~n
e e
I 4
x=O 846. Ymtn=O when x=O; Ym.•~ ~e2
845. Ymin = - - when x=-1.
e
when x=2 847. Ymin=e when x= 1. 848. No extremum. 849. Smallest
I
value ism=-~ for x=-1; greatest value, M=."2 when X=l. 850. m=O
I n
when x=O and x= 10; M =5 for x=5. 851. m=
2 when x=(2k +I)
4 ;
kn
M=l for x= (k=O, ±1, ±2, ... ). 852. m=O when x=l; M=n when
2
X=-1. 853. m =-1 when x= -I; M =27 when x=3. 854. a) m-- -6
when x = 1; Mo..= 206 when x = 5; b) m = -1579 when x = -10; M = 3745 when
x = 12. 856. p = -2, q o..= 4. 861. Each of the terms must be equal to a2
862. The rectangle must be a square with side f.
863. Isosceles. 864. The
side adjoining the wall must be twice the other side 865. The side o! tht
cut-out square must be equal to ]-. 866. The altitude must be half the
base. 867. That whose altitude is equal to the diameter of the base
868. Altitude of the cylinder, ~~; radius of its base R v;' where I(

is the radius of the given sphere. 869. Altitude of the cylinder, R Jl2
where R is the radius of the given sphere. 870. Altitude of the cone, ~F
410 Answers

where R is the radius of the given sphere. 871. Altitude of the cone, ~ R,
where R is the radius of the given sphere. 872. Radius of the base of the
cone -}r, where r is the radius of the base of the given cylinder. 873. Thaf
whose altitude is twice the diameter of the sphere. 874. cp = n, that is, the cross-
section of the channel is a semicircle. 875. The central angle of the sector
is 2n yf . 876. The altitude of the cylindrical part must be zero; that

is, the vessel should be in the shape of a hemisphere. 877. h= (t: -d: ) : .
878.
2~+ Y =1.
Xo 2 Yo
879. The sides of the rectangle are a"V2and bY2; where
a and b are the respective semiaxes of the ellipse. 880. The coordinates of
the vertices of the rectangle which lie on the parabola ( ; a; ±2 v-¥).
881. ( ± ~ ,
3
! ). 882. The angle is equal to the greatest of the numbers

V pVfi ' .
1
2rc cos T and arc t an dh . 883 . AM =a + J/ q 884. y·2"

885, a) X=Y= y2";


d
b) X=
d
y:f; Y=d V/2
-.
3. 886. V/2 aqQ
x __ -.

Pmin = Y2aqQ. 887. Y Mm. Hint. For a completely elastic impact of two
sphcros, the velocity imparted to the stationary sphere of mass m, after
impact with a sphere of mass m2 moving with velocity o is equal to
2 2
'+n v . 888. n= -. /N R (1f this number is not an integer or is not a divbor of
m, m2 V r
N,we take the closest integer which is a divisor of N). Since the internal resistance
of the battery is n;,
2
the physical meaning of the solution obtained is as
follows: the internal resistance of the battery must be as close as possible to the
2
external resistance. 889. y= h. 891. (- oo, 2), concave down; (2, oo ),
3
concave up; M (2, 12), point of inflection. 892. (- oo, oo ), concave up.
893. (-oo, -3), concave down, (-3, oo), concave up; no points of inflectiOn.
894. l-oo, -6) and (0, 6), concave up; (-6, 0) and (6, oo), concave down;
points of inflection M, (-6, -~).0(0, 0), M 2 (6, {). 895. (-oo,
- Y3i and (0, y3), concave up;(- V3, 0) and (Y3. oo), concave down;
tJOints of inflection M 1, 2 ( ± y:f, 0) and 0 (0, 0). 896. ( (4k + l) -; ,
(4k+3>i). concave up; (<4k +3) -;. (4k+5>-;), concave down (k=O,

±1, ±2, ... );points of inflection, (<2k+1lf. 897. (2kJt, (2k+l)n), o)-
concave up; ((2k-l) n, 2kn), concave down (k=O, ±1, ±2, ... ); the abscis-
.sas of the points of inflection are equal to x= kn. 898. ( 0, ~"ii). concave
Answers 411

rlown; ( ;ea, oo} concave up; M ( Jea' _ :ea) is a point of inflection.

899. ( - oo, 0), concave up; (0, oo), concave down; 0 (0, 0) is a point of
inflection. 900. (-oo, -3) and (-1, oo), concave up; (-3, -1), concave
down; points of inflection are M 1 ( -3, ~) and M 2 ( -1, ~ )· 901. x=2,
y=O. 902. X= I, x=3, y=O 903. X= ±2, y= I. 904. y=x. 905. y=-x,
left, y=x, right. 906. y=-1, left, y= I, right 907. x= ±I, y= -x, left,
y=x, right 908. y=-2, left, y~2x-2, nght. 909. y-=2 910. x-~o.
y= I, left, y=O, right. 911. x=O, y= I. 912. 1/=0. 913. x-=-1.
914. y=x-n, left; y=x+n, right. 915. y=a. 916. Ymax=O when X=O;
Ymin=-4 when x=2; point of inflection, M 1 (1, -2). 917. Ymax=l when
x=±Y3; Ymin=O when x=O; points of inflection M 1, 1 (±1, { )
918. Ymax=4 when X=-1; Ymm=O when x=l, point of inflection, M 1 (0, 2).
919. Ymax=B when X=-2, Ymin=O when x=2; point of inflection, M (0, 4).
920. Ymin=-1 when x--=0; points of inflection M 1 2 (± 0) and Y5.
M 3 , 4 ( ±1, - 64) .921.Ymax'=-2whenx=O;ymin=2when ' x=2; asymp-
125
totes,x=l,y~x-1. 922. Points of inflection M 1 , 2 (±1. =j=2); asymptote
x=O. 923. YmJx=-4 when x=-1; Ymm~4 when x=l; as~mptote. x=O.
-
924. Ymin=3 when x= I; potnt of tnllectton, M (- V 2, 0); asymptote,
x=O. 925.!/max={ when .>.=0, points of mflectton, M 1, 2 (±1, -{-}
asymptote, y=O 926. Ymax----2 when x~-0; asymptotes, x-~ ±2 and y=O.
927. Ymino~-1 when x=--1; Ym"x=l whenx=l; pomhof inflection, 0(0, 0)
and M 1 , l,r3) ;
±·T asymptote, y=O when X--"4-;
2 ( ±2Y3, 928. !/max= I

point of inflection, M ( 5, %) ; asymptotes, x=2 and u~=O. 929. Pot~j


of inflecti~n, 0(0, 0); asymptotes, X=±2 and y=O. 930. Ym.JX=-!6
when X=
3 : asymptotes, x--=0, x=4 and y=O 931. !im.x=-4 when
xo--1; Ymiu=4 \\hen X=l; asymptotes, X=O and y=3~ 932. A(O, 2)
and R (4, 2) are end-points; !lmax=2Jf"2 when x=2 933. A (-8, -4) and
B (8, 4) are end-pomts. Pomt of inflection, 0 (0, 0). 934. End-potnt,
A(-3, 0); Ymin=-2 when x=-2. 935. End-points, A(-V-3, 0), 0(0, 0)
and B ( V3. 0); Ymax = ¥2 when x= -I; point of inflection, M <V 3 + 2 V3,
~6 VI + ~3 ), 936, Ymax= I when x=O, points of Inflection,
M 1 , 2 (±I, 0). 937. Points of inflection, M 1 (0, I) and M 2 (I, 0); asymptote,
Y=-X. 938. !lmax=O when X=-1; Ymin=-1 (when X=O) 939. YmJx=2
when x=O; points of inflection, M 1 2 (±I, V2);
asymptote, y=O.
940. Ymin=-4 when X=-4; Ymax=4 wlien x=4; point of inflection, 0(0, 0);
asymptote. y=O. 941. Ymin= V-
4 when x=2, Ymln= 4 when x=4; v-
Ymax=2 when X=3. 942. Ymin=2 when x=O; asymptote, X= ±2.

943. Asymptotes, X= ±2 and y=O. 944. Ymin-= 'V¥3


2" when X= V3;
-
412 Answers

Ym•x = - ~~ when x= -3; points of inflection, M 1 ( -3, - ~), 0 (0, 0)

and M2 ( 3, ~); asymptotes, X=± 1 945. Ymin= V2" when X=6; point

of Inflection, M (12, v 12
100
): asymptote,x=2 946. !/max=_!_ when X= 1; point
e
ol tnflection, M ( 2, ~); asymptote, y=O. 947. Points of inflection,
1
M 1 ( -3a, ~:) and M 2 (-a, ~); asymptote, y=~- 948. Ymax=e2 when

x=4; points of inflection, M 1, 2 ( 8±2¥2 , e


2
2) ; asymptote, y=O.

A49. Yrnax = 2 when x = 0; points of inflection, M 1, 2 ( ± 1, ~). 950. !/max= 1


when x=±1; !lmin=O when x=O. 951. Ymax=0.74 when x=e 2 ::::::.1 39;
point of inflection, M (e"'• ::::::.14.39, 0.70); asymptotes, x=O and y=O.
2
a2 a . . . ( a 3a )
952. !/min= - 4 e when x = Ye, pomt of mflectton, M 4 ez . yes , -
953. Ymin = e when x = e; point of inflection, M ( e2 , e~ ) ; asymptote, x = 1;
4 I
y-+0 when X-+0. 954. Ymax= 2 ::::::.0.54 when X=~-1::::::.-0.86;
e e
!/min= 0 when x=O; point of inflection, M ( ~ -1 ::::::.-0.63; ~::::::. 0.37) ;
y 0 as x -+-1 + 0 (limiting end-point). 955.
""7 !/min= 1 when x = ± y2; points
of inflection, M 1, 2 (± 1.89, 1.33); asymptotes, x= ± 1. 956. Asymptote,
y=O. 957. Asymptotes, y=O (when x-++oo) and y=-X (as x--.-oo).
958. Asymptotes, x=-_!_, x=O, y=1; the function is not defined on the
e
interval I - -~ , 0] . 959. Periodic function with period 2n. Ymin = - ¥2
when x=
k .r- 11
4 n+2kn; Ymax= r 2 when x=-:r+2kn (k=O, ±1, ±2, .•. );

points of inflection, Mk ( { n + kn, 0). 960. Periodic function with


3 .r-
penod 2:rt. Ym!n = - 4 r 3 when
5
x= "3 n + 2kn; Ymax = 43 .r-
r 3 when
n
x= + 2kn (k=O, ±1, ±2, ... ); points of inflection, Mk(kn, 0) and
3
N k (arc cos ( - } ) + 2kn, ~ VIS). 961. Periodic function with period 2n.

·On the interval [ -:rt, :rt), Ymax = ! when x = ± ;: !/min=- 2 when


x= ±n; !/min =0 when x=O; points of inflection, M 1 1 (±0.57, 0.13) and
M1 , 4 (±2 20, -0.95). 962. Odd periodic function with' period 2n. On inter-
-val [0, 2n), Ymax= 1 when x=O; Ymin=0.71, when x=f: Ymax=1 when
Answers 413

~ 5·
X =2; Ymln=-1 when X=~; Ymax=-0.71 when X=4 ~;Ymin=-1 when
3
x =2 ~; Ymax= I when x=2~; points of inflection, M 1 (0.36, 0.86);
M 1 (1.21, 0.86); M1 (2.36, 0); M 4 (3.51. -0.86); M 1 (4.35, -0.86);
963. Periodic function with period 2~. Ymln = ~
2
M, (5.50, 0). when
~
x=4+2k~;
V2 when X=-
3
Ymax=---
2 4 ~+2k~ (k=O, ±1. ±2, ... );

asymptotes, x=! ~ + k~ 964. Periodic function with period ~; points of

inflection, Mk (~
-:r+k~.
¥2) (k=O,
- 2- ±1. ±2, ... );asymptotes,x= 3
4 ~+k~.
965. Even periodic function with period 2~ On the interval [0, .1t
4 I 4
Ymax =
3
y"3
when x =arc cos V ; Ymax = 0 when x = ~; Ymln = -
3
Y
3 3
when

x=arccos ( - :
3
} Ymin=O when x=O; points of inflection, M 1 (;, 0);
M 2 (arc sin ~2 , 4
~7 ); M1( ~-arc sin ~ 2 , -
4
~7 ) . 966. Even
per10drc function with period 2~. On the interval [0, ~1 Ymax =I when
x=O; Ymax= 3 ~6 when x=arccos(-:
6
); Ymin=- 3 ~6 when

x=arcros J 6
; Ymin=-1 when X=~; points of inflection, M 1 (; , 0):

M 2 (arccos Y~· ! y~); Ma(arccos(-yr~).


967. Odd function. Points of inflection, Mk (kn, k:rt) lk = 0, ±I, ±2, ... ) .
-{yM).
968. Even function. End-points, A, 2 {±2 83, -1 57) Ymax=l 57 when x=O
{cusp); points of inflection, M 1 ~ ( ± 1.54, -0. 34). 969. Odd function.
Limtting points of graph (-1, '-oo) and (1, +oo). Point of inflection,
0(0, 0); asymptotes, x=±i. 970. Odd function. Ymax= ~ -l+?k:rt when

x =~ + k~; Ymln = ~ ~ + I + 2k~ when x = ! ~ + k~; points of inflection,


2k+ I
Mk(k~. 2k~); asymptotes, X= - - n (k=O, ±I, ±2, ... ). 971. Even
2
function. Ymln=O when x=O; asymptotes, 1}=- ~ x-1 (as X-+--oo) and
~
y =2 x-1 (as X-+-+oo). 972. Ymin=O whenx=O (node); asymptote, y=l.

973. Ymin = l +~ 3
when x= 1; Ymax= ; -1 when X=-1; point of
inflection (centre of symmetry) (0, ~); asymptotes, y=x+2n (left) and y=x
(right). 974. Odd function. Ymin =I. 285 when X= I; Ymax = 1.856 when
x=-1; point of inflection, M ( 0, ~);asymptotes, y=]-+n (when
X
x ..... -oo) and y=2' (as x-++oo). 975. Asymptotes, x=O and y=x-ln2.
414 Answers

976. Ymln= 1.32 when X=± 1; asymptote, x=O. 977. Periodic function with
period 2n. Ymln= i when x= ~n+2kn; Ymax=e when X=~ +2kn
.r-
r5-1
(k=O, ±1, ±2, .•. );points of inflection, Mk ( arcsin---+2kn, e
- Vs-t')
2
2
Jf5-1 Vs+t\
and Nk ( -arc sin - 2- + (2k + 1) n, e 2 ) . 978. End-points, A (0, 1)
and B (1, 4.81). Point of inflection, M (0.28, 1.74). 979. Points of inflection,
M(0.5, 1.59); asymptotes, y=0.21 (as x-..-oo) and y=4.81 (as X-++oo).
980. The domain of definition of the function is the set of intervals (2kn,
2kn + n), where k =0, ± 1, ± 2, . . . Periodic function with period 2n.
n
Ymax=O when x=
2 + 2kn (k=O, ±I, ±2, •.. ); asymptotes, x=kn.

981. The domain of definition is the set of intervals [ ( 2k - {) n,

( 2k + ~) n] , where k is an integer. Periodic function with period 2n.


Points of inflection, Mk(2kn, 0) (k=O, ±1, ±2, ... ); asymptotes,
n
x= ± T+ 2kn. 982. Domain of definition, x > 0; monotonic increasing

function; asymptote, x=O. 983. Domain of definition, I x-2kn 1 <;


(k=O, ±1, ±2,
... ). Periodic function with period 2n Ym 111 =1 when
n
x = 2kn (k = 0, ± 1, ± 2, ... ); asymptotes, x = + kn. 984. Asymptote,
2
y= 1.57; y -+-1.57 as x ..... 0 (limiting end-pomt). 985. End-points,
I

A1, 2 (±1.31, I 57); Ymin=O when x=O. 986. Ymin=(f )e ::::::0.69 when
I
x =-::::::0.37; y -+1 as x ++0 987. Limiting end-point, A (+O. 0);
e
Ymax = e e:::::: 1.44 when x = e:::::: 2. 72; asymptote, y =I; point of inflection,
M,(0.58,0.12)andM 2 (4 35,140). 988.xmin=-lwhent=l(y=3);ym 111 =-l
when t =-I (x = 3) 989. To obtain the graph it is sufficient to vary t from 0 to 2n.
Xmin =-a when t =n (y=O); Xmax=a when t =0 (y=O); Ymin ~-a (cusp) when
3n n .
t=+y (x=O); Ymax=+a (cusp) when t=T (x=O); points of inflection
n 3n 5n 7n ( a a )
when t=-;r' 4 ' 4 ' 4 X=± 2Y.2' Y=± y·2 .
I I
990. Xmin=-- when t=-l(y=-e);ymax=- when t=l(x=e); points of
e e

inflection when t = - ¥2, i.e., ( - ::;.~, - V2eV2) and when t= V2,

- y'2'
( f2i'l, ev 2 ); asymptotes,x=Oandy=0.99t. Xmin=l andym 111 =l
i.e.,

when t=O (cusp); asymptote, y=2x when t -++oo. 992. Ymin=O when t=O.
Answers 415

.,93
u • dS =a
- d X,
y
COSil=-;Y
a
st'tt,.. x
= __
..,._
a
994. ds=-
a
r ya a-x
4
-c2 x2
2 • dx;

a Y a2 -x2 bx .r--
~os a= ; sin a=- , where c= r a 2 -b 2 • 995. ds=
Ya 4 -.c
2 2 X Va•-c•x•
1 . r2 - -2
=- r p +y dx; cos a= .r y .
; sma= .r p . 996. ds= v· a .
-dx,
J' p• + y• f p2 + yz X

; v~ .
y

cos a= v~ sin a= - 997. ds =cosh!... dx; cos a= _l_x ;


a cosh-
a

sinao...=tanh ~. 998. ds=2asinfdt; cosa=sinf; sina=cosf. 999. ds=

=3asintcostdt;cosa=-cost; sina=slnt. 1000. ds=a YI+!Jl1 d!p; cos~=


a .r-- 1 a
=y- I 2 .1001.ds=--ar I+IP d!p;cos~=- .r
2
.1002.ds=--d!p;
1 + IP
2
!Jl 1
r + <J! cos• _!!:
2
. (p
Sill~= COS 2. 1003. ds=aCOS ~ d!p; Sin~=COS ~. 1004. ds=
1 a
2
=rf1-j-(lna) 2 d!p; sin~ 1005. ds==-dlj); sin~=COS21jl.
Y I + (In a) 2 r
I a b 6
1006. K ~ 36. 1007. K = 3 ¥2. 1008. KA = b2; KR=(i2. 1009. K = 13 Yl3 .

1010. K-= a :.
2
at both vertices. 101 t. ( ~ , 3) and ( ~ , -3) •

In 2 V2 ) . _, (I +fix9x4 ) 3/ 2 1. _ (b'x 2 + a4y 2 )3f2


1012. ( - 2 , ~ 1013. R- 1014. R- a'b' .

I 4 I)., . 1016. R = I2'3asm. '2t I.


1015. R ~= (y• +y 1017. R =I at I. 1018. R=

=I r Yi+FI . !019. R =I} a cos ~ I· 1020. "'!e.1st =I p 1. 1022. (2,2).

t023. (-~a, 3 a). 1024. (x-3)


16 2
+(y-%Y=!. 1025. (x+2) 2 +

+(y-3) 2 =8. 1026. pY 2 =~(X-p) 1 (scmicubical parabola). 1027. taX) 1


+
2 4
1 1
+(bY) = c , where c2 = a2 -b 2 •
Chapter IV
In the answers of this section the arbitrary additive constant C is omit-
5 x4
ted for the sake of brevity. 1031. 7 a 2x7 • 1032. 2x 1 +4x 2 +3x. 1033. T+
/1-1

+ (a+b)
x' + abx' . 1034. a x+ T
2 abx4
+-y. b2 x1
1035.
2x .~
r 2px. 1036.
nx 11
n-l •
3 2 3
416 Answers

& 5 2 7

1037. JY -nx. 1038. a1 x-


g-
5
1
-
1
a x
g--
+]a 1 x
1
_
3
x•
. 1009.

1040. --13--
sx• v-x ax• v-x av- 7 6
x. 104 1.
2x2m
4m
-vx
+1
4xm+n Vx
2/1i + 2n + 1 +
2x•n Yx
4n +1 ·
1042. 2a .r- .r- 1
1
r ax-4ax+4x r ax-2x1 + 2x
X
.r- 1043. y"1 arc tan ¥1"
5 r ax
1044. 1 ~x- YV1o\ .
.r_!n 1045. In (x+ . r -1).
r 4+x 1046. arc sin x
2 r 10 x+ 10 2¥2 .
1047. arc Sin n-ln(x+ Yx 2 +2). 1048*. a) tanx-x. Hint. Put tan 2 x=
1
= sec 2 x-1; b) x- tanh x. Hint. Put tanh 2 x= 1 h2 • 1049. a) -cotx-
cos X

-x; b) x-coth X. 1050. ln(;er I . 1051. a In Ia c X I· Solution. 5a a X dx=

=-a5~(a-x)=-alnla-xl+
a-x
alnc=aln\-c-\· 1052. x+ln12x+11.
a-x
Solution. Dividing the numerator by the denominator, we get ;;!~ =
2
= I + 2x + 1 . Whence
5
2x + 3
2x + 1 dx = dx +
5 5 2 dx
2x + 1 = x +
5 d (2x + I)
2x + I =
3 11 x a
=x+lnj2x+li. 1053. - x+ 1nJ3+2xj. 1054. b-b2 1nla+bxl.
2 4
a ba-a~ x• x1
1055. ax+~lnlax+~i.1056.2+x+2lnlx-11. 1057. 2+2x+
x4 x•
+In I x+31. 1058.
4 + 3 +x +2x+31n I x-11. 1059. a•x+2ab In! x-al-
2

b2
x-a 1060.,
1
lnlx+1i+x+l" Hint.
5(x+J) = s
x dx
2
(x+ 1) - I
(x+ 1) 2 dx=

=5x~l-5(x!xt)•· -2bYl-y. 1061.


1062. -~·Y(•-bx) 8 •
1
1063. Vx•+t. Solution.J;dx =2 si·+l)=Vx2 +1.1064.2Yx+
. x2 + 1 x2 +I
+ 1nax
- 2 . 1065. .r- 1 arc
tan x ,V/ 3 I In x ¥7-2¥21
-5 . 1066. ,,......... .r ,r . I
rl5
1067. ,r I
2 r a2 -b 2
Va+b+x Y
In · ,r
r a+b-x
Ia-b
4r14
Ya-bl . 1068. x- .r-
xr7+2r2
x .
r 2 arc tan ,r-
r 2
1069. - ( ~· + ~~ In Ja 2 -x 1 J) 1070. x- fIn (x 2 +4) +arctan ~ .
1 ,r- .~ 1 ,/5 .
1071.
2
Y'21n (2 r 2x+ r 7 +8x 2). 1072. V'5 arc sin x Jf
7
1073. 3
l I 5 IX ¥"3--
1nl3x-21- YG1n xYJ+Y .1074.
2 2
¥21 Y335 arctan ,/5
V 1 x-
Answers 417

-i-ln (5x2 +7). 1075. ~ V5x 2 + I+ ;filn (xVS+ V5x 2 + 1).1076·. Vx"-4 +
+31nlx+rx 2 -4l.
.r-- 1077.
I
1njx1 -5j. 1078.
I
tn(2x 2 +3).
2 2
4
I I ax I x I

r
1079. - In (a 2x1 + b2 ) +-arc tan -b . 1080. - arc sin - 2 • 1081. - arctanx'.
2a a 2 a 3

1082. } In I x' + V x"-1 1. 1083. f V (arc sm x)'. 1084.


(arc tan i
4
1085. ! In (I+ 4x 2) - V(arc ~an (1'
2
1086. 21/ln (x+ fx1 + x2).
a -rnx 1088 . _:~_I_ 4•-ax l
1089. e l +e-. 1090. a e- a +2x-
1087. -me 1114 2
a -~ I (a" b")2
-2e a. 109 t.tna-lnb fiX-ax - 2x. 1092 · 31na Va'x+ Ina Va" ·
2 -

1093. - 2eX2+1 •
1094. 1 7""
21n 7 ·
1095. -e". 1096. ~
In 5
5 v-x ·
X 4

1097. In I ex-11 . 1098. - -~


,lb
V(a-be")' 1099. ~a (ea + 1)3. 1100.
1
- 31 ~ 2 ln(2"+3) Hint.
2,.~ 3 ,. ~ ( 1- }: ).
2 3
1101.
111
a arctan(a").

l102. I
-2bln )' +e-hx
l-e-hx. I 1103. arcsine1 1104. I
-bcos(a+bx) .
x
1105. •rr-2 ~111 y'_ tt06. 1
x-2a .r-.
cos 2ax. tt07. 2 sin r x. 1108. -In !Ox
2
X si•l 2~ . . l X
I I to.
x cos (log x) 1109.
2 - -4- . Hmt. Put sm• x =
2 (I-eos 2~). 2 +
sin 2t I cot ax
+ --
4
Hint. See hint in 1109 lttt. -tan(ax+b)_ tt12.
a
----x.
a
t113. alnjtanil· 1114. ~ln,tan(?;+:)l· titS.! lnltan uxtb
I ,r-2-

I I
1116. - tan(x 2).
2
1117.
2
1
cos(l-x). 1118. x-V_ cotxr
2
-V21n)tanx~2~.1119. -1n1cosx1. 1120. lnlsinxl. 1121. (a-b)X

I
xln sin a:_b I· tt22. Sin Isin~). lt23. -2ln I cos
a
Vx 1. tt24. ; lnx
x 6x Sl'J 4
xjsin(x +1)1.1
lt25. lnjtanxj. tt26. 2sin 2 a. lt27. ~·
I I I .r---
l128. • - - 1n(3+cos3x).
1129. 1130. - - r C<s2x.
4a su1 4 ax 3 2
x 2 .r--
1131. - g2 .r
r (1+3cos1 x)'. tt32.
3
4 tan';r. 1133. r tan•x.
3

1134. - 3 co~ 'x.1135.~ (tan3x+coslax)· 1136. ~(tn)tan'¥)+2smax)·


t4-t900
418 Answers

I 2 3 . x I
1137. 3ii In 1b-a cot 3x 1- 1138.
5 cosh 5x - 5 smh5x. 1139.- 2 + 4 slnh2.¥.
I
1140. In tanh ]- I· 1141. 2 arc tan ex. 1142. In I tanh xI . 1143. In cosh x.

1144. In 1sinhxl. 1145. - 5


12
v (5-x 2)'. 1146. 41 1nlx'-4x+ 1 . tl47. 1
4
I ys ."<
xarc tan f~. 1148. - ~ e-xz. 1149. Y~ arc tan ( x V'!)-
2
- 1._ln(x Y3+ Y2+3x 2 ). tl50.
x' - x +x-21nlx+ll.ll51. -}--.
Jf 3 3 2 r ex
1
(1nlsec3x+tan3xl+~ 3 3
1
tl52.lnlx+cosxl. tt53. sm x ). 1154. - - - -.
1nx
1155. ln 1tan x+ Y tan x-21 . 1156. Jf2 arc tan (x Y2)- 4 (2x! + 1) .
2

1157.
a s• n x
""fii(i'". 1158.
Vtx• + I) . 1
--2-. 1159. 2 arc sm (x ). 1160. a tan ax-x.
1 1 2

116t. x
2
-sin . tan-.1163.alntan
-2x-.1162.arcstn-2
x I (2a+T
x n ) I.1164. 3 V (l+lnx)
4
4

1165. -21nlcosYx-11- 1166. {lnltani/- 1167. earctanx+


2
+ ln (I+ x•) +arctanx. 1168. -lnlsinx+cosxl.1169. V2ln
4
- x
tan Y"2-
2
I I
-2x-V2cos V2' 1170. x+~ 2 1n/;~~!l· 117t.1nlxl+2arctanx.

1112. e
sin zx
. 1173.
5 . X Y3 .r--
Y'3arcsm--+r4-3x 2 • 1174. x-ln(1+ex).
2
1
1175 . •r-= arc tan x
• ya-b .r-- 1
- b .1176.\n(ex + r e2x-2). 1117.- lnltanaxl.
r~-~ a+ a
(arc cos ~ )
2
2 t
1178. -~cos(; +IPo)· 1179. {ln~~=:~;j. 1180. 2
1181. -e -tan" 1 . ( sin 2
. 11 82. 2 arcsm }72 .1183. -2cot2x. 1184. (arc
2
x)
sin x) 2 -

-Y1-x 2 • 1185. ln(secx+Vsec 2 x+l). 1186.


4r 5
~r-lnl rr 5
+<~in 2 x
r 5-sin2x

1187. :
2
arc tan ( t;;;) . Hint.
dx
S1 + cos 2 x
s dx
sin1 x +--=-2-co-s2-x-
dx
f ccsix
==Jtanzx+2' 1188. 3
2 y l In(x+ .r
r l+x'))'. 1l89. i sinh (x + 3).
1

1 Y2 whenx> .r-
1190 . - 1 3tanh x . 1191. a) .r7.arccos- r 2;
1n 3 r ~ x
Answers 419

c)
8~(5x -3) ;
1 8 d) ~ V<x+1)'-2y"X':j:l; e) ln(sinx+YI+sin 2 x).

tt92. ! 2
[< x];
5
>u- 5 (2'\T 5lu]. 1193. 2 ( v;- ~ +2 Yx~2ln1I+ ¥Xi).
tt94. In Irr2x+l-l I.
y 2x+l+l
1195. 2 arc tan Ye-" -1. 1196. In x-ln 21n lin X+
+2ln21. 1197. (arc~tlx)'. 1198. }<e-"-2) Ye-"+1. 1199. -}<cos•x-5)X

X YC<i'S'X. 1200. In I 1
I+Vx"+l
/· Hint. Put x=-tl . 1201. -i Yl-x + 2

.
+ 21 arcsmx. x• .r-- 14 . r - -2 .r-,--.
yx-a-
1202. -
3
r2--x-
3 r2-x. 1203.

-aarccos~.
X
1204. arccos..!.., if x>O, and arccos(-..!..) ifx<O*) Hint.
X X

Put x=+. 1205. Y x'+ l-In 1 1 + ~7+1~. 1206. - Y~. Note. The
1
substitution x = - may be used in place of the trigonometric substitution.
z
x .r--2 I .r- x .r--.
1207.
2 y 1-x +2' arcs!nx. 1208. 2arcsln
y x. 1210.
2 y x•-a +
+ 2a lnlx+ .y r
1
--
x•-a•1.
I
1211. xlnx-x. 1212. xarctanx- 1n(l-f-x').
2
1213. x arc sin x+ Y 1 x2 • 1214. sin x-x cos x. 1215. x si~ X +en~ X •
3 3

x+1 xln2+1 e""'


1216. --ex-· 1217. - 2 x1fi2T· 1218. 2f (9x 2 -6x+2). Solution. In place
of repeated integration by parts we can use the following method of undeter·
mined coefficients:

or, after differentiation,


x'e"' = (Ax 2 + 8x +C) 3e3-" + (2Ax + 8)e'"'·
Cancelling out e'-" and equating the coefficients of identical powers of x, we gcf:
I =3A; 0=38 +2A; 0=3C+ 8,

whence A=a; 2
I 8 = - -2 ; C=ijj· In the general form, s Pn(x)ea-"dx=_
= Q (x) eax, where Pn (x) is the given polynomial of degree n and Qn (x) is
a po1ynomial of degree n with undetermined coefficients 1219. -e-"' (x2 +5).
X
1
Hint. See Problem 1218*. 1220. -3e (x1 +9x2 +54x+162). Hint. See

*) Henceforward, in similar cases we shall sometimes give an answer tha\


is good for only a part of the domain of the integrand.
14*
420 Answers

Problem 1218*. 1221.


2 2 2
- x c~ x +sins x . 1222. x +
2
l~x + 11
sin 2x +

+
2
x: 5
cos 2x Hint. It is also advisable to apply the method of undeter·
mined coeffictents in the form
~ Pn (x) cos ~x dx = Qn (x) cos ~x + Rn (x) sin ~X,
where Pn (x) is the given polynomial of degree n, and Qn (x) and Rn (x) are
polynomials of degree n with undetermined coefficients (see Problem 1218*).
x8 x8 In x I
1223. ;rlnx- . 1224. xln 2 x-2xlnx+2x. 1225. -- x2- - xa·
9 2 4
.r- .r-
1226. 2 r x In x-4 r x. 1227. -
X
2
+- I arc tan x- x . 1228.
x
2
arc sin x-
I
X
2 2 2 4
X arcsinx+ ~ Yl-x 2 • 1229. xln(x+ Yl+x 2 )-Yl+x2 • 1230. -xcotx+
+In I sin x 1. 1231. -~+In tan
Sill X
I !...j.
2
1232. eX (sin x--cos x)
2
3x(s!nx+cosxln3) eax(asinbx:-bcosbx) ::_ . (I )-
1233. l+(ln3)• . 1234. a•+b• 1235 . 2 1sm nx
e-xz v- ( x3
-cos(lnx)). 1236. - --(x2 +1). 1237. 2e "(Jfx-1). 1238. -x 2 +
2 3
x 8 x• x2 - l 1-x ln 2 x 21nx 2
+3x ) Inx-g-+ 2 -3x. 1239. - 2-ln 1 +x -x. 1240. --x---x--x.
x8 x2 I l+x2
1241. (In (In x)-1]·1n x. 1242. arc tan 3x-T8+ 1n(9x2 +1). 1243. - -x
3 162 2
l
x(arc tan x) 2 -x arc tan x+ 1n (1 +x 2). 1244. x (arcsin x) 2 +2 Yl-x 2 X
2
xarcsinx-2x. 1245. arcsinx+ln\
x
x
1+ Yl-x 2

1246. -2Vt-xx

X arc sin ~rx+2 ~r-x. 1247. x tan 2 x+ In I cos 2x I xz 1248. e-"


r r 2 4 2· -2-x
x(cos 2x ~2sin 2x I). 1249. i + xcos (21n x)i;x sin (21n x)
X I X~
1250. - 2 (x•+ 1)+ 2 arcfanx. Solution. Putting u=x and dv=(x 2 +l)z'
1
we get du=dx and v= 2 (x•+l)' Whence (x 2 +l) 2
x2 dx x
2 (xz+l) +
s
dx x I 1 ( I x
+ 2 (xz+l)=- 2 (x 2 + 1)+2arctanx+C.
S 1251. 2az a-arctan-a+

+ x•~aa). Hint. Utilize the identity 1 aa ~~ [(x1 +a2 )-x2 ]. 1252. ix


~r-- a• x
2 arc sin ii". Solution. Put u= Ya• x and dv=dx; whence
-s
2
X, a1 -x1 +

ilu==-yxdx
~-~
andv=x;wehave f Ya• x2 dx=xYa2 -x1 .rxadx
r~-~
==
•1 y"•-x•-J<a•-x•>-aS,:tx=; Va•-x•-J Ya1 -x1 dx+a2J dx .
Va•-x• Ya -x 2 1
Answers 421

Consequent! y, 2 SYa -x dx = x Y a --x +a


2 2 2 2 2
arc sin ~ . 1253 .. ]- VA + x 2
+
A .r--
+2ln 1x+ r A +x2 J. Hint. See
x . r -2
Problem 1252*. 1254. -
2 r 9-x +
+
9 . x
arc sm . Hint. See Problem 1252*. 1255. I arc tan -x+- I . 1256. I X
2 3 2 2 2

I I
x 2 6x-l 1 "
x In x+ 2 . 1257. VII arctan y'"TI . 1258. 2 In (x -7x+ loJ)+ y'":3 x
2x-7 3
2

5
7

X arc tan V"3 . 1259. 1n (x 2 -4x + 5) +4 arc tan (x -2). 1260. x- In (x 2 +


2 2
9 2x+3
+3x+4>+y arctan y"1. 1261. x+3ln(x 2 -6x+l0)+8arctan(x-3).
7
1
1262. V arc sin~. 1263. arcsin(2x-l). 1264.Jnl x+f+ Yx 2 +px+q I·
1265.
2
3 Vx 2
-4x + 5. 1266. -2 Y l-x-x 2
-9 arc sin W,
1267. ~ V5x 2 -2x+ 1 +
5
:.
5 In ( x ¥5- ~S + Y5x 2
-2x+ I).

1268.lnl
1+
V \·l-x 2
1269.-arcsm
2

x r 5
~;_.1270. arcsin
(1-x)
2
-V 2
(x> V2).

1271. -arcsmx~l· 1272. xtl Vx'+2x+5+2ln(x+I+Vx 2 +2x+5).


2x -I . r - -2 1 2x + I .r .
1273. - - r x -x +B arc sin (2x-l). 1274. - 4 - r 2-x-x·+
4
+-arc sn1 2x+l
9
-- . 1275. -1 In lx'-31
- , -1 . 1276. 1
- .r- arc tan 3-~t•IX
.r .
8 ,1 4 x- r3 rJ
1277. In ( e-" +{ + V1 +e-" +e2-") 1278. -lnJcosx+2+ Vcos 2 x+ 4CI•SX + IJ.

1279.
· . 2+lnx
-V1-4lnx-ln 2 x-2arcsm V . 1280.
1
o-bln x+a.
/x+bl
5
1281. x+3lnJx-3J-3lnJx-21 1282. ..!..tn/(x-ll\~+4 3 1'1
12 ix+2) •
I 161

1283 • )nl (x-1)4(x-4)sll284


\X+ 3)7 • •
5x+ln x •(x-4) •
.2_ .
1285 • 1 +X+
I I
n
IX+x I
1 '
1
(x-1)

1286.
I I
4x+161n
I X
(2x-1)1(2x+1)"
11
j 1287.
2
x 11 8
2-(x-2)2- x-2 •
9
1288 · -2 (x-:-1)- 1
2 (x + 1) ·
1289 8 27 30
• 49 (x-5) 49 (x + 2) + ,J43 In
Ixx-5
+2
1
'

1290. - -2 (x 2 _ 3x+ 2)2 • 1291. x+lnl Yx~+ll 1:~:1-


1
1292. x++ln

- { arc tanx. 1293. 5~ In I x-31-do In I x-11+ 6~ In lx +4x+5>+ 1 ~0 X 1


422 Answers

X arc tan (x+ 2). 1294. ..!..


6 In X,<x-x+
1
+ >' + .)._arc
f 3
tan ~r-
f a
1
. 1295, 4 .~-X
f 2
1
X!+ X ¥2 + 1 ¥2 X Y2 1 X
2
+ X+ 1 I
x In + -4- arc tan -1- 1 • 1296. -;rln xz-x+ 1 + 2 .r-
Jtr 2+ 1 r 3
X
xl-x -x
x'-1 x arctanx 2x-1
xarctanx¥ . 1297. 2 (l+xz)+ 2 1298. 2 (xz+ 2x+ 2)+
3
x+2 5 2x+l
+arctan(x+l). 1299. lnJx+IJ+ 3 (xz+x+l)+ y"3arctan Y"3-
3
I 3x-17 I 15
- 2 In (x2 +x+ I). 1300. 2 (xz- 4x + 5) + 2 1n (x8 -4x+5)+ 2 arctan(x-2).
-x'+x I I 2 1
1301. 4 (x+ 1) (x 2 + 1) + 2 1n I x+ 11--;rln tx + 1)+-;r arc tan x.
3 x 3
1302. - 8 arctanx 4 (x•-I)+T61n x+l . 1303.
x- I I I
15x5 + 40x' + 33x
48 0+xz)• +
. 15 x-1
+ 48 arctanx. 1304. x x2 _ 2x+ 2 +21n(x 2 -2x+2)+3arc tan(x-1).
1 1
x1 +81-ln I x'+ 11 ). 1306.
2 lnlxt-11-
1305. 21 (81nj
1 ~ • I +12x 1-¥51
4
y 5 . 1307. 13 + -3-
2 (x- 4)2 x- 4
+
4 lnjx +x -11-
-- ,/-In
2 r 5 2xt + 1 +

~(2tnlx'x~ ~-~·-x•~ 1 ).
1 1
+21n~~-~~· 1308. 1309. x 1 +
,x-21
+In x - l .
1
1nlxl-ylnlx
1310. 7
+11. Hint. Put l=(x7 +1)-x7 •
. 1 1 1 1
1311. In I xl- 5 tn lx + 11 + 5 (xs+ 1). 1312. 3 arc tan (x+ 1)- 6 arc tanx
5

x+1 1313 1 1 1 I 1 1
1314 ' -5x 5 +3x·--x-
x-2-. · !;l(x-1)' 4(x-l) 8 7(x-1)1'
- arc tan x. 1315. 2 Vx=I[ 1 3 12
(x-; l' + (x5 ) + x 1316. l~a X 1. 2
x[2;Y(ax+bJ' -5bV(ax+bJ"l 1317. 2arctan Yx+l. 1318. 6 Vx-
-3 v- .r-
x+2r X-61n(1+ Jl
h ;-)
X. 1319, 6 I> r 6 n /;$ 3
"fX y X-5 V X 5 -2
v- r+

+2Yx-3Vx -6vx-3tnl1 + Vx/+6arctan vx.


1320. In I(v~~ 1-
x+2+
I)Z
x+l
.;_arc tan
r 3
2
3
+I. 1321. 2 Yx-2 Y2x ¥7
Yx'-1
~arc tan r
,/'X
2. 1322. -2 arc tan
. r-
f 1-x. 1323. 2
1
(x-2) +2 In I X+
.r-.- 1 2
z + z + 1 2 2z + 1
+ 2 ,2z_ 1 ,
+r x -lj. 1324. 3 tn (z-W+y- 3 arctan Y 3 where

z= V x+ 1
x-l· 1325.
V 2x
---J:-.
+ 3 2x + 3 .r 1
1326. - 6- r x2 -x+I+ In(2x-l+
16
Answers 423

1
+2 'r x1 -x+1). 1327• -
1
+ 4x + 3x• .r-1
15
8
r -x
5
- .. 1328. ( i6 x- x1 +
5
24
1
6 x X
s)
.r- 5 . r -2
X r 1 + x1 - i6 In (x + r 1 + X ). 1329.
( 1x• + Bx3 ) •r1 -X 2 - 1-8
4 1
3
arc sin
1
x.
1 .r-2- - I I 2x-l .r 2 19
1330. 2 (x+ i)• r x + 2x- 2 arc sin x+ 1 . 1331. - 4- r x -x+ I +sin X
1 l-!rx 2 1 vx-•+1+1
x (2x-1+2 Yx2 -x+l). 1332. -2 -;r-~-.
,. I+ 2x 1
1333. - In 4
4 x-• + 1-1
V
- { arc tan Vx- 4
+ I. 1334. (2x'-;~.Vf+Xi. 1335. 11
10
(z-l)z
n zx+z+ 1
+
+ ...
Y3
+--g- arc tan }73,
2z+1 where Z= V3 1 AO~.
1336. 1 4+3x'

V .+
-8 x(2-t-x1 )'/• •
4
1337. -2 (x 1)1 • 1338.

1 • sin • x _ sin 5 x. 1 1 x 1 x sin• x


- 5 cos1 x. 1340 1341.
4 cos 2 - 3 cos 1 2 . 1342. - 2- -
3 5
1 I . I 1343• 3x sin 2x + sin 4x
s---4- 32 •
- 2sin 2 x - 21 n smx ·
x sin 4x x sin 4x stn 1 2x 5 1 . 1 .
1344. -
32 . 1345. 16 -----sr+~· 1346. 16 x+ 12 sm6x+ 64 sml2x+
8
I cot• x 2 1
+ 144
sin 1 6x. 1347. -cot x- - 3 - . 1348. tan x +
3 tan• x + 5 tan
5 x.

cot" x cot5 x tan• x 1


1349. - -- - -- . 1350. tan x + - - - 2cot 2x. 1351.
2 tan
1
x+
3 5 3

-I 3 In 1tan x 1- 2 tanz 4
1 x . 1352. ~
3 x- tan
4
1 + 21n tan XI
2
I . 1353. -V2
- X
2
cos 2

X [In ltan~ l
l+ln tan ( ]-+ ~) IJ. 1354. 45~~!: -!~~~~:+~In Itan-i l·
1355.
sin 4x 3 sin 4x 3
lBcos• 4x+ 32 cos!4x+ 32 tn tan 2x+ 4
n ) I (
. 1356. 51 tan5x-x. I
cotz x 1 3 x
1 tan 2 +
1357. -
2 --lnjsinx!.
1358. -
3 cot x+cotx+x. 1359. 2 3
3
x x I I
+tan 1 x -3tan;r+3ln cos 3 +x. 1360. 4 - -8-
x• sin 2~1 coP x
1361. - - - .
3

1362. - f V cos• x +-} V cos•• x- ~ V cos 11


x. 1363. 2 Y tan x. 1364.
2
~ x
2
.r- 1 .r-
xln z•+z r 2 + 1- , 1 - arc tan~,
1
where Z= Vtanx. 1365.- C~'sBx +
z•-z(2+1 r 2 Z -1 16
cos 2x sin 25x sin 5x 3 5x x 3 x
+- 4- .
1366. ------w--+
10 . 1367. 5sin 6 +3sin 6 . 1368. 2
ct1s -
3
- ~ cos x. 1369• sln ~ + x co~ 2b • 1370 • t c~s <p _ stn (~ + <p) • 1371 • s•; x +_
4
424 Answers

tan.:..-2
sin 5x si'l 7x 1 1 1 1 2
+2J+2B. 1372. cos6x- cos4x- cos2x. 1373. 1n
24 16 8 4

~ tnjtan(]-+~)l·
1374.
2 1375. x-tan~. 1376. -x+tanx+secx.

tan.:._-5
1377. In ~ . 1378. arctan(1+tan ~)· 1379. Hx-~lnj2sinx+
tan -3
2
+ 3 C~'S x Solution. We put 3 sin x + 2 cos x =a. (2 sin x + 3 cos x) +
+~(2swx+3cosx)' Whence 2u-3~=3, 3a+2~=2 and, consequently,
12 5 s3sofJx+2c!'SX 125 5
a= 13 , ~=- 13 . We have 2 s,ux+ 3 cosxdx= 13 dx- 13 x

X
(2 ~i'l X+ 3J •"'"" v)' dx=-
12
x-
5
tnj2sinx+3cosxl. 1380. -lnfcosx-sinxf.
S2 ~~ I X -j- 1: >< 13 13
1381. -}arc tan ( ta~ x) Hint. Divide the numerator and denominator of the

fraction by cc:s 2 x 1382. ~!5 arctan ( J(~~n x). Hint. See Problem 1381.

. 1383 . _l_ln 12 t~n x+3- V_G,.


Hint. See Problem 1381. 1384. -1 In X
V3 2tanx+3+VU 5
tan v -5~ 1
X t
au x Hint. See Problem 1381. 1385. - ( -cosx )•· 1386. ln(l +sln 2 x).
2 1
I Jf2+~in2x 1 5-~inx 2
1387. .r-1n J( . . \388. - 1n - - . 1389. .r- arc tan X
2 r 2 2- Sill 2x 4 1-sm x r 3
' 2tan~-1 3tan.:_-l
2 1 2 identity
X --.r-- - .r- arc tan .r Hint. Use the
r:i r2 2r2
X
tan
1 2
. 1390. -x+ 21n . Hint.
(2-so.t x) (J-sm x) 2- Slll X 3 -smx X
tan +1
2
. 1-sin x +c~'S x 2 cosh 3 x
Use the ident1ty =-1+ . 1391. - --coshx.
1 + s1n x -cos x 1 + st•l x-cos x 3
. ~+smh2x+sinh4x sinh'x _ -~+sinh4x
1392 8 4 32 . 1393 . 4 . 1394 8 32 .
x j I tanh 2 x
1395. In tanh
I +cosh x . 1396. -2 coth 2x. 1397. In (cosh x)- - 2-
2 .
X
coth' x 2 (3 tanh2+2)
1398. x-cothx--- . 1399. arctan(tanhx). 1400. y]arctan
3 5
,r
.~)1
2

l . h
2

-1
( smh
2
x sinh 2x x

h ==(sinhx+coshx).
4
. .
or V'5 arc tan e" r 5 . 1401.--- - - - - . Hint. Use the tdentJty
2
1 .r-
1402 . • r-ln(r 2coshx+
,y1 cosh2x).
---
·SHI x-cos x y 2
Answers 425

1403. xt 1
Jf3-2x-x 2 +2nrcsin x; 1
.
x . r - -2 9 . r -2
1405.
2 r 9 +x - 2 In (x+ r 9+x ),
+{In(x-1+ Jfx 2 -2x+2) 1407.

In=- xne-x +nln_ 1; 110 =- e-x (X 10 + lOx'+


1 v"2cx-1)
+ l0·9x 8 + ... + 10·9·8 ... 2x + 10·9 ... 1). 1431. Vf4 arc tan Y7
1432. In Y x2 -2x + 2-4 arc tan (x-1). 1433. (x
2
12
) +i- 1n ( x+x+}) +
2

1 1
+ 2 arctan(2x+l). 1434. 5 Jn
-.1--r
Jl xt+S' lx+3j
1435. 21n x+ 2 -x+1 2 -x+J.
1

1436. i(1nl Vx~~ 1 ~-x~t)· 1437. i-(x~~ 2 +j7 arctan; 2 )~


426 Answers

1438.
1 (
4
2x ..L x+ 1
1-x• ' In x-1 .
I I)
1439.
1 x- 2
6 (xz-x + l)• +6 x•-x +I+
1 2x- I

+ 2 2x-l
,r-arctan ,r-. 1440.
x(3+2
,r
Yx"> . 1441.
I 4 I
- - - 3 , r - -21
-.
3 P' 3 P' 3 1-2 P' X X X P' X X

1442. ln(x+{+Yx'+x+1). 1443. ¥2X-! tf<2x)'. 1444. -V:+


1
.
2
1445. y x-l . 1446. -2 ( V5-x-l) 1 -41n (I+ V5-x).
4. 1 -2x+ I
x I -./l-x2 I
1477. InI x+ J! X 2
- 11 Yx 1
_ • 1448. - 2 V I+ x•· 1449. 2 x
1
. xz+
xarcsm 1
.r- .
r 2
1450. ,rx-1 .
rx'+l
1451. -81 1n ¥4=7-21 -
x
1
,r-X
Br3
I
x ,r--
. 2 (x + I) I
.xarcsm --x:t='"4 . Hint. x•+ 4x='4
I ( 1 1 )
x+ 4 . x-
1452. 2 r x 2 - 9 -
- 9 1n I x+ ,
2
;-
r x• -9 1 . 1453. I6I (8x-l) ,r--
r x-4x 2 +
I
arc sin (8x-l).
64
1454. In I2x + I + 2
x
Yx + x + 1
2
I· 1455 . (x 2 +2x + 2)
3
Y x• + 2x + 2

_{x~ I) VA-" +2x+2-} In (x+ I+ Yx 2 +2x+2). 1456.


X

y·IX2=l)i
3x'
1457. ~In YI=Xi-ll·
3 Vl-x'+ I
I 1458. -
1
3 In I z -11 +
1 1
+-6 In (Z2 +z+ 1)- .r- arc tan
2z+1
P'
where
3
v·-3 ' Z=
Vl+x'
X
5
1459. -2 X
1 , r - -4- 3x s1n 2x sin 4x
'Xin(x+rl+x). 1460. s+-4-+32· 1461. lnltanxl-cot 2 x-

- 41 cot 4 x. 1462. 2Vlcotx) 1 ,


-cot x 5 (cos I x- 6) v--~-
I 463. I2 cos x.

1464 ·
crs 5t • 3 cos 5x 3
3
5x
- 20 sw' Sx- 40 sm 2 5x + 40 In tan 2
I I· 1465 · tan1 x tan 5 x
- 3 - + -5- ·
.1466. { sin2x.
X
1467. tan• ( i ~) I + +2ln cos ( ~ + -i-) I· 1468. - .J3 x

Han 2-l 1 (2 tanx)


X arc tan V3 . 1469. ,r- arc tan
,. to
,r- . 1470. arc tan (2 tan x+ I).
,. 10
I 1 2 (tan~\ 1
1471. 21nltanx+secxl-2cosecx. 1472. vaxarctan ¥3 )-y2x

·xarctan ( tan~)
V"2 .
1
1473. lnltanx+2+Ytan 2 x+4tanx+ll. 1414.ax
.r 1 I ~
X In (sin ax + r a2 + sm 2 ax). 1475.
3 x tan 3x +g- In 1cos 3x I . 1476. 4 -
_ x s1n 2x _cos 2x e1 " I 3 x1 .r
__ 4 lS · 1477 • 4 (2x-1). 1478 3e" • 1479. 3' ·In ,. 1-x-
Answers 427

I x• x 1
x .r-- .r--
_61nlx-11-18- 12-6. 1480. r l+x1 arctanx-ln(x+ r l+r).
I 3 x l 5 x l ln- x I I
1481. -sln---sln---s . 1482. t . 1483. nll+cotxl-cotx.
3 2 10 2 2 2 1 + anx
sinh 2 x .r-- 1
1481. - - . 1485. -2cosh r 1-x. 1486. 5 tncosh2x. 1487. -xcothx+
2
I x I I ex-3
+In 1sinhx I. 1488. 2ex- 4 + 4 1n leX-21. 1489. 2 arctan - 2- ,
4 4 I 1 + 2x w-zx
1490. 7 V<ex
+ 1)7 -
3 Vlex
+ 1)1 • 1491. In 4 tn 1_ 2x · 1492. - 21 n 10 X
x- + -I - ) 2 .r-x-
r e +l+ln .r Vex+! -1 ,
x( x -l+
2
In 10 21n 10 2
. 1493.
r ex+ 1 + l
1494. In Iy X
1 + xz
1- arc tan
x
X •
1
1495. 4 (xc arc sin_!_+ x• +3
x
2 vxa=-r) .
I ( 2
1496. x (coslnx+slnlnx). 1497.
5
2
-x cos5x+ xsln5x+3xcos5x+
5
2
+?scos5x-: sin Sx). 1498. i [<x -2)arc tan (2x+3) +! In
1
(2x1 +6x+5>-::-

-fJ· 1499. { Yx-x2 +(x-;)arcsin Yx. 1500. xl xl,


2

Chapter V
T 2 2u-l
1504. lii'2
2 . 1503. 3.
1501. b-a. 1502. v0 T-g 1505. 156.
Hint. Divide the interval from x= I to x=5 on the x-axis into subtn-
tervals so that the abscissas of the points of division should form a geo-
b
metric progression: X 0 =1, X1 =x0 q, x2 =)( 0q 2 , . . . , Xn=x 0 qn. 1506. In-.
a
Hint. See Problem 1505. 1507. I-eos x. Hint. Utilize the tormula
sinu+sin2u+ ... +sinna=~ [cos~-cos(n+~)aJ. 1508. 1) ~=
2sm
2
I d/ I --
=-filii; 2)db=TiJb· 1509. lnx. 1510. -Yl+.t4 • 1511. 2xe-x
4
-e-x•.
cos X I I 3
1512. .r-+-zcos 2 . 1513. x=nn (n=l,2,3, ... ). 1514. ln2. 1515.-- •
2 I' X X X 8
1516. ex-e-x=2sinhx. 1517. slnx. 1518. }. Solution. The sum sn=
I 2 n-1 -1(1 2 n-1) mayberegardedasthe mtc-
=--z+--z+···+-
n n n 2- = nn -+- n +···+- n-
gral sum of the function f(x)=x on the interval [0,1]. Therefore, lim sn=
n -+<X>
I
I I I
= Sxdx=-2I . 1519.1n2.Soluti on.Thesumsn=-- +-- + ... + - - =
n+ I n +2 n+n
0

==*(-l+n 1 1
2 ...
- 1 + -- -+
I+;&
+~)may
l+n
be regarded as the integral sum of
428 Answers
~--------------------~~~~------------------------

the function f (x) = ~ x on the interval [0,1) where the division points have
1
I

the form Xk=l+~ (k=l, 2, ••• , n). Therefore, lim sn=S dx = ln2.
n n~~ 1 +x
0
100 I 7 7 16 2
1520. p+l. 1521. 3. 1522. 3=333. 1523. 4. 1524. 3. 1525. -3.
1 2 9
1526. In
2. 1527. In B
3 . 1528. 35 1 - 32ln 3. 1529. arc tan 3- arc tan 2 =
15
I 4 n 1 " n n
1' . 1532. 1- YJ. la33.
=arc tan 1530. In
3 . 1531. i6. 4 . 1534. 2 .
. 1 1+¥5 ll 1 2
1535.
3
In - -- .
2
1536. a+ 4. 1537. 3· 1538. In 2. 1539. I-eos 1.

~ +~ .
1540. 0. 1541.
9 3 1542. arc tane-~. 1543. sinh 1 = ~ ( e- ~) .
1544. tanh(ln3)-tanh(ln2)=!. 1545. -~+! sinh2n. 1546. 2. 1547. Di-

verges, 1548.
1
1
p, if p< 1; diverges, if p:;;;:,: I. 1549. Diverges. 1550. ~ .
1
1551. Diverges. 1552. l. 1553. - - , if p>1; diverges, if pE:;.l. 1554. n.
p- 1
1555. Vs. 1556. Diverges. 1557. Diverges.
1558. 1; 2 1559. Diverges.
2
I . I n 1 I 2n
1560. In a. 1561. D1verges. 1562. 7i 1563. B . 1564. 3 + 4ln 3 1565. Y'3 .
3
1566. Diverges 1567. Converges 1568. Diverges 1569. Converges. 1570. Con·
verges. 1571. Converges. 1572. Diverges 157J. Converges. 1574. Hint. 8 (p, q)=
'/, I

=~ f(x)dx+~ f(x)dx, where f(x)=xP-•(1-x)q-•;since lim f(x)x 1 -P=l


x~o
1
0
and Jim (1-x)
,.
-qJ (x)= l, both integrals converge when 1-p<l and 1-q<l.
" -+I
I <r>

that is, when p>O and q>O. 1575. Hint. r (p) = ~ f (X) dx + ~ f (X) dx, wher~
0 I
f (X) =xP-•e-x. The first integral converges when p>O, the second when p i!
:t

arbitrary. 1576. No. 1577. 2 .r2


y
s
1
a

.r-t
y
dt. 1578
'
s
n
a

dt
y' 1 + Sll12 t . 1579.
In a

Jf
In a
dt
.
...
f (arc tan t) 9
1580.
5 l+t•
0
dt. 1581. X=(b-a)t+a. 1582.4-21n3. 1583.8-
2
YJn.
n n rr n .r- n
1584. 2-- • 1585 . • r- 1586.
2
.r . 1587. 1 -4- . 1588. r 3-- .
r 5 2 r I+ a2 3
1...91. In 7+2Y1
1
1589. 4-a. 1590. 1 1u 112.
, "
- - - . 1592. 1 + 4n . 1593.
na
5 9 2 8
Answe,., 42!}

n n e2 +3 I
1594.
2 . 1599.
2 -I. 1600. I. 1601. -
8
- . 1602.
2 (e" + 11. 1603. I.
«>

1604. ~b2'
a+ 1605. -/!-b2' 1606. Solution.
a+ r(p+I)=S xPe-"dx. Applying
0
the formula of integration by parts, we put xP = u, e-x dx = dv. Whence
du=pxP-• dx, v= -e-x
and
.
r (p + 1) = [ -xPe-"J:' + p ~ xP-•e-x dx = pr (p) (*)

lf p is a natural number, then, applying formula (•) p times and taking into
account that
..
r (!)= ~ e-XdX= 1,
0
we get:
r <P+ l)=pl
1 3 5 2 =2k 1's an even I
' .. · ( k-l) _::: 'f
1607 • I zk = ' 2·4·6 ... 2k 2 • t n
b
num er; zk+r=
2·4·6 . 21?
- il n= 2k + 1 is an odd number
-~·~·5 ... (2k+l)'
128 63n
lo=:H5; 1•o= 512 ·

1608. (p-l)l(q-l)l 1609 _!_s(m+..J. n+l\ H' t P t 1 2 -t


(p +q-l)! . . 2 2 , 2 ) . In, U S II X- ,
1610. a) Plus; b) minus; c) plus Hint. Sketch the graph of the integrand for
values of the argument on the interval of integration 1611. a) First; b) second;
I I 3 . I
c) first. 1612. 1613. a. 1614. . 1615. 8 . 1616. 2 arc Sill .
2 3
3
- 2 2 2 2 n2
t617. 2<l<f5. 1618.
9 <1<
7 . 1619.
13 n<l< 7 n. 1620. 0<l<a •
2
Hint. The integrand increases monotonically. 1621. {<I< ~"2. 1623. s=¥.
1624. I. 1625. ; Hint. Take account of the sign of the function. 1626. 4 !.
1627. 2. 1628. In 2. 1629. m2 ln 3. 1630. ncr. 1631. 12. 1632. ~p 2 • 1633. 4 ~.
2 32 n I I
1634. 10
3 . 1635.4. 1636.
3 .1637.
2 -3· 1638. e+e--2=2(coshl-1) .
•r- .r-
1639. ab [2 ,- 3-ln (2+ ,- 3)}. 1640.
3
na1 • Hint. See Appendix VI, Fig. 27.
8
1641. 2a2e-•. 1642. ~ a 2 • 1643. 15n. 1644. ~ In 3. 1645. I. 1646. 3na1 • Hint.

See Appendix VI, Fig. 23. 1647. a 2 ( 2+ ~). Hint. See Appendix VI, Fig. 24.
4 4 16 4 Jf3 32 4 Jf3 3
1648. 2n+
3 and 6n-
3 . 1649.
3 n - -- and
3
n + -- . 165o. n:af1.
3 3 8
430 Answers

1651. 3na1 • 1652. n W+2ab). 1653. 6na1 • 1654. ; a2 • Hint. For the loop, the
parameter t varies within the limits 0 E:;;; t E:;;;+ oo See Appendix VI, Fig. 22.
1655. i na 2 • Hint. See Appendix VI, Fig. 28. 1656. 8n'a1 • Hint. See Appen-
11 1 11 1
dix VI, Fig. 30. 1657. : • 1658. a 1 • 1659. : • Hint. See Appendix VI,
.
F1g. 33. 1660.
9
2 r~. 1661.
14-8
3
¥2 az. r~p 1
1662. (1-ea)"f•. 1663. a
•(1[3 + -¥3)
2- •
•rr 2. Hint. Pass to polar coordinates. 8 .r-
1664. 11 1665.
27 oo r 10-1).
1666. Yh•-a•. Hint. Utilize the formula cosh1 a-sinh1 a= 1.
1
1667. ¥2 +In (1 + Y2). 1668. Y1 +e 2- ¥2 +In ( Yf'.tei-t)( V2 + ) .
e
1669. 1 +! In ~.
1670. In (e + Ye -1). 1671. In (2+ y3) 1672. {<e1 +1).
2

a .r- e•b-1 sinhb 1 2


1673. aln-b. 1674. 2a r 3 . 1675. ln~ +a-b=ln-.-h-. 1676. - aT.
e - 1 sm a 2
1 1
4(a -b )
Hint. See Appendix VI, Fig. 29. 1677. ab . 1678. I6a. 1679.
--
1 + 4r~2 + nay
+ ~ Y1 +4r~ 2 ). 1680. 8a. 1681. 2a [Y2 +In (y2 + 1)]. 1682. ~f)+
In (2n+
3+ ¥5 a Yl +m' 5
+In . 1683.
2
m . 1684.
1
(4 +In 3}. 1685. na . 1686. 4 r~ab 2 .
2 30 3
1687. a:Jt (e1 +4-e- 2). : 1!1 • 1689.
1688. Vx=:. 1690. Vy= ~ Jt.
1
Jt }fi:rta 32 I 4 I 3
1691. Vx=
2 : Vv=2:rt. 1692. - -. 1693. 15na. 1694. 3np. 1695.
5 10 n.
1696. n;• (15-tfi.ln 2). 1697. 2n 2a 1 • 1698.
11
~ H.
2

1699. :~ nh 1a. 1701. a) 5n 2a 1 ;


b ) 6n I aa.• c) 9 I )fi). 32 na I . 8 I 1704 . 4 na.
6 (n -
1101 I
1702.
105
1703.
3 na. 21
1705. h ( AB+ Ab+aB +ab ) . 1706. -nabh - . 1707.
128 a. 1
1708. 8 na 2b.
3 2 3 105
2
3
I 16 - ( h \ 4
2 na h.1710. 3 a•. 1711.na y'"pq. 1712.r~abh i+3c2)·1713. 3 nabc.
2 1
1709.

1714. ;[Yt7•-t]; ~ r~a 2 [5Y5-8]. ms. 2n[Y2+1n(Y2+1)].


8 6

1716. n(Y5-t''2)+n1n 2 (Y2"+I). 1717. n[Y2+ln(l+Y2}].


vs+t
:rta 2 na1 12 n
1718. T (e1 +e-•+4>=T (2+sinh 2). 1719. 5 na 2 • 1120. a<e-1) (e 2 +e+4).
1721. 4n 2ab Hint. Here, y=b·± Va 2 -x1 • Taking the plus sign, we get the
external surface of a torus; taking the minus sign, we get the internal sur-
2
2 1 8
face of a torus. 1722. 1) 2nb 2 + 1Wb arc sine; 2) 2na1 + :rtb In + • where
s e 1-e
8=
Y a1 - b1
a
64na1
(eccentricity of ellipse). 1723. a) -3-; b) 16n1a 1 ; c)
32 2
na • a
Answers 43t

1 8
1724. ~ na 2 • 1725. 2na 1 (2- Y2). 1726.
18 2
: na • 1727.
a.~ ab 2 a2 b
My='2 r a1 +b2 • 1728. Ma=2; Mb=y. 1729.
a 3 2 - - 2
x=y= . 1730. Mx=My= a ; x=y= a. 1731. 2na 2 • 1732. x=O~
3 5 5
- a 2 +sinh 2 - a sin a - - - 4 - 4a
y=-;r sinh 1 . 1733. x =-a-; y=O. 1734. x=na; y= 3 a. 1735. x= 3n:
-y= 4bn. 1736. -x=y=
- 9 . 1737. x=na;
- -y= 5 a. 1738. ( o,o, a ) . Solu-
3 20 6 2
tion. Divide the hemisphere into elementary spherical slices of area da by
horizontal planes. We have da = 2na dz, where dz is the altitude of a slice.
a
2n ~ az dz
Whence z =
o
2
na• = 2a . Due to symmetry, x=y=O. 1739. At a dis-

tance of ! altitude from the vertex of the cone. Solution. Partition the
cone into elements by planes parallel to the base. The mass of an elemen-
tary layer (slice) is dmi=YlT.Q 2 dz, where y is the density, z is the distance
of the .
cuttmg plane from the vertex of the cone, Q= h' z. Whence
h

n S;: z• dz
z=
1
°
-nr 2 h
! h. 1740. ( 0; 0; + : a) . Solution. Due to symmetry,
3
x= y=O. To determine 2 we partition the hemisphere into elementary
layers (slices) by planes parallel to the horizontal plane. The mas5 of such
an elementary layer dm = ynr 2 dz, where y is the den<;i ty. 2 is the distance
of the cutting plane from the base of the hemisphere, r = Va 2 -z 1 is the
a
n ~ (a 2 - z1 ) z dz
3
radius of a cross-section. We have: z =
0
2
=sa. 1741. l=na'.
-na•
3
I 1 34 1 1
1742. la=
3 ab'; lb=:ra'b. 1743. l=15hb • 1744. la= 4 nab ; lb=4na'b.
1745. 1=-}n (R!-R:). Solution. We partition the ring into elementary
concentric circles. The mass of each such element dm = y2nr dr and
R.
the moment of inertia I =2nS r• dr= ~ n (R!-R:); (y=l). 1746./ =
1~ nR Hy.
4

R,
Solution. We partition the cone into elementary cylindrical tubes parallel
to the axis of the cone. The volume of each such elementary tube is
dV=2nrhdr, where r is the radius of the tube (the distance to the axis of
the cone), h = H ( 1-; ) is the altitude of the tube; then the moment of
432 Answers

R
Inertia l=y 52nH ( 1- ~) r dr= yn~'H, where 1
y is the density of the
0

ocone. 1747. I= ; Ma2 • Solution. We partition the sphere into elementary


~ylindrical tubes, the axis of which is the given diameter. An elementary

volume dV=2nrhdr, where r is the radius of a tube, h=2a ...


V/1- ':
a
a
is its altitude. Then the moment of inertia I= 4nay
Jl y 1- ': r 1 dr = ! na 5y,
a 15

!
0

where y is the density of the sphere, and since the mass M = na1 y, it fol-
2 - - 2
lows that J = Ma 2 • I 748. V = 2n 2a 2 b; S=4n 2ab. 1749. a) x=y=r; a;
5
- - 9 -
b) X=y= wP· 1750. a) x=O, y-=3
4 r
n
Hint. The coordinate axes are cho-
sen so that the x-axis coincides with the diameter and the origin is the
- ,,
centre of the circle; b) x= Solution. The volume of the solid-a double
3
cone obtained from rotating a tnan;.(le about ib ba~e. is Pqual to V r>bl!2, ={
where b is the base, lz is the altitude of the tnan[!le. By the Gul1lin theo-
-1 -
rem, the same volume V = 2n x b'1. where x is the d1stance of th-' centre
2
of gravity trom the base. \\hence x = 3h 1751.

1752. 2g ~ln (1+~).


c· 1753. X= ~~lll(t)t;
(r}
v av --- n2u 0
1754. s~ lO'm.
1755. v=~ lnC1 a bt); h=~ x (bt,-(a-bltJlna~btJ. 1756. A=

=]Y R H 2
Hint. The elementary force (force of grav1ty) is equal to the
2

weight of water in the volume of a layer cl th1cknPss dx, that 1s, dF =


=ynR 2 dx, where y is the weight of unit volumL' of water. Hence, the ele-
mentary work of a force dA ~ ynR• (H -x) dx, wh<>re x is the water level.
11
A= ~ yR 2 H 2 •
1757.
11 1758. A--o Y R'TM:::::::079-10'=079·!07
4
kgm.
1759. A=ynR H. 1760. A= mghh ; Aoo=mgR. Solution. The force acting
l+R
on a mass m is equal to F= k m~
where r is the distance from the centre
r
,
of the earth. Since for r= R we have F=mg, it follows that kM =gR 2 • The
R+h

sought-for work will have the form A= k ~~ dr = kmM ( ~ - R ~h)= ' 5


mgh R
""'--h-. When h=® we have A«>=mgR. 1761. 1.8·10' ergs. Solution.
l+R'
Answers 433

The force of interaction of charges is F = e0~ 1 dynes. Consequently, the work


X

performed in moving charge e1 from point x 1 to x1 Is A =e,e1 S::


%a

%,
=

=e0 e1 (..l..-_!_)=1.8·10 4 ergs. 1762. A=800 nln2 kgm. Solution. For an


X X1 2
iSothermal process, pv = p 0 V 0 • The work performed in the expansion of a gas
v,

Jr p dv =PoVo In Vo
from volume Vo to volume VI is A= VI. 1763. A:::::: 15,000 kgm.
Vo
Solution. For an adiabatic process, the Poisson law pvk = p 0 v!. where
v,
k:::::: 1.4, holds true. Hence A=SPoV~dv-: PoVo [t-(Vo)k-1].
vk k-I v 1
Vo
4
1764. A =3 nJ.LPa. Solution. If a is the radius of the base of a shaft, the !I

the pressure on unit area of the support p = J:.....


na
The frictional force of a
2
ring of width dr, at a distance r from the centre, is 1-L;' rdr. The work per-
a
formed by frictional forces on a ring in one complete revolution is

dA= 4nJ.LP 41!J.LP


~r 2 dr. Therefore, the complete work A=(i!X 5 a
r 1 dr= 4 1lJ.LPa.
3
1765. i
MR 2 ro 2 • Solution. The kinetic energy of a particle of the disk
0

mv 2 Qr 2ro 2
dK= =- dcr, where da is an element of area, r is the distance of it
2 2
from the axis of rotation, Q
M
is the surface density, Q = nR• . Thus.
R
Mro• 2 Mro"5 MR•ro• 3
dK = nR r da. Whence K=T r'dr=--- 1766. K= xMR•ro•.
2 2 4 20
0

1767. K=~ R 2ro 2 =2.3·10'kgm. Hint. The amount of work required is equal
. bh 2 (a+ 2b) h1
to the reserve of kinetic energy. 1768. P=s. 1769. P= - ---::::::11.3·101 T
6
t 770. p =abyJth. 1771. P = nR;H (the vertical component is directed upwards).
I hb 1 p kMm
1772. 633 gm 1773. 99.8 cal. 1774. M=-~gf em. 1775. a(a+l) (k Is the
3 2
gravitational constant). 1776. npat
8J.Ll . Solution. Q =
sa
v2nrdr = -2np
4
s
a
J.LI (a1 - r1 ) rdr=

= J.Ll [a•r•
np
2 y-T rt J npat
= SJ.Ll. a
0
5 ab'
1777. Q= vadY=aPiif
•• 0
2
0

Hint. Drawthex-axls

15-1900
434 Answers

along the large lower side of the rectangle, and the y-axis, perpendicular to
v,
it in the middle. 1778. Solution. S= 5! dv; on the other hand, :~=a,
t•,
whence dt=l.dv, and consequently, the acceleration time is t= .)\~=S.
a
a
X

Mx=-Si-<x-t)dt+tx=-~ [xt- ;]:+%x=~x(t--j-).


1
1779.
0
X

1780. Mx=- S(x-t) ktdt+Ax=~W-x ). 1781. Q =0.12 TRI: cal.


2 Hint.
0
Use the Jemle-Lenz law.
Chapter VI
2 2 2 .r~~~~
2
3 (x+y) r 4z -t-3(x-y)"·
2
S=
1782. V=
3 (y -x )x. 1783.
1 ) 5 2
y -x2 x2 -y 2 y•-x•
1784. f ( -2' 3 =-3; f(l, -1)=-2. 1785. xy- ,
-2 -2 xy- , .,-'
-~xy
2xy R' Yf+Xi
x•-y•· 1786. f(x, x2)=1+x-r. 1787. z=I-"Rz· 1788. f(x)= x

Hint. ~epresent the given function in tile form I( ~)= y (; r +I and

replace f by x. 1789.
2
f(x, y)=x -;xy. Solution. Designate x+y=u,

x-y=v. Then
2
u+.v
x= -- , y =
2
u-v
-- ; f(u, v)=-
2
u+v
2
- • u-v
- -+ - -
2
2
= (u-v)
· u•-uv
= -.-2- . It r&mains to name the arguments u and v, x and y. 1790. f (u) =
=u 2 +2u; z=x-l+Vy. Hint. In the identity x=1+f(Yx-l) put
Yx-l=U; then x=;=(u+l) 2 and, hence, f(u)=u 2 +2u. 1791. f(y)=
= V1+y 2 ; z= Vx 2 +y 2 Solution. When x=l we have the Identity
Vl +Y 2 =1·f ( t)' i.e., f(y)= Yl+y•. Then f ( ~ )= -v-1 +(~rand
z=x v r I+ ( *" = Y x•+y". 1792. a) Single circle with centre a\ origin,
including_ the circle (x 2 +y•.;;;; I); b) bisector of quadrantal angle Y=x; c) half-
plane lo"ated above the straight line x+y=O (x+y > 0); d) strip contained
betv.:_een the straight lines y= ± I, including these lines (-I <y.;;;;; I); e) a
square formed by the segments of the straight lines x= ±I andy=± I, includ-
ing its sides (-1 <;x.;;;; 1, -1 .;;;;y.;;;;; 1); f) part of the plane adjoining the
x-axis and contained between the straight lines y = ± x, including these lines and
excludi.ng the coordinate origin (-x<y<x when x>O, x.;;;;y.;;;;-x when
x<O); g) two strips x;;::;.2, -2,;;;;y.;;;;2 and x<;-2, -2.;;;;;y,;;;;;;2; h) the
ring contained between the circles x2 +y 2 =a 2 and x"+y 2 = 2a2 , including the
boundaries; i) strips 2n:rto;;;;;x,.;;;;;(2n+ 1) :rt, y ;:;::;.0 and (2n+ 1) no;;;;x.;;;;(2n+2) n,
y...;;;; 0, where n is an integer; j) that part of the plane located above the
A11swers 435

par!! bola y = - x• (x1 + y > 0); .k) the entire xy-plane; I) the entire xy-plane,
with the exception of the coordinate origin; m) that part of th!' plane located
above the parabola y 2 =x and to the right of the y-axis, including the poi-nts
of the y-axis and excluding the faints of the parabola (x:;:;,o, y > Vx\
n) the entire place except points o the straight lines x= I !lnd y = 0; o) the
family of concentric circles 2rck,.;;;;;x 2 +y 2 ,.;;;n(2k+I) (k=O, I, 2, ... ).
1793. a) First octant (including boundary); b) First, Third, Sixth and Eighth
octants (excludmg the boundary); c) a cube bounded by the planes x= ± I,
y= ±I and z= ±I, including its faces; d) a sphere of radius I with centre
at the origin, including its surface 1794. a) a plane; the level lines are
straight lines parallel to the straight line x+y=O; b) a paraboloid of revo-
lution; the level lines are concentric circles with centre at the origin;
c) a hyperbolic paraboloid; the level liMes are equilateral hyperbolas;
d) second-order cone; the level lines are equilateral hyperbolas; e) a parabolic
cylinder, the generatrices of which are parallel to the straight linex+y+ I =0;
the level lines are parallel lines; f) the lateral surface of a quadrangular
pyramid; the level lines are the outlines of squares; g) _!eve! lines are parap-
olas y =- Cx 2 ; h) the level lines are parabolas y = C Yx; 1) the level Hmis
are the circles C(t 2 +y 2 )=2x. 1795. a) Parabolasy=C-x2 (C>O); b) hyper-
bolas xy=C (I C 1,.;;;;; I); c) circles x +y =C ; d) straight lines y=ax+C;
2 2 2

c) straight lines y~Cx (x :f= 0). 1796, a) Planes parallel to the plane
x--t-y+z=O; b) concentric sphere~ with centre at origin; c) for u>O,
one-sheet hyperboloids of revolution about the z-axis; for u < 0, two-sheet
hyperbol01ds of revolution about the same axis; both families of surfaces
are divided by the cone x2 +y 2 -z2 =0 (u=O). 1797. a) 0; b) O;c) 2;
d) e1'; e) limit does not exist; f) limit does not exist. Hint. In Item(b)
pass to polar coordinates In Items te) and Cf), consider the variation of x
and y along the straight lines y = kx and show that the given expression
may tend to different limits, depending o-n the choice of k. 1798. Continuous.
1799. a) Discontinuity at x = 0, y = 0; b) all points of the stra1ght line
x=y (line of discontinuity); c) line of discontinuity 1s the circle
x 2 +y 2 = I; d) the tines of discontinuity are the coordmate axes.
1800 Hint. Putting y = y 1 =canst, we get the function '~'• (x) = xy, , which
2
x•--t- y~
is continuous everywhere, since for y 1 :f= 0 the denominator x 2 + y~ :f= 0, and
when y 1 =0, lf' 1 (x)=0. Similarly, when x=x 1 =const, the function
2
q> 2 (y) = x,y is everywhere continuous. From the set of variables x, y, the
x~+ y•
function z is discontinuous at the point (0, 0) since there is no lim z. Indeed,
X -+0
l/-+ 0
passing to polar coordinates (x= r cos cp,y = r sin q>), we get z =sin 2q>, whence it is
evident that if x-+ 0 and y-+ 0 in such manner that q> =canst (0,.;;;; q>,.;;;;; 2:rt),
then z- sin 2<p. Since these limiting values of the function z depend on the
direction of cp, it follows that z does not have a limit as x-+ 0 and y--+ 0.
az 2 az 2 az 2y az 2x
1801. a-~3(x -ay), a-=3(y -ax). 1802. -a =-(- ·- •. a-=--(--)•.
x y x x+y) y x+y
1803. az=- !/2 , i)z=_!_ 1804. az- X , ~=- y
ax X ay X ax Vx•-y• ay Vx•-y•.
1805. i)z y• az xy 1806 az- 1 az-
ax <x•+y•>·t. • ay <x2+u•)·t. · · ax- Vx•+y' · ay-
Y az y az X az
vx• + y• (x + vx• + y 2 ).
1807 - = - - -1 - - - -2 1808 --yxY-•
• ax x• +y • oy- x• + y ' • iJx- •
436 Answers

0 sin .1L • sin J!....


~Yz=xYinx. 1809. ..!=_JL,
1
xcos 1!.. oz=.!..e xeoslL. 1810. ~ ...
v OX X X ' oy X X OX
xy• Y2x1 -2y 1 oz yx 1 Y2x 1 -2y 1 oz l x+a
- IYI(x4-y4) ' ay=- lyl(x4-y4) . 1811. ax=yy- cot Yy'
g.z = - x ~ra- cot x.;~. 1812. ~u = yz (xy)z- 1 , ~u ==xz(xy)z- 1 , ~=(xy)z'tn(xy).
uy 2y y y y y uX uy OZ
&
'J813. ax=yzXYJnz,
&
oy=XzXYJnz,
&
az=xyzXJ-I.
'
1814. fx(2,J)=2 •
I

I z I
111 (2,

1)=0.
I
1815. fx(l, 2, 0)=1, fy(l, 2, 0)=2• f (I, 2, 0)=2 ·
x y x2
1820. - , • 1821. r. 1826. z =arctan-+ cp (x). 1827. z = - +
1
~+~+~' X 2
1 1
2. 4 ; 2) tana.-=oo,
tan~=oo,
1
+y lnx+siny- 1828. I) tana=4, tanv=
t as 1 as 1 as 1
tan ~=4, tan y=-:r· 1829. aa=2"h, ab='ih, ah=2"(a+b). 1830. Hint.
Check to see that the function is equal to zero over the entire x-axis and
the entire y-axts, and take advantage of the definition of partial derivatives.
Be convinced that t:(o, O)=f~(O, 0)=0. 1831. M=4~x+~y+2~x 2 +
+2~x ~Y+ ~x ~y;
2
df=4dx+dy; a) M-df=B; b) M-df=0.062.
1833. dz=3{x 1 -y)dx+3(y 2 -x)dy. 1834. dz=2xy 3 dx+3x 2 y 2dy. 1835. dz=
4 2 2 1836.dz=sln2xdx-sin2ydy. 1831.dz=y 2xY- 1dx+
(A 2 +112) 2 (xy dx-x ydl/).
.
(xdx+ydy). 1839.df=-+ (ax-~dy \ ·
2 1
+xY(l+ylnx)dy. 1838.dz= +
X2 y2 X y y 1
'184!). dz=O. 1841. dz=~ ( dy- ~ dx). 1842. df (1, I)= dx- 2dy.
X Slfi J!..
X

1843. du = yz dx + jX dy + xy dz. 1844. du Y xz+I y2+z2 (x dx + y dy + z dz).

1845. du= (xu+; r- 1


[ ( u+;) zdx+ ( 1- :.) xzdy + ( xu+f) In·~
x (xu+;) dz]. 1846. du= xzy:~ 24 (u dx+xdy- 2:Y dz )· 1847. df (3, 4, 5)--_.,
I
= t5dz-3dx-4dy). 1848. d/=0.062 em; ~/=0.065 em. 1849.75 em 3 (rela-
25
tive to inner dimensions). 1850. ! em. Hint. Put the diiTerential of ll~e area
of the sector equal to zero and find the diiTerential of the radius from that.
1851. a) 1.00; b) 4.998, c) 0.273. 1853. Accurate to 4 metres (more exactly,
4.25 m). 1854. :nag:-;~ 1 . 1855. da.=J..(dycosa-dxsina). 1856. _ddzt=
. g, lg Q
e' (t In t -1) du t x ( x ) du
-= t lnzt . 1857. di= YY cot yy· 6- 2y• • 1858. dt=2t lnt tan t+

_'Litfl+l)tant+(t2+1)\nt 1859 ~=O 1860 '!!_=(' )cosx( t ·-


-:r t cosz t · · dt · · dx sm x cos x co x
Answers 437

-slnx1nslnx). 1861. ~:=-x•~y•; :~= 1 +x•· 1862. :;=·vxY-•; :;=


y] iJz
=Xy [ <p'(x)lnx+X .1863. OX=2xfu(u,v)+ye"Yfv(U, v); y:c:-2yfu(U,V}+
• • iJz ,
0
+ xe"Yf~ (u, v).:~ =0, 1864 ;; = l. 1865. :; =y ( 1- ~~) f' ( xy + ~) ;

:; = ( x+ ~) f' ( xy+ ~). 1867. :~=f~(x, y, z)+q>' (x)f~ (x, y, z)+


+ f~ (x, y, z) (ljJ~ (x, y) + ljJ~ (x, 1873. The perimeter increases at
y) q>' (xl).
1
a rate of 2 m/sec, the area increases at a rate of 70m 2 /sec. 1874. 1+2t
Y +3t'4 .
1 + t• + t
187s. 20 V5-2 .ry 2 km/hr. 1876. - 9Y3 V2. 1879. - Y3-
- - . 1877. l. 1878. _ -.
2 2 3
68 Cf'S a + Cf'S ~ + C0S y . . .
1880. . 1881. . 1882. a) (2. 0), b) (0, 0), and (1, 1),
13 3
1
c) (7, 2, 1). 1884. 91-3). 1885. (51-3)). 1886. 6i +3J+2k. 1887.1 grad uJ=6;
4
2 2 1 3
cosa=a I cos~=- 3. cos v= 3' 1888. cos <p= VTO. 1889. tan <p::::::: 8.944;

q>::::::: 83037'. a•z =1891. abcu• . a•z = - abcxy . a•z =-


ax• (b•x•+a•y•>''•' axay (b•x•+a•y•)'l•' ay•
abcx 1 a•z 2 (y -x 2 ) . -iJ1-
1892 - = 2
z 2( a•z 1
wx• + ay•)''· .
2 ax• (X + y) 2 ' ax ay= -(x-1-+ -y)·2 ' ay•-- - - -1 - -1
(x + y) •
a•z
1893. a-a =
X Y
XI/
(2xy + yl) • 0
a•z
0
( y
a•r r 1 - X 1
•1 . 18!14. - - = o. 1895. -a•= - 1-
X T
a•u a•a
t896. a~=-a
X y
.=
a•tl
=--0,
_ . a•tl
--=--=--=1
a•u o•tl a•tl
1897. ---=aP.yx"- y~- zl-l 1 1
az• ax oy ay r)z az ax . ox ay iJz I' •

1898.
a•z 2
axay·=-.>. ycos(xy)-2xsin(xy). 1899. t: (0, 0) = m(m -1);
t: 11
(0, 0) = mn; t;,,, (0, 0) = n (n -1 ). 1902. Hint. Usmg the rules of differcn·
tiation and the defimtion of a partial derivative, verify that 1; (x, y) =
x•- 11 1 2 1
41 1/ ] •
=Y [.>.'+y'+(x"+y•)•
1
(when -t +y 1 :;40), fx(O, 0)=0 and, consequently,
that for x = 0 and for any y, (0, y) ~~- y. Whence t: (0, y) = -1; in par· t:,,
ticular, 1
(0. r:,
0)=-1. Similarly, we find that (,x\0. 0)=1.
iJiz - ' I ,, • , I , •
1903. "if:!-2fu(u, v)+4xfuu<u,v)+4.xyf 11 v(u, v)+Yfvl•(u. v),
a•z ' .
ox iJy =fv (u, v)+ 4xyfuu (u, v) +2 (x1 +y 1 ) fuv (u, v) + xyfr·v(u, v);
, "
(Jiz _
aiJ• -- 2{u (u.
'
v) + 4y zf•uu (u, v)
• •
+ 4xyf uv (u, v)

+ x•fvv (u, v).
au2 ,. ,. , " , 1 , ,
1904. iJ.A.I =fxx+2fi(l'Px+fu l'P.xl +fl'Pu·
438 Answers

iJ2z - n I I " I ' , I z , " I , •


1905. iJxl -fuu<'Pxl +2fuv'PxWx+fvv(Wxl +fu'Pxx+fvWxx•
iJ2 z n I I II I I I I " I I I " I "

ax iJy = f uu 'Px'Pu+ f uv (cpx¢y+Wx<i>y) + fvvWxW II+ f u 'Pxu + fvWxy;


iJ 2z II I
1
II I I n I
2 I '' I "

oyz = f uu (cpy) + 2f uv'PyWy + fvv (¢y) + f u 'Pyy + fv¢1/Y'


1914. u(x, y)=cp(x)+¢(y). 1915. u(x, y)=xcp(y)+¢(y). 1916. d 2z=e"Y X
x [(ydx + xdy) 2 + 2dxdy]. 1917. d2u = 2(xdydz + ydzdx + zdxdy).
1918. d2z=4q>"(t)(xdx+ydy) 2 +2cp'(t)(dx2 +dy 2 ). 1919. dz= (;)"Y x
x (ylnexdx+xln!._dy);
y ey
d2z= ( !._)"Y
\ y
[(y ln 2 1 !0.+
y
Y )dx2 +
x
+2 (xu In ex In~+ In!._) dx dy + (x ln ey.=__!._)
2 2
dy J. 2
y ey y y
1920. d2z = a2 f:u (u, v) dx 2 + 2abf:v (u, v) dx dy + b2 f~v (u, v) dy 2 •
1921. d2 z = (ye"f~+ e8Yf:u + 2ye"+Yf;w + y 2e2"f:v> dx 2 +
+2 (eYf: + e"f~+xe2Yf:u +e"+ Y (I +xy) f;,v + ye 2"f:v) dx dy +
+ (xeYf: +x2e1Yf:u + 2xe"+Yf:v + e2"f~v) dy 2• 1922. d 1 z = e" (cosy dx 1 -
-3 sin y dx 1 dy - 3 cosy dx dy 2 + sin y dy 1).1923. d1 z = - y cos x dx 1 -
-3sinxdx 2 dy-3cosydxdy 2 +xsinydy 1 • 1924. df(l, 2)=0; d2 f(l, 2) =
= 6dx 2 +2dxdy+4. 5dy 2• 1925. d 2f (0, 0, 0) =2dx2 +4dy2 +6dz 2 -4dxdy +
ya X
+Bdxdz+4dydz. 1926. xy+C. 1927. x•y-- +sinx+C. 1928. - - +
3 x+y
1 X X
+In (x + y) +C. 1929. - In (x 2 + y 2 ) + 2 arc tan-+ C. 1930. - + C.
2 y y
1931. Yx 1932. a=-1, b=-1, z= ~-+y 2 +C. 1933.x2 +y 2 +z 2 +
2 +y 2 +C.
X y
+xy+xz+yz+C. 1934. x1 +2xy 2 +3xz+y 2 -yz-2z--f-C. 1935. x 2 yz-3xy 2 z-+
+ 4x2 y 2 +2x+y+3e+C. 1936. ~+..!L+~+C. 1937. Yx 2 +y 2 +z 2 + C
y Z X
1938. "-= -1. Hint. Write the condition of the total differential for the
XI/

expression Xdx+Ydy. 1939. t;=f~. 1940. u=Sf(z)dz+C. 1941. ~=


a
b2x. d 1 y b' d1 y 3b 8x . . .
= --az • d--.= -""''i; -d1 = -45. 1942. The equation defimng y ts the
y x ay x ay
equation of a pair of straight lines. 1943. ddyx = y" In Y 1944 dy- _1!__ ·
1-xy"- 1 ' • dx-y-1 •
(d ~)
2
day1 _ _Y_1 1945. (dy) =3 or -1; =8 or -8.
dx - (l-y) • dx x=t dx x=t
1946. ~=x+a!l. day (aa+ I) (xa+ yz) dy y . day-~
d x ax-y ' d--.=
x· (ax-y )a 1947. d-=
x -- x' d x z- x 2 ·
8
~-6y 2 -3xz-2 ~=zsinx-cosy,
1948. oz x -yz2 1949 . ox oz
ih=xy-z ; oy- 3(xy-z1 ) · cosx-ysinz' a;,=-
1950 . ~=-l· 1951. iJz=_c 2x; ~=-c"11.
2
xsiny-cosz oz I
cosx-ysinz ox ' ay=2· ox a z iJy b2 z •
Answers 439

1953. dz
dx=

x y y 2 -a1 xy
..p~
1954. dz=- -dx - - dy;
z z
d2z = -1 - dx 1 - 2 -1 dx dy
z z
+
1
x -a1
4
+- -dy -
21
1•
1955. dz=O; d 2 z= l5(dx1 +dy 2). 1956. dz=
1
z
_
2
(dx+dy);
2 2
z (d 1 2d d d 2
d zz=(l-z)• dy . dz 1 . d y . 'd z 4
x + xu+ y). 1961. dx=oo, iiX=s· cJox•=oo, dx'=25 ·
y (z-x) z (x-y) 2 1 a
1962. dy=-(--)dx; dz= x (y-z )dx; dy =-dz = - a(
x y-z x y-z )• X
au au a•u a2u a•u
+ 0
+
x ((x-y)z (y-z) 2 (z-x) 2] dx 2 • 1963. -= -= I; a----"~ =a-a = ----z=O;
X 0y X XY 0y
~-- ·
ox- 1' ov
ay=O;
a•v
axz=2;
a•v
axay=1;
iJ2 v
oy• =0. 1964. du=l+ydx+
y

v I v 2
+ 1 +Ydy; dv= 1 +Ydx- 1 +Ydy; 1
d u= -d1 v=(-1-+-y-)2 dxdy-

2v d z 1965.
'lj)~dX-qJ~dy - 'lj); dX -j- qJ; dy
1 1
- ( I +y)2 y.
du= ~'ll~,'ll~~ dv= ~qJ~!j)~~
Wu Wv Wu Wv
1966. a) ~=-cs~nv' ~~=cc~sv b) ~={<u+u), ~={<v-u);
I
C) dz= e211 (e 11 - v(v+u)dx+e 11 +V(v-U)dyj. az I
1967. ax=f,(r, qJ)COSqJ-
2
Sin qJ , az COS qJ az
1
-Fcp(r, qJ)-,-, ay=F,(r,
I
cp)sincp+FIP(r, I
qJ)-,-. 1968. a;=
c az c . d2 y dy d1 y
=-aCOSqJCOt'lj); ay=-bSIOqJCOt'lj). 1969. ;nz+;n+y=O. 1970. dtZ =6.
d2 x dx d3x r
1971. a) dy 2 -2ydy=0; b) dy'=O. 1972. tan~=dr.
dqJ
, 2 + 2 (dr )•-r d r
2

az
dqJ d<p 1 az a•u
1973. K= [r•+(:~rr/• 1974. au=O. 1975. uau-z=O. 1976. arz +
1 a2u 1 au a•z 1 az iJw azw
+-a-z+-a-=o.
r• rp r r 1977. -uav = 2-a-· t978. -a =0. 1979. a-r=O.
0 u v v v
azw 1 x-1 y+2 z-5
1980. au• =2. 1981. a) 2x-4y-z-5=0; -2-= -4 = -1; b) 3x+4y-
x-4 y-3 z-4 x-R cos a
-6z=O,· - - 3 = - 4- = --6- ·• c) xcosa+ysina.-R=O, .
cos
== a.
y-Rslna. z-R a• b2 c3
_ - . 1982. ± .r ; ± ,1 2 2 ; ± .r ·
sin a 0 r a• +b2 +c• r a +b + ct r a• +b1 +c•
440 Answers

1983. 3x+4y+ 12z-169=0. 1985. x+4y+6z= ±21 1986. x+ y +z =


= ± y·a 2 +b1 -t--c 2 1987 At the points (I, ±I. 0), the tangent planes are
parallel to the xz-plane; at the points (0, 0, 0) and (2. 0, 0), to the yz-plane.
There are no points on the surface at which the tangent plane is parallel to
2 0
the xy-plane. 1991. n . 1994. Projection on the xy-plane: { ;'+ 2
3 x y -xy- 1=.
0

Projection on the yz-plane: { ;:=:; 22


_
1
=O Projection on the xz-plane:

jy=O
3x2 Hint. The line of tangency of the surface with the cylin-
\ 4 -t--z1 -1=0.
der projecting this surface on some plane is a locus at which the tangent
plane to the given surface is perpendtcular to the planE:' of the proJeclJOn
1996. f(x+h, y+ k)=ax 2+2bxy+cy 2+ 2 (ax+ by) h + 2 (b~ +cy) k -j--ah 2 +
+ 2bhk+ck 2 1997. f(x, y)= 1-(x+2l 2 +2(x+2)(y-1)+3(y-1)2.
1998. l'lf(x,y)=2h+k+h 2 +211k+h 2 k. 1999. f(x, y, z)=(x-1)2-j-(y-1) 2 +
+
+ (z-1)2 2(x-l )(y-1)-(y-1) (z-1). 2000. f (x + h, y + k, z+ l) =
= f (x, y, z)+ 2 (h (x- y - z) + k (y- x- z) + I (z- x- y)) + f (h. k, 1).
3..:zy-y' xz+yz x4+Gx2y2+y•
2001. u+xu+ .2002.1--- + . 2003. I+(y-1)+
31 21 41
+ (x- I) (y- 1). 2004. I+ [(.t- 1) + (y + !)] + [(x-1) -~! (y + 1)] +
2

+ ((x- 1Hlll+l)]' 2005 a) arctan 1 +a.._~+_!_(a+R)_.!...(a 2 -R')·


31 . 1- ~ ~ 4 2 t' 4 t' '

-./(I +a)m+(l +~)n =::::I+..!.. (ma+n~)+..!.. [(3m 2 - 4m) a 2 - 2mna~ +


b)
v
+ (3n -4n)
2
2 4 32
~ 2 ). 2006. a) 1.0081; b) 0.902. Hint. Apply Taylor's formula for
the functions: a) f(x, y)= Yx· Vi/
tn the neighhourhood of the point (I, I);
b) f (x, y) = y" in the neighbourhood of the point (2, 1). 2007. z = 1 + 2 (x-I)-
- (y-1)-8 (x-1) 1
+
lO (x-1) (y-1)-3 (y-1) 2 + ... 2008. Zmm =0 when X= I,
y=O 2009.Noextremum. 2010. Zmin=-1whenx=1,y=0. 2011. Zmax=l08
whenx=3,y=2.2012.Zmin=-8 when x= Y2~·y=- Vi and when x=
.r- .r- .
=- r 2, y= r 2. There ts no extremum for x=y=O. 2013. 2max=--.;r-at
ab
3 r 3
. a b a b ab
the )::Otnts X= y:f. y= Y3- and X=- ¥3' y =- ¥3; Zmln =- 3 V3
a a
at the points X= ¥3. y =-
b
¥3 and X=- V3. Y= v:r·
b
2014. Zmax=l
when x=y=O. 2015. Zmln=O when X=y=O; nonrigorous maximum
(z = ~) 2 2
at points of the circle x + y = I. 2016. Zmax = when x =I, y =-1. ¥3
4 2 1
2017. Umln=- when X=-3, y=-3• t=l. 2018. Umin=4 when
3
x= ~,
y=l, z=l. 2019. The equation defines two functions, of which one
has a maximum (2max = 8) when x =I, y = -2; the other has a minimum
(Zmjn=-2Jwhenx=l.y=-2, at points of the circle (x-J)2+(y+2J 2 -o25,
each of these funct10ns has a boundary ext·emum (z = 3). Hint. The func-
tions mentioned in the answer are explicitly defined by the equalities
Answers 441

z=3± V23-(x-1) 2 -(y+2) 2 and con5equently exist only inside and on


the boundary of the circle (x-1)2 + (y + 2l =23, at the points of which both
functions assume the value z = 3. This value is the least for the first function
and is the greatest for the second. 2020. One of the functions defined by the
equation has a maximum (~max--~- 2) for x = - I, y = 2. the other has a
minimum (Zmin =I) for x = - I, y = 2, both functions have a boundary extremum
at the points of the curve 4x 3 -4y 2 -12x+l6y-33=0. 2021. Zmax={ for
I
x=y=2. 2022. Zmax=5 for X= I, y=2; Zmin·:o-5 for x=-1, y=-2
36 1s 12 2 t- ¥2- 7n
2023. Zmin = T3 for x = I3 , y = I3 . 2024. ZmJx =
2
for x = B kn, +
9rt 2 - V2 3n 5n
y=s +kn, zmin-=--2- for x= +kn,
8 Y= g+kn. 2025. r1m 111 =
=-9 fJr X=-1, y=2, Z=-2, Umax=9 for x=l, y=-2, z=2.
2026. llm•x_oa for X=±a, Y=Z=O; Umin=C for x=y=O z=±c.
2027. Umax=2·4 2 ·63 for X=2, y=4, z=6. 2028. Umax=4 4 / 27 at the po1nts
4 4 7)
(4 4) 7 4 4) (7
( 3 • 3 • 3 ; 3 • 3 • 3 ; 3. 3. 3 ; Umtn = 4 at the pmnts (2,
2, I) (2, I, 2) (I, 2, 2). 2030. a) Greatest value z=3 for x=O, y= I;
2
b) smallest value z=2 for x=I, y=O. 2031. a) Greatest value z=---;r- for
3 y 3

X=± Y"i-. y= vi: smallest value Z=-


3
~;r for X=± vi,
y-=- v~; b) greatest value z=l for X=± I, y-=0; smallest value

z=- I for x=O, y= ± I. 2032. Greatest value z =


3
~3 for x = y = ~ (in-
ternal maximum); smallest value z=O for X=!f-=0 (boundary m111tmum).
2033. Greatest value z=l3 for x=2. y=-1 (boundary maxiJnum); smallest
value 2'=-2 for X-=Y=i (internal minimum) and for x=O, y=-1 \boun-
J/- J/- I J/-
rlary minimum). 2034. Cube. 2035. v 2V, v 2V, '2 v 2V. 2036. Isosceles

triangle. 2037. Cube. 2038. a= Va·- v- a · l4/a-- · v a. 2039. M (-T, TI I ) .


2040.Sidesofthetriangleare -3 p, 3 p, and !!... . 2041. x=-"m,x,+mzxz+m,r,,
4 4 2 m,+m 2 +111a
y = m,11, + tnzllz + m,y, . 2042. _.:. + 1!... + !_= 3. 2043. The dimensions of the
m, + m2 + m1 a b c
.
paralleleptpedare v-·3
2a 2b
.r-··
y 3
2c .
.r_,wherea, b, and care the semi·
f 3

axes of the ellipsoid. 2044. x=y=2~+ V -2V, Z=2.


x
2045. X=± y-a2-,
y ,-= ± : 2046. Major axis, 2a = 6, minor axis, 2b = 2. Hint. The square of
2
ihe distance of the point (x, y) of the ellipse from its centre (coordinate origin)
IS equal to x 2 + y 2• The problem reduces to finding the extremum of the function
x2 +y 1 provided 5x 2 +8xy+5y 2 =9. 2047. The radius of the base of the cylinder
442 Answers

is ~ y 2+ :
5
, the altitude R y 2- ;
5
, where R is the radius of the

spkere. 2048. The channel must connect the point of the parabola ( ~ , !)
with the point of th~
straight line ( - its length is ~, ~l ; ~~ 2=
V2730. 2050. s~n~=~. Hint. Obviously, the point M, at which
1
2048.
14 Stn I' V1
the ray passes frem one medium into the other, must lie between A 1 and 8 1 ~
AM= _a_ , BM =
cos a
~
cos I'
, A1M= a tan a, B 1 M = b tan~- The duration of motion

a_+~. The problem reduces to finding the minimum


of the ray is __
V 1 COS a V 2 COS I'

of the functien f (a, ~)=--a-+~


v cos a v cos I'
provided that a tan a+b tan ~=c.
1 2

2051. a=~. 2052. / 1:/ 2 : / 1 = ~~: ~~: ~~. Hint. Find the minimum of the
function /(/ 11 / 2 , / 1)=/~R 1'+I!R 2 + I!Ra provided that 11+1 2 +1 1 -=;-/.
2053. The isolated point (0, 0). 2054. Cusp of second kind (0, 0). 2055. Tac-
node (0, 0). 2056. Isolated point (0, 0). 2057. Node (0, 0). 2058. Cusp of first
kind (0, 0). 2059. Node (0, 0). 2060. Node (0, 0). 2061. Origin is isolated point
if a> b; it is a cusp of the first kind if a= b, and a node if a < b.
2062. If among the quantities a, b, and c, none are equal, then the curve
does not have any singular points. If a= b < c, then A (a, 0) IS an isolated
point; if a< b = c, then B (b, 0) IS a node; if a= b = c, then A (a, 0) is a cusp
of the first kind. 2063. y= ± x. 2064. y 2 =2px. 2065. y-= ± R. 2066. x'la +
+ y'i•=l'i•. 2067. xy= ~ S. 2068. A pair of conjugate equilateral hyperbolas,
whose equations, if the axes of symmetry of the ellipses are taken as the
coordinate axes, have the form xy = ± 2: . 2069. a) The discriminant curve
y=O is the locus of pomts of inflection and of the envelope of the give11
family; b) the discriminant curve y = 0 is the locus of cusps and of the envelope
of the family; c) the discriminant curve y = 0 is the locus of cusps and is not an en-
velope; d) the discriminant curve decomposes into the straight lmes: x = 0 (locu!
v• • I
of nodes) and x=a (envelope). 2070. y = ° - gx . 2071. 7 . 2072. J! 9 + 4n•.
2g 2v~ 3
,r
2073. r 3 (e 1 -I). 2074. 42. 2075. 5. 2076. Xd- z8 • 2077. In 10
11-\---
9
2079. a) Straight line; b) parabol3; c) ellipse; d) hyperbola. 2080. I) ~ao

2) ad~ ; 3)~a +a~~ • 2081. : (abc)=(~~bc )+(ad!c)+(ab~~)


0
0
0

1
2082. 41 (t'+ I). 2083. x = 3 cost; y = 4 sin t (ellipse); for t =0, fJ = 4}, w = -3l; fo
n 3 ¥2 .r-- 3 Jllr - n
1=4,fJ=---t+2 r 2), w = - --i-2 f2j;fort=2 ,'0=-3i, 'W=
2 2
=-4}. 2084. x=2cost, y=2sint, z=3t (screw-line); fJ=-2isint-t
+2Jcost-/-3k; V= yf3 for any t; w=-2lcost-2jsint; w=2for any t
for 1=0, fJ=2j-j-3k, w=-2l; fort=-]-. 'll=-2i+3k, w=-2J
Answer~ 443

2085. x =cos a cos rot; y =sin a cos rot; z =sin rot (circle); t1 = - ffil cos a sin rot -
--roj sin a sin rot +rok cos rot; t1 =I ro(; w =- ro 2l cos a cos rot-ro•jsin a cos rot-
2 2
- U l ksinrot; w=ro • 2086. tl= tl! o +t~u1 o +<tlxo -gt)l; Wx=Wy=O; W::=-g;
V
6
W=g. 2088. roVa 1 +h2 , where ro=~t istheangular speed of rotatien of the
: y--
screw. 2089. Va 2Ul 2 +t~!-2arotJ 0 sinrot. 2090. 't=-1 (l+k); 'Y=-J; P=

= ~2- (l-k). 2091. 't= ~:f [(cost- sin t) l + (sin t +cos t)j + k]; v =
=-~ 2 ((sint+cost)l+(sint-cost)j); cos('t-:'z)= ~3 ; cos(v~)=O.
2092 . 't=l+4J+2k; V=-4l+_5J-8k; ll=-21_=!-k. 2093 . x-a~ost=
V21 V105 V5 -a sm t
y-asint z-bt (tanaent)-, x-acost y-asint z-bt (binormal)·
= a cos t - -b- "' b sin t - b cos t -a- '
x-acost
cos = y-asint
sm z-bt
- - - . 1 norma I) . Th e d'tree t'10n cosmes of
(pnnctpa
1 1 0
a sin t a cos t b
the tangent are cos a =- ,r 2 ; cos ~ = ,r 2 1 ; cos y = ,r-:-;--:-;:;; •
r a + b2 r a +b r a2 + b2
The direction cosines of the principal normal are cos a, =COSt; cos~~= sin t;
cos y 1 = 0. 2094. 2x-z = 0 (normal plane); y-1 = 0 (osculating plane);
x-2 y-4 z-8
x+ 2z-5=0 (rectifying plane). 2095. - - = - -=!2 (tangent); x +
1 4
+ 4y + 12z-114 = 0 (normal plane); 12x-6y + z-8 = 0 (oscutating plane).
12 11 12 t~ 11 1:
x-4 y-3 z-2 x-4 y-3 z-2
2096. - 12- = -1- = -1- (tangent); t'+ 2t = I-t'= _ 21 ,_ 1 (prin-
x--

8
14
4
cipal normal); - - -
)
1
t'
y--
t2
z--
= _ 2~-=~ (binormal); M (! , ~);
x-2 y+2 z-2- (tangent); x+ y=O (osculat-
1 -+,
M2 ( 4, - 3 , 2 . 2097. - 1- =-=-r=- 2
x-2 y+2 z-2 x-2 u+2 z-2
ing plane); - -=-=1= _ (principal normal); +T=-- = - 0-
1 1 1
R R
. I 1 x-2 y-2
(btnorm~~!__ cos a 2 = V2'; cos ~~ = V2' , cos Ya = Q. 2098. a) - 2- = - 0 - =
z--2-R , - x-1 y-1 z-2
= _ V (tangent); x r1 2 -z= 0 (normal plane); b) - - = - - = - 4-
2
x-2 y-2 V3 z-3
(tangent); x+y+4z-l0=0 (normal plane); c) ,r-=
1
.r
2 f 3 -2 f 3
(tangent); 2 Jf3 x+y-2 V3i = 0 (normal plane); 2099. x+y = 0. 2100. x-
-y-zY2-=0. 2101. a) 4x-y-z-9=0; b) 9x-6y+2z-18=0;
c) b 2x!x-a2y~y + (a2 -b 2 ) z!z = a1 b2 (a 2 -b 2 ). 2102. 6.t-8y-z+ 3 = 0 (osculat·
x-l y-l z-1 x-1 y-1 z-1
ing plane);
31 = 26 =_ 22 (principal normal); -=-5=-8 -=-r
444 Answers

(binormal). 2103. bx-z-= 0 (osculating plane); ; : ~· } (principal normal);


x + bz = 0, } . l + bk - bl + k
u= 0
(bmormal); 'I'= .r
rltb•
; p= .r
rl+b 1
; \1 = J. 2106. 2x +
+ 3y + 19z-27 = 0. 2107. a) f2; b) - - . 2108. a) K =
-- ¥6 e- 1 ¥2 ; e-t
T= 3 l
4 3
b) K= T = 2acosh 1
1t ·
2109. a) R - Q - (y + a)• • b) R - Q -
- --a-' - -
(p' + 2x')'
8p'x1
av•
2111. a• + bl . 2112. When t = 0, K = 2, w~ = 0,
Wn =2; when t = 1, K= f y~~, w~ = ;
14
, Wn = 2 Y~ ·
Chapter VII
2 25 n 9 na•
2113. 4
3 . 2114. In 2"4 2115.
12
. 2116. 4'. 2117. 50.4. 2118. T. 2119. 2.4.

2120.
n u•
6' 2121. x=-:r-1: x=2-y; y=-6; y=2. 2122. y=x 2 ; u=x+9;
X
X= 1; X=3. 2123. u=x; Y= 10-x; y=O; !1=4. 2124. u=:r; y=2x;
X=l; X=3. 2125. y=O; u=Y25-x•; x=O; x=3. 2126. u=x 2 ; y=x+2;
I I I I

x=- 1; X=2. 2127. S dy Sf (x, y) dx= S dx Sf (x, g) dy.


0 0 0 0
I I I X I 2-y

2128. S dy Sf (x, y) dx= S dx S f(x, y) dy. 2129. S dy S f (x, y) dx =


0 y 0 0 0 0
I I I 1-X I 2X+I

= Sdx Sf (x,
0 0
y) dg + Sdx S I (x,
I 0
g) dy. 2130. Sdx
I
S f (x, g) dg =
2%

C 2 I I 7 I

= S dg S f(x, g) dx+ S dg S f(x, y) dx+ Sdg S f(x, y)dx.


C I I ~
I

2131. ss
o
dg
-y
f (X, g) dx + yr Yisy• dg
-Yz-y•
f (x, g) dx=
-1
s YI dx
-x
x• I (x, !/) dg+

1 Yt- x• 1 2 2 V~
+ S dx S f (x, y) dy. 2132. S dx S f (x, g) dg = S dy S f (x, y) dx.
2
0 -J 2X 0 --~-
-v f
Answers 445

- 1 V4=X> 1 - v~ 1 v~ - ""
2133. S dx S f(x, y)dy+ S dx S f(x,y) dy+ S dx S f(x.gkly+
-I_ V 4- ,t2 - I - Jf~ - I V~
a V4=X> -1 V4 -u• 1 - vt=jji
t- Sdx S f(x, y)dy= S dy S f(x, y)dx+ S dy S f(x.y)dJC+
- V4 - x• - 2
- V 4 -- 'I' - 1
- V 4 - u•
1 V • - ,,. a V' - u•
+5 dy S f (x, y) dx + 5 dy S I (x, g). dx...
- 1 v...--::go - v. -u•
-2 Ye -x• 2 l'1+ .t 2

2134.5dx 5 f(x,y)dy+5dx 5 t<x.y)dy+


-• -l'e--x• -z -l'i""+Xi
Ve-.t• -1 -Vy•-1
5 f (x, y) dy = 5 dy 5 f (x, y) dx+
1 _ v....-=-x; -VI - v. -y•
- I v. - ,,. I v. - "' Jl'l - Vii>=~

t- sdy s
-- VI V 11 • - 1
f (x, y) dx+ s s
- 1
dy
- V • - y•
f (x, y) dx+ 5 dy s
- V • - u•
f(x, y) dy.J-

vI v. - ,,. I I - X I 1 - y

+s dy
v~
5 f(x, y)dx. 2135. a) s dx 5 f(x, y) dy= 5dy
o
5f(x, y) dx:

a V u• - x• a V a• - 11• 1 V x - x•
b) 5 dx 5 f (x, y) dy= 5 dy 5 f (x, y) d.1.; c) 5dx S f(x, q)dV-=
-a - V~ -a - ~- u' - 11' o - Jf x -- x 1
I+V~
I I I 1J
''·
5 dy S 2
f (x, y) dx; d) 5 dx 5 f(x, y) dy= 5 dy 5 f(x, g)dx;
- 'I• 1 - v.--.;;; -1 X --1 -1

a y + 2a a x 2a a 1a a
e) 5 dy 5 f (x, y) dx = 5 dx 5 f (x, y) dy + 5 dx 5 f (x, y) dy + 5 1x 5 f(x, y)dy.
o y {Jf o o f a o 1a x- aa
48 2 1 1

2136. 5 dy 5 f (x, y) dx. 2137. 5dy Sf (x, y) dx +5dy 5f (x, Y> !h..
0 II 0 II I .J!._
12 J I
a
2 Vu• -u• a Ya• -y•
2138. 5dy S f (x, y) dx +S dy S f (x, y) dx.
0 Vu• - aay !!. o
I
444 Answers

(binormal). 2103. bx-z- 0 (osculating plane); ; : ~· } (principal normal)~


x+bz=O,} . l+bk -bl+k
u= 0
(bmormal); 1= .r ; IS= V
, 1 tb• l+b•
; \1 = j. 2106. 2x +
+ 3y+ 19z-27 = 0. 2107. a) V2; b) - - . 2108. a) K =
-- ¥6 e- 1 ¥2
; T=
e-t
3:
4 3
b) K = T = 2a c!sh•t . 2109. a) R = Q = (y +aa)• ; b) R = Q = (p~~;t>'
av•
21tt. a•+bi. 2112. When t=O, K=2, w~=O,

Wn =2; when t = 1, K= +v:~' w~ = ;14 ' Wn = 2 v~ .


Chapter VII
2 25 n 9 na1
2tt3. 4
3 . 2ll4. In
24 . 21111. !2 2116. 4 . 2117. 50.4. 2118. 2 . 2119. 2.4.
2120. lf' 2121. x=y: -1; X=2-y; y=-6; y=2. 2122. y=x 2 ; y=x+9;
X
X=l; x=3. 2123. y=x; Y=IO-x; y=O; y=4. 2124. u=:r; y=2x;
X= I; X=3. 2125. y=O; y= V25-x 2 ; x=O; x=3. 2126. y=x 2 ; u=x+2;
I I I I

x=-1; x=2. 2127. S dy S f(x, y)dx=S dx Sf(x, y) dy.


0 0 0 0
I I I X I 1-y

2128. S dy Sf (x, y) dx= S dx Sf (x, y) dy. 2129. S dy S f (x, y) dx =


0 y • 0 0 0
1 1 I 1-X 2 2X+I

= Sdx Sf (x, y) dy + S dx S f (x, y) dy. 2130. Sdx S f(x, y)dy=


0 0 I 0 1 2X

.1!...
4 I I I 7 I

= S dy Sf(x, y) dx+ S dy S f(x, y) dx+ Sdy S f(x, y)dx.


4 I I ~
I
I II Yl Yi-u• 0 Ya- x•
2131. ss
0
dy
-y
f (x, y) dx+ s s dy
- Ya -u•
f (x, y) dx= S dx
-I
s
-x
f (x, y) dy+

I Ya- x• I I I {+
+ S dx s f (x, y) dy. 2132. S dx S f (x, y) dy = S dy S f(x, y) dx.
0 X -I ax• 0

-v+
Answers «5

-1 V~ 1 -V~ 1 ~-~

2133. S dx S f (x, y) dy + S dx S f (x, y) dy + Sdx S f(x.gldy+


- 2 - V4 - x• - I - V4 - x> - I V~
I V~ -I V4 -y> I -VJ="iii
+ Sdx 5 f(x, y)dy= 5 dy 5 f(x, y)dx+ Sdy S f(x.y)d~+
- V4 - x• - 2 - V 4 -- ,,. - 1 - V 4 - u•
I v4 _,,. I V4 -u•
+S dy S f (x, y) dx + dy S 5 f (x, g). b...
-1 VI -y• _y,_IJ.
- 2 v ii""=Xi 2 vi+Xi
2134. 5 dx 5 f (x, y) dy +5 dx S ftx, y) dy+
- a _ V ,.-::x; - 2 _ I' i"""'+X2
v~ -1 -Vu•-1
5 f (x, y) dy = 5 dy 5 f (x, y) dx+
a -Vg-:-x2 -VI -Vo-u•
V u - ,,. V 8 - u• Vi - Vy;-:=-j
s vs
- 1 1

t- 5dy 5 f (X, y) dx+ dy f (x, y) dx+ 5 dy 5 f(x, y} dy~


-- V& v y• - I - I - 8 - !!' - v.....-:-y;
VI V8 -•1' I 1-X I 1-y

+5 dy 5 f (x, y) dx. 2135. a) Sdx 5 f (x, y) dy = 5 dy S f (x, y) dx~


v~ o
a Va2 - x> a V a2 - IJ' I vx-::-x:.
b) S dx 5 f(x, y) dy= 5 dy 5 f(x,y)dx;c) 5dx 5
-a - V a• - x' -a - V a' - y' 0 - V x -- x•
I+ VJ:4iji
.,. 2 I I I !1

S dy 5 f (x, y) dx; d) 5 dx
-1
Sf (x,
X
y) dy = 5 dy 5 f (x, y} dx;
--1
-'{> I- J/~ -1

a y + 2a a x aa a aa a
e) S dy 5 f (x, y) dx = 5 dx 5 f (x, y) dy + dx 5 f (x, 5 y) dy + 5-tx5 f(x, y)dy.
o u Y ~~ o o f a o tax- 2a
2 8 1

2136. 5dy 5 f (x, y) dx. 2137. 5 dy Sf (x, y) dx + 5dy 5 f (x, Y) dx,_


.!!._ 0 1J I J!_
12 • •
a
1 V u' - y' a V a• - u•
2138. 5 dy 5 ftx, y) dx+ dy S S flx, y) dx.
o Vu• -zuy a o
I
446 Answers

a 'VI
a a a a

2139. S dy S f (x, y) dx + S dy S f (x, y) dx.


a a 'VI a- 'V a• - y2
2 2

a a- 'VQq a za 2 'VIa za

2140. ~ dy ~ f(x, y)dx+ sdy s f(x, y)dx+ s dy sf (x, y)dx.


!!!_ a+ Va•-y• !!:
&a 4a
2..
0 'VI-X' I 1-X 2 'V2X

2141. S dx S f(x, y) dy+ Sdx S f(x, y)dy. 2142. S dx S f (x, y) dy +


-I 0 0 0 0 0
R'VI
'VI I 'VI- 'V;::x>

+ Sdx Sf(x,y)dy + S dx S f (x, y) dy. 2143.


y
f(x, y) dx.
'V2 0

1 n- arc sin y
1 ll ll
2144. S dy S f (x, y) dx. 2145. { . 2146. 6 . 2147. 2 a. 2148. 6
arc •In y
1 4 15n-16 2
2149. 6. 2150. . 2151. In 2 2152. a) 3; b) 150 ; c) 2 5 ·
2
a 'VI -(X - 2)'
8 V2 5
2153. -2,-p. 2154. S dx S xydy=~. 2155. ~a V2a.
0
mR y=f(~l

2156. inR 1
• Hint. SSudxdy= S dx
(S) o
S
o
ydy=

lit R (I-COS t)

= 5 R (I-eos t) dt 5 y dy, where the last integral Is obtained from


0
R4 2158. I .
. t). 2157. SO.
R (t-sm
the preceding one by the substitution X=
6
1

s
&
cos Ql
2159 •. rf (r cos q>, r sin cp) dr+
2160.
Sdq>
0
0

It 1 :!.. 2
1 sin Ql " cos Ql

+ S dq>
11
5 rf (r cos q>, r sin q>) dr.
0
2161. Sdq>
0
5 rf
0
(r 2 ) dr.

T
Answers 447

111 1 11 • sin cp
4 sin cp 4 ces• cp

2162. S dcp S rf(rcoscp rsincp)dr. 1 2163. Sf (tan cp) dcp S r dr +


11 0 0 0

111 1 sin cp
' sin cp 11 ces• cp

+Sf (tan cp) dcp S r dr + S f (tan cp) dcp S r dr.


11 0 111 0
4

~
4 a V cos 1cp 111 a VCciS'ZiP
2164. S dcp S rf (r cos <P1 r sin cp) dr + S d cpS rf (r cos cp, r sin cp) dr.
n 111 o
-4
n
1 a cos cp
na1
2165. S dcp S r
2
sin cp dr =~. 2166. 3 '
2na. 2167.
3'

2168. (~+ i) aa. 2169. 2170. ( :::._16


3
¥2 -20) ~
9 2 •
2
2171. nab. Hint. The Jacobian is I= abr. The limits of integration are
3
fl c
t+fl 1-V

O~cp,.;;;2rr, O~r,.;;;l. 2172. S dv S f(u-uv, uu)udu. Solution. We


a 0
t+a
have x=u(l-v) and y=uu; the Jacobian Is l=u. We define the
limits u as functions of u: when x=O, u (1-v)=0 1 whence u=O
(since 1-v =f;. 0); when x=c, u =c- . Limits of variation of v: since
1-v
y=a.x,it follo\\S that uv=a.u(l-v), whence v=-a.-; for y=~x we find
1+a.

2173. I=~[~du ~tt(u~v. u2v)dv+

1
2

+ Sdu S f
2 •• II

u -2
(u ~
0
1 u 2 °) dv J ~ ~· dv
= [ _ ]> (~ u v 1 u2 °) du +
+J(~ dv zs-v f (u+v
-2 -~ u 2 °) du]. Hint. After change of variables, the equa-
0 v
tions of the sides of the square will be u=v; u+v=2; u-v=2; u=-v.
2 2
2174. ab aJii-Jiib ) arc tan ak ab] . Solution. The equation of the curve
bh+hk
[(
448 Answers

~-r(:: cos1 tp-:: sln1 tp). whence the lower limit for r wi\1 be 0 and

.the upper limit, r=


az bz
y
~: cos• tp- :: sin1 tp , Since r must be real, It
follows that hz cos 2 tp- ill sln 2 tp;;;:. 0; whence for the first quadrantal angle
ak
we have tan tp..;;; bh . Due to symmetry of the region of integration relative
1
lo lhe axes, we can compute of the entire integral, confining ourselves
4
ak
arc tan /iii r
, ; · a•
/ji cos• q>-
b•
iii sln'cp
'lo lhe ftrst quadrant: 55 dxdy=4 5 dtp 5 abrdr.
(S) o o
1 Vy a V/J aV~

~171. a) 4 ~; 5dy 5 dx+ 5dy 5 dx; b) :n:•-a;; Sdx S dv.


o _ Jfy y-2 a-J&

!178. a) {, b) ( 2+ ~ ) a2 • 2177. ~;~. 2178. ~a2 • 2179. n Hint.

-1<x<L 21so. ~ViS. 21s1. a(~+{). 2182.


4
;-v:r.
tl83. { :rta1 • 2184, 6. 2185. IO:rt. Hint. Change the variables x-2y = u,
I I b
b+4v=u. 2186. :r<b-a)(~-a). 2187. a<~-a)lna.
l l l "

!118. u = sdy So-x> l!x= Sdx So-x> dy.


0 y 0 0
2193. ~· 2194. i. 2195. i
a1 n'r 4 48 .r6 88 a• b
2196. '3. 2197. 4a. 2198. - ; - . 2199. 105 . 2200. 1S. 2201. a c .
3
2202. na (a-~).1
2203.
4 1 .r- 4 3 .r-
3 :na (2 r 2-l). 2204. :na (r 2-1).
3
2205. ":. 2206. j nabc. 2207.
11
~
1

(6 )13 -5). 2208. ~ a3.


2209. Ita (1 _ , -
J?l)
. 2210 . 3nab. 2211. 3 V3
2 -2 . Jf"2 (2 .r--
2212. -2- f 2 -I).
2
tlint. Change the variables xy=u, ~ =u. 2213. ~ Ya"b2 +b 2c2 +c2a2

2214. 4 (m-n) R•. 2215. ~'2 a 1


• Hint. lntegrafe in the yz-plane. 2216. 4a2 .

2217. lalarc sin a.


b
2218. 3'l :rta1 (3 .r-
r 3 -1). 2219. 8a1 • 2220. 3:rta1 • Hint.

"Ill to polar coordinates. 2221. a=~ rra 2


[ ( I+ ~: ) -i- - I ] . Hint. Pass to
Answers 449

polar coordinates. 2222. ~ a• and Sa'. Hint. Pass fo polar coordinates.


a a a

.-8aS arcsin .r~.11.


1 1 1

2223. 8a1 arctan Y5"2


·
Hint.cr=S dxS y a -x
ady
2 -y 2 2 r a -x 2 2
0 0 0

Integrate by parts, and then change the variable x= a ~"3 sin t; transform
the answer. 2224 n 2 2
(b 2 2
Vb +c -a Ya +c +c In b+ ~il+Ci). Hint.
2
4 a+ a 1 +c1
. 2n6R 2 a'b a2 b' - 12 -n•
Pass to polar coordmates 2225. - - . 2226. 12; 2T . 2227. x= ( -:n:) ;
3 3 4
- n - 5 - - 2'1 sin a - - 2
y= ( -:n:). 2228. x=
6 4 6 a; y=O. 2229. X=3a; y=O. 2230. ~=5';
n n
y=0. 2231. lx=4 2232. a) / 0 co
32
(D4 -d4 ); b) lx=
64
(D'-d').
a YiiX
2 8
2233. £=3 a'. 2234. 5 a'. Hint. I=- Sdx S (y + a)• dv.
o -Y<U
2235. 16\n 2-9} . Hint. Th~ distance of the point (x, y) from the straight line
x= y is equal to d = ~;=.~ and is found by means of the normal equation
" 2
of the straight line. 2236. I= ~ ka 5 [7 V 2 +3 In (Y2 + 1)), where k is the
4
proportionality factor. Hint. Placing the coordinate origin at the vertex, the
distance from wh1ch is proportiOnal to the density of the lamina, we direcf
the coordinate axes along the sides of the square. The moment of inertia is
determmed relative to the x-axis Pass111g to polar coordinates, we have
n n
c a sec rp a cosec

J +I
1 1p

I"= S dcp kr (r sin q>) r dr


2
dcp ~
na' 35 '
2238. / 0 =T. 2239. na'. Hint. For the variables of lnfegratlon take 1 and
12
I 1-X l-X-y

y (see Problem 2156). 2240. \ dx Sdy S I (x, y, 1) d1


0 •
R YiF"="i> H

2241. S dx S dy Sf (x, y, 1) dJ.


- R - YRi"="ii o
ab y --
a•- x•
2242.
-a
s
-ab yat-xl
--
dy f (%, "' 1) cfl.
450 Answers

1 'Vi"="Xi 'Vt-x• -u•


2243.
-1
S dx s
- 'VJ-'Xi
dy
s
0
f (x, y, z) dz.

8
2244. 15 (31 + 12 .r- .r-
.. 2 -27 .. 3 ). 2245.
41t
-3-.
¥2 2246.
1t1a1
8 1
2247. 720 .

2248. ~ In 2- ~ . 2249. "'t ( 18 ¥3- 9: ) . 2250. ::a nR'. 2251. 1ta!c' .


2 2
2252. 4 1t11bc. - R
2253. nh -. 2254. 1tR 1 • 2255. 98 a2 • 2256. 38 r' ( ;t- 34 ) .
5 4
4 32 3
2257. iS"' R1 . 8
225. To·
"' 2259.
9a
2
h. 2260 •
4
3
1ta. Se Iu t'wn. V=
x•+y• :rt r•
za 'V aax- x• aa 2 aa cos Ql aa

=2S dx
0
:rt
s dy
s
0
:rt
dz=2 S d<p
0 0
s rdr S dh=
0

a w cos Ql a
'~: = ~ S ( a c~s CJI) d<p
2
4
= 2 S d<p S
2
=} 1ta'. 2261. "'a' Y'i
3
Hint. Pass
0

to spherical coordinates. 2262. ~ :n:. Hint. Pass to cylindrical coordinates.


a• abc b +c.
) 66 2'4(6c-a-
ab z z bz) .
2263. g (3n-4). 2264. nabc. 2265. T(a+ 22.

2267. x = 0; y = 0; z=
2
5 a. Hint. Introduce spherical coordinates.
2
- 4 - - na h
2268. x ='3, y =0, z =0. 2269. ) 2 (3a2 +4h 2 ). Hint. For the axis of
the cylinder we tl\ke the z-axis, for the plane of the base of the cylinder,
the xy-plane The moment of inertia is computed about the x-axis. After
passing to cylindrical coordinates, the square of the distance of an element
r dcp dr dz from the x-axis is equal to r2 sin 2 <p +z 2
• 2270. n~~aa (211 2 + 3a2).
Hint. The base of the cone is taken for the xy-plane, the axis of the cone,
for the z-axis. The moment of inertia is computed about the x-axis. Passing
to cylindrical coordinates, we have for points of the surface of the cone:
r =Ji- (11- z); and the square of the distance of the element r dcp dr dz from
+
the x-axis is equal to r 2 sin 2 <p z2 • 2271. 21tkQh (I-eos a), where k is the
proportionality factor and Q is the density. Solution. The vertex of the cone
is taken for the coordinate origin and its axis is the z-axis. If we introduce
spherical coordinates, the equation of the lateral surface of the cone will be
'IJ=~ -a,
2 and the equation of the plane of the base will be
From the symmetry it follows that the resulting stress is directed along the
r=~.
Slll 'iJ
z-axis. The mass of an element of volume dm=Qr 2 cos1jJdc:pd1jJdr, where Q
is the density. The component of attraction, along the z-axis, by this element
of unit mass lying at the pomt 0 is equal to k ,~m sin 1jJ = kQ sin 'ljJ cos 1jJ d¢ d<p dr.
Answers 451

:t
2:t ~-a hcosec1jl

The resulting attraction is equal to 5 dcp 5 d1jl 5 kQ sin 1jl cos 1jl dr.
0 0 0
2272. Solution. We introduce cylindrical coordinates (Q, cp, z) with origin
at the centre of the sphere and with the z-axis passing through a material
point whose mass we assume equal to m. We denote by ~ the distance of
this point from the centre of the sphere. Let r= YQ 2 +(~-z) 2 be the dis-
tance from the element of volume dv to the mass m. The attractive force of
the element of volume dv of the sphere and the material point m is directed
along r and is numerically equal to -kym ~, where v= ~ is the
r -nRa
3
density of the sphere and dv = Qdcp dQ dz is the element of volume. The pro-
jection of this force on the z-axis is
kmydv "' !:- z
dF = - - -
2
- cos (rz) = - kmy -·-,- Q dcp dQ dz.
r r
Whence
z:t R YR' -z'
F=-kmy5dcp S (~-z)dz 5 Qr~Q=kmy4nR 3 ;!.
-R
5y e-XY dy-e-x
00 •

kMm 2273. -
But since 34 ynR'~= M, it follows that F=---v-. 2 2 3
_

2 -1~ Rt ~+p Ra
1
2275. a) __!_ (p > 0); b) - - for p >a; c) (p > 0); d) (p > 0)
p p-a P -~, P P

2276. _ _!_. 2277.!. l:lint. Dil!erentiate


n• pa
fJ e-pt dt =_!_p twice. 2278. In~.
a
0

2279. arc tan 1_ -arc tan~ . 2280. n In (I+ a). 2281. n (l1 l-a 2--l).
m m 2
a 1 :rt :TT •
2282. arc cot lf. 2283. 1. 2284.
2 . 2285. T. 2286.
4
a2 • Htnt. Pass to

~n. ~ • 2289. Converges. Solution. Eliminate


2
polar coordinates. 2287. 2288.
from S the coordinate origin together with its e-neighbourhood, that is,
consider / 1 = ~~In Y x2 +y2 dxdy, where the eliminated region is a circle of
(S)
radius e with c~ntre at the origin. Pa5sing to polar coordinates, we have
2:t I f:t I

/ 1 = Sdcp Sr lnr dr= 5 [';In r ~~ -~ 5 rdr] dcp=2n (~-~In e-{).


o e o e
Whence lim I,=-~ . 2290. Converges for a> I. 2291. Con\'erges. Hint. Sur-
e~o 2
round the straight line y=x with a narrow strip and put
cl Vdxdy =
S
<S> (x-y)
2
452 Answers

=lim
e-+o
s I

dx
X-1!

f
JV
dy + lim f dx f
(x-y)• il-+oJ J
I I

dy
V(x-y)•
. 2292. Converges for
0 0 0 X+iJ
3 v-5+3 ab(a 1 +ab+b2 ) 2'16 1
a > 2' . 2293. 0. 2294. In
2
. 2295.
3
(a+ b) . 2296.
15 a .
I + m•
Y5m
5
a• [ .! ] a 1
2297. 3 (I +4n 2) 1 - l . 2298. . 2299. a•V2. 2300.54 (56 ¥1-·
-1). 2301.
Y a•ab+ b 2
arctan a . 2302.
2rrb 16 .r-
2rra•. 2303. 2] (10 r 10-1). Hint.

~ f (x, y) ds may be interpreted geometrically as the area of a cylindrical sur·


c
face with generatrix parallel to the z-axis, with base, the contour of integra
tion, and with altitudes equal to the values 'of the integrand. Therefore,
S = ~ x ds, where C is the arc OA of the parabola y = ~- x 2 that connects the
c 8
points (0, 0) and (4,6). 2304. a ¥3. 2305. 2 (b 2
+ .r a•b
r a2 -b 2
arc sin v~)
a
.
2306. v a• + b 2 ( 1t Va• + 4rrb 2 + ;i In 21Tb + vr-:-=-·._,+.:...4.,--rc-=-'b~.) . 2307. ( i a,~ a)-
2308 · 2rca• . r--
r a1 +b 2 •
kMmb
2309. Y(a•+b•)•. 2310. 403() ·
19 2311 2 a
· - rca·
4
2312. a) ; b) 0; c) 12 ; d) -4; e) 4. 2313. In all cases 4. 2314. -2rc. Hint.
3 5
Use the parametric equations of a circle. 2315. ! ab 2 •2316. -2 sin 2.
x.
2317. 0. 2318. a) 8; b) 12; c) 2; d) ~ ; e) In (x+ y); f) ~ cp (x) dx +
x,
112
+ ~ 'ljl (y) dy. 2319. a) 62; b) I; c) {+In 2; d) I+ ¥2. 2320. Yl +a•-
y, _ _
- Y I+ b2, 2322. 2
a) x +3xy-2y 2 +C; b) x 1 -x2 y+xy 2 -y1 +C;
c) eX-Y(x+u>+C; d) lnlx+ui+C. 2323. -2rra(a+b). 2324. -rcR 2 cos 2 a

1~
2
2325. (i+rc 1 2326. a) -20; b) abe-l; c)5 V2; d) 0. 2327. 1=
)R •
= SSy dx dy. 2
2328. - ~ . 2329. rc:'. 2330. - { . 2331. 0. 2332. a) 0;
(Sl
b) 2nrc. Hint In Case (b), Green's formula is used in the region between the
contour C and a circle of sufficiently small radiu~ with centre at the coor-
dinate origin 2333. Solution. If we consider that the direction ol the tangent
coincides with that of positive circulation of the con1our, then cos (X, n) =
=cos(Y,t)=1Jt. hence, pcos(X, n)ds=:f'1fds=pdy=O 2334. 2S,where
s c c s c
S is the area bounded by the contour C. 2335. -4. Hint. Green's formula is
not applicable. 2336. nab. 2337. ~ rca1 • 2338. 6na1 • 2339, fa•. Hint. Put
Answers 453"

a•
11= tx, where t is a parameter. 2340. 611R 2 for 60. 2341. 1t (R + r) (R +2r);
R=r Hint. The equation of an epicvcloid is of the form x=<R+r)cost-
-rcosR+rt. y=(R+r)slnt-rsinR+rt, where tis the angle of turn of
r r
the radius of a stationary circle drawn to the point of tangency.
2342. 11 (R-r) (R-2r), i 11R 2
for r= ~ Hint. The equation of the hypo-
cycloid is obtained from the equation of the corresponding epicycloid (see
Problem 2341) by replacing r by -r 2343. FR. 2344. mg (z 1 -z 2 ).
2345. ~ (a1 -b 2 ), where k is a proportionality factor, 2346. a) Potential,
J.L ~t
U = mgz, work, mg (z 1 -z 2); b) potential, U =-, work, Jf ;.
r a 1 +b2 +c1
2
k k2 8
c) potential, U=-2(x2 +y 2 +z1 ), work, 2 (R 2 - r 1 ). 2347. al'ta•.
211a1 Jfa· + b2 4 na• 3
2348.
3
2349. 0. 2350.
3 11abc. 2351.
2 . 2352. 4.
25 V5 + I
2353. ( ,r ) a. 2354. 11- Jf2
2
4 •
- h. 2355, a) 0, b) - SS(cos a + cos ~ +
lO 5 r 5-l (S)

+cos y) dS. 2356. 0. 2357. 411. 2358. -11a 2, 2359. -a•.


oR iJQ
2360. iJy = iJz •
aP oR aQ ap rr
oz=ax. ax=ay• 2361.0. 2362. 2jJ(x+u+z)dxdydz.
(V)
rr d~ dy dz c~·s (a•u a•u a•u)
2363. 2
S.lJ v
(V)
,~.a+ y• + z'. 2361. J ax• + iJyl + i)zl dx dy dz.
I )

a•11a2 b2 . 12 •

2 . 11a .
1
2365. 3a4 2366. 2363. - - 2367.
2371. Spheres, cylmders.
5 2
2372. Cones. 2373. Circles, x1 +y 2 =c~, z=c 1 • 2376. gradU(A)=9l-3j-3k;
I grad U (A) I= Y99=3 JfTI; z 2
=xy; x=y=z. 2377. a) !..r ; b) 2r. c)-~
r
:

d) f' (r)!..
r
2378. grad (cr) = c; the level surfaces are planes perpendicular to

the vector c. 2379.


au 2u a,-=JgradUJ
or=-;-· au when a=b=c. 2380.
au
ar=
=-
CClS (1, r)
forlj_r. 2382.-
;
au =0
-
2
2383.
. 2 I
dtva=-f(r)+f(r).
1
01 ' r '
2385. a) divr=3, rotr=O; b) div(rc)=~. rot(rc)=rxc;c)div(f(r)c)=
r r
=f'(r)(c,r), rot(f(r)c)=f'(r)cxr. 2386. divt1=0; rott1=2m, where
' '
m = wk 2387. 2wn°, where no is a umt vector parallel to the axis o! rotation.
2388.
. a•u a·u a•u
dtv grad U = a~• + oy• + oz• ; rot grad U =0. 2391. 311R H.
2

2392. a) k11R 2 H (3R 1 +2fi2); b) ~nR 2 H (R 2 +2H 2). 2393. div F=O at all
points except the origin. The flux is equal to -4nm. Hint. When calculatine
454 Answers

the flux, use the Ostrogradsky-Gauss theorem. 2394. 2n 2 h 2 • 2395. -~R'


r
'2396. V= S rf(r)dr. 2397. ~. 2398. a) No potential; b) U=xuz+C;
To
c) V=xy+xz+yz+C. 2400. Yes.

Chapter VIII
l n l n+2 2n
2401. 2n_ 1 . 2402. 2n . 2403. 2n_ 1 • 2404. ili. 2405. (n +
) • 2406. n
12 3 2
. +
1 1·3·5 ... (2n-l) "+1
2407. n(n+!)' 2408. 1 _4 _7 ... ( n- 2). 2409. (-l)n+ 1. 2410. n<-11
3
2416. Diverges. 2417. Converges. 2418. Diverges. 2419. Diverges. 2420. Diverges.
2421. Diverges. 2422. Diverges. 2423. Diverges. 2424. Diverges. 2425. Con-
verges. 2426. Converges. 2427. Converges. 2428. Converges. 2429. Converges.
2430. Converges. 2431. Converges. 2432. Converges. 2433. Converges. 2434. Di-
verges. 2435. Diverges. 2436. Converges. 2437. Diverges. 2438. Converges.
2439. Converges. 2440. Converges. 2441. Diverges. 2442. Converges. 2443. Con-
verges. 2444. Converges. 2445. Converges. 2446. Converges. 2447. Converges.
2448. Converges. 2449. Converges. 2450. Diverges. 2451. Converr~es. 2452. Di-
verges. 2453. Converges. 2454. Diverges. 2455. Diverges. 2456. Converges.
2457. Diverges. 2458. Converges. 2459. Diverges. 2460. Converges. 2461. Di-
verges. 2462. Converges. 2463. Diverges. 2464. Converges. 2465. Converges.
2466. Converges. 2467. Diverges. 2468. Diverges. Hint. all+t > l 2470. Con-
an
verges conditionally. 2471. Converges conditionally. 2472. Converges absolute-
ly 2473. Diverges. 2474. Converges conditionally. 2475. Converges absolutely.
2476. Converges conditionally. 2477. Converges absolutely. 2478. Converges
absolutely. 2479. Diverges. 2480. Converges absolutely. 2481. Converges con-
ditionally. 2482. Converges absolutely. 2484. a) Diverges; b) converges abso-
lutely; c) diverges; d) converges conditionally. Hint. In examples (a) and (d)
"'
·consider the series~ (a 2k_ 1 +a 2k) and in examples (b) and (c) investigate
k=1
"'
1ieparately the series ~ a2k_ 1 and ~ a2k.
"' 2485. Diverges. 2486. Converges
k=1 k=1
absolutely. 2487. Converges absolutely. 2488. Converges conditionally. 2489.
Diverges. 2490. Converges absolutely. 2491. Converges absolutely. 2492. Con·
~ 1+(-l)n
verges absolutely. 2493. Yes. 2494. No. 2495. ~ 3n ; converges. 2496.

L. n (2n- 1): converges.


"'
2
1 n=1
2497. Diverges. 2499. Converges. 2500. Converges.
n=1
a
1 1 ·
2501. I R4 1< 120 , I R 1 1 < 720 ; R 4 <0, R1 >0. 2502. Rn < 2 n+
1 = 2n ( 2 n+
1) nl
Hint. The remainder of the series may be evaluated by means of the sum of
1 1
a geometric progression exceeding this remainder: Rn =an [ - -
2 n+I
+
+(iY(n+l)\n+2)+ ... ] <a,. [{-·tJ~l+(~Y·(n~l) 2 + ... ] ·
Answers 455

2503 R n+ 2 i Solution.
RJo<3.JO -a .2504.n+l<Rn<-n·
I
· n<(n+l)(n+l)l
R =--~- __I_ I I
n (n+l) 2 +(n+2) 1 + ... >(n+l)(n+2)+(n+2)(n+3)+···=
-(-1 I ) ( I I ) I I
- n+l-n+!l + n+2-n+3 + ... =n+l'Rn<n(n+l)+
I I
2
=n .
+..,.(n_+___,.l-=)-(n-+-) + · · · 2505. For the given series it is easy to find the
exact value of the remainder:

Rn=fs(n+!~) (}yn-z.

Solution. Rn = (n +I) ( ~ yn + (n + 2) ( +
rn+z + .••

We multiply by (! )':
~Rn=(n+l) (+ rn+z+(n+2>(i)'n+4+•••
Whence we obtain
~
16 Rra
=
n
(__l_)an
4
(__l_)an + (__l_)!n+a
+ 4 4 + (1..)tn+~
4 +
_
.. · -

_ (__1_)211
-n 4 +
({)'n-( ~)(__l_)"n
I - n+ 15 4 ·
1-16

the series S-= (


I
a -=-- I
--
~ r.
From this we find the above value of Rn· Putting
2506. 99; 999.
11

2507. 2; 3; 5.

2509. Seed when x > 0, S =-1 when x < 0; S=O when


=0, we find the sum of
2508. S = 1. Hint.

" n n-t-1
x=O. 2510.Convcrges absolutelyfor x>l, diver~esfor x..;;l.25tt.Converges
absolutely for x >I, converges conditionally for 0 < x..;; I, diverges for
x..;;;O. 2512. Converges absolutely for x>e, converges conditionally for
I <x..;;;c, diverges for xo;;;;l. 2513. -oo <x< oo. 2514. -oo <x< o:l.
2515. ConvergLs absolutely for x > 0, diverges for x,;;;; 0. Solution. I) I a11 j.;:;;
..;; ~x; and when x > 0 the series with general term ).: converges; 2) ).: ~ l
for x..;; 0, and cos nx does not tend to zero as n __., oo, since from cos nx-+ 0
it would follow that cos 2nx-+ -I; thus, the necessary condition for conver·
gence is violated when x.;;;;O. 2516. Converges absolutely when 2kn < x <
< (2k + I) n (k = 0, ±I, ± 2, ... ); at the remaining points it diverges. 2517.
Divergcseverjwhere. 2518. Converges absolutely for x :1:0. 2519. x >I, x.;;;;-1.
I 2
2520. x>3, x< 1. 2521. x::;:,l, x..;;-1. 2522. x~5 ,
3
x<43. 2523.

x >I, x <-I. 2524. -1 < x < - ~, ~ < x <I. Hint. For these values
1
L xk and
00

L 21lxk
00

of x, both the series the series converge. When I xI ;;;:.1


k=l k=l
456 Answers

1
1x 1..;;;
11nd when
2 , the general term of the series does not tend to zero
2525. -l<x<O, O<x<l. 2526. -1<x<l. 2527. -2<x<2.
2528. -1 < x < 1 2529.- Y1 .;;;;;x.,;;;;
1
Y"2. 2530.-l <x..;;;1. 2531. -1<x<1
2
2532. -l<x<1. 2533. -oo<x<oo. 2534. x-=0. 2535. -oo<x<oo.
I I
2536. -4 <X< 4. 2537. -
3 <X<
3 . 2538.-2 <X< 2. 2539.-e <X <e.
2540. -3..;;;;; x < 3. 2541. - I < x < I 2542. -1 < x < 1 Solution. The diver·
gence of the series for 1 xI~ I is obvious (it is interesting, however, to note
that the divergence of the series at the end-points of the interval of conver-
gence X=± I is detected not only with the aid of the necessary condition
of convergence, but also by means of the d'Aiembert test). When I xI< 1 we
have
lim l(n+l)tx<"+llJI= lim l(n+1)x" 1 "1<lim(n+l)lx/"= lim n+I=O
.n-+:s~ n!x"' II-+<» 11-+ex> II-+~~~~~~
(this equality is readily obtained by means of !'Hospital's rule).
2543. -I.,;;;; x,.;;; I Hint. Ustng the d' Alembert test, it is possible not only
to find the interval of convergence, but also to investigate the convergence
of the given series at the extremities ol the interval of convergence. 2544.
-1..;;;x.;;;;;l. Hint. Ustng the Cauchy test, it is rosstble not only to find the
interval of convergence, but also to investigate the convergence of the given
series at the extremities of the interval of convergence. 2.>45. 2 < x,;;:;;; 8.
2546. -2.;o;;x<8. 2547. -2<x<4. 2548. 1.;o;;x.;;;;3 2549. -4.;;;;x,;;;;-2.
5 13
2550. x=-3 2551. -7<x<-3 2552. O,;;;;x<4. 2553. - <x< . 4 4
2554. -e-3<x<e-3. 2555. -2,.;;;x..;;;O. 2556.2<x<4 2557.1 <x..;;;3.
1 I 1
2558. -3.,;;;;x.;;;;;-1 2559. 1--<x<l+- Hint.Forx=1±- the
e e e
• 1 )"'
( 1+-
series diverges, since lim
"-+"'
--:--=.)
e re
f: 0 2560. -2 < x < 0

2561. l<x.;;;;;3 2562. 1.;o;;x<5. 2563.2...-;;;xc;;;;4.2564.1zl<1 2.<;63./zl<1


.r-
2566.lz-2tl<3 2567.1zl< r 2 2568.z=0 2569.1zl<oo.2570.1zl<
I
2
2576. -ln(l-x) (-l.;;;;;x<l) 2577. ln(l+x) (-l<x..;;;;l).
1 1 tx I
2578. 2ln 1_x (I xI< 1) 2579. arc tan x (/xI,;;:;;; 1). 2580. (x-1)2 (I X/ < 1).
1-x 2 2 x
258t. (1 +x 2 ) 2 (I xI< 1) 2582. (1-x)' (J xI< 1). 2583 (x- 1)2 (I x 1 > 1).

2584, 1(arc tan x- 21In 1-x)


2 1
+x (J xI < 1). 2585. nY3
- -
6
Hint. Consider the
. I
sum of thesenes x- 3x' + x5 - ... (see Problem 2579) f()r x=¥"3'
(IJ

2586. 3. 2587. ax= 1 + .t.,.,.


"" x" -n- - a , - oo < x < oo. 2588. sin ( x + T
In" :rt ) =
1

~2 [ l+x-;:-~+~=:~- ... -t-(-1) f+ ... ]·


11

= -·-.--"
Answers 457

x2 x' x.4
2589. cos (x+ a) =COS a-x sin a- 2f cos a+ 3f sin a+4T cos a+ ...
x" .
... +nrsm
ra+ (n +21) nl + .... -oo<x<oo.2590.s!n x= 2t21 - 41
2 x4 25x'
+
61 - ...
2
2 3

zcn-t xtn X
... -j-(-1)"-' ,Zn)! -j- .... -oo<x<oo. 2591 ln(2-j-x)=ln2+- -
2
x2 x' x"
- . 2+ _ , - .. -1-(-1)"-' n·Z"-j- ... , -2<x~2. Hint. When investi-
2 2 3 2
gating the remainder, use the theorem on integrating a power series
2x-3 ~ 3x-5
2592. (x- )2 = - ...::.,.<n+3)x", lxl< I. 2593, x 2 _ 4 x+ __
1 3
11=o
~ ~
2) L(-1)"-'2"-'x 11
=-~ ( 1-1- "+' x".lxl< l. 2594. xe-u-=x+
."'- 3 (n-1)! •
11=0 11 =2
® :.n oo x:.n t •
- oo < ~< oo. 2595. ex'= 1 + ~ xnl , - oo < x < oo 2596. '\~ -=---
...::.,. ...::.,. (2n + 1)1
n=• 11=o
11211 211
I+'""~
2
(-oo<x<oo) 2597. (-1) x . 2598. 1-j-..!_'\.'; (-l)"(2x) "
~ t2n)l 2 ~ (2n)l
n=J '1=1

"'~ (n + 2) 32n·x"'+'
-oo<x<oo. 2599. 2~(-1\ 11
(2n+l)l l-oo<x<oo).
n=o
"' x•"+'
2600. ,L<-l)"gn+• (-3<x<3). 2601.
n=o
1·3·5X 1 1·3·5 . . (2n-1) x 111
-j- 2 •4 •6 27 +. -j- 2 · 4 •6 21l 22n +I -j- " . t- 2 < X < 2)

2602.
~ x•n+t
2 ...::.,. 211 + 1
(I
xI< )
I 2603 ~
. ...::.,.
( -1)"+1 2"-1 n ( - .!_
n x 2 < x< 2
.!.) .
11=o 11=1
oo x" "" x•"+'
2604 x+l:(-l)"(n-l)n (lxl.,;;;;l). 2605. L(-I)"Zn-j-l (lxlo;;;;l).
n=t n=o
I x' 1·3X1 1·3·5 ... (211-l)x 2"+'
2606 · x+2·3+2.45+ ... + 2-4·6 . . 2n 2n+l+... (lxjo;;;;l).
2
I x1 1-3x 5 11 1·3·5 . . (2n-1)x "+'
2607 · x-2·3+2.45- ... -j-(-l) 2·4·6 ... 2n 2n+l-j- ... (lxj~l).
"' 2m-• 2n "' 1
2608. ,L<-1)"1·' (Zn~ (-oo<x<oo). 2609. I t-,L(-1)"-'nnl x"
11=1 11=1

"' 1 -j-211 -j-3 11 - 1


(-oo<x<oo). 2610. 8+3_L nf x" (-oo<x<oo).
ll=t
2-x 2x 2·5x1 11 _ 1 2·5·8 ... (311-4) x
11
2611 • 2 +2 2 ·3·11-25 ·3 2 ·21+28 ·:3 3 ·31+ ... + ( - 1
) --zan-•.an:n-,- t ...
458 Answers

00

(-oo <x <oo). 2612. 1


6- L ( 1 I )
2n+l+3n+l xn (-2<x<2).
n=1
3 .., (1 +3zn-t) x•n
~613. 1 +4 L (
2 n)l (I X I< oo). 2614. L"' 4x'nn+t (.!xI < Y2).
n=t n=o
oo n oo
2615.ln2+L(-1) n-t(l+2-n): (-l<x.so;;1). 2616. I.<-1)nx
n=l n=o
00
x2n+t x2n+l
x(2n+1)(2n+I>' <-oo<x<oo). 2617. x+L<-t>n<zn-r-I>ti,uxl<oo>.
n=l

2619. 1 $
X+ 2·5 X +2 2 ·9·21 X +. •.
1·3 • +
n=l
1·3·5 ... t2n-1) 4n+l+ (lxl<l). 2620. +xa+2xs+
+ 2n(4n+1) nl x "' x 3 1f ...
5 4
2621. x- x' +-2x - ... 2622. e ( 1- x• + x
4
2623. 1+ x• + 5x + ...
)
3 15 2 6 - .... 2 24
2624. - ( ~ + ~ + :; + ... ) . 2625. x + x• + ~ x + . . . 2626. Hint. Proceed·
8

jng from the parametric equations of the ellipse x=aCOS<f, y=bsincp, com-
pute the length of the ellipse and expand the expression obtained in a series
of powers of e. 2628. x1 -2x2 -5x-2= -78+59 (x+4)-14 (x+4) 2 +
+<x+W (-oo < x < oo). 2629. f(x+h)=5x 3 -4x2 -3x+2+
+(15x 2 -8x-3)h+(15x-4) h 2 +5h 3 (-oo <x<oo; -oo <h< oo).
"'
2630. I . (-
I n
on-t
(x n ) (0 <X,.;;;; 2). 2631.
"'
L. (
-l)n (X-l)n (Q <X< 2).
n=l n=o
2632. L."' (n+ 1) (x+ l)n (-2 < x < 0). 2633. L"' (2-n- -3-n- 1 1
) (x+4)n
n=o 11=0

(-6<x<-2). 2634. L(-1)n(X


3
oo
t 2)"n
1 (-2-Y3<x<-2 +Y3).
n=o

2635. e-•[ l+t(x~12)n] (lxl<oo). 2636. 2+x ~/-! 42


(x 24 ) +

-1-~ (x-4) 8 _1·3·5 (X-4) 4 + +(- 1)n-l 1·3·5 ... (2n-3) (x-4)n+
4·6 21 4·6·8 28 • •• 4·6·8 ... 2n zrn ...
"' ( x-~ rn-t
(O..;;x<;8). 2637. ~(-l)n ( n.:_l)!
2
(lXI < oo). 2638. ~+
n=1

n:o1
..,
+ ~ (-l)n
4n-l ( x-;!_ yn-t
(2n-~)l tl XI< oo). 2639. -2
oo

L. 2n ~I
n=o
c +;rn+l
(0 <x < oo).
Answers 459-

Hint. Make the substitution !+; t = and expand In x in powers of t.

~ 640 ' I :. X+ }( I ~X)


2

+ ~:! ( ~X) + ...


I
1

+ =(
~:!: ~: ::~~: ~~ I ~X) n + ' ''
... (- ~ ~X< 00 ) • 2641. I R I < ~ < ~. 2642. I R I < 1\ . 2643. ~ :::::

:::::::{+; (~)' +~:! (~y :::::::0.523. Hint. To prove that the error does not
exceed 0.001, 1t is necessary to evaluate the remainder by means of a geo-
metric progression that exceeds this remainder. 2644. Two terms, th3t is,
I
z
I - ~ , 2645. Two terms, i.e.,
a
2646. Eight terms, i.e., 1 +
7
x-1r.
iiT . L
n=l
2647. 99; 999. 2648. 1.92 2649. 4.8 I R I< 0.005. 2650. 2.087. 2651.1 X I< 0.69;
I 1
XI< 0.39; I XI< 0.22. 2652.1 X1<0 39; I XI< 0 18 2653. 2-2'·3·3!::::::: 0.4931.
2654. 0.7't68. 2655. 0.608 2656. 0 621 2657. 0.2.'J05 2658. 0.026.
00 ( 2n
2659.1-1-~(-1) 11 x-y) (-oo<x<oo; -oo<y<oo).
k- (21!)!
11=1

""
2660. ~(-l)"(x-y)'"-(x-1-y)zn (-oo<x<oo; -oo<y<oo).
k- 2·(2n)l
11== 1
00
( 2 I_ 2)~" -I
2661. ~(-1) 11 - 1 x -,y ····- (-oo<x<oo; -oo<y<oo).
k- (2n-1)!
rl=l
.n
. 1-x+y 2
2662. 1-1-2
L (y-x)";
11=1
lx-yl<l Hmt.
1 -1-x-y
=-1+ (
1- y-x
)'Use

"' x"+r" 1 (-I~ x < 1·


a geometric progression 2663. - "-..., -·- - -1~y<1).
~II '
11=1

Hin:. 1-x-y+xy=(l-x)(l-y). 2664. LJJ


(-!)"
XZII+I

2 ,~: 1
I 2n+l
(-1~~1;
11=0
x+y
-1 ~y ~ 1). Hint. ~rc tan - -=arc tanx+arc !any (for 1 xI< I, 1y I< 1).
1->.y
2665. f (x+ h, Y-1- k) =ax2 +2b xy+cy 2 +2 (ax -!-by) h -1-2 (bx-1-cy) k +ah 2 +
+
I· 2bh +ck 2 • 2666. f (I+ lr, 2 k) -f (I, 2) = 91! -2\k 311 2 -!-3/tk -12k 2 112 - + +
00 II
00 [ X+ ( y -:rt- ) ]211
-2k'. 2667. I+ L[(x- ):,<!1-1- )] . 2668. I+ ~(-I)"
2 2 2
( 1!)! .
2
n=t n=l
xz- yz x'- 3xyz I z
2669. l+x+-21-+ 31 + ... 2670. I +x-j-xy+"2 x !!+ ...

267 1. c1 +c2 _ 2 (c 1 -c2 ) ,..., sin (21! +I) x. S (O)= c, +c 2 ; S ( ± n)='' +.':! C2 •
2 :rt ""-' 2n + I ' 2 •
11=0
-460 Answers

•2672. b-an- 2 (b-a) -\.... cos (2n+ 1) x+(a +b ~ (- 1)"_ 1 sin nx.
4 n ~ (2n+ 1)1 ) ~ n '
n=o n=1

b-a
.s<± n>=-
2
- n. 2673.
n
3 +4
1
:E <-1)"/il:
"" cos nx
s <± n>=n•. 2674. -n2 slnhanx
n=1

1 ~ (- 1)" ] 2 sin an
X [ 2a+ ~a'+n1 (acosnx-nslnnx) ; S(±n)=coshan. 2675. -n- X

..
x ~(
R=1

-1)" n :•" n; if
.
a is nonintegral; sin ax if a is an Integer; S ( ± n) =0.
~ a-n
n=1
2 sin-
2676. - an [ -+
I ~ a CfiS
"' (-1)"- nx] 1f
. a IS
. nomntegral;
. . a Is an
cosax 1f
n 2a ~ 1--
a-n 2

integer; S ( ± n) =cos an.


n=1

2677. 2 sinh
.
an~ ( -l)n-1 n ~i+nn~; S'<± n) =0.
n ~ a n

t (-I)":~~~:]; t
n=1

2678.
2
sl:h an [
2~ + S ( ± :n:) =cosh an. 2679. sinnnx .

2680.
..
L "~ " ~~ ~/) x; a)
1 1
~; b) ~; c) V • 2681. a) 2
.
L (-1)"- 1
X
2 3
n=1 n=1

sinnx, b)~-_! ~cns(2n-l)x. n• ~b


X n • 2 n ~ (2n-1)2 • s· 268 2. a)~ nsinnx, where

2:n: 8 n . n2
n=1

~ 11 cos nx . :n: 2
..
n=1

ba~~_ 1 = 2 k-l-n( 2 k-l)' and bzk=-T• b) 3 +4 ~(-I) fiZ• I) 6 .


n=1
CD

2) ~12•. 2683. '1) ~ "\.'"" [I-( -·I)" ean] n sin nx. b)


11 L.. 1 a•+n '
n=1
CD nn
I n an • I -COS -
~ ~
CD
((- ) e -I] ens nx 2684 ) ~ ~ 2 . . b) _!_+
+n ~ a• + n• · •a n i.J n sm nx, 2
n=1 n=1
g)nn
2 ~., stn 2
CD

+- 1 1 - - cos nx. 2685. a) _i..~(-l"_ 1 sin(2n-l~. b) n


11 ~ ) (2n-1) 2 ' 4-
:n: n=1
- n
_ _! ,., crs 2 (2'1-1) x . ;..... k I
n ~ (Zn-l)" 2686 2... bnsln nx, where b2k=(-1) - 1 k, b1k+ 1=
2
n=1 n=1
Answers 461

2689. ~ (i +i: sl~;h cos nx) . 2690. ~


n•t
[i +±C'~;h r
n=t
nx]. cos

2691. 1-~+2
2
~ ( -l)n-1 COl
~ n• I
nx. 2692 . ..!. [..!.. + ~ (-l)n-1 C"S
n 2 ~
2nx]
4n - l · 1
n~• n•J
1'1
1'1 •

2694. Solution. I) a 1 n=~ Sf(x)cos2nxdx=~ S f(x)cos2nxdx+


1'1 0 0

+~ Sf (x) cos 2nx dx. If we make the substitution t""' ; -x in the first
1'1
1
lntegr al and t = x- ; in the second, then, taking advantage of the assumed

identity f( ; +t ) = - f ( ~ - t ), it will readily be seen that a2n =0


(n = 0, I,""'!, ... ); n
1'1 I 1'1

2) b1n = !Sf 0
(x) sin 2nx dx = !Sf (x) sin 2nx dx + ~ Sf (x) sin 2nx dx.
1'1

The same substttutwn as in Case (1), with account taken of the as~umed
ldentityf(~+t )=t(~-t) leads to theequalitiesb 2 n=O(n=l,2, ... ).

2695
. _!_ _ _! ~ ~(2n + I) nx . 2696 . 1 _ ~ ~~ sin 2mtx.
2 n2 ~ (211 j- I ) 2 n .......,. n

2697. sinh/ [
11=o

1 \-,
T 1-2 ~(-1)"
I cos T-nn
z•+n•n•
sin T] .
n=t

tl=l
"' . mr:x
10~ 51115 . a) "'
_i. ~sin 2 (n-1) 1'1\:.
2698. - (-I)"-- 2699 b) I 2700
1'1 I! n ~ 211-l '
11=1
11=1
at • 1!1'1X "' (2•1-l) llX "-
2/ ~ Sill -~- l 4/ ~ COS l - "' II~
a)- (-1) 11 + 1 - - ; b ) - - 2 ( • . 270t.a))b"sm- 2 .
1t II 2 1t 21!- 1)• ......
n=•
where
n=t

b~kt 1 =n
8 [ n
2

2k+l-~2k+l)'
1 4
bs,=-k;
4n
n=J

b)
4 rr:"
0
"' nx
COS -
"' .
Sl 11
(2 ·1 + I) llX
~----'-=-'--

-16 \ 1 (-1) 11 -
2
I.J
1 --
n• ·
2702 ) !
· a n 2 I.J
'""(-I)" ?
t2.1 + I)2
b)
n=• 11=o
_ _! ~ "' cos (211 + I) nx 2703
.
n2 ~ (2n+ 1)1 •
n=o
462 Answers

Chapter IX
2704. Yes. 2705. No. 2706. Yes. 2707. Yes. 2708. Yes. 2709. a) Yes; b) no.
2710. Yes. 2714. y-xy'=O. 2715. xy'-2y=0. 2716. y-2xy'=O. 2717.
xdx+ydy=O. 2718. y'=y. 2719. 3y 2 - X 2 =2xyy'. 2720. xyy'(xy 2 -j-l)=l.
2721. y=xy' In~. 2722. 2xy" -j-y' =0. 2723. y"-y'-2y=0. 2724. y" +4y=0.
y
2725. y"-2y'+y=0. 2726. y"=O. 2727. y"'=O. 2728. (l-j-y' 2)y"'-3y'y"2 -'0.
2729. y 2 -x 2 =25. 2730. y=xe 2x. 2731. y= -cosx. 2732. Y=
I
=s (-5e-x -j-9eX-4e 2x). 2738. 2.593 (exact value y=e). 2739. 4.780 [exact

value y=3 (e-1)]. 2740. 0.946 (exact value y= 1). 2741. 1.826 (exact value
Y =Y3) 2742. coPtJ=tan x-j-C. 2743. x=y lCy
2 ; y=O. 2744. x2 -j-y2 =
. + l/ 2
=In Cx 2 • 2745. y=a+ Cx-. 2746. tan y=C (1-ex)s; x=O. 2747. y =C sinx.
1 +ax
!1'
2
2748. 2e1 = Ve (1 +ex). 2749. 1 +u•= x 2 • 2750. y= I. 2751.
1
arc tan (x+y)=x+C. 2752. 8x-j-2y+ l =2 tan (4x+C). 2753. x+2y+
+31nl2x-j-3y-71=C. 2754. 5x+lOy+C=31ni!Ox-5y-t-6 1. 2755. Q=
1
= 1 C =• - -lnjcoscp!-1-C or lnlxl-
or y 2 =2Cx+C 2 • 2756. l n Qcos-
-cos 'P 2 cp
-::.=C. 2757. Straight line y = Cx or hyperbola y=: . Hint. The seg-
2
ment of the tangent is equal to yy•+(ff.Y·2758.y 2 - X =C.2759.y=
X

X
~ xydx
2 0 3
=Cea. 2760. y -=2px. 2761. y=ax 2• Hint. By hypothesis -x--=4X,
~ ydx
0
Differentiating twice with respect to x, we get a differential equation.
2762. y2 = } X •
•r - - 2- V 4-x•
2763. y= r 4-x 2 -j-2ln . 2764. Pencil of lines y=kx. 2765. Fa-
x
mily of similar ellipses 2x -j-y =C • 2766. Family of hyperbolas x 2 -y 2 =C.
2 2 2

2767. Family of circles x2 -j-(y-b)2=b 2 • 2768. y=xln~. 2769. y=~-~ .


X X 2
2770._x=Ce Y. (x·-C) 2 -y2 =C 2 ;
2771. (x-2) 1 -y 2 =4; y = ± x. 2772.
,fX C 1
V -y+1njyi=C. 2773. u= 2 x2 - 2C; x=O. 2774. 1
(x -j-y 2) 3 (x-j-y) 2 C.

2775. Y=X Vt-! x. 2776. (x-j-y-l)'=C(x-y-j-3). 2777. 3x+y+2x


Xln!x+u-li=C. 2778. ln!4x+Bu+51+8y-4x=C. 2779. x2 =1-2y.
Answers 463

2780. Paraboloid of revolution. Solution. By virtue of symmetry tile sought-


for mirror is a surface of revolution. The coordinate origin is located in the
source of light; the x-axis is the direction of the pencil of rays. If a tangent
at any point M (x, y) of the curve, generated by the desired surface being cut
by the xy-plane, forms with the x-axis an angle q>, and the segment connect-
ing the origin with the point M (x, y) forms an angle a, then ta-n a= tan 2q> =
2
t~n ~ . But tan a =it. ; tan <p = y'. The desired differential equation is
1- an <p x
y-yy' 1 =2xy' and its solution is y 1 =2Cx+C 1• The plane section is a para-
bola. The desired surface is a paraboloid of revolution. 2781. (x-y) 1 -Cy=0.
2782. x 2 =C (2y+C). 2783. (2y 2 -x 2 ) 1 =Cx 2 • Hint. Use the fact that the area

s
X

is equal to y dx. 2784. y=Cx-xlnlxj. 2781l. y=Cx+x 1 . 2786. y=


II

'""'~ x'+~2 • 2787. x Yl+y 2 +cosy=C. Hint. The equation is linear with
dx I e" ab-e 11
respect t~ x and d-. 2788. x = Cy 2 - - . 2789. y = - + - - . 2790. y=
y y X X

= - (x
I .
rr -
1-x 2 +arc sin x)
y'l+x
l-x. 2791. y = x
--. 2792. u (x 2 + Cx) = 1.
2 COS X -

2793. y 2
c
=xln-. 2
2794. x =-+C2 •
1
2795. y'(3+Cecosx)=x 2797. XIJ=
X y y

=Cy2 +a1• 2q98. u'+x+ay=O. 2799. x=ylnlt.. 2801.


a
x•
2
-l-y 2 -Cy+a 2 =0. 2802.
A
2 +xu+u'=C. 2803. 2804.

X
x2 - y2 x x2 1
?807. +ue !I =2 2808. In I x 1-- =C. 2809. - + - =C. 2810. - In x+
2 X !I 2 !I
I
+ 2Y2"'oC. 2811. (xsln y+y cosy-sin y) c~ =C. 2812. 2
(A 2C + 1-2Cy)x
2 2 1 =0.
2
X(x + C -2Cy) = 0; singular integral x - y 2813. Qeneral Integral
(y + C) 2 =x'; there is RO singular integral. 2814. General integral ( f- u+C) X

X ( x- r +C) =0; there is no singular integral. 2815. General integral

y 2 -j-C 2 ~2Cx; singular integral x2 -y 1 =0. 2816. y=~cosx ± ~-jsinx. 2817.


x=sinp+lnp,
{ y =p sin fl+cosp+ P+ C. 2818·
{ x=eP+peP+C, 2819 . x=2p--p~ +C, I
y=p 2eP. \
t y=p 2 +2lnp.
X
Sin~ular solution: y=O. 2820. 4y=x2 +p2 • lnlp-x/=C+--.
p-x
y=~.
2
2821. In Yp 2 +y 2 +arctan.!!.=C, x=lnY +P'. Singular solution:
y 2p
464 Answers

equation from which x is defined as a function of p ts homogeneous. 2826.


x'
y=Cx+C 2 ; y=- 4 . 2827. y=Cx+C; no singular solutiOn 2828. yo.Cx+

+ Jfl -J-C 2 ; x1 +y = l. 2829. y=Cx+ ~; y =4x. 2830. xy=C 2831. A circle


1 1

and the family of its tangents. 2832. The astroid x211 + y 211 =a 2 / 1 . 2833. a) Homo-
geneous, y=xu; b) linear in x; X=uv; c) linear in y; y=uv; d) Bernoulli's
equation; y = uv; e) with variables separable; f) Clairaut's equation; reduce
to y=xy' ± Jfy' 1 ; g) Lagrange's equation; differentiate with respect to x; h)
Bernoulli's equation; y = uv; i) leads to eq uatwn with variable~ separable;
u =x+ y; j) Lagrange's equation; differentiate with respect to x; k) Bernoul-
li's equation in x; x=uv; I) exact differential equation; m) linear; y=uv;
n) Bernoulli's equation; y=uv. 2834. a) sin}!._= -In 1x 1+ C; b) x= y·eCY+ 1 •
X
X I
1
2835. x'+y'=Cy 1• 2836. y=x'+C' 2837. xy(C- 2 1n x)=l. 2838. y=

=Cx+C InC; singular solution, y=e-V'+ll, 2839. y=Cx+ V -aC; singular


. a I x1 - l I I
etx-eY-arc tany- ,
solution, Y=4x· 2840. 3y+ln (y+I) 1 =C. 2841. 2
I
1 -
- 2 1n(l-l-Y1)=C. 2842. y=x'(I+Cex). 2843. x=y 1 (C-e-Y). 2844. y=
=Ce-slnx+sinx -:-1. 2845. y=ax+CYI-x •. 2846. y=x~l(x+lnjxi-J-C).
11011 x•
2847. x=Ce -2a(I-J-slny). 2848. 2'+3x-J-y+ln [(x-3) 10 ly-li"]=C.
I
y-l 2 --
2849. 2arctan-2-=lnCx. 2850. x1 =1--+Ce
y
11 .2851. x•=CeY-y-2
X

v
2852 ... /!_+In I x 1 =C. 2853. y=x arc sin (Cx). 2854. y'=Ce-tx +.!sin
X 5
x+

+}cosx. 2855. xy=C(y-1). 2856. x=CeY-{(siny +cosy). 2857. py-·.-


3 3 3
=C (p-1). 2858. x'=Ce'Y-y•- 4 y 1 -B y- 32 2859. (xy+C) (x 2y-J- C)=O.

x
V1 +P 2c 1
+-.In (p-!-
1 -
- X=2-
2860. Vx +y 2
--=C. 286t.xeY-y 2
=C. 2862. 2p S-- 2p
Y +
{ r I -l-P 2),
y = 2px VI + p 1.+
2
2863. y=xecx. 2864. 2e"-y'=Cy•. 2865. lnly+21+2arc tanY+ =C.2866.
x- 3
Answers 465

_tC I .!!..
y2 +Ce • +--2=0. 2867. x•.y=Cea.
X
2868. x+~=C. 2869. Y=
y
C-x 4
• a 2 ln(x+ Ya 2 +x"}+C
Jf
4 ( x •-!)•·, 2 .2870,y=CS111X-a.2871.y= x+ a•+x"
2872.

1
(y-Cx}·(ye-x•+C}=O. 2873. y=Cx+c•· u= 2 3v·~-
· 2x 2 • 2874. x 3 +x"y-
-y•x-y3=C. 2875. p"+4y",=Cy•. 2876. y=x-1. 2877. y=x. 2878. y=2.
2879. y=O. 2880. u={(sinx+cosx). 2881. u=i-(2x 2 +2x+l). 2882. y=
~,e-x +2x-2. 2883. a} y=x; b) y=Cx, where Cis arbitrary; the poinf (0,0)
is a siu~.;ular potnt of the differential equation. 2884. a} y 2 =x; h) y 2 =2px;
(0,0) is a singular pomt. 2885. a} (x -C) 2 + y 2 = C 2; b) no solution; c) x 2 + y 2 = x;
X

(0,0) is a singular point. 2886. y ~eu. 2887. yo=(Y2u ± 2. 2888. Y"'-" Yx}
-d-e-". 2889. r=Cea<. Hint. Pass to polar coordtnates. 2890. 3y 2 -2~=0
2891. r -krp 2892. x 2 -l-(y-b}"~b". 2893. y 2 16x=O. 2894. Hyperbola
.... 1
+
y 2 -x 2 .~C or c1rcle x 2 l-y 2 ---C 2 • 2895. u= (e''+e-x). Hint. Use th~ fact
2
X X
tlut the area 1s equal to ~ ydx and the arc length. to ~ l 1-t-y'2 1.-:.
a•
2896. x~-+CtJ. 2897. y 2 -4C(C-:-a-x). 2898. !-lint. Use the f.tct tl::.:t th~
If '
rl'~ultanl of the force oi t;raYII~ and !he cenlr1fugal force is nornnl to tlw surface.
'lak111~ the y-ax1~ as the :1x1s of rotatton and denot111~ by l•J the an~ular ve-
locity-of rotatllln, \\ e ~et for the plane ax1al cross-sec !ton of the dcstred sur-
face the differential equation g~~=l•l~X.
p...ce-Mu~ml•. Hint. The prc:s-
2899.
sure at each IL-vel of a Yertical column of mr may be considered <•S due solely
to the pre;'>utc of the upper-lytng layers Usc the law of BoylP-J\Ltr utte, <:c-
cordi!H~ to \\ luch the dcnsily ts proportional to the pressure. The sought-for
l
llifll'n·nltal cquaiJOntsclp--kpdlt. 2900. s~ kl~- Hint. Equdlion ds-=
2
.okwl-~xdx. s=(r;-{w)kl.
Olle hour. 2904.
2901.

(ll'--' 100 ( ~ r rpm. 2905, 4


2902. T=a+(T 0 -a)e-k1• 2903. In

2~o of the initial quantity Q 0

will drcay in 100 years. Hint. Equation dd~ =kQ. Q ..c: Q0 ( ~ y t


600 . 2906. t:::::::

:::::::35.2 sec. Hint. Equation n(h 2 -2h}dh=n(~)"vdt. 2907. d


1 24
. Hint.

dQ=-kQdh. Q=Q 0 ( 1 ).!!. .2908.u-.. ..V/ gm


1 .
Tast--+oo(ktsapropor-
2
tionality factor). Hint. Equation m~~=mg-ku 2 ; v= V g: tanh (t y ~).
2909.18.1 kg. Hint.Equation~~=k ( ~ - 3~0 ). 2910. i= R':L'w 1 [(Rsinwt-

16-1900
466 Answers

_!i t
-Loocosoot)+Lroe L ]. Hint. Equation Ri+L::=Esinoot. 2911. y=

=xlnlxi+C 1x+C 2 • 2912. I+C 1 y 2 =(C 2 +~;)·· 2913. y=lnje•x+C.I-

-x+C2. 2914. y=C 1 -t-C 2 lnixl. 2915. y=C 1ec,x 2916. y= ± YC.x+C~
2917. y=(I+C~)Inlx+C 1 I-C 1 x-t-C 2 • 2918. (x-C 1) =alnlsiny C
11
2

2919. Y=; (In I X 1) 2 + c.lnlxl +C•. 2920.x=~ In Iy:c. 1·1 c.: y=C. 2921. y=
= C1ec,x + ~2 • 2922. y = ± i [x V C~ -x + C~ arc sin~]
X
2
+ C2 2923. II=
-+t e
=(C 1ex+l)x+C 2 • 2924. y=(C 1 x-C~)ec, -j-C 2 ; u=
2 x•-t-C (singular solu-
~~ +
tion). 2925. y = C1x (X- C1) + C2 ; y=
~
=i2 + 2
~
+ c.x In I X I +C.x+C•. 2927. y= ±sin (C.± x) + c.x+
C (singular solution).

c,.
. 2926.

2928. y ~
II=

I
=x'+3x. 2929. y·= 2
-t-l). 2930. y=x+l. 2931. y=Cx 2 • 2932. y~c.x
2 (x
X
1-t-C,ex
1-C.eX; y=C. 2933. x=C,+Jn ,y-C I
u+C •1. 2934. X=C.-c-.ln
2 \ y+C
!I
•I .
2935. x=C 1y2 -t-ylny-j-C 2 • 2936. 2y 2 -4x 2 =1. 2937. y=x-/-1. 2938. y~~
x2 - l e2 - l 1-x• e2 +1 I
(e•-t) - 4 -lnjxjory= 2 (e'+l)+ 4 -1nlxl. 2939. y - 2 x•.
2
2
I 3 -1
2940. u= x•. 2941. y=2ex. 2942. X=- (y-f-2) . 2943. y ·ex.
2 2
2944.
e e-x
u·=·e-1+1-e" 2945.
2
y=-:~-x·-~·
Jf2 +- 8
2946.y·~
~ ~
= +eax. 2947. y=sec x. - I .
2
2948. y~sinx+l 2949. y
2 4 2
2950. x=-fe-Y•. 2951. No solution. 2952. y=ex. 2953. y=o·21nlxl-~-.
(x+ c• + 1)2 4 -=-
2954. y= ~ -t- C1 (x+ I) 2 -j-C 2 • Singular solution, y=C. 2955. y =
3
(C 1 -C~)x-j-C 2 •
2
13
·==C 1 ; + Singular solution, y=(xt ) +C. 2956. y=
2
1
·=i2(C 1 +x) 4 -t-C 2x-t-C 3 • 2957. y=C 1 +C 2ec,x; y=l-ex;y=-l+e-x;

·singular solution, Y=c 4


x. 2958. Circles. 2959. (x-C 1 ) 2 -C 2 y 2 -f-kC~ ~o.
x-x
2960. Catenary, y=a cosh T· Circle, (x-x 0) 2 -j-y 2 =a 2• 2961. Parabola,
2 2
(x-x 0) =2ay-a • Cycloid, x-x 0 =a (t-sin t), y=a (I-eos t). 2962. eaY+C,=
C H !Lt 1 H _!i_x
=sec (ax-t-C 1). 2963. Parabola. 2964. y= <} qeH + 2C, q e H +C 2 =aX
Answers 467

xcosh x-t-C -f-C 2 , where H 1s a constant honzontal tension, and !!_=a. Hint.
a q

The differential equation :;;=!Ji Vt + (:~r. 2965. Equation of motion,


ds
2
gt• m
dt 2 =g(sina-)1cosa). Law of motion, s= (sinu-)1cosu) 2966.
2 s=k x

xlncosh(t yg !). Hint. Equation of motiOn,m:;!=mg-k (~r- 2967. In

6.45 seconds. Hint. Equation of mot10n, ft d"x


300
dt·=-10 v. 2968. a) No, b) yes,
c) Yl'S, d) yes, e) no, f) no, g) no, h) yes 2969. a) 1/'-f-y=O; b) y"-2y'+y~O;
c) x'y"-2xy' -t-2y=O, d) y"'-3y"-t-41J'-2y~O 2970. u=3x-5x 2 -t-2x 3 . 2971. y=
=_!_(C 1 smx-t-C 2 cosx). Hint. Use the substitutiOn y=y,u. 2972. y=C 1x-f-
x
2
-f-C 2 1nx. 2973. y=A-t-Bx'+x 3 • 2974. y=!_ -t-Ax+'!_· Hint. Particular so-
3 x
I
lullons ~ the homoi-(eneous equatiOn y, ~= x. 'I··
• •
_cc-
X
• By the method of the

variation of parameters we ftnd: c,~i-t-A. C2 =-f+B 2975. y=A+


-I B Sill x -1 C cos x -t-In/ sec x +tan xI+ ~in x In! cos\" 1-x ,·os x. 2976. y = C,e•x +
+ C 2e -~ 3
2977. !I ~c C,e- 3 " -1- C,e". 2978. y = C 1 C 2 e" 2979. y = C, cosx C 2 sitlX.+ +
2980. y~e-~(C 1 cosx-I-C 2 sHIX) 2981. y=e-•x(C 1 co~3r-I-C 2 sm3x) 2982. !J=

- (C, + Ccx) e-x. 2983. !I 0~ e 2


·' (C.\e t I 2 + c.e -xl'2). 2984. If fl > 0, y=
=- C,e"' 1 k + C,e-.tl -;;; 1f k < 0. y-~C, cosY -kx + C 2 sm Y -kx.
--
x 1·--:;-
-X
5

--- X -
< ( .r-
I' II
r- )
~ II
298'i.y ~e 2 2 2 6
(C 1e -I-C 2e ) 2986. y=e C, cos --yx+C 2 sin--x .
6

. ~ 1e"' +e'"'. 2988.1/=C-x.


2987.11 . 2989. y=sin2x. 2990. y= 1. 2991. y=a cosh!_.
a
2992. IJ~o() 2993. y=Csinnx 2994. a) xezx(Ax 2 -t-Bx+C); b) Aco~2<1-
·I-B'iin2x; c) Aco~2\"-I-Bsnl2r-\-C\" 2e 2 x; d) ex(Acosx-\-B~in\), c) eX;<
X (!h" -I- BY+ C) -j- xe"~ (Dr+£); f) xe"' [(Ax 2 Br +C) c:os 2x+ (Dx'+Ex I-F) X +
X sm 2x) 2995. !I= (C, -I C,x) e•x + ~ (2x 2
+ 4x + :3). 2996. y =e+ ( C,cosx ~3 +
+C 2
.· X -
stn- Y 3 ) +x
2
$ ., 2
-1-·>.\. 2997 . y=- (C 1 - 1- c 2 X•) ~ -X - 1- 9I t' ·-~ ·
2998. y ..oC 1ex I-C 2e 7 -~-l-2 2999. y~c,ex-j-C 2e-·"-l-{xe·~. 3000. !J=C,cosx+

-f- C 2 Sill X -1- ~ X Sill X. 3001. !J = C 1t'x -j- C2e- 2-"- ~ (:1 Slll 2X -f-COS 2x). 3002. !f =

= C 1e•-~ + C e-ax + x (\ Tii-


2
x 2SI ) e•·"' . 3003 . y -- (C 1 + C x) ex + 2I
2
• x.
cos + x•4 ex -

- *e-x 3004. y = C 1 -j- C 2e-X -f-i-X -j- ~ (2 COS 2x-sin 2X). 3005. y =eX 'X,

x (C 1 cos 2x +C 2 sin 2x) + Tex sin 2x. 3006. y-= cos 2x +-} (sin x+ sin2x).
16*
468 Answers

A
-8007. 1) X-== C1 COS cot -t- C1 sin cot -t- - 1- -1 sin pt; 2) X= C1 COS cot -t- C1 sin cot-
co-p
A x x• x•
- 2co t •COS cot. 3008. Y= Cle " + Cae'"-xe"'· 3009. y = cl + Cae 2" + 4-4- 6 .
1

I 5
-3010. y=e" (C 1 -t-CzX-t-x 2 ). 3011. y=C1 -t-Cae 2" -t- xe 2" - 2 x. 3012. y =
2
=C 1e- 2"-t-C2e'"-! e"+ ~ (3cos2x-t-sin2x). 3013. y=C 1 -t-C 2e-"+e" +

+ 26 x -5x.
2
3014.y=C 1 -t-C 2e"-3xe"-x-x 2 • 3015.y= ( C1 -t-C1 x-t- I x 2 .
2 )x
1 I X
xe-x -t-4e". 3016. I/=(C 1 cos 3x-t-C1 sln 3x) e" -t- (sin 3x-t-6cos3x)-t-~ .
37
x-t-I 1
3017. y=(C 1 -t-C 1x-t-x1 ) e2" -t---. 3018. y=C 1 -t-C 2e'"-ro (cos x-t-3sinx)-
8
x• x I x• x 2 x
- - . 3019. Y=ge1"(4x-t-I)- - -t- . 3020. y=C 1e"-t-C 2e-"-
6 9 6 4 4
e2"
- x sin x-cos x. 3021. y = C1e- 2" + C2e2" - (sin 2x + 2 cos 2x). 3022. y=
20
c: cl cos 2x + Ca sin 2x- f (3 sin 2x + 2 cos2x) +~. 3023. y =e" t~l cos X+
+ C1 sin x-2x cos x). 3024. y=C 1e" + C2e-" +~ (x 1 -x)e". 3025.y=C1 cos3x-t-
l . I I _ I
+C1 sin3x+ x sm x- cos x+ (3x- I) e•". 3026. y =C 1e1" -t-C 2e x + x
4 16 54 9
1 3 3
(2x 2 -x) e•". 3027. y=C 1 -t-C 2e2"-2xe"-
4 x- 4 x 3028. y =
1
X (2-3x)-t-
16
-= ( C1 -t-C1x + x~) e2". 3029. y=C 1e-•" +Cae"-~ (2x1 -t-x)e-•" +~X
X1
sin x- B cos 3x +
X X
)( (2x1 -t-3x) e". 8030. 11= C1 cos x + C2 sin x -t- cos x +
4 4
+ ~sin ax. Hint. Transform the product of cosines to the sum of cosines.
8031. y= C1e-x YJ:" -t-C 2e" Yl -t-xe" sin x +e" cos x. 3032. y =C 1 cos x + C1 sinx+
i
+ cos x In / cot ( + ~ ) /· 3033. y = C1 cos x + C1 sin x +sin x ·In tan ~ I I·
3034. y=(C1 -t-C 2x)e" -t-xe" In I xI· 3035. y=(C 1 -t-C 2x)e-"-t-xe-x In 1x j.
3036. y= C1 cos x-t- C1 sin x + x sin x -t-cos x In 1cos xI 3037. y =-=C 1 cos x +
+ C1 sin x-x cos x +sin x In I sin xI· 3038. a) y = C1e" + Cat-x + (e" -t-e-")X
y- y-
xarc tane"; b) y=C 1e" 1 +C.e-x 1 -t-e"1 • 3040. Equation of motion,
i-(~:.)=2-k(x-t-2); (k=l); T=2n: Y-i sec. 3041. x-
2gsin30t-60 Ygsin Yi/
-= g-
900
em. Hint. If x Is reckoned from the position of
4
resf of the load, then Ix'=4-k(x,-t-x-y-l), where x0 Is the distance of
lhe point of rest of the load from the Initial point of suspension of the spring,
4 d1 x
I Is the length of the spring af rest; therefore, k (x0 -l) = 4, hence, - dt• -
d1 x g
--k(x-y), where ~-4, g=981 cmjsec 1 • 8042. mdil=k(b-x)-k(b-t-x)
Answers 469

andx=ccos(t f !)- 3043.6::~=gs;t= V~ln(6+Y35).3044.a)r=


2

=; (e"'1 +e-"'f); b) r= ;~ (emt_e-"'t) Hint. The differential equation of motion

is ::: = w2r. 3045. y = c, + c,ex + Cae12". 3046. y = c, + c.e-x + C,ex.

X(
3047. y=c,e-x+e2 c.cos--;x+C,sin
-v-· -v;-·/X )
3048. y = C, + C 2 x + C3 exl'2' -t- C 4 e-x l'z- 3049. y -=-ex (C, + C 2 x + C1 x2).
3050. y ~ex (C 1 COS X -j- C 2 sin x) +e-x (C1 COS X -j- C~ sin x)
3051. y = (C 1+ C2 x) cos 2x + (C1 + C4 x) sin 2x

3052. y =- C, + C~e -x -t-e~ ( C3 cos 2Jl3 x +C4 sm- . ¥3- x ) .


2
3053. y = (C 1 -j- C2x) e-.-.: -j- (C 1 -1- C4 ~)ex.
3054. y = C,eax + c-1!-ax +c. ['('S or+ c. sin ar
3055. y=(C 1 -j-C 2 x)e 1 ' 1 ~ +(C.-J C4 x)e_J'3'r. 3056. y=C 1 -j-C 2 x+
+c. CO~JIK -I- c. sin at. 3057. y ·° Cd- C2x -1- (Ca + c.t) e-X. 3058. y oo= (C, +
-I-C 1 x)cosx-j-(C 3 -j-C,x)sinx. 3059. Y--e-·-.:(C 1 -j-C 2 t-!- ... +C,.~:n- 1 ).
3060. y=C 1 -j-C 2 x l- ( C3 + C 4 x-j-~) ex.

3061. y=Cd-C.x-1 12x 2 -l·:lx 3 -j-~ .\ 4 -j-~.\ 5 i (C 3 -j-C 4 ~.:)cx.


I -~ (c y:l· C '>111 yr3 x ) -x •-:J,
306 2. y= C1ex -,-e 2 cos- -x-j
r::
2 3
2
3063. y=C,+C,x-j-C 3 x2 -j c.e-"+ 1 ~88 (4cns4x-sin4x)
30 64 .y._ C1c -x -1- C2 -,-1 C.
,x+ 3 x.2 -
2 3
I x.a + I
12 t
··I_J
Te
"(3. 15')
2x-4.
3065 . yo- C1e -x + C2 cc,sx-; C•''nx+e
. ~ ( 4-~
x :l) .
3066. y = C, + C2 cos x-!- C3 sin x -1 sec x +cos x In I cost 1-tan x sin x +x sin X.
-~2 ( J!.3 I Jl3 )
3067. y=e-x+e cos
2 -x+ y;rsin 2 x -\-x-2.
3068. y ,__ (C 1 + C2 In x) ,_I_ . 3069. y ~ C1 .t3 + ~.
X X
3070. 11 = C1 cos (21n x) + C 2 sin (21n x).
3071. y =' Cl'~: -1- C2x 2 -1- C8 x3 • 3072. y =- Cd- C2 (3x +2)-'/,,
3073. y~c,x:+c".
X
3074. y=C 1 cos(lnx)-!-C 2 sin(lnx).

3075. y=C 1x3 -j-C 2 x"+{x. 3076. y=(x-j-1) 2 [C 1 -!-C 2 1n(x-!-1)] t-(x-f-1) 3 •
3077. y=x(lnx-j-ln 2 x). 3078. y=C 1cosx-j-C 2 sinx, z=C 2 wsx-C1 sinx.
3079. y =e-x (C 1cos x+ C2 sin x), z = i, e-x [(C -2C1) cos x-(C + 2C
2 1 2) sin x).
3080. y= (C 1 -C1 -C1x)e-•x, z=(C1x-j-C 2 )e-•x.
470 Answers

3081. X=C 1 et +e -~ (c 2 COS


Y3 t +
2
c · 2Y 3- t) ,
a Stn

y = C1et+ e-{- (Cs ¥3-Ca cos -y3


- t - Ca Y3+Ca sm
. - - v:r t) ,
2 2 2 2
-c1e1 + e
z- -+(-c. ¥3-
2 2
· v3 t) .
C cos V:f t + c. V3-c •.sm
2
2 2
3082. x=C 1e- 1 +C 2e21 , yo...:C8e- 1 +C 2e21 , z=-(C 1 +C 8 )e- 1 +C 2e~ 1 .
3083. y=C 1 +C 2e2x-+ (x'+ x), z=C 2e2x-C1 ++ (x 2 -x-l).
3084. y= C1 +C 2x+ 2sin X, Z= -2C 1 -C2 (2x+ 1)-3 sin x-2cos X.
3085. y= (C 2 -2C1 -2C 2x) e-x-6x+ 14, z = (C 1 + C2 x) e-x+ 5x-9;
cl=9, c.=4,
y= 14 (l-e-x)-2x (3+4e-x), z= -9 (l-e-x) +x (5+4e-x).
3086. x= 10e21 -8e 81 -e1 + 6t--l; y = -20e2 1 +Be81 + 3e1 + 12/ + 10.
2C 1 _ C1 * (x 2 +y 2 )y C Z
3087. Y=(c )"' Z-c--. 3088. a) = I• -!/~c.;
2 -x 2 -x x ,.
2
b) In Yx 2 +y 2 =arc tan lt._+C 1 , Y
=C 2 • Hint. Integrating the homo-
x
x"+ y•
geneous equation !!!._= d+x , we find the first rntegral In Y x2 -f-p 2 ~~
x-y x y
= arcx tan lt..
X
+ C,. Then, us111g the properties of derivative proportions, we havl'
x dx + y dy
dz
-= (
Z
x dx
)
X X-y
= y (yX dy
-f- y
) X 2 + lj 2
I
Whence lnz=- ln(x 2 +Y 2 )-l-lnC 2 and,
2
2
hence, V =C 2 ; c) x+y+z=O, x2 +y'-f-z 2 =6. Hint. Applying Ibe-
x•+ y•
. . . . dx dtJ dz dx+du-1-dz .
properl!esofdertvatJve proporlions,wehave --=---'--=--=
y-z z-x x-y 0
whence dx+dy-f-d<t=O and, consequently, x+y+z=C 1 • Stmilarly, (xdx )=
x y-z
y dy z dz x dx + 11 dy + z dz ., X dX _,_ lj d I} -t Z d Z ·-__ 0 !Ill!1 X a+ lj z _
yz-x
( ) zx-y
( ) 0 1
· 1
+z 2 = C2 • Thus, the integral curves are the ctrcles x + y + z ~ C,, x• + y 2 + z2 - C.
From the initial conditions, X= I. y=l, Z=-=-2, we will have Cl=O, c.=G.
C x2
3089. y=C 1x2 +-f- (3ln 2 x-2lnx),
18
z = I-2C 1 x + :~ +-§- (3 In' x +In x-1).

3090. y=C 1 e~v.- +C 2e-xVi"+C3 cosx+C4 sinx+ex-2x,


x Vi- -xV2 C3 C, . I
z=-Ce I
-Ce
2 ---
4 cosx--smx--ex+x
4 2 •

3091. X=
v0mcosa (
k
_.!.!_
1-e m
1) m ( _k_
, Y=Jil(kv 0 sina+mg) l-e m
t) --k-.
mgt

Solution. m d:t = -kux; m ~:J = -kuy-mg for the initial conditions: when
Answers 471

t=O, x0 =y 0 =0, l'x


0
=v 0 cosa, v 1, 0 =v 0 sina. Integrating, we obtain Vx=
k k
- -- I -- t ll
=VoCOsae Ill' kvy+mg=(kvosina+mg)e m . 3092. X=acos Ymt, y=

v 0 Yin k x" k"11 2


= -- - sin--:_:: t, --;;+-·-~I. Hint. The differential equations oi motwn:
k mvz Vm u· 0
2
d''< .. d 1! 2
m dt" = -k·x, m dt;- -~ -k y.

3093. y~ -2-2'<-X2 • 3094. Y= (Yo+{) e2 IX-t)_~ x++.


_ I I . I I 3 9 21
'3095
' • y- 2 + -4 X + 1f + X
2
j6 X 32 + X
4
+ 320 X 5
-/- • • •

3096 2
. II c ' 3I X -7.9 I
X +
3 7 II
7. II . 27 X - •••

x2 x• x•
3097. y~x.+j:2+ . + • +
2 3 3 4 ... ;the series converges for -l<x<l.
2 3 4
3098. Y---'X-(I-;) 2 • 2 +( 2 ~) 2 • 3 -( 3 ~) 2 • 4 \- ... ; the series conven~es for -oo<
<x< + oo. Hint. Use the method of undetermmed coeffictents.
I 1·4 I· 4 ·7
+
3099. 11 . .=.1 - :fi x 3 6! x6 - - -1- x 9 ••• ; the serie~ converges for- oo <x<+ oo.
9
3100. 1/-= sin
• X
X. Hint. u~e the method of undetermined copffictrnts.

0
x' x
3101. y= 1-22+ 22 . 4 . - 2 ,. 4 ,. 6 ,-1- ... ; the senes converges for I'< I< ro.

Hint. u~e the mdhod of unddrrmined coefflctents.


2!
3102. x - a ( 1-_1__ 12 +
2 •-
+;nt
6
9!1 o + 55!t • - . . .) . 31 03 . r1= A cos-
8
a:rrt . :rrx H' t
-sntT. tn. Use the condi-
1
. ( 0) A . :rrx iJu ( x. 0) 0
tton~:u(O,I)c"O,u(l.t)~-O,ux, = smT, dt --.

2/
3104. u , -.-
L -I (I-eos
00

. n:rrat . mrx
rm) sm - - sm - - . Hint. Usc the conditions:
rr·a tt 2 1 1
tt=l

u(O, t)=O, u(l, 1)=0, u(x,0)-=0,


au (x.
dt
0)
I.

"' I
8/r ""' n:rr n::rat . rmx
3105. u = :n: 2 ~ fi2 sin Hint. Use the condiltons:
2 ens - 1- su1 - 1- .
n == 1
2/tr I
iJu (x, O) = 0
r)t - ' lL
(0 t)
' =
0
' !l
(I t)
' = 0' U (x, 0) = } (
f
x \
- -
1
for O<x ~ ~- ,
I
~ 2/r t - )
1 for-;y < x < l.
00

3106 • U
'\._,
= ~ A nCOS - +-I)-a:rrt- Sl1l. (2rt +I)
(2n- t :rrx , wl!erc tltc Clle
ffiICIPII t S llr&
• =-=
21 2
n==o
472 Answers

=y Sysm
x · (2n+I):nxd x.
2 H1'nt. Use the conditions
21
0
iJu(l, /) iJu (X, 0)_
u (0, t)=O, ax= O, U
(
X,
O) X
[' iJt - 0•
oo a2 n2 n 2 t
400~ I . n:nx -~
3107. u = 1ii" ~ na (I-eos n:rt) sm 100 • e .
n=t
Hint. Use the conditions: u(O, 1)=0, u(IOO, l)=O, u(x, O)=O.Oix(IOO-x).

Chapter X
3108. a)~ I"; ~0.0023°{0 ; b)~ I mm; ~0.26°/ 0 ; c)~ I gm; ~0.00160{ 0 •
3109. a) ~ 0.05; ~ 0.021°( 0 ; b) ~ 0.0005; ~ 1.45°/ 0 ; c) ~ 0.005; ~ 0.16°/ 0 •
3110. a) two decimals; 48·10' or 49·10 3 , since the number lies between 47,877
and 48,845; b) two decimals; 15; c) one decimal; 6· 102 • For practi~1 ·purpo~es
there is sense in writing the result in the form (5.9±0.1)·102 • 31 II. a) 29.5;
b) 1.6·102 ; c) 43.2. 3112. a) 84.2; b) 18.5 or 18 47 ±0.01; c) the result of
subtraction does not have any correct decimals, since the difference is equal
to one hundredth with a possible absolute error of one hundredth.
3113'~~. 1.8±0.3 cm 2 • Hint. Use the formula for increase in area of a square.
3114. a) 30.0±0.2; b) 43.7±0.1; c) 0.3±0.1. 3115. 19.9±0.1 m 2 •
3116. a) 1.1295±0.0002; b) 0.120±0.006; c) the quotient may vary between
48 and 62. Hence, not a single decimal place in the quotient may be comid·
erect certain. 3117. 0.480. The last digit may vary by unity. 3118. a) O.li29;
b) 277·10 3; c) 2. 3119. (2.05±0.01)·10 3 cm 2 • 3120. a) 1.648; b) 4.025±0.001;
c) 9.006±0.003. 3121. 4.01·101 cm 2 • Absolute error, 6 5 cm 2 • Relative error,
0.16°/ 0 .3122. The side is equal to 13.8+0.2 em; sina=0.44±0.0l,a~-"26°!5'±
±35'. 3123. 2 7 ±0.1. 3124. 0.27 ampere 3125. The length of the pendulum
should be measured to within 0.3 em; take the numbers :-t and q to three
decimals (on the principle of equal effects). 3126. Measure the radii and the
generatrix with relative error 1/300. Take the number :n to three decimal places
(on the principle of equal effects). 3127. Measure the quantity 1 to witlun
0.20/0 , and s to within 0.7°/ 0 (on the principle of equal efTects).
3128.

X
I y
I !J.y
I tJ.2y
I tJ..3y
I !J.•y
I l!sy

I
I 3
I 7
I -2
I -6
I 14
I -23
2
l 10
I 5
I -8
I 8
I -9
I
3
I 15
I -3
I 0
I -1
I I
4
I 12
I -3
I -1
I I I
5
I 9
I -4
I I I I
6
I 5
I I I I I
Answers 473

Jt29.

X
I y
I M I ().2y
I ().'y

I
I -4
I -12
I 32
I 4!l

--
3
I -16
I 20
I 80
I 4.'1

fi
I 4
I 100
I 12R
I ·18

7
I 104
I 228
I 176
I
9
I 332
I 40t
I I
II
I 7:36
I I I
••
3130.

I :l I -Hi'! I -1/)!

1--~--r -a~~-r----:!~.;- i -.,


,_
_
1 ti
- - - - - - - I- - -
~-~~~~--r- ;~~-- --=2~~- --,-~-
1 1
-·--

1 ~--~~,- -
1
1
~--~--;, 1 --, ~--1-:lti---!--1-~-,4------,~---~-:.!0___ T----;-;----
. ~------'----=~:~~~~~-----:l8--7-l-2-9·1---'--l-l-·ll---,- - - -

~ I ---: ~~ I ;):'. .! I 43 ~ I I
~~ --~---__::;:;:;--1- 7' 0 I I ,---
--~;--l- 2:lu-l I f I
Hint. Compute the first live values of y and, after obtaining (). 4y 0 ~~24, repeat
the number 24 throughout the column of fourth differences. After this the
remaining part of the table is tilled U1 by the operation of addit.o.J (moving
from right to left).
474 Answers

3131. a) 0.211; 0.389; 0.490; 0.660; b) 0.229;0.399;0.491;0.664. 3132.0 1822;


0.1993; 0. 2165; 0 .2334; 0.2503. 3133. I +x+ x2 +x'. 3134. y =
1 4
96
x - !!
x3 +

+ ~~ x 2 -~ x + 8; y =.:::: 22 for x = 5.5; y = 20 for x=-::::5.2. Hint. When computing


xfor y=20 takey 0 =11. 3135. The interpolating polynomial isy=x 2 -IOx+I;
y =I when x = 0. 3136. 158 kgf (approximate! y). 3137. a) y (0.5)-=- I,
y(2)=11; b) y(0.5)=-!~· y(2)=-3. 3138. -1.325 3139. 1.01.
3140. -1.86; -0.25; 2.11. 3141. 2.09. 3142. 245and0 019. 3143. 0.:~1 and4
3144. 2.506. 3145. 0.02. 3146. 0 24. 3147. I 27 3148. -1.88; 0 35; I 5:)
3149. 1.84. 3150. 1.31 and -0.67. 3151. 7.13. 3152.0.165.3153. 1.73and0.
3154. 1.72. 3155. I 38 3156. X=0.83; y=O 56; X"--'-0.83; !f=-0.56
3157. x= 1.67; y =I 22. 3158. 4 493. 3159. ± 1 1997 3160. Bv the trapezoi-
dal formula, 11.625: by Simpson's formula, II 417. 3161. -0 995; -1; 0.005;
0.5°/ 0; Ll-=0.005. 3162. 0.3068; il=l.:~.J0- • 3163. 0 69 3164. 0.79.
5

3165. 0.84. 3166. 0.28. 3167. 0.10. 3168. I 61. 3169. 1.85 3170. 0.09.
3171. 0.67. 3172. 0.75. 3173. 0.79. 3174. 4.93. 3175. 1 2!). Hint. Make use
of the parametric equation of the ellipse x~cost, y-==0.6222 sintr.:,.d tram-
n
2

fotm the formula of the arc length to the form ~ YI-e2 cos 2 t·dt, where e
0
xa xs x1 xs
is the eccentricity of the ellipse. 3176. Y1 (x) = 3, Yz (x) = 3 +53, Ya (x)-= 3 +
x1 2x 11 x 15 x2 x3 3x2 x•
+ 63-1-2079 +59535· 31 77. y 1 (x)=2-x+ I, Yz(X)=6+2 -x -1- I, Ya(X) ~-- 12-
3x' x' 7x 3
- 6 -r- 2 -x-t-1; z.(X)=--'3x-2, z2 (x)= 6 -2x2 +3x-2, z3 (x)= 6 -
X3

x3 x3 x 5
-2x2 -f-3x-2. 3178. y.(x)=x, y 2 (X)=x-6, Y 3 (x)~x-6+
120 .
3179. y(l)=3.36. 3180. y(2)=0.80. 3181. y(l)=3.72; z(l)o=2.72
3182. y=l.80. 3183. 3.15. 3184. 0.14. 3185. y(0.5)"~3 15; z(O 5)-'-3 15.
3186. y(0.5)=0.55; z(O 5)=-0.18. 3\87.1.16.3188.0 87.3189.x(n) -3.58;
x' (n) =0.79. 3\90. 429-1- 1739cos x-10:37 Sill x-6:321 ros 2x-t-1263 sm 2x-
-1242cos3x-33sm3x. 3191. 6 49-1 96cosx--j-2.14smx-1.6!lcos2x+
+
-1- 0.53 ~in 2x-1.13 cos 3x -1-0.04 sin 3x. 3192. 0. 960 0.851 cos x -f-0.915 sm x ·I·
+0.542cos2x--j-0.620sin2x-f-0.27lcos3x-!-0.100sin3x. 3193. a) 0 6U8~1nx 1-
+ +
0.076 sin 2x -1-0.022 sin 3x; b) 0.338 0.414 cos x -f-0.111 cos 2x 1-0.056 cos 3x.
APPENDIX

I. Greek Alphabet
Alpha-Aa Iota-h Rho-PQ
Beta-B~ Kappa-Kx Stgma-~cr
Gamma- ry Lambda-A/.. Tau- Tt
Delta-M Mu -Mft Upsilon- fu
Epsi\on-E<. Nu-Nv Phi-•D<p
Zeta-Z~ Xi-SE Cht-XX
Eta-Ill] Omicron-Oo Psi- '1'~,
Theta-eo Pi- Iln Omega- Qlu

II. Some Constants

Quantity
I X
I log x
I Quant It)
I
X
I log x

I
:t :1 \.t\5'1 0.4971:> -e 0.36788 i 56571
2:t 6.28318 0.79818 ez 7.3890fi 0.86!l59
Jl
--
2
l.fl7080 0.19612 v-e 1.64872 0.21715
l1
-
4
0.78540 I .89509 v-; 1 39561 0.14-176
I i .63778
-- 0.31831 1.50285 ,\J =loge 0.4342)
Jt
I 0.36222
Jt2 9.86960 0.99130 M =ln!O 2.30258
v-n I. 77245 0.24857 I radian 57°17' 45"
v; e
1.46459
2.7182R
0.16572
0.43429
arc I o
g
0.01745
9.81
2.24188
0.99167
476 Appendix

Ill. Inverse Quantities, Powers, Roots, Logarithms

X I I I
I
X
x• x' I I IV: IVtoxl v-llogx I
Vx Vtox IOOx (t,nan-
ttssas)
In x

1.0 1.000 1.000 1.000 1.000 3.162 1.000 2.154 4.642 0000 0.0000
1.1 0.909 1.210 1.331 1.049 3.317 1.032 2 224 4.791 0414 0.0953
1.2 0.833 1.440 I. 728 1.095 3.464 1.063 2.289 4.932 0792 0.!823
1.3 0.769 1.690 2.197 1.140 3.606 1.091 2.351 5.066 1139 0.2624
1.4 0.714 1.960 2.744 1.183 3.742 1.119 2.410 5.192 1461 0.3365
1.5 0.667 2.250 3.375 1.225 3.873 1.145 2.466 5.313 1761 0.4~55
1.6 0.625 2.560 4.096 1.265 4.000 1.!70 2.520 5.429 2041 0.4 00
1.7 0.588 2.890 4.913 1.304 4.123 I !93 2.571 5.540 2304 0.5306
1.8 0.556 3.240 5.832 1.342 4.243 1.216 2.621 5.646 2553 0 5878
1.9 0.526 3.610 6.859 I 378 4.339 I. 239 2.668 5.749 2788 0.6419
2.0 0.500 4.000 8.000 1.414 4.472 1.260 2.714 5.848 30t"V 0.6931
2.1 0.476 4.410 9.261 1.449 4.583 I. 281 2.759 5.944 3222 0.7419
2.2 0.454 4.840 10.65 1.483 4.690 1.301 2 802 6.037 3424 0.7885
2.3 0.435 5.290 12.17 1.517 4.796 1.320 2.844 6.127 3617 0.8329
2.4 0.417 5.760 13.82 1.549 4.899 1.339 2.884 6.214 3802 0.8755
2.5 0.400 6.250 15.62 1.581 5.000 1.357 2.Q24 6.300 3979 0.9163
2.6 0.385 6.760 17.58 1.612 5.099 1.375 2.962 6.383 4150 0.9555
2.7 0.370 7.290 19.68 1.643 5.196 1.392 3.000 6.463 4314 0.9933
2.8 0.357 7.840 21.95 1.673 5.292 1.409 3.037 6.542 4472 1.0296
2.9 0.345 8.410 24.39 1. 7035.385 1.426 3.072 6.619 4624 1 0647
3.0 0.333 9.000 27.00 I. 7325.477 1.442 3.107 6.694 4771 I 0986
3.1 0.323 9.610 29.79 1.761 5.568 1.458 3.141 6.768 4914 1.1314
3.2 0.312 10.24 32.77 I. 7895.657 1.474 3.!75 6.840 5051 1.1632
3.3 0.303 10.89 35.94 1.817 5.745 1.489 3.208 6.910 5185 I. !939
3.4 0.294 II. 56 •39.30 1.844 5.831 1.504 3.240 6.980 5315 1.2238
3.5 0.286 12.25 42.88 1.871 5.916 1.518 3.271 7.047 5441 1.2528
3.6 0.278 12.96 46.66 1.897 6.000 1.533 3.302 7.114 5563 1..2809
3.7 0.270 13.69 50.65 I. 9246.083 I. 547 3.332 7.179 5682 1.3083
3.8 0.263 14.44 54.87 1.949 6.164 1.560 3.362 7.243 5798 1.3350
3.9 0.256 15.21 59.32 1.975 6.245 1.574 3.391 7.306 5911 1.3610
4.0 0.250 16.00 64.00 2.000 6.325 1.587 3.420 7.368 6021 1.3863
4.1 0.244 16.81 68.92 2.025 6.403 1.601 3.448 7.429 6128 1.4110
4.2 0.238 17.64 74.09 2.0~9 6.481 1.613 3.476 7.489 6232 1 4351
4.3 0.233 18.49 79.51 2.0 4 6.557 1.626 3.503 7.548 6335 1.4586
4.4 0.227 19.36 85.18 2.098 6.633 1.639 3.530 7.606 6435 I. 4816
4.5 0.222 20.25 91.12 2.121 6.708 1.651 3.557 7.663 6532 1.5041
4.6 0.217 21.16 97.34 2.145 6.782 1.663 3.583 7.719 6628 1.5261
4.7 0.213 22.09 103.8 2.168 6.856 1.675 3.609 7.775 6721 1.5476
4.8 0.208 23.04 110.6 2.191 6.928 1.687 3.634 7.830 6812 1.5686
4.9 0.204 24.01 117.6 2.214 7.000 1.698 3.659 7.884 6902 1.5892
5.0 0.200 25.00 125.0 2.236 7.071 1.710 3.684 7.937 6990 1.6094
5.1 0.196 26.01 132.7 2.258 7.141 1. 721 3.708 7.990 7076 1.6292
5.2 0.192 27.-04 140.6 2.280 7.211 1.732 3.733 8.041 7160 1.6487
5.3 0.189 28.09 148.9 2 302 7.280 1.744 3.756 8.093 7243 1.6677
5.4 0.185 29.16 157.5 2.324 7.348 1. 754 3.780 8.143 7324 1.6864
Appendix 411

Continued

X
I I I
I
X ~' x'
I v-x I Iv-lv-!v-j•ogx I
Vtox x · lOx lOOx (~an-
tss~as)
In x

5.5 0 182 30 25 166 4 2.345 7.416 I. 765 3.803 8.193 7404 1.7047
5 6 0.179 31 36 175.6 2 366 7 483 I 776 3.826 8.243 7482 I 7228
5.7 0.175 32 49 185 2 2 387 7 550 1.786 3.849 8.291 7559 I. 7405
5.8 0.172 33.64 195.1 2 408 7 616 1.797 3.871 8.340 7634 1.7579
5.9 0 169 34 81 205.4 2 429 7.681 1.807 3 893 8 387 7709 1.7750
6.0 0.167 36.00 216.0 2.449 7 746 1.817 3.915 8.434 7782 I. 7918
6.1 0.164 37 21 227.0 2.470 7.810 I 827 3.936 8 481 7853 I. 8083
6.2 0 161 38.44 2:~8 3 2 490 7 874 1.837 3.958 8 527 7924 1.8245
6.3 0 159 39.69 250 0 2.510 7.937 1.847 3.979 8 57:3 7993 1.8405
6 4 0.156 40 96 262.1 2.530 8.000 1.857 4.000 8 618 8062 1.8563
6.5 ,;~154 42.23 274 6 2.550 8.062 1.866 4 021 8.662 8129 1.8718
66 0.151 43.56 287 5 2 569 8.124 I 876 4.041 8.707 8195 1.8871
67 0.149 44 89 300 8 2 588 8.185 I 885 4 062 8 750 8261 I 9021
68 0.147 46 24 314 4 2 608 8.246 1.895 4.082 8 794 8:325 I 9169
69 0 145 47 61 328 5 2.627 8 307 1.904 4. 102 8.837 8388 I 9315
7.0 0.143 49.00 343 0 2.646 8.367 1. 91:3 4.121 8 879 8451 I 9459
7.1 0. 141 50.41 357.9 2 665 8 426 1.922 4. 141 8 921 8513 1.9601
7.2 0.139 51.84 373.2 2.683 8.485 1.931 4.160 8.963 8573 I. 9741
7.3 0 137 53.29 389.0 2.702 8.544 1.940 4.179 9.004 8633 1.9879
7.4 0.135 54 76 405.2 2.720 8 602 1.949 4.198 9.045 8692 2.0015
7.5 0.133 56.25 421.9 2.739 8.660 1.957 4.217 9.086 8751 2.0149
7.6 0.132 57.76 439 0 2 757 8.718 1.966 4.236 9.126 8808 2.0281
1.7 0 1:\0 .')9 29 456 5 2 775 8.775 1.975 4.254 !).166 8865 2.0412
7.8 0 128 60.84 474 6 2. 793 8.832 1.983 4 273 9.205 8921 2.0541
7.9 0 127 62.41 493.0 2.811 8.888 I 992 4 291 9.244 8976 2.0669
S.O 0 125 64.00 512 0 2.828 8.944 2.000 4.309 9.283 9031 2.0794
8.1 () 12~ 6ri.61 531.4 2 846 9.000 2 008 4.327 9.322 9085 2'.0919
8.2 0.122 67.24 551.4 2.864 9 055 2 017 4.344 9 360 9138 12.1041
8.3 0.120 68.89 571.8 2.881 9.110 2 025 4.362 9.398 9191 2.1163
8 4 0 119 70.56 592.7 2.898 9.165 2.033 4 380 9.435 9243 2.1282
8.5 0.118 72.2.3 614.1 2.915 9.220 2.041 4.397 9.473 9294 2.1401
8.6 0.116 7:3.96 636.1 2 933 9.274 2 049 4.414 9 510 9345 2.1518
8.7 0.115 75.69 658.5 2.950 9.327 2.057 4.431 9.546 9395 2.1633
8.8 0.114 77.44 681.5 2.966 9.381 2.065 4.448 9.583 9445 2.1748
89 0 I 12 79 21 705.0 2 983 9.43·1 2 072 4.465 9 619 9494 2.1861
9.0 0. Ill 81.00 729.0 3.000 9.487 2.080 4.481 9.655 9542 2 1972
9.1 0.110 82.81 753.6 3.017 9.539 2.088 4.498 9.691 9590 2.2083
9.2 0.109 84.64 778.7 3.033 9.592 2.095 4.514 9.726 9638 2.2192
9.:3 0.108 86.49 804.4 3.050 9.644 2.103 4.531 9.761 9685 2.2300
9.4 0.106 88.36 830 6 3 066 9.695 2.110 4.547 9.796 9731 2 2407
9.5 0.105 90.25 857.4 3.082 9.747 2.118 4.563 9.830 9777 2.2513
9.6 0.104 92.1'6 884.7 3.098 9.798 2.125 4.579 9 865 9~23 2.2618
9.7 0.103 94.09 912 7 3.114 9.849 2.133 4.595 9 899 9868 2 2721
9.8 0.102 96.04 941.2 3.130 9.899 2.140 4.610 9.933 9912 2.2824
9.9 0.101 98.01 970.3 3.146 9.950 2.147 4.626 9.967 9956 2.2925
10.0 0.100 100.00 1000.0 3.162 \0.000 2.154 4.642 10.000 0000 2.3026
478 Appendix
--------------'--'---------------
IV. Trigonometric Functions

x"

0
I
X
(rad oans)

0.0000
I •In

0.0000
A
I tan x

0.0000
I
col x

00
I COS X

1.0000
I
1.5708
l_
90
l 0.0175 0.0175 0.0175 57 29 0.9998 l. 55:33 89
2 0.0349 0.0349 0.0349 28.64 0.9994 l 5:i5q 88
3 0.0524 0 .052:i 0.0524 19.08 0.9986 1.5184 87
4 0.0698 0.0698 0.069::! 14 30 0.9CJ76 \.ii0\0 86
5 o.o8n 0.0872 0.0875 \1.43 0.9962 1.48:35 85
6 0.!047 0.1045 0.1051 9.514 0.994fi 1.4661 84
7 0.1222 0.1219 0.1228 8.144 0.9925 1.4~86 83
8 0.1396 0.1392 0.1405 7.115 0.9903 I 4:ll2 82
9 0.!571 0.156t 0 1584 6 314 0.9877 I 41:l7 81
10 0.!745 0.1736 0.1763 5.671 0.9848 I 3963 80
II 0.1920 0.1908 0.1944 5.145 0.9816 1.3788 .. 79
12 0.2094 0.2079 0.2126 4.703 0 9781 I 3614" 78
13 0.2269 0.2250 0.2309 4.3:31 0.974-t 1.34:i9 77
14 0.244'{ 0.2419 0.2493 4.011 0.9703 1.3263 76
15 0.2618 0.2588 0.2679 3.732 0.9659 1.3090 75
16 0.2793 0.2756 0.2867 3.487 0.96J:i 1.2915 74
17 0.2967 0.2924 0.3057 3.271 0.9563 1.2741 73
18 0.3142 0.3090 0.3249 3.078 0.9511 I 2566 72
19 0.3316 0.3256 0.3443 2.904 0.945G I 2:392 71
20 0.3491 o.:H2o 0.3640 2.747 0 9.197 I 2217 70
21 0.3665 0.3584 0.3839 2.605 0. 9:l:i6 1.2043 69
22 0.3840 0.3746 0.4040 2.475 0.9272 1.1868 68
23 0.4014 0.3907 0.4245 2.356 0.9205 1.1691 67
24 0.4189 0.4067 0.4452 2.246 0.9135 1.1519 66
25 0.4363 0.4226 0.4663 2.!45 0.9063 I 1345 6S
26
27
0.4538
0.4712
. 0 4384
0.4540
0 4877
0.5095
2.050
1.963
0.89R8
0.8910
I. 1170
1.0996
64
G:i
28 0.4887 0.4695 0.5.117 1.881 0.8829 1.0821 62
29 0.5061 0 4848 0.5543 1.804 0.8746 1_0647 !)I
30 0.5236 0.5000 0.5774 I 732 0.8660 1.0472 60
31 0.5411 0.5150 0.6009 1.66-t:l 0.8572 1.0297 59
32 0.5585 0.5299 0.6249 l.G00:3 0.8480 1.012:l 58
33 0.5760 0.5446 () 64Q4 1.5399 0.8:387 0.9948 57
34 0.5934 0.5592 0.6745 1.4826 0 8290 0.9774 56
35 0.6109 0.5736 0.7002 1.4281 u.8tq2 0.9599 55
36 0.6283 0.5878 0.7265 1.3764 0.8090 0.9425 54
37 0.6458 0.6018 0. 7536 1.3270 0. 7986 0.9250 53
38 0.6632 0.6157 0.7813 1.2799 0.7880 0.9076 52
39 0.6807 0.6293 0.8098 1.2349 0.7771 0.8901 51
40 O.ti981 0.6428 0.8391 1.1918 0.7660 0.8727 so
41 0.7156 0.6561 0.8693 1.1504 0.7547 0.8fi52 49
42 0.7330 0.6691 0.9004 1.1106 0.7431 0.8378 48
43 0.7505 0.6820 0 9325 1.0724 0.7314 0.8203 47
44 0. 7679 0.6947 0.9657 1.0355 0.7193 0.8029 46
45 0.7854 0.7071 1.0000 1.0000 0.7071 0. 7854 45

I I COS X

I
c·ot x
I
tan x
I
sin x
I (rad1a li~) I ""
Appendix 479

V. Exponential, Hyperbolic and Trigonometric Functions

e• .-3) cosh x tanh x sln x


X
I I I Slllh X
I I I I
cosx

0 0 loOOOO I 0000 000000 1.0000 000000 000000 1.0000


001 I. 1052 0 9048 001002 100050 000997 000998 009950
002 I 2214 0 8187 00201:3 I 0201 001974 001987 009801
0 :~ I.:H99 007408 00304.') 1.0453 002913 Oo29fi3 009553
I 004 I 04918 006703 004108 1.0811 003799 Oo3894 009211

05 I 6487 006063 0 fi211 lol276 004621 004794 008776


() 6 108221 00!1488 0 6367 101853 005370 005646 008253
0 7 200138 004!)66 0 7fi86 102552 006044 006442 007648
Oo8 2022;);) 0 4493 0 8881 I. 3374 006640 007174 0.6967
0 9. 2 4596 004066 100263 104331 0.7163 Oo7833 0.6216

I 0 2 0718:3 003679 I. 1752 I. 5431 0 7616 0 8415 005403
1.1 300042 0 :3329 1.3356 1.6685 008005 0 8912 004536
I 2 3 3201 003012 lo509fi I. 8107 0 8337 009320 0.3624
I 3 3 66 3 0 272.') 106981 lo!J709 008617 0 9636 002675
14 4 0352 0 2466 I. 9043 201309 0 8834 009854 001700

I 5 4 4817 () 22:~1 2012lrl 203524 009051 009973 0007071


l.tl 40 9:i:3o () 2019 2 3756 2 fi77'S 0 9217 009996 -000292
17 ~>047:39 () 1827 206456 2 828o3 009354 009917 -0.1288
I8 6o!Wl6 () 1653 209422 :3. 1075 009468 Oo 97:38 -0 2272
I 9 b 68!19 0 J.\'16 :3.2682 3 4177 () 9~162 () 9463 -003233

2 () 7 3801 () I:l:J:l :l 6269 :\ 7622 009640 009093 -0.4161


2 I 8 1662 0 122~1 4 021!1 4ol44:l 000704 008632 -0 5048
2 2 q 02SO () 11118 4 4S71 4 567!) 009757 0 8085 -005885
2 ;; 9 9742 () 1003 409370 .')00372 !UISOI 0 H:17 -006663
2 4 II 02:12 0 0907 504662 505569 00 98:37 0 67:13 -G 7374

2 G 12 1823 000821 600.'102 60132:3 009866 005985 -008011


26 1:30 4li.l7 0 0743 6 6'l47 607690 0 ~)890 li 5155 -008569
27 1108797 0 0672 704063 7 047:~5 0.9910 004274 -009041
208 16 4446 0 0608 8.1919 8 2:127 009926 00:3:350 -009422
29 1801741 000550 900596 9.1146 0.9940 0.23()2 -009710

:3.0 200085!1 000498 10 00179 1000677 009950 0.1411 -009900


301 220 1979 0.0450 11.0764 II. 1215 0.9959 0.0416 -009991
302 2405325 000408 1202459 12.23()6 0.9967 -000584 -009983
303 2701126 0003()9 1:3.537!) 13.5748 009973 -0 1577 -0.9875
3 4 29.9641 ()0 0334 14.9654 14.9987 0.9978 -002555 -009668
305 33.1154 0.0302 16.5426 16.5728 1 0.9982 -·003508 -0.9365
480 Appendix

VI. Some Curves (for Reference)


y
r

0 X
1. Parabola, 2. Cubic parabola, 3. Rectangular
y=x2, y=x'. ,.
hyperbola,

Y'-0-x·
'

-1 0 1 X

4. Graph of a fractional 5. The witch of Agncsi,


function, I
1 Y= 1 +x2.
y =--xs.

V-x.
6. Parabola (upper 7. Cubic parabola,
branch),
u=
u=Yx.
Appendix 481

y
y

8b Sem icubical
parabola,
y 2 =X 3 or I X=i 1'
X \ y=t•. X

Sa. Neile's p arabola,

• •

-f

9. Sine curve and cosine curve,


y-cs,ax antltJ=COSX.

10. Tangent curve and cotangent curve,


y =tan x and y =cot x.
Appendix

y
y=sec x y=cosec T
I '-.,I'-\ l.¥-1 l ..1 l _L_ I

I \
--'
J
\ i I \ \ L
I

~ /~ . .J /'Z .J
2
\ \ .....
' f
X
31t -1C
-2
1f 0 r!12 1t Jrr z:rr 5!r Jtr
-2 2 2

_L
I
/
r; :\ -t

-2
.1
I
/
''({_ \ I
...

·/ I \ \' I
I\ \' I
I I I -3
11. Graphs of the functions
•..
y -=sec x and y =cosec x..

y=arc sin x
X

12. Graphs of the inverse trigonometric functions


y =arc sin x and y =arc cos x.
Appendix 483

13. Graphs of the 1nverse tri!,;onometric functions


y=-arc tanx and y=arccotx.

14. Graphs of the exponential functions


y=e" and y=e-x.
486 Appendix

X 25. Bernoulli's lemniscate,


(x2 + y2)" = a• (x•- yz)
or r 2 = a 2 cos 2<p.
y

24. Strophoid,
z 2a+x
y =X a=x·
y

27. llypocyclotd (astroid),


x=acos•t,
{ y=a sin1 t
X 2

26. cycloid,
or A a + y a =a~-.
x=a (t-sin t), y
{ y=a(l-cost).

28. Cardioid, 29. Evolvent (involute) of the circle


r=a (1 +cos cp). x=a (cost +t sin t),
{ y=a (sin t-t cos e).
Appendix 487

.30. Spiral of Archimedes, 31. Hyperbolic spiral,


t=aq>. a
r=-.
q>

32. Logarithmic sp1ral, 33. Three-leafed rose,


t =ea·;. r=a sin 3q>.

34. Four-leafed rose,


r =a sin 2cp.
INDEX

A B
Absolute error 367 Bending point 84
Absolute value Bernoulli's equation 333
of a real number II Bernoulli's lemniscate 155, 486
Absolutely convergent series 296, 297 Beta-function I 46, I 50
Acceleration vector 236 Binormal 238
1\.dams' formula 390 Boundary conditions 363 •
Adams' method 389, 390, 392 Branch of a hyperbola 20, 480
Agnesi Broken-line method
Witch of 18, 156, 480 Euler's 326
Algebraic functions 48
Angle between two surfaces, 219
Angle of contingence 102, 243 c
Angle of contingence of second kind
243 Cardioid 20, 105, 486
Antiderivative 140, 141 Catenary 104, 105, 484
generalized 143 Catenoid 168
Approximate numbers 367 Cauchy's integral test 295
addition of 368 Cauchy's test 293, 295
division of 368 Cauchy's theorem 75, 326
multiplication of 368 Cavalieri's "lemon" 165
powers of 368 Centre of curvature 103
roots of 368 Change of variable 211-217
subtraction of 368 in a definite integral 146
Approximation in a double integral 252-254
successive 377, 385 in an indefinite Integral 113
Arc length of a curve 158-161 Characteristic equation 356
Arc length of a space curve 234 Characteristic points 96
Archimedes Chebyshev's conditions 127
spiral of 20, 65, 66, 105, 487 Chord method 376
.\rea in polar coordinates 155, 256 Circle 20, 104
Area in rectangular coordinates 153, of convergence 306
256 of curvature 103
Area of a pi ane region 256 osculating 103
Area of a surf ace 166-168, 259 Circulation of a vector 289
Argument 1 I Cissoid 232
Astroid 20, 63, 105, 486 of Diocles 18, 485
Asymptotl' 93 Clairaut's equation 339
1Pft horizontal 94 Closed interval 11
1t>ft inclined 94 Coefficients
right horizontal 93 Fourier 318, 393, 394
right inclined 93 Comparison test 143, 293, 294
vertical 93 Composite function 12, 49
Index 489

Coneave down 91 probability 19, 484


Concave up 91 sine 481
Concavity tangent 481
direction of 91 Cusp 230
Conchoid 232 Cycloid 105, 106, 486
Condition
Lipschit1. 385
Conditions D
boundary 363 D' Alembert's test 295
Chebyshev's 127 Decreasing function 83
Dirichlet 318, 319 Definite integral 138
initial 323, 363 Del 288
Conditional extremum 223-225 Dependent variable II
Conditionally (not absolutely) Derivative 43
convergent series 296 left-hand 44
Contingence logarithmic 55
angle of 102, 243 nth 67
Continuity of functions 36 right-hand 44
Continuous function 36 second 66
prop•er.ies of 38 Derivative of a function
Convergence in a given direction 193
circle of 306 Derivative of functions
interval of 305 represented parametrically 57
radius of 305 Derivative of an implicit functir.n 57
region of 304 Derivative of an inverse function 57
uniform 306 Derivative of the second order 66
Convergent improper integral 143, Derivatives
270 of higher orders 66-69
Convergent serie~ 293 one-sided 43
Coord111ales table of 47
of centre of gravity 170 Descartes
genrrali?ed pol;1r 255 folium of 20, 21, 232, 485
Correct decimal places in a broad Determinant
sense 367 functional 264
Correct decimal places in Determining coefficients
a narrow sense 367 first method of 122
Cosi ue curve 481 second method of 122
Cotangent curve 481 Diagonal table 389
Coupling equation 223 Difference of two convergent
Cntical point of the second kind 92 series 298
Critical points 84 Dift'erenti al
Cubic parabola 17, 105, 234, 480 of an arc 101, 234
Curl of a vector field 288 first-order 71
Curvature higher-order 198
centre of 103 principal properties of 72
circle of 10:3 second 198
of a curve 102, 242 second-order 72
radius of 102 total, integration of 202-204
second 243 Differential equation 322
Curve homogeneous linear 349
cosine 481 inhomogeneous linear 349
cotangent 481 Differential equations
discriminant 232, 234 first-order 324
Gaussian 92 forming 329
integral 322 higher-order 345
logarithmic 484 linear 349, 351
490 Index
Differential equations of higher powers coupling 223
first-order 337 differential 322
Differentials Euler's 357
method of 343 exact differential 335
of third and higher orders 72 first-order differential 324
Differentiating a composite function homogeneous 330, 351, 356
47 homogeneous linear differential 332,
Differentiation 43 349
of implicit functions 205-208 inhomogeneous 349, 351, 356
tabular 46 Lagrange's 339
Diodes Laplace's 289, 291
cissoid of 18, 485 linear 332
Direction of concavity 91 of a normal 60, 218
Direction field 325 of a tangent 60
Dirichlet of a tangent pi ane 218
conditions 318, 319 with variables separable 327, 328
function 40 Equivalent functions 33
series 295, 296 Error
theorem 318 absolute 367
Discontinuity 37 limiting absolute 367
of the first kind 37 limiting relative 367
infinite 38 relative 367
removable 37 Euler integral 146
of the second kind 38 Euler-Poisson integral 272
Discontinuous function 270 Euler's broken-line method 326
Discriminant 222 Euler's equation 357
Dicriminant curve 232, 234 Even function 13
Divergence of a vector field 288 Evolute of a curve 103
Divergent improper integral 143, 270 Evolvent of a circle 486
Divergent series 293, 294 Evolvent of a curve 104
Domain II Exact di!Terential equation 315
Domain of definition II Exponential functions 49, 55, 483
Double integral 246 Ext.remal point 84
in curvilinear coordinatrs 253 Extremum
in polar coordinates 252 conditional 223-225
in rectangular coordinates 246 of a function 83, 83, 222
Double point 230
f
E Factor
integrating 335
Elimination Field
method of 359 direction field 325
Ellipse 18, 20, 104, 485 nonstationary scalar or vector 288
Energy potential vector 289
kinetic 174 scalar 288
Envelope solenoidal vector 289
equations of 232 Field (cont)
of a family of plane curves 232 stationary scalar or vector 288
Epicycloid 283 vector 288
Equal effects Field theory 288-292
principle of 369 First-order differential 71
Equation First-order di!Terential equations 324
Bernoulli's 333 Flow lines 288
charactrristic 356 Flux of a vector fie! d 288
Clairaut's 339 Folium of Descartes 20, 21, 232, 485
Index 491

Force lines 288 logarithmic 49 _


Form transcendental, integration of 135
Lagrange's 311 trigonometric 48
Formula trigonometric, integrating 128, 129
Adams' 390 Fundamental system of solutions 349
Green's 276, 281, 282
Lagrange's 145
Lagrange's interpolation 374 G
Leibniz 67 Gamma-function 146, 150
Maclaurin's 77, 220 Gaussian curve 92
Newlon-Leibniz 140, 141, ~75 General integral 322
Newlon's interpolation ~72 General solution 359
Ostro!.!radsky-Gauss 286-288 General solution (of an equation) 323
parabolic 382 General term 294
Sun p~on 's 382-384 Generalized antiderivative 143
Stokes' 285, 286, 289 Generalized polar coordinates 255
TaYlor's 77, 220 Geometric progression 293, 294
lu1"pezoidal 382 Gradient of a field 288
Formula~ Gradient of a function 194, 195
reduction 130, 135 Graph of a function 12
Fumier_., c•_,effictents 318, 39:3, :394 Greatest value 85, 225, 227
Fourter series 318, 319 Green's formula 276, 281, 282
Four-leafed rose 487 Guld1n's theorems 171
Fraction
proper rational 121 H
l'unctton II
composite 12. 49 Hamtltonian operator 288
con! lf!UOUS 36 Harmonic series 294, 296, 297
continuous, properties of :38 Htgher-order d tfTerentl at 198
decreasing 83 Higher-order dtiTerential equatiom 345
Dtnchlet 40
Htgher-order parti at deriv alive 197
dt~conltnuous 270 Hodograph of a vector 235
('VCll 13
Homogeneous equations 330, 351, 356
of a function 12 Homogeneous linear diiTerenttal
equation 332. 349
impltctt 12
llyperbola 17, 18, 20, 48')
increasinl.( 83
Lagrange 223, 224 rectangular 480
multtple-valued II Hyperbolic functions 49, 484
periodic 14 i ntegr ali on of 133
sinl'le-valued II Hyperbolic spiral 20, 105, 487
vedor 235 Hyperbolic substitutions 114, 116, 133
Functional determinant 264 llypocycloid 283, 486
Functional series 304
Functions
algebraic 48 Implicit function 12
equivalent 33 Improper integral
P\ ponenli ::~I 49, 55, 483 conver~ent 270
hyperbolic 49, 484 divergent 270
hyperbolic, integration of 133 lmproprr multiple integrals 269, 270
inverse 12 Incomplete Fourier series 318, 319
Functions (coni) Increasing function 83
inverse circular 48 Increment of an argument 42
invrrse hyperbolic 49 Increment of a function 42
inverse trigonometric 482, 483 Independent vari<~ble II
linearly dependent 349 Indeterminate forms
linearly independent 349 evaluating 78, 79
492 Index
Infinite discontinuities 38 Interpolation formula
Infinitely large qt~antities 33 Lagrange's 374
Infinitely small quantities 33 Newton's 372
lnfinites 33 Interval
lnfinitesimals 33 of calculations 382
of higher order 33 closed II
of order n 33 of convergence 305
of the same order 33 of monotonicity 83
Inflection Interval (cont)
points of 9I open II
Inhomogeneous equation 349, 351, 356 table interval 372
Inhomogeneous linear differential Inverse circular functions 48
equation 349 Inverse functions 12
Initial conditions 323, 363 Inverse hyperbolic functions 49
Integral 322 Inverse interpolation 373
convergent improper 143 Inverse trigonometric functions 482,
definite 138 483
divergent improper 143 Involute of a circle 20, 106, 486
double 246 Involute of a curve 104
Euler 146 Isoclines 325
Euler-Poisson 272 Isolated point 230 •
general 322 Iterative method 377, 378, 380
improper multiple 269, 270
line 273-278
particular 322 J
probability 144 Jacobian 253, 264
singular 337
surface 284-286 I(
tri pie 262
Integral curve 322 Kinetic energy 174
Integral sum 138
Integrating factor 335 L
Integration Lagrange's equation 339
basic rules of 107 Lagrange's form 311
under the differential sign 109 Lagrange's formula 145
direct 107 Lagrange's function 223, 224
by parts 116, 117, 149 Lagrange's interpolation formula 374
path of 273, 274, 280 Lagrange's theorem 75
region of 246-248 Laplace equation 289, 291
by substitution 113 Laplace transformation 271
Integration of differential equation Laplacian operator 289
by means of power series 361, 362 Lamina
Integration of functions coordinates of the centre of gravity
numerical 382, 383 of a, 261
Integration of ordinary differential mass and static moments of a 260
equation moments of inertia of a 261
numerical 384-393 Least value 85
Integration of total differentials 202- Left-hand derivative 44
204 Left horizontal asymptote 94
Integration of transcendental functi· Left inclined asymptote 94
ons I35 Leibniz rule 67, 269
Interpolation Leibniz test 296, 297
of functions 372-374 Lemniscate 20, 105, 232
inverse 373 Bernoulli's 155, 486
linear I3, 372 Level surfaces 288
quadratic 372 L'Hospital-Bernoulli rule 78·82
lnde:t 493

Lima<;on of successive approxi_mation 384,


Pascal's 158 385, 389
Limit of a function 22 of tangents 377
Limit on the left 22 of undetermined coefficients 121,351
Limit on the right 22 of variation of parameters 332, 349,
Limit of a sequence 22 352
Limiting absolute error 367 Minimum of a function 84, 222
Limiting relative error 367 Minimum point 84
Lim its Mixed partial derivative 197
one-sided 22 Moment
Line of inertia 169
straight 17, 20 static 168
Line mtegral Monotonicity
application of 276, 28:3 intervals of 83
of the first type 273, 274, 277, 278 Muliiple-valued function II
Line integral of the second type 274, Multiplicities
275, 278-281 root 121
Linear difierential equations 349, 351
Linear equation 332 N
Linear interpolation 372
of a f~nrtion 13 nth deriYative 67
Linearly dependent functions 349 Nabla 288
Linearly independent functions 349 Napier's number 28
Lines Natural tnhedron 238
flow 288 Necessary condition for convergence
force 288 29:3
vector 288 Necessary condition for an extremum
Lipschitz condition 385 222
Logarithmic curve 484 Newton
Logarithmic derivative 55 tr1dent of 18
Logarithmic functions 49 Newton-Leibniz formula 140, 141, 275
logarithmic spiral 20, 21, 105, 106, Newton's interpolation formula 372
487 Newton's method 377, 379
M
Newton's serpentine 18
Niele's parabola 18, 234. 481
Maclaurin's formula 77, 220 Node 230
Mnclaurin's series 311, 31:3 Nonstationaryscalar or vector field 288
Maximum of a function 84, 222 Normal 217
Maximum point to a curve 60
Mean value of a function 151 equations of 218
Mean-valuc theorems 75, 150 principal 238
Mcan rate of change 42 Normal plane 238
.\\cthod Number
Adams' 389, 390, 392 Napier's 28
chord method 376 real II
of difTerentials 343 Number series 293
of elimination 359 Numerical integration of functions 382,
Method (cont) 383
Euler's broken-line 326 Numerical integration of ordinary
iterative 377, 378, 380 differential equations 384-393
Milne's 386, 387, 390
Newton's 377, 379 0
Ostrogradsky 123, 125
Picard's 384, 385 One-sided derivatives 43
reduction 123 One-sided limits 22
Runge-Kutta 385-387, 390 Open interval II
494 Index
Operator critical 84
Hamiltonian 288 stationary 222, 225
Laplacian 289 Polar subnormal 61
Order of smallness 35 Polar subtangent 61
Orthagonal surf aces 219 Potential (of a field) 289
Orthagonal trajectories 328 Potential vector field 289
Osculating circle 103 Power series 305
Osculating plane 238 Principal normal 238
Ostrogradsky-Gauss formula 286-288 Principle
Ostrogradsky-Gauss theorem 291 of equal effects 369
Ostrogradsky method 123, 125 Runge 383, 386
of superposition of solutions 353
p Probability curve 19, 484
Probability integral 144
Parabola 17, 20, 104, 105, 480, 485 Product of two convergent series 298
cubic 17, 105, 234 Progresswn
Niele's 18, 234, 481 geometric 293, 294
safety 234 Proper rational fraction 121
semicubical 18, 20, 234, 481 Proportionate parts
Parabolic formula 382 rule of 376
Parameters
variation of 332, 349, 352 Q
Parametric representation of
a function 207 Quadratic interpolation 372
Quadratic trinomial 118, !19, 12:~
Parti a! derivative
hirheg-order 197 Quantity
"mixed" 197 infinitely large 33
second 197 infinitely small 33
Partial sum 293
Particular integral 322
Particular solution 339 R
Pascal's limac;on !58 Radius of convergence 305
Path of integration 273, 274, 280 Radius of curvature 102, 243
Period of a function 14 Radius of second curvature 243
Periodic function 14 Radius of tor~ion 243
Picard's method 384, 385 Rate of change
PI nne of a function 43
normal 238 mean 42
osculating 238 Ratio (of a geometric progre<.sion) 294
rectifying 238 Real numbers II
tangent 217 Rectangular hyperbola 480
Point Rectifying plane 238
bending 84 Reduction formulas 130, 135, 150
critical (of the second kind) 92 Reduction method 123
of discontinuity 37 Region of convergence 304
double 230 Region of integration 246-248
extremal 84 Relative error 367
of inflection 91 Remainder 311
isola ted 230 Remainder of a series 293, 304
maximum 84 Remainder term 311
minimum 84 Removable discontinuity 37
singular 230 Right-hand derivative 44
stationary 196 Right horizontal asymptote 93
of tangency 217 Right inclined asymptote 93
Points Raile's theorem 75
characteristic 96 Root multiplicities 121
Index 495

Rose Solenoidal vector field 289


four-leafed 487 Solution (of an equation) 322
three-leaf!"d 20, 487 general 323, 359
Rotation (of a vector field) 288 particular 339
Rule Spiral
Leibniz 67, 269 of Archimedes 20, 65, 66, 105, 487
I'Hospitai-Bernoulli 78-82 hyperbolic 20, 105, 487
of proportionate parts 376 logarithmic 20, 21, 105, 106, 487
Runge-Kutta method 385-387, 390 Static moment 168
Hunge principle 383, 386 Stationary point 196, 222, 225
Stationary scalar or vector field 288
Stokes' formula 285, 286, 289
s Straight line 17, 20
Safety parabola 234 Strophoid 157, 232, 234, 486
Scalar field 288 Subnormal 61
Scheme polar 61
twelve-ordinate 393-39§ Substitutions
Second curvature 243 hyperbolic 114, 116, 133
Second derivative 66 trigonometric 114, 115, 133
Second dJ.fl'erenti al 198 Subtangent 61
Second-orde~ difierential 72 polar 61
Second partial derivative 197 Successive approximation 377, 385
Segment of the normal 61 method of 384, 385, 389
Segment of the polar normal 61 Sufficient conditions (for an extremum)
Segment of the polar tangent 61 222
Sef!:ment of a straight line 20 Sum
Segment of the tangent 61 mte;.;ral 138
Semicircle 20 partial 293
Semicubical parabola 18, 20, 2.l4, 41'1 of a series 293, 304
S!"ries of two convergent sen!"s 298
absolutely convergent 296. 297 Superposition of solutwns
with complex terms 297 pnnciple of 353
conditionally (not absolutely) Surface inte~ral of the first type 284
convergent 296 Surface integral of the second type 284
convergent 293 Surface intl"grals 284-286
Series (cont) Surfac!"s
Dirichlet 295, 296 level 288
divergent 293, 294 orthogonal 219
Fourier 318, 319
functional 304
harmonic 294, 296, 297 T
Incomplete Fourier :318, 319 Table
Maclaurin's 311. 313 diagonal table 389
number series 293 of standard integrals 107
operations on 297 Table interval 372
power 305 Tabular diiTE"rentiation 46
Taylor's 311, 313 Tacnode 230
Serpentine Tangency
Newton's 18 point of 217
Simpson's formula 382-384 Tangent 238
Sine curve 481 Tangent curve 481
Single-valurd function II Tangent plane 217
Singular integral 337 equation of 218
Singular point 230 Tang!"nts
Slope (of a tangent) 43 method of 377
Smallest value 225, 227 Taylor's formula 77, 220
496 Index
Taylor's series 311, 313 computing volumes by means of 268
Term evaluating a 265
general 294 in rectangular coordinates 262
remainder 311 Trochoid 157
Test Twelve-ordinate scheme 393-395
d' Alembert's 295
Cauchy's 293, 295 u
Cauchy's integral 295
comparison 143, 293, 294 Undetermined coefficients
Leibniz 296, 297 method of 121, 351
Weierstrass' 306 Uniform convergence 306
Theorem
Cauchy's 75, 326 v
Dirichlet's 318
Theorem (cont) Value
greatest 85, 225, 227
Lagrange's 75
Ostrogradsky-Gauss 291 least 85
Rolle's 75 mean (of a function) 151, 252
Theorems smallest 225, 227
Guldin's 171 Variable
dependent II
mean-value 75, 150
independent II
Theory Variables sep.arable
field 288-292 an equation with 327, 328
Three-leafed rose 20, 487
Torsion 243 Variation of parameters 332, 349, 352
Tractrix 161 Vector
acceleration 236
Trajedories
orthogonal 328 of binormal 238
Transcendental functions of principal normal 238
integration of 135 of tangent line 238
Transformation velocity 236
Vector field 288
Laplace 271 Vector function 235
Trapezoidal formula 382
Trident of Newton 18 Vector lines 288
Trigonometric tunctions 48 Velocity vector 236
integrating 128, 129 Vertex of a curve 104
Trigonometric substitutions 114, 115, Vertical asymptote 93
Vertices of a curve 104
133 Volume of a cylindroid 258
Trihedron
natural 238 Volume of ~olids 161-166
Trinomial
quadratic 118, 119, 123 w
Triple integral 262 Weierstrass' test 306
applications of 265, 268 Witch of Agnesi 18, 156, 480
change of variables in 263 Work of a force 174, 276, 277

You might also like